Sie sind auf Seite 1von 502

1-Which rectus muscle receives only one ciliary artery:

medial rectus
lateral rectus
inferior rectus
superior rectus

Explanation

All rectus muscles receive two ciliary arteries except the lateral rectus which receives just
one.

2-Which statement is false of the cavernous sinus:

occurs on either side of the body of the sphenoid bone

contains the internal carotid artery

is bound posteriorly by the ethmoid air sinus

contains the oculomotor nerve

Your answer was INCORRECT

Explanation

The cavernous sinus is bound posteriorly by the sphenoid air sinus. Other statements are
true.

3-The blood aqueous barrier is maintained by:

tight junctions of the outer pigmented ciliary epithelium

hemidesmosomes of the inner non-pigmented ciliary epithelium

tight junctions of the inner non-pigmented ciliary epithelium

zonulae occludentes of the inner non-pigmented ciliary epithelium

Your answer was INCORRECT


Explanation

The blood-aqueous barrier is maintained by the tight junctions of the inner non-
pigmented ciliary epithelium.

4-The anterior lens capsule contains:

crystallin alpha

crystallin beta

laminin

fibrillin

Your answer was INCORRECT

Explanation

Fibrillin is a major component of the lens capsule as well as the zonules. This helps
provide them with elastic properties.

Question 5
Which of the following is essential for unique genetic fingerprinting:
incomplete dominance
variable number tandem repeats
restriction fragment length polymorphisms
point mutations

Your answer was INCORRECT

Explanation

Genetic fingerprinting involves the detection of variable number tandem repeats (VNTR),
which are highly polymorphic sequences of DNA, the pattern of which on blotting is
unique for each individual.

Score: 17 %
Question 6 Part 3 – Tutor Mode
Which statement is FALSE regarding dark adaptation of the human eye:

the rods are more sensitive than cones in the dark


it is related to the time required to build up rhodopsin stores

it is biphasic in the normal population

wearing red glasses can shorten the dark adaptation time

it takes a shorter time than light adaptation

Your answer was INCORRECT

Explanation

Light adaptation takes about 5 minutes and dark adaptation about 20 minutes in the
normal population. It is biphasic. The dark adaptation time is related to the time required
to build up rhodopsin stores. Wearing red glasses can speed up dark adaptation because
red light stimulates rods only minimally in the light.

Score: 14 %
Question 7 Part 3 – Tutor Mode
Which of the following is FALSE regarding Leber's hereditary optic atrophy:

the enzyme affected is ATPase 6

it is mitochondrially inherited

it results in demyelination of the optic nerve

it is the result of point mutations

the papillomacular bundle is the last to be affected

Your answer was INCORRECT

Explanation

LHOA affects the papillomacular bundle most severely. Other options are true.

Score: 25 %
Question 8 Part 3 – Tutor Mode
Which statement regarding contact lens correction is FALSE:

in myopes, contact lenses reduce the need for convergence compared with spectacle
truncation of lenses provides greater stability

correction of astigmatism requires a toric lens

contact lens power is calculated from the spectacle correction and the back vertex
distance

keratoconus can be corrected by a hard contact lens

Your answer was CORRECT

Explanation

Contact lens correction of myopia increases the need for convergence because of the loss
of the base-in effect of the spectacle lenses. All other statements are correct.

Score: 22 %
Question 9 Part 3 – Tutor Mode
Which of the following statements is TRUE about macrophages:

they express constitutively high levels of MHC class II molecules

they secrete cytokines such as TNF/IL1

they circulate in the blood

they are short-lived cells

they are unimportant in the pathogenesis of autoimmune disease

Your answer was INCORRECT

Explanation

Macrophages are the tissue form of blood monocytes. They are long-lived phagocytic
cells, and are important mediators of the innate immune response. Macrophages are able
to phagocytose foreign particles, microbes and injured or dead host cells. They have
receptors on their surface - pattern recognition receptors, which recognise microbial
components. These cells also have surface receptors that recognise the Fc portion of
immunoglobulin as well as complement components. Binding of these receptors to their
ligands enhances phagocytosis of antigen; a process termed opsonisation. Phagocytosed
microbes are killed by macrophages that secrete reactive oxygen intermediates and
possess potent microbicidal properties. These cells also produce pro-inflammatory
cytokines such as IL1, IL6 and TNF.

Macrophages are also important antigen presenting cells. They express high levels of
MHC class II molecules on their surface when activated. These molecules present
exogenously derived antigen to helper T-cells. They also express co-stimulatory
molecules which play an important role in T-cell activation. These cells secrete IL12
which is the cytokine required to direct T-cell differentiation to the Th1 subset.

The macrophage response can cause host cell damage, and they play a major role in the
pathogenesis of diseases like rheumatoid arthritis, sarcoid, tuberculosis and tuberculoid
leprosy.

Score: 20 %
Question 10 Part 3 – Tutor Mode
Regarding HLA proteins:

HLA class II antigens are absent from macrophages

HLA tissue typing is carried out in all forms of transplantation to prevent rejection

HLA class I antigens are expressed on all cells with nuclei

HLA proteins are found within the cytoplasm of the cells

Your answer was INCORRECT

Explanation

HLA typing is not necessary for routine corneal grafts. HLA antigen is coded by
chromosome 6. It is found on the cell surface and can be divided into two classes:

 Class I is found on the surface of all nucleated cells. Its main function is to allow
cytotoxic T-lyphocytes (CD8+) to recognize and eliminate virus-infected cells.
 Class II is found on macrophages, dendritic cells, B lymphocytes and some
endothelial cells and its function is to initiate the immune response by interaction
with T helper cells (CD4+)
Score: 18 %
 Question 11 Part 3 – Tutor Mode
 Where is the image formed by an object lying between the centre of curvature and
the principle focus of a concave mirror:

 between the principle focus and the mirror

 outside the centre of curvature

 behind the mirror

 between the centre of curvature and the principle focus
 Your answer was INCORRECT
 Explanation
 For an object lying between the centre of curvature and the principle focus of a
concave mirror, the image formed is real, inverted and enlarged; and it lies behind
the centre of curvature
Score: 17 %
 Question 12 Part 3 – Tutor Mode
 Which of the following best describes the mode of action of the fluoroquinolones:

 inhibit cell membrane synthesis

 inhibit protein synthesis

 inhibit nucleic acid synthesis

 inhibit cell wall synthesis
 Your answer was INCORRECT
 Explanation
 Fluoroquinolones such as ciprofloxacin inhibit DNA replication by their action on
bacterial DNA gyrase.
Score: 15 %
 Question 13 Part 3 – Tutor Mode
 The long ciliary nerve arises from:

 the anterior ciliary nerve

 the optic nerve

 the ciliary ganglion

 the nasociliary nerve
 Your answer was INCORRECT
 Explanation
 The long ciliary nerve, which passes through the choroid and transmits sensory
fibres to the cornea, iris and ciliary body (as well as sympathetic fibres to the
dilator pupillae muscle) is a branch of the nasociliary nerve, itself a branch of the
ophthalmic nerve. The short ciliary nerve arises from the ciliary ganglion and
carries sensory (from the nasociliary), sympathetic and parasympathetic fibres.
Score: 21 %
 Question 14 Part 3 – Tutor Mode
 Branches of the pterygopalatine ganglion enter the orbit through:

 the inferior orbital fissure

 the foramen ovale

 the optic canal

 the superior orbital fissure
 Your answer was CORRECT
 Explanation
 Branches of the pterygopalatine ganglion enter the orbit through the inferior
orbital fissure
Score: 20 %
 Question 15 Part 3 – Tutor Mode
 The nerve of the second pharyngeal arch is:

 the maxillary nerve

 the mandibular nerve

 the ophthalmic nerve

 the facial nerve

 the lacrimal nerve
 Your answer was INCORRECT
 Explanation
 The facial nerve is the nerve of the second pharyngeal arch.

Score: 19 %
Question 16 Part 3 – Tutor Mode
Which structure forms part of the medial wall of the orbit:

the frontal bone

the maxillary bone

the zygomatic bone

the greater wing of sphenoid

Your answer was INCORRECT


Explanation

The medial wall is made up of the ethmoid bone, the lacrimal bone, a small part of the
lesser wing of the sphenoid, and the tip of the maxilla

Score: 18 %
Question 17 Part 3 – Tutor Mode
Which of the following is a recognized side-effect of acetazolamide:

metabolic alkalosis

renal calculi

hyperkalaemia

hypernatraemia

Your answer was INCORRECT

Explanation

Acetazolamide is a carbonic anhydrase inhibitor. It can cause metabolic acidosis,


hypokalaemia, hyponatraemia, renal calculi, fatigue, paraesthesia of limbs and Steven-
Johnson's syndrome

Score: 17 %
Question 18 Part 3 – Tutor Mode
Which is the site of pathology in Coats disease:

the basement membrane of capillaries

the arteriolar and venular endothelium

the pericytes

the media of medium and small vessels

the intima of large vessels

Your answer was INCORRECT

Explanation
The vascular abnormality in Coats disease is the result of an abnormal endothelium in
arterioles and venules. This causes massive leakage of lipid-rich plasma into the retina
and the subretinal space.

Score: 16 %
Question 19 Part 3 – Tutor Mode
The conjunctival epithelium consists of:

10-15 layers of cells

single layer of cuboidal cells and goblet cells

2-5 layers of cells

50-100 layers of cells

Your answer was INCORRECT

Explanation

The conjunctival epithelium consists of 2-5 layers of epithelial cells. It varies from a
stratified squamous non-keratinising epithlium to a stratified columnar epithelium.

Score: 15 %
Question 20 Part 3 – Tutor Mode
What colour excitation filter does a fluorescein angiography camera have:

green

blue

orange

yellow

Your answer was INCORRECT

Explanation

Fluorescein sodium is an orange dye which is excited by blue light of wavelength 465-
490nm. It then emits yellow-green light (520-530nm). For this reason, an angiography
camera has a blue excitation filter to illuminate the fundus with blue light for maximal
fluorescein absorbance. The camera then has a yellow-green barrier light which blocks
blue light and exposes the camera only to the yellow-green light from the fluorescein.
Score: 19 %
Question 21 Part 3 – Tutor Mode
Which statement is TRUE regarding prisms:

convergence insufficiency is the commonest therapeutic use of prisms

the Maddox rod is an objective measure of squint

the prism cover test is a subjective measure of squint

prisms can be stacked one on top the other to produce an additive effect

Your answer was CORRECT

Explanation

Prisms cannot be stacked on top the other for additive effect because the angle of
incidence changes in successive layers. The prism cover test is an objective measure of
squint while the Maddox rod is a subjective measure of squint. Convergence
insufficiency is the commonest therapeutic use of prisms.
Question 22
Which is FALSE regarding uveal melanoma:

the most common site of metastasis is the brain

it is more common in the choroid than the iris

iris melanomas may cause secondary glaucoma

the tumours can be non-pigmented

Your answer was CORRECT

Explanation

The most common site of metastasis of uveal melanoma is the liver. Other statements are
true

Score: 26 %
Question 23 Part 3 – Tutor Mode
If the parents of a child with a congenital eye defect were normal but found to be first
cousins, what is the most likely mode of inheritance:

autosomal recessive
mitochondrial inheritance

autosomal dominant

X-linked recessive

Your answer was CORRECT

Explanation

When parents are phenotypically normal but produce an affected child it implies a
recessive gene. Consanguinity increases the risk of autosomal recessive disorders because
of the smaller gene pool creating more likelihood of homozygous gene loci in the
offspring.

Score: 25 %
Question 24 Part 3 – Tutor Mode
All are TRUE of the human pituitary gland EXCEPT:

it is ectodermal in origin

it is connected to the hypothalamus by the infundibulum

the anterior and posterior lobes have a common vascular system

it derives its blood supply from the circle of Willis

Your answer was INCORRECT

Explanation

The pituitary gland has independent vascular systems for the anterior and posterior lobes.
Other options supplied are true.

Score: 24 %
Question 25 Part 3 – Tutor Mode
Which statement is TRUE regarding the lacrimal drainage system:

the inferior punctum is located more medially than the superior punctum

in 50% of people, the lacrimal canaliculi open directly into the lacrimal sac

the nasolacrimal duct may be closed at 6 months post delivery


the lacrimal drainage duct is lined by a single layer of columnar epithelium

Your answer was INCORRECT

Explanation

The superior punctum is located more medially than the inferior punctum. In only 10% of
people do the lacrimal canaliculi open directly into the lacrimal sac in the remaining 90%
they open into the common canaliculus. The nasolacimal duct may be closed for the first
12 months of life after this time consideration should be given to surgery. The lacrimal
drainage duct is lined by stratified ciliated columnar epithelium.

Score: 23 %
Question 26 Part 3 – Tutor Mode
Which statement is FALSE about Descemet's membrane:

it consists of a banded anterior third and a non-banded, homogenous posterior two thirds

it is 50 micrometers thick

it is rich in basement membrane glycoproteins

it is the modified basement membrane of the corneal endothelium

Your answer was INCORRECT

Explanation

Descemet's is 8-12 micrometers thick. Other options are true

Score: 22 %
Question 27 Part 3 – Tutor Mode
Which is FALSE regarding light waves:

waves that are in phase create constructive interference

anti-glare spectacle coatings use destructive interference

constructive interference is used in the corneal stroma

waves that are exactly in phase are called coherent

Your answer was INCORRECT


Explanation

Light waves that are exactly in phase are called coherent, while out of phase waves are
termed incoherent. Waves that are in-phase produce constructive interference where there
is summation of amplitude while waves that are out of phase produce destructive
interference where there is subtraction of amplitude. Destructive interference is used in
the corneal stroma to reduce scatter and therefore maintain clarity; it also used in anti-
glare coatings on spectacles.

Score: 21 %
Question 28 Part 3 – Tutor Mode
Regarding immune hypersensitivity reactions:

Arthus reaction is a type IV reaction

contact dermatitis is a type IV reaction

anaphylaxis occurs in patients who have had no previous antigen exposure

positive Mantoux test is a type III reaction

Your answer was INCORRECT

Explanation

 Type I: anaphylaxis, urticarial


 Type II: haemolytic anaemia (antibody binding to antigen presented on the cell
surface)
 Type III: Arthus reaction (immune complex mediated)
 Type IV: Mantoux and contact dermatitis (cell-mediated)

Score: 24 %
Question 29 Part 3 – Tutor Mode
Which is FALSE regarding bacterial virulence factors:

streprokinase initiates clot formation by its effect on plasmin

coagulase facilitates deposition of a protective fibrin coat on the bacteria

collagenase allows bacterial spread by disrupting connective tissue

hyalurondiase dissolves extracellular matrix hyaluronate

Your answer was CORRECT


Explanation

Streptokinase activates fibrinolysin, causing release of plasmin and resulting in


dissolution of clots, allowing bacteria to spread through tissues.

Score: 23 %
Question 30 Part 3 – Tutor Mode
In diabetic retinopathy, which of the following changes are commonly observed:

macroaneurysms

increase in number of pericytes

cotton wool spots due to retinal nerve fibre ischaemia

thinning of the capillary basement membrane

Your answer was INCORRECT

Explanation

In diabetic retinopathy, the capillary basement membrane is thickened. There is a


decrease in the number of pericytes. Microaneurysms not macroaneurysms are a feature
of diabetic retinopathy. Cotton wool spots represent RNFL ischaemia.

Score: 23 %
Question 31 Part 3 – Tutor Mode
The sphenoid bone transmits all of the following structures EXCEPT:

optic nerve

internal carotid artery

middle meningeal artery

mandibular branch of the trigeminal nerve

Your answer was INCORRECT

Explanation

The internal carotid artery grooves the body of the sphenoid but does not enter the
cranium through it. The ICA enters the skull through the carotid canal in the petrous
temporal bone and travels to the middle cranial fossa via the foramen lacerum.
Score: 22 %
Question 32 Part 3 – Tutor Mode
Which of the following is TRUE about the visual pathway:

the visual pathway synapses in the medial geniculate body

congruous hemianopia occurs if there is a lesion in the optic tract

the posterior cerebral artery supplies most of the optic radiation and the visual
cortex

the inferior colliculi are the relay stations for the light reflex

Your answer was INCORRECT

Explanation

The lateral geniculate body is the centre for visual fibre synapses. Incongruous
hemianopia occurs in lesion of the optic tract. Congruous hemianopia occurs if the lesion
is in the optic radiation or the visual cortex. The calcarine sulcus on the medial aspect of
the occipital lobe is the main visual centre. The superior colliculi are involved in the light
reflex. The posterior cerebral artery supplies most of the posterior visual pathway.

Score: 21 %
Question 33 Part 3 – Tutor Mode
Differential diagnosis of leucocoria includes all EXCEPT:

persistent hyperplastic primary vitreous

Coats disease

Stargardt's disease

retinopathy of prematurity

Your answer was INCORRECT

Explanation

Stargardt's disease is an inherited juvenile macular degeneration and is not associated


with leucocoria.

Score: 21 %
Question 34 Part 3 – Tutor Mode
Which cells are responsible for signal modulation reaching the ganglion cells of the
retina:

bipolar cells

astrocytes

amacrine cells

Muller cells

Your answer was INCORRECT

Explanation

Amacrine cells, together with horizontal cells, are believed to play a crucial role in signal
modulation (mostly inhibitory) reaching the ganglion cells. Bipolar cells serve as a
connector or intermediary between the photoreceptor and the ganglion cells. Astrocytes
and Muller cells are glial cells and perform a supportive role both structurally and
nutritionally.

Score: 20 %
Question 35 Part 3 – Tutor Mode
All of the following drugs impair accommodation EXCEPT:

phenothiazine

topical atropine

topical pilocarpine

topical cocaine

Your answer was INCORRECT

Explanation

Accommodation occurs under parasympathetic control. It is impaired by parasympathetic


antagonists e.g. atropine, cylopentolate, tropicamide and phenothiazine. Pilocarpine is a
parasympathomemetic and enhances accommodation.

Score: 19 %
Question 36 Part 3 – Tutor Mode
The following are true about the muscles of facial expression EXCEPT:
procerus is responsible for frowning

the frontalis is deficient medially

paralysis of the orbicular oculi results in widening of the palpebral fissure

corrugator is responsible for vertical glabellar wrinkles

Your answer was INCORRECT

Explanation

The frontalis is deficient laterally and therefore lateral brow ptosis is commoner in the
elderly.

Score: 22 %
Question 37 Part 3 – Tutor Mode
Which statement is FALSE regarding respiratory airway resistance:

it increases with the radius of the airway

it increases with the length of the airway

it can be measured using the peak-flow meter

it is increased by histamine

it is greater in expiration than inspiration

Your answer was CORRECT

Explanation

Airway resistance is directly proportional to the length of the airway and is indirectly
proportional to the fourth power of the airway radius. It is increased by histamine which
causes bronchoconstriction. It is greater in expiration than inspiration. Peak-flow meter
can be used to measure airway resistance.

Score: 24 %
Question 38 Part 3 – Tutor Mode
A patient has a successful allogeneic bone marrow transplant. He now develops a skin
rash and abnormal liver function tests. This is likely to represent:

Type IV hypersensitivity
Type I hypersensitivity

Type II hypersensitivity

Type III hypersensitivity

Your answer was CORRECT

Explanation

The patient has developed graft versus host disease. This disorder occurs in
immunocompromised individuals post allogeneic bone marrow and solid organ
transplant. T-lymphocytes from the graft react to allogeneic antigens, and cause cell-
mediated damage by activation of effector cells. Acute GVHD presents with skin
manifestations, hepatitis, and enteritis developing within 100 days of allogeneic
haematopoietic cell transplantation (HCT). Chronic GVHD describes a syndrome
developing after day 100. Graft versus host disease is a type IV (TH1 mediated)
hypersensitivity disorder.

Score: 26 %
Question 39 Part 3 – Tutor Mode
Regarding the meibomian glands which is FALSE:

they secrete lipid droplets in an apocrine manner

they are modified sebaceous glands

they number approximately 30 in the upper tarsal plate

their secretions aid in tear film stability

Your answer was CORRECT

Explanation

The meibomian glands secrete lipid droplets in a holocrine manner. Other options are true

Score: 28 %
Question 40 Part 3 – Tutor Mode
Which statement is FALSE regarding retinoblastoma:

metastatic dissemination is usually via the lymphatics

it may be successfully treated with radiotherapy and chemotherapy


it appears macroscopically as a smooth white mass

it is due to mutation in a tumour suppressor gene

Your answer was CORRECT

Explanation

Metastatic dissemination of retinoblastoma is via the blood stream. Other options are
true.

Score: 27 %
Question 41 Part 3 – Tutor Mode
Which statement is FALSE of the parotid gland:

it receives post-ganglionic parasympathetic fibres from the otic ganglion

its sheath is innervated by the second cervical nerve

it contains a duct which opens into the mouth opposite the upper canine tooth

it is composed of serous acini which contribute to the saliva

Your answer was INCORRECT

Explanation

The parotid gland duct opens opposite the upper second molar (not the canine). The other
options are true.

Score: 26 %
Question 42 Part 3 – Tutor Mode
Which of the following is TRUE about coronary circulation in the left ventricle:

at rest the blood in the coronary vein is 50% saturated with oxygen

flow is greater during systole than in diastole

it receives an equivalent supply of blood compared to the right ventricle

noradrenalin causes vasodilatation of the coronary artery

Your answer was INCORRECT


Explanation

The right ventricle receives less blood than the left ventricle. This is related to the work
load each ventricle performed being higher in the left than the right ventricle. During the
systole, vessels are compressed by the myocardium and this reduces flow. Coronary
venous blood is only about 25% saturated with oxygen in the resting person.

Score: 26 %
Question 43 Part 3 – Tutor Mode
Which is FALSE regarding Southern Blotting:

gel electrophoresis is utilized to separate fragments

restriction endonucleases cut the initial DNA into fragments

fluorescent labeled DNA probes are required

it is a technique for analyzing DNA

Your answer was INCORRECT

Explanation

Southern blotting is a technique for analyzing DNA. The steps are as follows:

 restriction endonucleases cut human DNA into fragments


 fragments are separated by gel electrophoresis
 the fragments are denatured to single strands and blotted on nitrocellulose
 specific radio-isotope labeled DNA probes are used to bind complementary
strands on the blotted fragments

Score: 25 %
Question 44 Part 3 – Tutor Mode
Ocular effects of adrenergic agonists include all EXCEPT:

mydriasis

contraction of Muller's muscle

ciliary muscle relaxation

increased outflow of aqueous

dilation of conjunctival and episcleral vessels


Your answer was INCORRECT

Explanation

Adrenergic agonists cause:

 mydriasis
 ciliary muscle relaxation
 increased aqueous formation (though selective alpha-2 stimulation will reduce
formation)
 increased aqueous outflow
 constriction of conjunctival and episcleral vessels
 contraction of Muller's muscle

Score: 24 %
Question 45 Part 3 – Tutor Mode
Goblet cells secrete:

aqueous

mucous

lipid

oil

Your answer was INCORRECT

Explanation

Goblet cells, such as are found in the conjunctiva, secrete mucous which is an important
constituent of the tear film.

Score: 24 %
Question 46 Part 3 – Tutor Mode
All of the following antibiotics inhibit bacterial cell wall synthesis EXCEPT:

Ceftriaxone

Vancomycin

Flucloxacillin

Erythromycin
Your answer was INCORRECT

Explanation

Macrolides such as erythromycin inhibit protein synthesis by binding to a subunit of the


bacterial ribosome. Penicillins (e.g. flucloxacillin), vancomycin and cephalosporins all
work by inhibiting cell wall synthesis.

Score: 23 %
Question 47 Part 3 – Tutor Mode
The corneal epithelium obtains its oxygen from:

the air

the aqueous

the precorneal tear film

the conjunctival

Your answer was INCORRECT

Explanation

The corneal epithelium obtains its oxygen from the precorneal tear film. The endothelium
and the keratocytes in the deep stroma by contrast obtain their oxygen supply from the
aqueous humour.

Score: 23 %
Question 48 Part 3 – Tutor Mode
All of the following are true of the facial nerve EXCEPT:

it supplies secretomotor fibres to the submandibular and sublingual glands

it exits the skull through the styloid foramen

it does not contain sensory nerve fibres

it supplies the second pharyngeal arch muscles

Your answer was INCORRECT

Explanation
The facial nerve contains sensory (taste) sensation from the anterior two-thirds of the
tongue and a small area of the external ear. Other options are true.

Score: 24 %
Question 49 Part 3 – Tutor Mode
The corneal epithelium obtains its glucose directly from:

the stroma

the ciliary vasculature

the limbal vasculature

the tear film

Your answer was CORRECT

Explanation

The glucose metabolized in the corneal epithelium comes from the stroma by way of the
aqueous and the limbal vessels.

Score: 24 %
Question 50 Part 3 – Tutor Mode
Which statement is FALSE about the conjunctiva:

its epithelium is continuous with the corneal epithelium at the limbus

it is bound loosely to the tarsal plate

its epithelium is continuous with the lid skin at the eye lid margin

it is responsible for the production of the mucous component of the tear film

Your answer was INCORRECT

Explanation

The palpebral conjunctiva is firmly adherent to the tarsal plate. The bulbar conjunctiva is
also tightly adherent at the limbus. The conjunctival epithelium is continuous with both
the corneal epithelium at the limbus and with the lid skin at the muco-cutaneous junction
of the lid margin. Goblet cells within the conjunctiva are responsible for the mucous
component of the tear film.

Score: 24 %
Question 51 Part 3 – Tutor Mode
In A-scan biometry, a one diopter IOL power error could result from an axial length error
of:

4 mm

0.4 mm

8mm

1 mm

0.04 mm

Your answer was INCORRECT

Explanation

0.4mm compression error can result in a 1 diopter error in the calculated IOL power.

Score: 23 %
Question 52 Part 3 – Tutor Mode
Above the annulus of Zinn, the superior orbital fissure transmits:

the optic nerve

the trochlear nerve

the abducens nerve

the oculomotor nerve

Your answer was INCORRECT

Explanation

The trochlear nerve together with the lacrimal nerve, the frontal nerve and the superior
ophthalmic vein pass within the superior orbital fissure but above the annulus of Zinn.

Score: 23 %
Question 53 Part 3 – Tutor Mode
An increase in the number of chromosomes occurs in:

Retinoblastoma
Fragile-X syndrome

Turner's syndrome

Klinefelter's syndrome

Your answer was INCORRECT

Explanation

Klinefelter's syndrome is xxY - there is an extra X sex chromosome. Turner's syndrome


is one chromosome short, XO. The number is normal for fragile-X syndrome and
retinoblastoma.

Score: 22 %
Question 54 Part 3 – Tutor Mode
The lamina papyracea of the orbit refers to:

the orbital floor

the medial wall

the lateral wall

the orbital roof

Your answer was INCORRECT

Explanation

The lamina papyracea refers to the paper-thin medial wall, which is semi-transparent, the
ethmoidal air sinuses being easily seen through the dried skull.

Score: 22 %
Question 55 Part 3 – Tutor Mode
Which immunoglobulin class has the highest individual molecular weight?

IgA

IgG

IgE

IgD
IgM

Your answer was INCORRECT

Explanation

IgM is a pentamer, consisting of five immunoglobulin units, making it the largest of the
antibody classes.

Score: 21 %
Question 56 Part 3 – Tutor Mode
The ciliary ganglion:

contains parasympathetic nerve fibres that innervate the dilator pupillae

if damaged, produces a pupil that is unreactive to light but is normal to accommodation

contains preganglionic axons from the Edinger-Westphal nucleus

lies medial to the optic nerve

Your answer was INCORRECT

Explanation

The ciliary ganglion lies lateral to the optic nerve. It contains parasympathetic nerve
fibres that innervate the sphincter pupillae. If the parasympathetic fibres are damaged, the
pupil fails to react to light and responds to accommodation sluggishly.

Score: 21 %
Question 57 Part 3 – Tutor Mode
Which is FALSE regarding retinal vascularization:

the new vessels always leak on fluorescein angiography

it occurs under the control of factors including VEGF

it can occur both within and on the surface of the retina

blood vessels form from the arterial side of the capillary bed

Your answer was INCORRECT


Explanation

New vessels arise from the venous side of the capillary bed. Other options are true.

Score: 21 %
Question 58 Part 3 – Tutor Mode
Which is FALSE regarding polymerase chain reaction (PCR):

it usually involves 20 to 50 cycles to amplify the DNA

it can be used to diagnose viral infections

it can be used to determine genome mutations

the first stage involves the use of primers

Your answer was INCORRECT

Explanation

The first stage of PCR involves denaturation of the target DNA. In the second stage
primers are used to hybridize to the target nucleic acid region. In the third stage,
amplification of the target region is achieved by multiple cycles involving the DNA
polymerase enzyme. The amplified sequence can then be analysed for mutations using
techniques such as single-strand polymorphisms or restriction fragment length
polymorphisms.

Score: 20 %
Question 59 Part 3 – Tutor Mode
Put the following corneal tissues in order from lowest energy consumption to highest:

epithelium, endothelium, stroma

stroma, endothelium, epithelium

stroma, epithelium, endothelium

epithelium, stroma, endothelium

Your answer was INCORRECT

Explanation
The stromal keratocytes have the lowest energy consumption. The endothelium cells have
the highest energy consumption, about 5 times that of the corneal epithelium, due to the
activity of the endothelial pump.

Score: 22 %
Question 60 Part 3 – Tutor Mode
Which immunoglobulin molecule has the longest serum half-life?

IgG

IgA

IgD

IgM

IgE

Your answer was CORRECT

Explanation

IgG has the longest half-life, 21 to 23 days. IgA is second at about 6 days, followed by
IgM (5 days), IgD (3 days), and IgE (2 days).

Score: 21 %
Question 61 Part 3 – Tutor Mode
An epidural haematoma is most likely to be caused by:

embolus of the anterior cerebral artery

laceration of the middle meningeal artery

laceration of the superior cerebral bridging veins

rupture of a berry aneurysm

Your answer was INCORRECT

Explanation

An epidural haematoma is most often caused by laceration of the middle meningeal


artery.
Score: 21 %
Question 62 Part 3 – Tutor Mode
Which of the following tests provides a subjective measure of latent horizontal deviation:

the cover test

the alternate cover test

the horizontal Maddox rod

the vertical Maddox rod

Your answer was INCORRECT

Explanation

A horizontal Maddox rod tests for subjective latent horizontal deviation, while a vertical
rod tests for subjective latent vertical deviation. The cover and alternate cover tests are
subjective measures.

Score: 22 %
Question 63 Part 3 – Tutor Mode
In a histological section of the retina, which layer occurs immediately above (inner to)
the outer plexiform layer:

inner nuclear layer

inner plexiform layer

outer nuclear layer

outer segment

ganglion cell

Your answer was CORRECT

Explanation

The eight layers of the neurosensory retina from outer to inner, that is from the RPE
upwards are:

 outer segment
 inner segment
 outer nuclear
 outer plexiform
 inner nuclear
 inner plexiform
 ganglion cell
 nerve fibre layer

Score: 22 %
Question 64 Part 3 – Tutor Mode
Natural killer cells:

are small agranular lymphocytes

can recognise MHC class II and peptide antigen

are activated by interferons

only kill target cells in the absence of antibody

are important in the early response to bacterial infection

Your answer was INCORRECT

Explanation

Natural killer (NK) cells are large agranular lymphocytes that can kill virally infected
cells by antibody-dependent and -independent mechanisms. They are not phagocytic. NK
cell killing is especially important in the early response to viral infection. They also play
an important role in the control of the early phases of infection with intracellular
pathogens such as Leishmania and Listeria monocytogenes. NK cells do not express T-
cell receptors and do not recognise MHC class II molecules (cf. T-cells). They express a
number of non-specific adhesion molecules, which mediate attachment to target cells.
Human NK cells express membrane proteins that recognise specific allelic forms of MHC
class I molecules bound to self-peptide. On binding their ligand, these receptors inhibit
NK cell killing. If the concentration of class I molecules on a cell is reduced, or the range
of peptides is markedly altered (as can occur in viral infection), NK cells are no longer
inhibited and cytolysis occurs. These cells release granules containing perforin and
granzymes, which lyse target cells and then induce apoptotic cell death. NK cells also
express receptors for the Fc region of IgG and are important mediators of antibody-
dependent cellular cytotoxicity. The killing activity of NK cells is enhanced by IFN-
gamma, IFN-beta, and IL-12.

Score: 22 %
Question 65 Part 3 – Tutor Mode
Which of the following is TRUE about medical testing:
a positive predictive value is also known as the true positive rate

a test with a high sensitivity has few false negatives

the positive predicitive value of a test will be the same in any population

a test with a low specificity will have many false negatives

the positive predictive value of a test does not depend on the pretest probability

Your answer was INCORRECT

Explanation

The positive predictive value is the probability that if you test positive, you have the
disease. The positive predictive value depends upon prevalence of the disease and pretest
probability is the same as disease prevalence.

Sensitivity means true positive rate. A test with high sensitivity will have few false
negatives.

A test with low specificity will have many false positives.

Score: 23 %
Question 66 Part 3 – Tutor Mode
Which is FALSE regarding transcription:

it occurs in the cytoplasm

it proceeds in a 5 to 3 direction

it begins by the binding of a promoter

it is catalyzed by RNA polymerase

Your answer was CORRECT

Explanation

Transcription is the process whereby the DNA code is read in the nucleus and a
complementary messenger RNA is produced which leaves for the cytoplasm where
protein synthesis can occur. Transcription occurs under the action of RNA polymerase in
a 5 to 3 direction.
Score: 24 %
Question 67 Part 3 – Tutor Mode
Which statement is FALSE regarding Muller's muscle:

it is composed of thin striated muscle

it is attached to the superior portion of the tarsal plate

it receives sympathetic innervation

it occurs on the inferior aspect of the levator palpebrae superioris

it assists in elevating the lid

Your answer was CORRECT

Explanation

Muller's muscle is composed of a small band of smooth muscle. Other options supplied
are true.

Score: 24 %
Question 68 Part 3 – Tutor Mode
The volume of aqueous humour in the anterior chamber is:

6 microlitres

400 microlitres

15 microlitres

250 microlitres

3 microlitres

Your answer was INCORRECT

Explanation

There are 250 microlitres of aqueous in the anterior chamber.

Score: 23 %
Question 69 Part 3 – Tutor Mode
The suprachoroid is continuous with which ocular layer:
the lamina densa

the lamina cribrosa

the lamina vitrea

the lamina fusca

Your answer was INCORRECT

Explanation

The suprachoroid is the outermost layer of the choroid. It is an avascular transition layer
between the choroid and sclera comprising melanocytes, firoblasts and connective tissue
fibres. It is continuous with the lamina fusca, which is the inner layer of the sclera. The
lamina vitrea is another term for Bruch's membrane. The lamina cribrosa transmits the
optic nerve. The lamina densa is part of the basement membrane of the corneal
epithelium.

Score: 23 %
Question 70 Part 3 – Tutor Mode
Base-in prisms:

can be used to treat convergence insufficiency

can be used to measure horizontal fusional amplitude

can be used in the management of esophoria

can be used in the management of VIth nerve palsy

are used in the diagnosis of microtropia

Your answer was INCORRECT

Explanation

Base-in prisms can be used to measure horizontal fusional amplitude. However, other
options are incorrect as base-out prisms are used in the management of esophoria,
convergence insufficiency and VIth nerve palsy. A 4 prism-dioptre base-out prism is used
to examine for the suppression scotoma of microtropia.

Score: 24 %
Question 71 Part 3 – Tutor Mode
Which statement is FALSE regarding the beta-blockers:

they decrease the intraocular pressure by increasing the aqueous outflow

they have negative inotropic and chronotropic effects

they cause hypotension

those with intrinsic sympathetic activities are less likely to cause bradycardia

Your answer was CORRECT

Explanation

Beta-blockers decrease IOP by decreasing aqueous flow. Other options supplied are true.

Score: 24 %
Question 72 Part 3 – Tutor Mode
Which of the following is the most probable site of action for ADH?

proximal convoluted tubule

distal convoluted tubule

collecting duct

descending limb Loop of Henle

ascending limb Loop of Henle

Your answer was INCORRECT

Explanation

The collecting duct is normally impermeable to water, it becomes permeable under the
actions of antidiuretic hormone (ADH). As much as three-quarters of the water from
urine can be reabsorbed as it leaves the collecting duct by osmosis.

Score: 23 %
Question 73 Part 3 – Tutor Mode
Which statement about Down's syndrome is FALSE:

amniocentesis is a more accurate prenatal test for Down's than hormonal tests
the risk of Down's is associated with the age of the mother

it is most commonly caused by chromosomal disjunction during meiosis

it is associated with congenital glaucoma

Your answer was INCORRECT

Explanation

Down's syndrome is associated with cataract, myopia and keratoconus but not glaucoma.

Score: 24 %
Question 74 Part 3 – Tutor Mode
Corneal endothelium has a density in middle-aged adulthood of :

2500 cells/mm2

6000 cells/mm2

1000 cells/mm2

4000 cells/ mm2

Your answer was CORRECT

Explanation

The corneal density is about 2500 cells/mm2 in middle age.

Score: 24 %
Question 75 Part 3 – Tutor Mode
A 16-year-old patient with AIHA (autoimmune haemolytic anaemia) is about to have a
therapeutic splenectomy.

Which prophylactic treatment do you NOT need to give her:

pneumococcal vaccine

meningococcal vaccine

penicillin prophylaxis

HIb vaccine
tetanus booster

Your answer was INCORRECT

Explanation

One complication of splenectomy is infection with encapsulated bacteria such as


pneumococcus, haemophilus influenza and meningococcus. This is because the spleen is
important in the induction of protective immune responses to thymus-independent
antigens. Macrophages in the marginal zones of lymphoid follicles in the spleen trap
polysaccharides, which then persist for long periods on the surface of these cells. These
antigens are recognised by specific B lymphocytes, which mount an immune response in
a T cell-independent fashion.

The patient will maintain her immunity to tetanus.

Score: 24 %
Question 76 Part 3 – Tutor Mode
An anterior communicating artery aneurysm is most likely to produce:

a homonymous lower quadrantanopia

a homonymous upper quadrantanopia

a bitemporal lower quadrantanopia

a bitemporal upper quadrantanopia

Your answer was INCORRECT

Explanation

An anterior communicating artery aneurysm is most likely to compress the upper portion
of the optic chiasm, producing a bitemporal lower quandrantanopia.

Score: 23 %
Question 77 Part 3 – Tutor Mode
Melanocyes may be found in all the following layers EXCEPT:

the choroid stroma

the corneal epithelium

the conjunctival epithelium


the iris stroma

Your answer was INCORRECT

Explanation

The corneal epithelium is devoid of melanocytes. It is also rare to find immunocompetent


cells in the cornea such as dendritic cells, especially further from the limbus.

Score: 23 %
Question 78 Part 3 – Tutor Mode
Which of the following is FALSE regarding the orbicularis oculi:

its lacrimal component on contraction draws the lacrimal papillae medially

the orbital portion draws the skin of the forehead and cheek towards the orbit

the palpebral portion acts solely under involuntary control to close the eyelids

it is innervated by branches of the facial nerve

Your answer was INCORRECT

Explanation

The palpebral portion of orbicularis acts under both voluntary and involuntary control to
close the eyelids during normal blinking. The lacrimal portion draws the lacrimal papillae
medially and dilates the lacrimal sac during blinking. The orbital portion draws the skin
of the forehead, temple, cheek and orbital margin towards the medial angle of the orbit
firmly closing the lids.

Score: 23 %
Question 79 Part 3 – Tutor Mode
Regarding the optic nerve which statement is FALSE:

the dura and arachnoid mater surrounding the optic nerve blend with the sclera

the orbital portion of the optic nerve extends backward and laterally from the eye to
the optic canal

the orbital portion of the optic nerve is covered by 3 layers of meninges

optic nerve fibres are myelinated by oligodendroytes posterior to the sclera


Your answer was INCORRECT

Explanation

The orbital portion of the optic nerve extends backward and medially from the back of
the eye to the optic canal. Other statements are true.

Score: 24 %
Question 80 Part 3 – Tutor Mode
Which is FALSE regarding restriction fragment length polymorphisms:

they represent artificially induced variations

they are created from restriction endonucleases

they can be utilized in gene linkage studies

they are most commonly point mutations in the genome

Your answer was CORRECT

Explanation

Restriction fragment length polymorphisms represent the natural variation in the human
genome, most commonly point mutations. As a result of these natural variations,
restriction endonuclease enzymes cleave the human DNA in different positions in the
laboratory creating different lengths of DNA, which can then be detected by Southern
Blotting. RNFLs are useful for tracking gene transmission in families among other uses.

Score: 23 %
Question 81 Part 3 – Tutor Mode
Which of the following nerves travel through the lateral wall of the cavernous sinus at the
level of the pituitary fossa:

4th, 5th and 6th cranial nerves

6th cranial nerve

3rd, 4th, 5th and 6th cranial nerves

3rd and 4th cranial nerves

3rd, 4th and 5th cranial nerves


Your answer was INCORRECT

Explanation

The 3rd, 4th and 5th cranial nerves travel through the lateral wall of the cavernous sinus.
The 6th nerve travels through the cavernous sinus but goes through it rather than
involving the lateral wal

Score: 23 %
Question 82 Part 3 – Tutor Mode
Regarding the synthesis of neurotransmitters:

cholinesterase synthesises acetylcholine

dopa decarboxylase is involved in adrenaline synthesis

catechol-o-methyl transferase (COMT) synthesises dopamine

monoamine oxidase synthesises noradrenaline

Your answer was INCORRECT

Explanation

Cholinesterase breaks down acetylcholine. MAO and COMT break down adrenaline and
noradrenaline (MAO in the synapse, COMT intracellularly). Dopa decarboxylase is
responsible for the synthesis of dopamine and by extension adrenaline and noradrenaline.

Score: 23 %
Question 83 Part 3 – Tutor Mode
The polymerase chain reaction:

cannot be used for prenatal diagnosis as insufficient sample is usually collected

replicates deoxyribonucleic acid (DNA) but not ribonucleic acid (RNA)

cannot be used to diagnose viral infections

requires the temperature to be cycled between 55 and 94 degrees

relies on the maintenance of a folded DNA structure at all times

Your answer was INCORRECT


Explanation

The polymerase chain reaction (PCR) can be used to exponentially amplify specific
intended segments of DNA. It is based on the principle that the two strands of DNA
denature i.e. separate (and thus unfold) at high temperatures, and that complementary
strands of DNA recombine when cooled. The DNA sequences bordering the intended
target must be known in order to design primers. The sequence of the primers is
complementary to the 5' ends of the region to be amplified. This is because DNA is
synthesised in a 5'-3' direction. These primers need to be specific and bind strongly,
therefore the optimal length is 17 - 20 bp. Heating to 95 degrees C pulls the two DNA
strands apart. The temperature of the reaction is then dropped to < 50 degrees C, to allow
annealing to occur. The annealing temperature differs depending on the nature of the
primers and the template DNA. The temperature is elevated to 70 degrees C, the optimal
temperature for the DNA polymerase. This polymerase is derived from volcanic bacteria.
The cycle is then repeated; this process results in the exponential amplification of the
target DNA. Reverse transcriptase PCR can be used to amplify RNA. As the PCR
reaction exponentially amplifies DNA, very small amounts of initial template DNA are
required for this technique. It can therefore be used for prenatal diagnosis of genetic
conditions and congenital infection.

Score: 23 %
Question 84 Part 3 – Tutor Mode
Free nerve endings in the skin transmit:

proprioception

pain and temperature

light touch

vibration

Your answer was INCORRECT

Explanation

The cutaneous receptors are as follows:

 Meissner's corpuscles: light touch


 Pacinian corpuscles: pressure and vibration receptors
 Merkel discs: light touch
 Free nerve endings: pain and temperature

Score: 22 %
Question 85 Part 3 – Tutor Mode
Hip flexion was measured in a sample of adults and compared with their age. The
correlation coefficient was found to be -0.64 (p = 0.03) and regression coefficient -0.012
(p = 0.12).

Which of the following is TRUE?

we can use the regression coefficient to calculate hip flexion in children

the null hypothesis is that the correlation coefficient equals 1

regression coefficients can take any value between -1 and 1

for a year's increase in age, hip flexion decreases by 0.64 units

correlation coefficients measure the strength of the association between one variable
and another

Your answer was INCORRECT

Explanation

Regression coefficients are the gradient of the line of best fit and can take any value.
Correlation coefficients range between -1 to +1. The regression coefficient cannot be
used to calculate for children because we cannot assume the gradient stays the same
outside our sample range.

Score: 22 %
Question 86 Part 3 – Tutor Mode
Which nerve passes close to the petrous part of the temporal bone:

V nerve

VI nerve

IV nerve

III nerve

Your answer was INCORRECT

Explanation
The sixth nerve exits the brainstem at the junction of the pons and medulla. It runs
upward in the pontine cistern. It pierces the dura and at the tip of the petrous temporal
bone makes a sharp turn forward to enter the cavernous sinus.

Score: 22 %
Question 87 Part 3 – Tutor Mode
All the following are true of retinitis pigmentosa EXCEPT:

individuals are typically affected in early adult life

it can lead to tunnel vision

retinal atrophy is evident with fine and course strands of pigmentation

retinal vessels are relatively unaffected

it causes night blindness

Your answer was INCORRECT

Explanation

RP is a heterogenous group of conditions which can be caused by a host of mutations,


some of which code for rhodopsin. In addition to peripheral retinal atrophy with streaks
of pigmentation (bone-spicule appearance) microscopy reveals evidence of retinal vessel
attenuation and hyalinization.

Score: 22 %
Question 88 Part 3 – Tutor Mode
Which drug and side-effect pair is NOT correctly matched:

corticosteroids: osteoporosis

tacrolimus: gingival hyperplasia

cyclosporine: bone marrow suppression

azathioprine: diarrhea

Your answer was INCORRECT

Explanation

Cyclosporin and tacrolimus both inhibit interleukin-2. They do not cause bone marrow
suppression (unlike other immunomodulators such as azathioprine and methotrexate) but
they can cause hypertension, hyperlipidameia, glucose intolerance, hirsutism and gingival
hyperplasia.

Score: 21 %
Question 89 Part 3 – Tutor Mode
All are true of Human Papilloma virus EXCEPT:

serotypes HPV-16 and HPV-18 are implicated in cervical carcinoma

it induces epithelial proliferation

it can cause conjunctival papillomas

it is an RNA virus

Your answer was INCORRECT

Explanation

HPV is a DNA virus. Other options are true.

Score: 21 %
Question 90 Part 3 – Tutor Mode
HLA B27:

is a class II major histocompatibility antigen

reduces the risk of anterior uveitis

is found on antigen presenting cells

is found in 8% of the general population

Your answer was INCORRECT

Explanation

HLA-B27 is a class I major histocompatibility antigen and is found in most cells. Class II
HLA antigens and not class I HLA antigens are expressed on antigen presenting cells like
the B lymphocytes. It is associated with ankylosing spondylitis (90% of patients have
HLA B27) which is associated with increased risk of anterior uveitis. It is also found in
8% of the general population.

Score: 22 %
Question 91 Part 3 – Tutor Mode
Which statement is FALSE about the sclera:
it is covered posteriorly by the conjunctiva

it allows passage of the optic nerve through the lamina cribrosa

it consists of three layers, the innermost of which is called the lamina fusca

it is composed of irregularly arranged collagenous lamellae

Your answer was CORRECT

Explanation

The sclera is covered anteriorly by the conjunctiva and posteriorly by the fascia bulbi also
known as Tenon's capsule.

Score: 22 %
Question 92 Part 3 – Tutor Mode
How many ganglion nerve fibres are there in the human eye:

120 million

1.2 million

100,000

12 million

6.5 million

Your answer was INCORRECT

Explanation

There are 1.2 million ganglion cells in the eye.

Score: 22 %
Question 93 Part 3 – Tutor Mode
Which is FALSE regarding the histology of the conjunctiva:

its stroma contains a rich vascular network

mucus-secreting goblet cells are most abundant in the bulbar conjunctiva


its epithelium varies from a stratified squamous to a stratified columnar epithelium

its epithelium contains MHC class II dendritic cells

Your answer was INCORRECT

Explanation

The conjunctival epithelium contains mucus-secreting goblet cells which are most
abundant in the fornices and the plica semiluminaris. Other options are true.

Score: 21 %
Question 94 Part 3 – Tutor Mode
The lateral ventricles are connected to the third ventricle by:

the magna cistern

the foramen of Munro

the aqueduct of Sylvius

the great cerebral vein of Galen

Your answer was INCORRECT

Explanation

The interventricular foramen of Munro connects the lateral ventricles to the third
ventricle. The aqueduct of Sylvius connects the third and fourth ventricles.

Score: 21 %
Question 95 Part 3 – Tutor Mode
Pick the incorrect statement about apoptosis:

it can be induced by cytokines and NK cells

it results in the generation of an inflammatory response

it is inhibited by the oncogene bcl-2

it is characterized by condensation of nuclear chromatin

it results in cell death without necrosis


Your answer was INCORRECT

Explanation

Apoptosis is programmed cell death and occurs in embryonic development and healthy
tissue. The whole process is under genetic control and is influenced by a number of
proto-oncogenes, oncogenes and tumor suppressor genes. Cancer could be considered a
failure of apoptosis. The commonest translocation in hematological malignancies is
Chr14/18 creating the oncogene bcl-2 that blocks apoptosis.

Score: 21 %
Question 96 Part 3 – Tutor Mode
Which is FALSE regarding open angle glaucoma:

temporal nerve fibres are more susceptible than nasal fibres

fibres from the papillomacular bundle tend to be spared

the optic cup is enlarged more horizontally than vertically

mutations have been identified in the GLCA1 gene in some families

Your answer was INCORRECT

Explanation

The optic cup is enlarged more vertically than horizontally. Other options are true.

Score: 22 %
Question 97 Part 3 – Tutor Mode
Which condition results in the accumulation of bi-refringent crystals in the conjunctiva,
cornea, choroid and retina:

cystinosis

Tay-Sachs

Gaucher's

phenylketonuria

homocystinuria

Your answer was CORRECT


Explanation

Cystinosis occurs due to a disturbance in membrane transport of cystine. Ocular


manifestations include the accumulation of cystine crystals in the conjunctiva, cornea,
choroid, RPE and retina.

What is the transposition of +4.0 DS/+1.50 DC at 90 degrees:

+4.00 DS/ -1.50 DC at 180

+6.00 DS/ -1.50 DC at 180

+5.50 DS/ -1.50 DC at 180

-4.00 DS/ -1.50 DC at 90

Your answer was INCORRECT

Explanation

To transpose cylinders, sum the sphere and cylinder, change the sign of the cylinder and
rotate the axis of the cylinder through 90 degrees.

Score: 21 %
Question 99 Part 3 – Tutor Mode
The Northern blot is a technique used in molecular biology research to study:

mitochondrial DNA

double-stranded DNA

RNA

single-stranded DNA

Your answer was INCORRECT

Explanation

Northern blot is a technique for analysing RNA. Southern blot analyses DNA.

Score: 21 %
Question 100 Part 3 – Tutor Mode
The sphincter pupillae muscle:
is composed of striated muscle fibres in a circumferential ring

is responsible for pupillary dilatation

is innervated by the short ciliary nerves

is derived from mesoderm

Your answer was INCORRECT

Explanation

The sphincter pupillae muscle is composed of smooth muscle fibres in a circumferential


ring. It is derived from neuroectoderm. It is responsible for pupillary constriction and is
supplied by parasympathetic fibres via the short ciliary nerves.

Score: 22 %
Question 101 Part 3 – Tutor Mode
Which statement is FALSE regarding Acanthamoeba:

it is the most common cause of corneal ulcer in contact lens wearer

acanthamoeba keratitis is difficult to treat with conventional antibiotics

swimming with contact lenses is a risk factor for acanthamoeba keratitis

it can be cultured from the nasopharynx of normal people

Your answer was CORRECT

Explanation

Acanthamoeba corneal infection is rare and usually occurs in contact lens wearers. It is
not the most common infection in contact lenses wearers, however. The most common
cause of corneal ulcer in contact lens wearer is bacterial infection. Swimming with
contact lenses is a risk factor for acanthamoeba. It can be killed by chemical means such
as hydrogen peroxide or by boiling contaminated water for 10 minutes at a temperature of
70-80 degrees centigrade. Acanthamoeba keratitis is difficult to eradicate.

Score: 23 %
Question 102 Part 3 – Tutor Mode
All of the following are branches of the nasociliary nerve EXCEPT:

the supratrochlear nerve


the anterior ethmoid nerve

the long ciliary nerves

posterior ethmoid nerve

the sensory root to the ciliary ganglion

Your answer was CORRECT

Explanation

The supratrochlear nerve supplies the skin of the medial brow and forehead and is a
terminal branch of the frontal nerve, itself a branch of the ophthalmic nerve.

Score: 22 %
Question 103 Part 3 – Tutor Mode
Which of the following materials is LEAST likely to cause a granulomatous tissue
reaction:

sebum

cotton suture

silicone

wood

vegetable matter

Your answer was INCORRECT

Explanation

Silicone is relatively inert immunologically hence its use as implant material e.g.
intraocular lenses and glaucoma drainage tubes.

Score: 22 %
Question 104 Part 3 – Tutor Mode
A nationwide cross-sectional survey was conducted to compare the prevalence of
respiratory symptoms among university students who have smoked compared to those
who have not smoked.

Which one of the following tests is best used to compare its prevalence?
Logistic regression analysis

Spearmann's correlation

Chi square test

Kaplan Meier curve

Mann Whitney test

Your answer was INCORRECT

Explanation

In order to compare the prevalence in two groups, the chi square test is most appropriate.

Score: 22 %
Question 105 Part 3 – Tutor Mode
Regarding prions, pick the FALSE statement:

they are resistant to the action of proteases

they are derivatives of normal cell membrane proteins

they are a cause of Alzheimer's disease

they are resistant to ionizing radiation

they contain no genetic material

Your answer was INCORRECT

Explanation

Prions are glycoproteins derived from a gene on chromosome 20, which codes for a
normal membrane protein of indeterminate function. A modified version of this protein
can occur and lead to spongiform change of the neural tissue. Unlike its normal
precursor, the prion protein is resistant to heat, ionizing radiation, proteases, disinfectants
and formaldehyde. Diseases caused by prion proteins affect humans and animals. They
manifest as progressive fatal neurodegenerative conditions. Included are Kuru,
Creutzfeldt-Jakob disease (CJD) and Gerstmann-Straussler-Scheinker (GSS) in humans;
bovine spongiform encephalopathy (BSE) in cattle; and scrapie in sheep.

Score: 22 %
Question 106 Part 3 – Tutor Mode
A focimeter uses which colour of light:

blue

green

yellow

red

Your answer was INCORRECT

Explanation

A focimeter uses green light in order to eliminate chromatic aberration.

Score: 21 %
Question 107 Part 3 – Tutor Mode
Parasympathetic agonists instilled topically to the eye can cause all EXCEPT:

conjunctival toxicity

iris cysts

reduced salivation

bradycardia

Your answer was INCORRECT

Explanation

Parasympathetic agonists such as pilocarpine can cause conjunctival toxicity, iris cysts
and cataracts locally. Systemically, they can cause muscarinic effects including increased
salivation, bradycardia, sweating and vomiting.

Score: 21 %
Question 108 Part 3 – Tutor Mode
The region of DNA coding for a specific protein product is termed:

primer

exon
allele

promoter

intron

Your answer was INCORRECT

Explanation

An exon refers to the sequence of DNA coding for a specific product. Between exons
(remember: exons are expressed) are non-coding sequences called introns. Introns are
spliced out of the code during transcription.

Score: 21 %
Question 109 Part 3 – Tutor Mode
The inferior oblique inserts on which quadrant of the globe:

anterolateral

posterolateral

posteromedial

anteromedial

Your answer was INCORRECT

Explanation

The inferior oblique inserts on the posterolateral quadrant, mostly below the horizontal.

Score: 21 %
Question 110 Part 3 – Tutor Mode
Which of the following drugs is most likely to cause Stevens-Johnson syndrome:

phenothiazines

amiodarone

sulphonamides

penicillin
Your answer was INCORRECT

Explanation

Stevens-Johnson syndrome can occur as a consequence of a severe drug reaction and is


typically associated with sulphonamides.

Score: 21 %
Question 111 Part 3 – Tutor Mode
Which statement is FALSE about the corneal endothelium:

the cells contain large numbers of mitochondria

the cells become more homogenous with age

the cells have a low regenerative capacity

the cells are polygonal in shape

Your answer was INCORRECT

Explanation

The corneal endothelial cells become more heterogenous with age, taking on a variety of
shapes, rather than the regular hexagonal pattern typical in the young.

Score: 21 %
Question 112 Part 3 – Tutor Mode
Which statement is FALSE regarding the nerve supply to the lacrimal gland:

it receives sympathetic innervation from the superior cervical ganglion

it is supplied with sensory innervation via the lacrimal nerve

the lacrimatory nucleus receives input from the hypothalamus

its parasympathetic innervation travels along the lesser petrosal nerve

Your answer was INCORRECT

Explanation

Parasympathetic innervation to the lacrimal gland travels via the nervus intermedius to
the greater petrosal nerve, where it joins with the deep petrosal nerve in the pterygoid
canal before synapsing in the pterygopalatine ganglion. From here fibres reach the
lacrimal gland via the retro-orbital plexus. The lesser petrosal nerve synapses with the
otic ganglion and is responsible for parasympathetic innervation to the parotid gland.

Score: 20 %
Question 113 Part 3 – Tutor Mode
Which nutrient medium is NOT correctly matched with the organisms it encourages to
grow:

Lowenstein-Jensen: tuberculosis

non-nutrient e.coli-enriched agar: pseudomonas

Theyer-Martin: gonococcus

Sabouraud's agar: fungus

Your answer was INCORRECT

Explanation

Non-nutrient e.coli-enriched agar is used to cultivate acanthoemeba. Other options are


correctly matched.

Score: 20 %
Question 114 Part 3 – Tutor Mode
Options for microbial sterilization include all of the following EXCEPT:

gamma irradiation

160 degree Celsius dry heat for 30 minutes

ethylene oxide

high temperature steam of 134 degrees Celsius for 3 minutes

Your answer was INCORRECT

Explanation

Dry heat sterilization requires a temperature of 160 degrees Celsius for a duration of at
least 120 minutes. Steam sterilization can be achieved at 120 degrees Celsius in 15
minutes or at 134 degrees Celsius in 3 minutes.

Score: 21 %
Question 115 Part 3 – Tutor Mode
Which is FALSE regarding Leber's hereditary optic atrophy:
it has an insidious onset

it affects boys more than girls

peripapillary telangiectasia are characteristic

it is mitochondrially inherited

Your answer was CORRECT

Explanation

LHON is characterized by rapid onset visual failure. It affects boys more than girls for
reasons that are not understood. On presentation, there is papilloedema and peripapillary
telangiectasia progressing to optic atrophy and vision loss.

Score: 21 %
Question 116 Part 3 – Tutor Mode
Which of the following drugs is a cytochrome P450 enzyme inhibitor:

nicotine

chloramphenicol

rifampicin

phenytoin

Your answer was INCORRECT

Explanation

Chloramphenicol is a P450 enzyme inhibitor while the other options supplied are all
P450 enzyme inducers.

Score: 21 %
Question 117 Part 3 – Tutor Mode
Which is FALSE regarding orbital tumours:

astrocytic hamartomas are associated with tuberous sclerosis

rhabdomyosarcoma is the most common orbital malignancy in childhood


optic nerve gliomas carry a good prognosis

optic meningioma is more indolent in children

Your answer was INCORRECT

Explanation

Optic meningioma is more aggressive in children than adults. Other options are true.

Score: 20 %
Question 118 Part 3 – Tutor Mode
The sixth cranial nerve nucleus is in:

the cephalic pons

the midbrain

the caudal pons

the medulla

Your answer was INCORRECT

Explanation

The sixth nerve nucleus resides in the caudal pons anterior to the fourth ventricle.

Score: 20 %
Question 119 Part 3 – Tutor Mode
At what level of protein structure can an alpha-helix be formed:

tertiary structure

quinternary structure

secondary structure

primary structure

quarternary structure

Your answer was INCORRECT


Explanation

There are four levels of protein structure; primary, secondary, tertiary and quarternary.

 Primary structure refers to a simple polypeptide chain in its simple, unfolded


state.

 Secondary structure shows slightly more complex arrangements, such as alpha-


helixes and beta-sheets.

 Tertiary structure refers to a folded protein which assumes a three dimensional


structure.

 Quarternary structure involves two or more polypeptide chains interacting to form


a dimer.

Score: 20 %
Question 120 Part 3 – Tutor Mode
The fifth cranial nerve:

provides sensory innervations to the whole face

arises from the midbrain

is involved in the blink reflex

supplies parasympathetic fibres to the salivary gland

innervates the muscle of facial expression

Your answer was INCORRECT

Explanation

The fifth cranial nerve provides sensation to the face except for a region around the angle
of the mandible which is supplied by C2 and C3 in the form of the great auricular nerve.
The mandibular branch of the fifth nerve supplies the muscles of mastication, but the
seventh nerve supplies muscles of facial expression. Corneal sensation is provided by the
ophthalmic branch of the fifth nerve and is involved in the blink reflex. The fifth cranial
nerve does not contain parasympathetic nerves fibres of its own.

Score: 21 %
Question 121 Part 3 – Tutor Mode
Where does myelination of the ganglion nerve fibres commence:
upon exiting the lamina cribrosa

within the retinal nerve fibre layer

at the optic nerve head

in the distal third of the intra-orbital optic nerve

Your answer was CORRECT

Explanation

Ganglion nerve fibres become myelinated by oligodendrocytes immediately on exiting


the lamina cribrosa.

Score: 20 %
Question 122 Part 3 – Tutor Mode
Which statement is FALSE regarding the lateral geniculate body (LGN):

the LGN sends the bulk of its fibres to cortex area 17

it is composed of six laminae or cell layers

upper retina fibres synapse more medially in the LGN than lower retina fibres

crossed fibres from the nasal retinae terminate on layers 2, 3 and 5

Your answer was INCORRECT

Explanation

Crossed fibres from the nasal retinae terminate on layers 1, 4 and 6. Un-crossed fibres
(those from the ipsilateral eye) terminate on layers 2, 3 and 5. Other options provided are
true.

Score: 20 %
Question 123 Part 3 – Tutor Mode
The lateral corticospinal tracts decussate at:

the spinal level

the medullary pyramids

the graceate and cuneate nuclei


the thalamus

Your answer was INCORRECT

Explanation

The lateral corticospinal tracts mediate voluntary, skilled motor activity. Fibres originate
in the motor cortex, descend through the medullary pyramids where over 90% of fibres
decussate to form the lateral corticospinal tracts (the remainder form the ventral
corticospinal tracts). The lateral corticospinal tracts then synapse with second order motor
neurones in the ventral horn of the spinal column which directly innervate skeletal
muscle.

Score: 20 %
Question 124 Part 3 – Tutor Mode
Which of the following viruses can potentially cause malignancy by binding to p53 and
inactivating it:

HIV

HPV

EBV

HBV

HCV

Your answer was INCORRECT

Explanation

A number of viruses have been associated with malignancy. HPV produces a protein
called E6 that binds to and inactivates p53. This therefore means that DNA replication
can occur even if the DNA molecules are damaged, and this of course increases the
chances of a malignancy. EBV produces a protein making the cell more resistant to
apoptosis, again increasing the likelihood it will undergo oncogenic change. Hep B and
Hep C are associated with hepatocellular carcinoma. HIV is associated with Kaposi's
sarcoma.

Score: 20 %
Question 125 Part 3 – Tutor Mode
The rate of production of aqueous humour is approximately:
30 microlitres per minute

3 microlitres per minute

3 millilitres per minute

2 millilitres per minute

Your answer was INCORRECT

Explanation

The rate of production of aqueous is between 2 and 4 microlitres per minute

Score: 21 %
Question 126 Part 3 – Tutor Mode
Which statement is FALSE about the medial canthus:

it lies closer to the eyeball than the lateral canthus

it forms a less acute angle than the lateral canthus

it helps form a triangular zone called the lacus lacrimalis where tears accumulate

it can have different shapes in different races

Your answer was CORRECT

Explanation

The medial canthus is further away from the globe than the lateral canthus. It is about
6mm from the eyeball and is separated by a triangular zone called the lacus lacrimalis,
where tears accumulate for drainage.

Score: 21 %
Question 127 Part 3 – Tutor Mode
Which statement is TRUE about cones:

blue cones are the least common of the three types of cones

they are found only in the macula

all cones are coded by DNA found on the X-chromosome


absence of red cones results in protanomaly

Your answer was CORRECT

Explanation

Absence of red cones results in protanopia. Cones are found outside the macula but in
much smaller numbers. Red and green receptors are coded by DNA on the X-
chromosome but the blue is on Chromosome 7.

Score: 21 %
Question 128 Part 3 – Tutor Mode
The excretory ducts of the lacrimal gland secrete tears into:

the superomedial fornix

the inferomedial fornix

the superotemporal fornix

the inferotemporal fornix

Your answer was INCORRECT

Explanation

The lacrimal gland, situated in the lacrimal fossa superotemporally, secretes tears into the
superotemporal fornix via approximately 12 excretory ducts.

Which statement is TRUE about reflection:

a concave mirror always produces an inverted, real, enlarged image

for a spherical reflecting surface the focal length is equal to the radius of curvature

the angle of incidence is the angle between the incident ray and the reflecting surface

reflection at a plane surface produces a virtual image

a convex mirror always produces an erect, virtual, enlarged image

Your answer was INCORRECT

Explanation
The angle of incidence is the angle between the incident ray and the normal to the
reflecting surface. For a spherical reflecting surface the focal length is equal to half the
radius of curvature. A convex mirror always produces an erect, virtual, diminished image.
The image produced by a concave mirror is only inverted, real and enlarged if the object
is between the centre of curvature and the principal focus.

Score: 21 %
Question 130 Part 3 – Tutor Mode
The effects of corticosteroids include all EXCEPT:

increasing the catabolism of immunoglobulins

increasing the amount of lymphoid cells in spleen and lymph nodes

promoting the breakdown of proteins

inhibiting the peripheral utilisation of glucose

Your answer was INCORRECT

Explanation

Corticosteroids are used to control inflammation because of their ability to reduce


immune responses. Some of these abilities include:

 reducing the lymphoid contents within the lymph nodes and spleen
 decreasing certain prostaglandins production such as leukotriene
 reducing the macrophage-mediated lysosomal contents
 impairing the function of macrophages and increasing the catabolism of
immunoglobulins.

Inhibition of peripheral glucose utilisation (resulting in impaired glucose tolerance and


diabetes) and increased protein breakdown are some of the known side effects of
corticosteroids.

Score: 21 %
Question 131 Part 3 – Tutor Mode
Gram negative rods include all EXCEPT:

Proteus vulgaris

Haemophilus influenza

Streptococcus pneumoniae
Pseudomonas aeruginosa

Your answer was INCORRECT

Explanation

Streptococcus is a Gram positive cocci, not a Gram negative rod. All other options are
Gram negatives which can cause ocular disease.

Score: 21 %
Question 132 Part 3 – Tutor Mode
The anterior spinal artery is derived from:

the vertebral artery

the basilar artery

the ascending aorta

the subclavian artery

Your answer was CORRECT

Explanation

The anterior spinal artery is formed by union of branches from the two vertebral arteries.

Score: 21 %
Question 133 Part 3 – Tutor Mode
All of the following are ocular features of Trisomy 21 except:

keratoconus

cataract

Bruschfield spots

hypermetropia

epicanthal folds

Your answer was INCORRECT

Explanation
The ocular features of Down's syndrome include:

 keratoconus
 cataract
 myopia
 Brushfield spots
 epicanthal folds

Score: 21 %
Question 134 Part 3 – Tutor Mode
Which is FALSE regarding reflection of light at a plane surface:

the image is upright

the image is real

the image is laterally inverted

the angle of incidence is equal to the angle of reflection

Your answer was INCORRECT

Explanation

The image produced on reflection at a plane surface is virtual, upright and laterally
inverted.

Score: 21 %
Question 135 Part 3 – Tutor Mode
Cyclosporin:

decreases the production of interleukin-2 (IL-2)

suppresses mainly the humoral immunity

is derived from bacteria

causes bone marrow suppression as its major adverse effect

Your answer was CORRECT

Explanation
Cyclosporin is derived from a soil fungus. It suppresses mainly the cell-mediated immune
reactions and the humoral immunity is only little affected. It works by binding to proteins
within the cells called cyclophilin which inhibits the production of a number of
interleukins especially IL-2. IL-2 is the primary chemical stimulus for T-lymphocyte
proliferation. Cyclosporin, unlike other immunosuppressants, does not cause bone
marrow suppression. Its main adverse effect is nephrotoxicity. It is metabolised by the
liver and excreted in the bile.

Score: 21 %
Question 136 Part 3 – Tutor Mode
Which is FALSE regarding the rod photoreceptors:

the cilium connects the inner segment to the outer segment

the inner half of the inner segment is known as the ellipsoid

the ellipsoid contains numerous mitochondria

they are separated by an interphotoreceptor matrix

Your answer was INCORRECT

Explanation

The inner half of the inner segment is known as the myoid while the outer half of the
inner segment is known as the ellipsoid. The myoid contains numerous organelles
including Golgi apparatus and smooth ER while the ellipsoid contains numerous
mitochondria.

Score: 21 %
Question 137 Part 3 – Tutor Mode
Virulence factors include all of the following EXCEPT:

Ig A-proteases

Capsular polysaccharides in Haemophilus influenzae

Streptococcal M protein

Beta lactamases

Your answer was INCORRECT

Explanation
Virulence factors are molecules expressed and secreted by pathogens that enable them to
achieve the following: colonization, immunoevasion, immunosuppression, and entry into
and out of cells. All of the above are virulence factors except beta lactamases, which
hydrolyse penicillin (and therefore play a role in antibiotic resistance) but which have no
direct effect on host tissue or defenses.

Score: 22 %
Question 138 Part 3 – Tutor Mode
Choose the FALSE statement regarding parathyroid hormone:

it decreases calcium absorption from the gastrointestinal tract

it increases 1,25 dihydroxy-vitamin D3 production by hydroxylation in the kidney

it increases calcium release from bone

it is a peptide hormone

Your answer was CORRECT

Explanation

Parathyroid hormone is a peptide hormone. It raises serum calcium concentration by


increasing calcium release from bone and increasing gastrointestinal absorption of
calcium. It increases the renal hydroxylation of Vitamin D3.

Score: 22 %
Question 139 Part 3 – Tutor Mode
The dorsal column of the spine transmits:

autonomic motor pathways

pain and temperature

volitional motor pathways

vibration, touch and proprioception

Your answer was INCORRECT

Explanation

The dorsal column of the spine transmits vibration, light touch and proprioception. Fibres
ascend the column, synapse in the gracile and cuneate nucleus of the medulla where the
decussate before ascending to the thalamus.
Score: 22 %
Question 140 Part 3 – Tutor Mode
The superior oblique inserts on which quadrant of the globe:

posterosuperior

anteroinferior

posteroinferior

anterosuperior

Your answer was CORRECT

Explanation

The superior oblique inserts on the posterosuperior quadrant of the globe.

Score: 22 %
Question 141 Part 3 – Tutor Mode
Most of the refractive power of the eye comes from:

the lens

the cornea

the vitreous

the iris

Your answer was INCORRECT

Explanation

The surface of the cornea with associated tear film is responsible for most of the
refraction of the eye.

Score: 23 %
Question 142 Part 3 – Tutor Mode
The anterior and posterior ethmoidal nerves are branches of:

the nasociliary nerve

the maxillary nerve


the abducens nerve

the trochlear nerve

the oculomotor nerve

Your answer was CORRECT

Explanation

The anterior and posterior ethmoial nerves are derived from the nasociliary nerve.

Score: 23 %
Question 143 Part 3 – Tutor Mode
Which statement is NOT true about the vitreous:

the central vitreous is more dense than the cortical vitreous

it has a high viscosity roughly 2-4 times that of water

Type 2 collagen and hyaluronan are major constituents

it is composed of 98% water

Your answer was CORRECT

Explanation

The central vitreous is more liquid than the cortical zone, which consists of more densely
arranged collagen fibrils. Other options are true.

Score: 24 %
Question 144 Part 3 – Tutor Mode
The nasociliary nerve:

supplies the sphenoidal air sinus

gives off the supratrochlear nerve which innervates the medial forehead

innervates the cornea but not the sclera

carries within it parasympathetic fibres from the internal carotid plexus


Your answer was CORRECT

Explanation

The nasociliary nerve innervates both the cornea and sclera. It supplies the anterior and
posterior ethmoidal air cells and sphenoid sinus. It carries sympathetic fibres from the
internal carotid plexus. The supratrochlear nerve is a branch of the frontal nerve.

Score: 23 %
Question 145 Part 3 – Tutor Mode
The nasolacrimal duct is lined by:

simple cuboidal epithelium

stratified columnar ciliated epithelium

stratified squamous non-keratinised epithelium

stratified squamous keratinized epithelium

Your answer was INCORRECT

Explanation

The nasolacrimal duct is lined by stratified columnar ciliated epithelium


Score: 23 %
Question 146 Part 3 – Tutor Mode
A father suffers from an X-linked recessive condition. What is the chance that his son
will also be affected:

100%

0%

50%

75%

33%

Your answer was INCORRECT

Explanation
The answer is 0%. His son inherits the Y chromosome from his father and therefore will
not be affected. We recommend drawing a 2 x 2 box of chromosomal probabilities.

Score: 23 %
Question 147 Part 3 – Tutor Mode
Which branch of the external carotid artery contributes to the blood supply of the lateral
palpebral margin:

ophthalmic artery

angular artery

lacrimal artery

transverse facial artery

Your answer was INCORRECT

Explanation

The eyelids are supplied by the palpebral arcades (marginal and peripheral arcades)
which are themselves supplied from both medial and palpebral ends by feeder arteries as
follows:

 medial palpebral artery: supplied by the angular artery (branch of facial artery,
branch of external carotid) and the dorsalis nasi (branch of ophthalmic artery,
branch of internal carotid)

 the lateral palpebral artery: supplied by the transverse facial artery (branch of
external carotid) and the lacrimal artery (branch of ophthalmic artery, branch of
internal carotid)

Score: 23 %
Question 148 Part 3 – Tutor Mode
A 22-year-old patient is known to have recurrent Neisserial infections. Which of the
following is he most likely deficient of?

C4

C1

C3

C5
C2

Your answer was INCORRECT

Explanation

C5-9 are complements involved in the terminal pathway (membrane attack complex) and
deficiency predisposes to Neisserial infections.

Score: 23 %
Question 149 Part 3 – Tutor Mode
As a diagnostic test polymerase chain reaction suffers MOST from which limitation:

false-negatives

inter-observer variation

false-positives

low reproducibility

time consuming

Your answer was INCORRECT

Explanation

PCR is fast, fully automated and highly reproducible. However, because it amplifies
DNA material to such a scale, it suffers from high false-positives due to contamination.

Score: 23 %
Question 150 Part 3 – Tutor Mode
All of the following visual acuity tests use letters EXCEPT:

Sheridan-Gardiner

Kay's

LogMAR

Snellen

Your answer was INCORRECT


Explanation

The Snellen letter chart is the most widely used visual acuity chart, though it is being
replaced in many centres by the LogMAR test. The LogMAR is more precise because of
the regular spacing of letters. The Sheridan-Gardiner and the STYCAR charts use letters,
but they are restricted to the letters that children are most familiar with (such as T, X, V,
O, H). The Kay's is a picture card test where the test-taker matches images; it is useful for
pre-school children. The Cardiff test also uses pictures. They are suitable for 18 months
to 3 years. The Catford drum uses oscillating dots on a drum and is used for babies.

Score: 23 %
Question 151 Part 3 – Tutor Mode
Which is FALSE regarding gene cloning:

restriction endonucleases are utilised

bacteria replicate the required segment of DNA

DNA ligase cleaves sections of human DNA

vectors transfer the material to bacteria

Your answer was INCORRECT

Explanation

In the cloning process:

 restriction endonucleases cleave sections of human DNA


 DNA ligase incorporates human DNA into a vector
 the vector transfers the DNA to bacteria
 bacteria replicate the required segment of DNA.

Score: 23 %
Question 152 Part 3 – Tutor Mode
Which statement is FALSE regarding the cell membrane:

it is freely permeable to ions

it contains a lipid bilayer

the presence of gap junctions allows the transfer of molecules between cells

microvilli are usually found in absorptive cells


Your answer was CORRECT

Explanation

The cell membrane is permeable to lipid molecules but impermeable to ions. Other
options supplied are true.

Score: 23 %
Question 153 Part 3 – Tutor Mode
The pH of tears is:

6.5

7.5

Your answer was INCORRECT

Explanation

The normal pH of tears is 7.5.

Score: 23 %
Question 154 Part 3 – Tutor Mode
All statements regarding Type I collagen are true EXCEPT:

it is the main type of collagen found in basement membrane

it is the most common collagen

it is found in skin and bone

it is found in cornea

Your answer was CORRECT

Explanation

Type 4 is the main type of collagen found within the basement membrane. Other options
are true.

Dot hemorrhages are due to rupture of capillaries in which layer:


Bruch's membrane

the nerve fibre layer

the internal limiting membrane

the inner plexiform layer

the outer plexiform layer

Your answer was INCORRECT

Explanation

Dot hemorrhages are a result of rupture of capillaries in the outer plexiform layers. They
are smaller and more circumscribed than flame hemorrhages.

Score: 23 %
Question 156 Part 3 – Tutor Mode
Regarding the interleukins choose the FALSE statement:

they can function as growth and differentiation factors

the major pro-inflammatory interleukins are IL-1, IL-6 and TNF

they are all pro-inflammatory to some degree

they are synthesized mainly by T cells

they are associated with the development of hypotension

Your answer was INCORRECT

Explanation

An interleukin is a cytokine produced mainly by T-cells. IL-4, IL-10 and IL-13 act to
suppress inflammatory cytokines and the immune response. Interleukins also function as
growth and differentiation factors.

Score: 24 %
Question 157 Part 3 – Tutor Mode
The polymerase chain reaction:
requires the use of oligonucleotide primers

has low sensitivity and specificity

does not require prior knowledge of the DNA sequence being assayed

cannot be used to detect RNA of interest

Your answer was CORRECT

Explanation

PCR is an in vitro amplification of a specific DNA segment that allows for the synthesis
of millions of copies of that DNA segment in a short span of time. It is necessary to know
at least some of the DNA sequence in the region to be assayed before amplification can
be performed. The PCR consists of a three-step cycle: denaturation, primer binding, and
DNA synthesis. Oligonucleotide primers are essential to detect the DNA of interest. RNA
can also be detected with PCR using reverse transcriptase. The technique is highly
specific and sensitive. The high sensitivity is one of its major drawbacks because of the
major risk of false-positive reaction caused by contamination.

Score: 23 %
Question 158 Part 3 – Tutor Mode
The following structures enter the circle of Zinn EXCEPT:

nasociliary nerve

trochlear nerve

abducent nerve

superior branch of the oculomotor nerve

Your answer was INCORRECT

Explanation

Four nerves pass through the circle of Zinn:

 abducent nerve
 inferior branch of the oculomotor nerve
 superior branch of the oculomotor nerve
 nasociliary nerve
The trochlear nerve passes into the orbit outside and superior to the circle of Zinn.

Score: 23 %
Question 159 Part 3 – Tutor Mode
Which process produces the most net ATP per molecule of glucose:

gluconeogenesis

glycolysis

citric acid cycle

pentose phosphate pathway

Your answer was INCORRECT

Explanation

The citric acid cycle produces the most energy- the equivalent of 30 molecules of ATP
for each glucose molecule that enters. Glycolysis produces 6 molecules of ATP for each
glucose molecule. The pentose phosphate cycle produces pentose for DNA/RNA
synthesis and NADPH for biosynthetic reactions, while gluconeogenesis is an anabolic
(not catabolic) process involving the production of glucose from precursors.

Score: 24 %
Question 160 Part 3 – Tutor Mode
Which is FALSE regarding the synaptic terminals of the photoreceptors:

spherules are broader than pedicles

pedicles have more numerous connections than spherules

photoreceptor nuclei are connected to pedicles and spherules by an inner rod fibre

the synaptic terminal of the cone is termed the pedicle

Your answer was CORRECT

Explanation

The synaptic terminal of the cone is termed the pedicle and the terminal of the rod is
termed the spherule. Pedicles are broader than spherules and have more neuronal
connections. The terminals are connected to the cell body by an inner rod fibre

Score: 24 %
Question 161 Part 3 – Tutor Mode
Which statement is FALSE about the corneal epithelium:

it responds rapidly to disruptions by amoeboid sliding movements

the epithelial cells rest on a basal lamina comprising the lamina lucida and lamina densa

new cells are derived from mitosis in the limbal basal layer

adhesion between epithelial cells and the Bowman's is facilitated by Type II collagen

Your answer was INCORRECT

Explanation

Adhesion between epithelial cells and Bowman's layer is maintained by


hemidesmosomes and by a complex mesh of anchoring fibrils and plaques composed of
Type VII and Type VI collagen (not Type II).

Score: 24 %
Question 162 Part 3 – Tutor Mode
Which is FALSE of a 20 dioptre condensing lens used for indirect ophthalmoscopy:

it produces an erect and laterally inverted image

it produces more magnification than a 28 dioptre lens

the refractive state of the patient's eye affects the size and position of the image

it is held further away from the patient than a 28 dioptre lens

Your answer was CORRECT

Explanation

The image produced by a condensing lens is inverted both vertically and laterally. Other
options are true.

Score: 24 %
Question 163 Part 3 – Tutor Mode
All statements about the levator superioris are true EXCEPT:

it is attached to the lesser wing of the sphenoid bone

it turns into aponeurosis where the superior transverse ligament of Whitnall is found
it is attached to the circle of Zinn

it originates at a point superolateral to the optic foramen

Your answer was INCORRECT

Explanation

The levator superioris is not attached to the circle of Zinn. All other statements are true.

Score: 24 %
Question 164 Part 3 – Tutor Mode
The following are true about pain sensation EXCEPT:

it ascends the spinal cord in the dorsal column tracts

local anesthetics that block sodium channels in nerve fibers can reduce pain sensation

it arises by stimulation of free nerve endings

it is transmitted to the central nervous system by unmyelinated C fibers

the thalamus is important in the perception of pain

Your answer was CORRECT

Explanation

The majority of pain sensation arises from the stimulation of free nerve endings. Pain
sensation is transmitted to the CNS in small unmyelinated C fibers. It is then transmitted
to the thalamus in the spinothalamic tracts. Local anaesthetics work by blocking sodium
channels in nerve fibers.

Score: 24 %
Question 165 Part 3 – Tutor Mode
The maxillary artery:

arises from the internal carotid artery

gives off its main branches in the pterygopalatine fossa

gives rise to the inferior alveolar artery which supplies the maxillary gingival circulation
supplies the maxillary incisor teeth by way of its infraorbital branch

Your answer was INCORRECT

Explanation

The maxillary artery is a branch of the external carotid artery. The maxillary artery gives
off its main branches within the infratemporal fossa: the middle meningeal artery and the
inferior alveolar artery that supply the mandibular gingiva. It enters the pterygopalatine
fossa and splits into small branches that accompany the maxillary nerve. The infraorbital
artery is a branch of the maxillary artery and its own branches supply the maxillary
incisor teeth.

Score: 24 %
Question 166 Part 3 – Tutor Mode
Which statement is true about chloroquine:

it causes reversible toxic maculopathy

it is safer than hydroxychloroquine at equivalent dose

it causes posterior subcapsular cataract

it binds to the melanin of the RPE

Your answer was INCORRECT

Explanation

Chloroquine is more toxic to the retina dose-for-dose than hydroxychloroquine. It can


cause corneal deposition and anterior subcapsular cataract neither of which is of visual
significance. It is bound to the melanin in the retinal pigment epithelium which is
believed to be the reason for its toxicity. It causes Bull's eye maculopathy which may
progress despite stopping the medication

Score: 24 %
Question 167 Part 3 – Tutor Mode
Human lymphocyte antigens (HLA):

are found on red blood cells

play an important role in antigen recognition by the T lymphocytes

matching is important for routine corneal graft


are coded by genes found on the X-chromosome

Your answer was INCORRECT

Explanation

HLA genes are found on chromosome 6. They are found on cells with nuclei and
therefore not on the red blood cells. HLA are essential for antigen recognition by the T
lymphocytes. Matching of HLA is important for organ transplant but routine corneal graft
(by virtue of its lack of blood vessels) are not usually needed.

Score: 24 %
Question 168 Part 3 – Tutor Mode
Which statement is FALSE regarding chloramphenicol:

it inhibits peptidyltransferase

it is bactericidal

it is effective against both Gram-positive and Gram-negative bacteria

it acts by inhibiting bacterial protein synthesis

Your answer was INCORRECT

Explanation

Chloramphenicol acts by inhibiting bacterial peptidyltransferase necessary for protein


synthesis. It is effective against both Gram-positive and Gram-negative bacteria. It is
bacteriostatic rather than bactericidal. It is one of the commonest topical antibiotics
available. Its systemic use is limited by its potential to cause a fatal aplastic anaemia.

Score: 24 %
Question 169 Part 3 – Tutor Mode
Which of the following statements is FALSE about nitric oxide:

it promotes platelet aggregation and adhesion

it exists in a free radical form

it causes cell death by interference with lipid membranes

it can act as a neurotransmitter

its production is stimulated by TNF


Your answer was CORRECT

Explanation

Nitric oxide (NO) or EDRF is synthesized from the oxidation of nitrogen atoms in the
amino acid L-arginine by the enzyme nitric oxide synthase. It is produced in many tissues
and has several diverse biological functions. Nitric oxide dilates arteries and arterioles,
regulates cardiac contractility and inhibits platelet aggregation and adhesion. Nitric oxide
has been identified as a neurotransmitter in the CNS and PNS and is thought to play a
role in memory formation. Excessive nitric oxide production mediates hypotension in
septic shock. Synthetic nitrates such as GTN and sodium nitroprusside act after
conversion to nitric oxide.

Score: 24 %
Question 170 Part 3 – Tutor Mode
ALL of the following events occur after 24 hours of fasting EXCEPT:

lipolysis

the brain switches from using glucose to ketone bodies

muscle breakdown

the blood glucose concentration is maintained

hepatic gluconeogenesis

Your answer was INCORRECT

Explanation

After 24 hours of fasting:

 the liver undergoes gluconeogenesis as the storage of glycogen is usually


insufficient to maintain the blood glucose level
 lipolysis occurs
 the blood glucose concentration is usually maintained
 the brain switches from using glucose to ketone bodies which are derived from
fatty acid oxidation in the liver.

Note: muscle breakdown does not occur until much later.

Score: 24 %
Question 171 Part 3 – Tutor Mode
The following are features of malignant tumours EXCEPT:
cellular pleomorphism

nuclear hyperchromatism

nuclear pleomorphism

increased cytoplasmic: nuclear ratio

Your answer was INCORRECT

Explanation

Malignant cells typically shows cellular and nuclear pleomorphism with nuclear
hyperchromatism. There is a decrease in the cytoplasmic: nuclear ratio

Score: 24 %
Question 172 Part 3 – Tutor Mode
Which statement is FALSE about the dural venous sinuses:

they are lined by endothelial cells

they drain venous blood from the brain and cranial bones

their walls contain smooth muscle cells

they are valveless

Your answer was INCORRECT

Explanation

The walls of the dural venous sinuses are lined by endothelial cells in common with other
veins but they do not contain smooth muscle cells. Other options supplied are true.

Score: 24 %
Question 173 Part 3 – Tutor Mode
The most common site for ocular metastases is:

the cornea

the retina

the choroid
the ciliary body

Your answer was INCORRECT

Explanation

The choroid is a highly vascular tissue and is the most common ocular site for metastases.
Common primaries are the breast, prostate, lung and colon.

Score: 24 %
Question 174 Part 3 – Tutor Mode
The actions of the following antibiotics are correct EXCEPT:

chloramphenicol inhibits bacterial protein synthesis

metronidazole interferes with DNA synthesis

sulphonamides interferes with folic acid synthesis

fusidic acid interferes with protein synthesis

tetracycline inhibits cell synthesis

Your answer was INCORRECT

Explanation

Tetracycline interferes with bacterial protein synthesis. All other statements are true.

Score: 23 %
Question 175 Part 3 – Tutor Mode
Which is TRUE regarding rod photoreceptors:

their discs are not enclosed by cell membrane

their concentration is lower in the periphery

peak sensitivity to light at wavelength of 496nm

the rod:cone ratio is 10:1

Your answer was INCORRECT


Explanation

Rod photoreceptor discs are enclosed within a cell membrane (unlike those of cones).
The rod concentration is higher in the periphery relative to the centre. The ratio of
rods:cones is 20:1. The peak sensitivity of rods is at 496nm.

Score: 23 %
Question 176 Part 3 – Tutor Mode
Which is the only extraocular muscle not to originate at the orbital apex:

the superior rectus

the superior oblique

the inferior rectus

the inferior oblique

Your answer was INCORRECT

Explanation

The inferior oblique is the only extrocular muscle to originate outside the orbital apex. It
originates behind the orbital margin lateral to the nasolacrimal duct and passes between
the eye and the lateral rectus.

Score: 23 %
Question 177 Part 3 – Tutor Mode
All of the following are true about Hepatitis B EXCEPT:

it is a DNA virus

it is associated with hepatocellular carcinoma

a carrier is identified by the presence of surface antigen in the presence of Ig G against


hepatitis B

it has an incubation period of 2 to 6 weeks

Your answer was INCORRECT

Explanation

Hepatitis B is a DNA virus. It has an incubation period of 2 to 6 months. About 10% of


infected individuals become chronic carriers. Hepatitis B surface antigen (HBsAg) is a
protein antigen produced by the virus. This antigen is the earliest indicator of acute
hepatitis B and frequently found in infected people before symptoms appear. HBsAg
disappears from the blood during the recovery. In some patients, chronic infection occurs
and the HBsAg remains positive. Chronic hepatitis B carrier is associated with a high risk
of developing hepatocellular carcinoma.

Score: 24 %
Question 178 Part 3 – Tutor Mode
The basal lamina of the inner layer of the optic cup differentiates into:

internal limiting membrane

Bruch's membrane

nerve fiber layer

retinal pigment epithelium

external limiting membrane

Your answer was CORRECT

Explanation

The basal lamina of the inner layer of the optic cup becomes the internal limiting
membrane. The basal lamina of the outer layer of the optic cup becomes incorporated
into Bruch 's membrane.

Score: 24 %
Question 179 Part 3 – Tutor Mode
Which statement regarding apoptosis is FALSE:

it causes inflammation which may damage surrounding cells

it causes the plasma membrane to undergo zeiosis

it is the normal physiological process of programmed cell death

it results in products that are removed by phagocytosis

Your answer was CORRECT

Explanation
Apoptosis is a normal physiological process of programmed cell death. Apoptosis does
not cause inflammation (unlike necrosis). The plasma membrane undergoes blebbing
called zeiosis. The products of apoptosis are removed by phagocytosis.

Score: 24 %
Question 180 Part 3 – Tutor Mode
A large research study was conducted to test a new drug in cancer. The risk of dying with
placebo is 15 percent and with the new drug is 10 percent at five years.

What is the number needed to treat to prevent one death in 5 years?

20

50

200

10

Your answer was CORRECT

Explanation

The absolute risk reduction is 5%. Definition of NNT is 100 / ARR, in this case It is 100 /
5 = 20

Score: 25 %
Question 181 Part 3 – Tutor Mode
Which statement about the trochlear nerve is FALSE:

it pierces the dura to enter the medial wall of the cavernous sinus

it travels anteriorly beneath the free edge of the tentrorium cerebelli

it enters the orbit via the superior orbital fissure outside the tendinous ring

it travels above the superior cerebellar artery and below the posterior cerebral artery as it
winds around the crus of the midbrain

Your answer was CORRECT

Explanation
The trochlear nerve pierces the dura to enter the lateral wall of the cavernous sinus. Other
options supplied are true.

Score: 25 %
Question 182 Part 3 – Tutor Mode
All are true of Neisseria gonorrhoeae EXCEPT:

it is a Gram-negative diplococcus

its growth is optimal on blood agar

it can cause Ophthalmia neonatorum

it is sensitive to penicillin

Your answer was INCORRECT

Explanation

Growth of Neisseria is optimal on enriched chocolate media such as Thayer-Martin.

Score: 25 %
Question 183 Part 3 – Tutor Mode
Allowing 5 minutes between two eye drops works on the principle of:

pharmacodynamics

saturation kinetics

bioavailability

pharmacokinetics

Your answer was INCORRECT

Explanation

Allowing time between drops ensures sufficient absorption, that is, it is a


pharmacokinetic consideration. Pharmacokinetics deals with the liberation, absorption,
distribution, metabolism and excretion of drugs. Pharmacodynamics refers to the
physiological mechanisms whereby drugs perform their actions. Bioavailability refers to
the concentration of a drug reaching the circulation (thus an iv drug has a bioavailability
of 100%).

Score: 24 %
Question 184 Part 3 – Tutor Mode
Which is FALSE regarding wound healing by secondary intention:

may occur when the wound becomes infected

may occur when there is irreparable skin loss

tends to occur more quickly than healing by primary intention

may occur when the wound dehisces

Your answer was INCORRECT

Explanation

Wound healing by secondary intention is slower than by primary intention where wound
edges are well opposed. Other statements are true.

Score: 24 %
Question 185 Part 3 – Tutor Mode
Which statement is FALSE regarding orbital pseudotumour:

it causes oedema of orbital tissues with fibrosis in the late stages

it responds poorly to systemic steroids

it is a non-granulomatous inflammation

infiltrates are predominantly lymphocytic

Your answer was INCORRECT

Explanation

Orbital pseudotumour usually responds rapidly to corticosteroids, though other


immunosuppresants such as azathioprine may be used if there is an inadequate response.

Score: 24 %
Question 186 Part 3 – Tutor Mode
When a patient stands up from a lying position, the following occurs:

peripheral vasodilatation

bradycardia
renin secretion is reduced

a significant drop of cerebral blood flow

cardiac output is reduced

Your answer was INCORRECT

Explanation

On standing from a lying position, there is a reduction in venous return, which in turn
decreases the cardiac output. To maintain the blood pressure, the baroceptors initiates
peripheral constriction. There is an increase in the secretion of renin and angiotensin.
Tachycardia occurs to maintain the cardiac output. The cerebral flow remains constant
due to autoregulation.

Score: 24 %
Question 187 Part 3 – Tutor Mode
Sex hormones exert their action by:

inhibiting translation

enhancing transcription after binding to receptors in the cytoplasm

releasing a second messenger

binding to specific membrane receptors

interacting with DNA directly

Your answer was INCORRECT

Explanation

Sex hormones (estrogen, testosterone, progesterone) are steroid hormones. All steroid
hormones bind specific cytoplasmic receptors, and then bind DNA. Thyroid hormones
also act this way. Contrast this with non-steroid hormones such as epinephrine, which
interact with the cell membrane and then release a 2nd messenger. As a result they act
more quickly.

Score: 24 %
Question 188 Part 3 – Tutor Mode
Which of the following statements regarding the analysis of trial data is correct?

parametric tests may be applied to non-normally distributed data


the t-test is a parametric test

one-tailed tests will suffice for the majority of clinical trials

intention to treat analyses are preferable because they require a smaller sample size

the Mann-Whitney and Wilcoxon tests are parametric tests

Your answer was INCORRECT

Explanation

One-tailed tests will not suffice for the majority of clinical trials because in clinical trials
we almost always perform two tailed tests. Parametric tests cannot be applied to non-
normally distributed data, but non-parametric tests may be applied to normal data (but
give less good results). The Mann-Whitney and Wilcoxon tests are non-parametric tests.

Score: 24 %
Question 189 Part 3 – Tutor Mode
Which of the following metals can produce a pyogenic response if retained in tissue:

iron

copper

magnesium

zinc

aluminium

Your answer was INCORRECT

Explanation

Retained tissue copper produces a pyogenic immune response for reasons that are not
understood.

Score: 24 %
Question 190 Part 3 – Tutor Mode
Which is FALSE regarding Von-Hippel-Lindau syndrome:

it is associated with capillary haemangiomata of the retina


it is associated with optic nerve glioma

it is associated with renal cell carcinoma

it is dominantly inherited

Your answer was INCORRECT

Explanation

Von-Hippel-Lindau syndrome is an autosomal dominant syndrome due to a mutation in


chromosome 3. It is associated with capillary haemangiomata of the retina, renal cell
carciona and haemangioblastoma of the cerebellum.

Optic nerve glioma is associated with NF-1.

Score: 24 %
Question 191 Part 3 – Tutor Mode
All are true of the lens epithelium EXCEPT:

it is a simple cuboidal epithelium

mitotic activity is maximal in the polar regions

the epithelial cell sinks deeper into the lens as further cells are laid down during mitosis

it produces lens fibres by a process involving elongation

Your answer was INCORRECT

Explanation

Mitotic activity of lens epithelial cells is maximal in the equatorial and pre-equatorial
regions known as the germative zone. Other statements are true.

Score: 23 %
Question 192 Part 3 – Tutor Mode
Ischaemia due to occlusion of a calcarine artery would be expected to produce:

incongruous contralateral homonymous hemianopia with macular sparing

incongruous contralateral homonymous hemianopia


congruous contralateral homonymous hemianopia with macular sparing

congruous contralateral homonymous hemianopia

Your answer was INCORRECT

Explanation

Ischaemia to a calcarine artery produces a congruous defect (more posterior the defect,
more congruous) and there is often macular sparing because of anastomoses between the
middle and posterior cerebral arteries at the very pole, where the macular fibres
terminate.

Score: 24 %
Question 193 Part 3 – Tutor Mode
Which cause and effect pair is NOT correctly matched:

ruptured lens capsule: phacomorphic glaucoma

melanoma of iris compressing angle: secondary angle closure glaucoma

central retinal vein occlusion: rubeotic glaucoma

peripheral anterior synechiae: secondary angle closure glaucoma

Your answer was CORRECT

Explanation

Phacomorphic glaucoma (not phacolytic glaucoma) refers to secondary angle closure due
to lens intumescence. Phacolytic glaucoma occurs when lens proteins are released into
the anterior chamber, which create an intense granulomatous reaction with macrophages.
These macrophages and lens material can block the trabecular meshwork leading to a rise
in pressure and the term phacolytic glaucoma.

Score: 24 %
Question 194 Part 3 – Tutor Mode
Regarding wound infections all of the following are TRUE except:

Staphylococcus aureus is a rare cause of surgical wound infections

MRSA wound infection can occur due to wound contamination by hospital staff

opportunistic infections occur in patients with reduced immune defences


necrotizing fasciitis is caused by streptococcus

Your answer was CORRECT

Explanation

Staph aureus is a commensal on the skin and is the most common cause of post-surgical
wound infection. Other options are true.

Score: 24 %
Question 195 Part 3 – Tutor Mode
Which is FALSE regarding Toxoplasma infection:

congenital infection may cause hydrocephalus and retinochoroiditis

congenital infection occurs when a woman is previously exposed to toxoplasma

congenital infection is more likely to be recurrent

ocular reactivation occurs at the border of a previous retinchoroidal scar

Your answer was INCORRECT

Explanation

Congenital infection occurs when a mother becomes infected for the first time during
pregnancy. Other options are true.

Score: 24 %
Question 196 Part 3 – Tutor Mode
Which of the following best describes the mode of action of sulphonamides:

inhibit cell membrane synthesis

inhibit cell wall synthesis

inhibit protein synthesis

inhibit nucleic acid synthesis

Your answer was INCORRECT

Explanation
Sulphonamides act by inhibiting folate metabolism, which is essential for bacterial DNA
synthesis and survival.

Score: 24 %
Question 197 Part 3 – Tutor Mode
Bruschfield spots are associated with:

Sturge-Weber syndrome

Neurofibromatosis Type 1

Neurofibromatosis Type 2

Down's syndrome

Your answer was INCORRECT

Explanation

Bruschfield spots are small nodules on the iris and occur in Down's syndrome. The ocular
features of Down's syndrome include:

 keratoconus
 cataract
 myopia
 Brushfield spots
 epicanthal folds

Score: 24 %
Question 198 Part 3 – Tutor Mode
Which is FALSE regarding the cone photoreceptor:

the outer segments are shorter in cones than rods

cone lamellae are not enclosed by a membrane

the synaptic terminals of cones are called spherules

cone lamellae have a longer lifespan than rods

Your answer was INCORRECT

Explanation
The synaptic terminals of the cones are called pedicles while those of the rod are termed
spherules. They synapse with bipolar and horizontal cells.

Score: 24 %
Question 199 Part 3 – Tutor Mode
MHC Class II cell surface antigens are expressed on all the following cells EXCEPT:

B cells

dendritic cells

erythrocytes

macrophages

Your answer was INCORRECT

Explanation

Major histocompatibility complex (MHC) class II molecules are expressed on the


surfaces of antigen-presenting cells - including macrophages, dendritic cells, and B cells.
They serve to stimulate CD4+ Th-cells and initiate cytokine release that regulates other
cellular processes associated with immune responses. The Dr, Dp, and Dq regions code
for these antigens.

Score: 24 %
Question 200 Part 3 – Tutor Mode
The following are true about the vortex veins EXCEPT:

there are usually 4 to 7 in each eye

they drain into the superior and inferior ophthalmic veins

they exit the eye at the equator

they drain the retina and the choroid

Your answer was INCORRECT

Explanation

The vortex veins drain the choroid, ciliary body and the iris (not the retina).

Score: 23 %
Question 201 Part 3 – Tutor Mode
In which part of the nephron does water absorption not take place?
collecting duct

ascending limb of Loop of Henle

descending limb of Loop of Henle

proximal convoluted tubule

distal convoluted tubule

Your answer was INCORRECT

Explanation

Unlike the descending limb, the ascending limb of Henle's loop is impermeable to water,
a critical feature of the countercurrent exchange mechanism employed by the loop. The
ascending limb actively pumps sodium out of the filtrate, generating the hypertonic
interstitium that drives countercurrent exchange. Though the collecting duct is normally
impermeable to water, it becomes permeable under the actions of antidiuretic hormone
(ADH). As much as three-fourths of the water from urine can be reabsorbed as it leaves
the collecting duct by osmosis.

Score: 23 %
Question 202 Part 3 – Tutor Mode
The lens is derived embryologically from:

neuroectoderm

mesoderm

ectoderm

endoderm

Your answer was INCORRECT

Explanation

The lens is derived from the ectoderm.

Score: 23 %
Question 203 Part 3 – Tutor Mode
Which is TRUE of Toxoplasma gondii infection:
it is more common in immunocompromised patients than cytomegalovirus

it can be transmitted through dog faeces

acquired infection usually causes a severe acute illness

it can be transmitted from undercooked meat

Your answer was INCORRECT

Explanation

Toxoplasma gondii is the commonest protozoal parasite to affect the eye. It is transmitted
from ingestion of undercooked meat or the ingestion of soil contaminated with infected
cat faeces. (Toxocara is transmitted through dog faeces). Congenital infection may result
in severe symptoms including hydrocephalus, intracranial calcification and mental
retardation. Acquired infection is often asymptomatic. It is less commonly associated
with immunocompromise than CMV retinitis

Score: 23 %
Question 204 Part 3 – Tutor Mode
Which of the following structures exits the cranium through the foramen ovale:

the greater petrosal nerve

the mandibular nerve

the oculomotor nerve

the main meningeal artery

sympathetic nerve fibres

Your answer was INCORRECT

Explanation

The mnemonic MALE is useful for remembering the structures that pass through the
foramen ovale: Mandibular nerve, Accessory meningeal artery, Lesser petrosal nerve and
Emissary vein

Score: 23 %
Question 205 Part 3 – Tutor Mode
The hemoglobin-oxygen dissociation curve shifts to the right in ALL cases EXCEPT:
increase in hydrogen ion concentration

increase in 2,3 diphosphoglycerate (2,3 DPG)

hypothermia

increase in pCO2

Your answer was INCORRECT

Explanation

Shifting the hemoglobin-oxygen dissociation curve to the right increases the release of
oxygen to the tissues. This shift occurs in acidosis, raised temperature and increased 2,3
DPG.

Score: 23 %
Question 206 Part 3 – Tutor Mode
The third cranial nerve arises from the brain stem between which two branches of the
basilar artery:

I. the posterior cerebral artery


II. the labyrinthine artery
III. the superior cerebellar artery
IV. the anterior inferior cerebellar artery

III and IV

I and IV

I and II

I and III

Your answer was INCORRECT

Explanation

The third cranial nerve arises from the brainstem between the posterior cerebral artery
and the superior cerebellar artery.

Score: 23 %
Question 207 Part 3 – Tutor Mode
Which of the following is TRUE about cerebrospinal fluid:
only the third ventricle contains the choroidal plexus which secretes CSF

cerebrospinal fluid contains the same concentration of glucose as the blood

its normal volume in humans is about 130 ml

it is found in the space between the dura mater and the arachnoid

Your answer was INCORRECT

Explanation

Cerebrospinal fluid is found in the subarachnoid space ie between the pia mater and the
arachnoid. The normal amount is 130 ml and the normal intracranial pressure is 100 ml of
water (varying between 60 to 150 ml of water, or about 7-15mmHg). It is produced by
the choroidal plexus of the lateral ventricle, third ventricle and the fourth ventricle. It
contains 60% of the concentration of glucose found in the plasma. Few cells and trace
protein are found in the normal state.

Score: 23 %
Question 208 Part 3 – Tutor Mode
Which statement regarding HIV is FALSE:

it can be passed from mother to foetus

it gains entry into cells with CD4 cell-surface receptor molecules

its core protein p24 can be used to monitor progression of AIDS

it is a single-stranded DNA virus

Your answer was INCORRECT

Explanation

HIV is a single-stranded RNA virus. Other options are true

Score: 23 %
Question 209 Part 3 – Tutor Mode
Which statement about the foramen lacerum is FALSE:

it transmits pre-ganglionic parasympathetic fibres


it occurs at the apex of the petrous temporal bone

it transmits the internal carotid artery

its lower end is covered by connective tissue

Your answer was CORRECT

Explanation

The foramen lacerum transmits sympathetic nerves associated with the internal carotid
artery but not parasympathetic fibres. Other options are true.

Score: 23 %
Question 210 Part 3 – Tutor Mode
What is the spherical equivalent of the toric lens of power +2.00DS/+2.00DC:

4.00DS

1.50DS

3.00DS

2.00DS

Your answer was INCORRECT

Explanation

The spherical equivalent of a toric lens is calculated by addition of the spherical power
and half the cylindrical power.

Score: 23 %
Question 211 Part 3 – Tutor Mode
Which stage of trachoma is characterized by fibrous replacement of inflamed tissue:

Stage IV

Stage I

Stage III

Stage II
Your answer was INCORRECT

Explanation

Trachoma has the following recognized stages:

 Stage I: involvement of the conjunctival stroma


 Stage II: involvement of the cornea with pannus formation
 Stage III: fibrous replacement of inflamed tissue
 Stage IV: contraction with entropion and trichiasis.

Score: 23 %
Question 212 Part 3 – Tutor Mode
The following are true of prisms EXCEPT:

they are used in the assessment of binocular function

a combination of base-up and base-down prisms are used to measure vertical deviation

they are an important part of a Javal Schiotz keratometer

the power in prism dioptres is half the apex angle in degrees

Your answer was INCORRECT

Explanation

The power in prism diopters is roughly equal to the apex angle in degrees. Other options
are true

Score: 23 %
Question 213 Part 3 – Tutor Mode
Which option is FALSE regarding the lens fibres:

older lens fibres are distinguished by larger nuclei

during elongation the lens cell nucleus migrates anteriorly as the cell is pushed deeper in
the lens

lens fibres are arranged with little intercellular spaces

during elongation the apical portion of the lens cell passes anteriorly

Your answer was CORRECT


Explanation

Older lens fibres are found deeper within the lens substance. The older lens fibres are
anucleate and are incapable of producing lens proteins known as crystallins unlike
younger more superficial lens fibres. Other options are true.

Score: 23 %
Question 214 Part 3 – Tutor Mode
Which of the following is NOT true of prisms:

the dipersive power is independent of the refractive index of the prism

the image formed by a prism is displaced towards the apex of the prism

decentering a convex lens induces image displacement towards the centre of the lens

the position of minimum deviation is when the angle of incidence equals the angle of
emergence

the Porro prism inverts an image and deviates the light path through 180 degrees

Your answer was INCORRECT

Explanation

Decentering a convex lens induces image displacement towards the periphery of the lens
which is the apex of the induced prism.

Score: 23 %
Question 215 Part 3 – Tutor Mode
Which is FALSE regarding the conjunctiva:

it contains parasympathetic nerves from the pterygoplatine ganglion

it contains accessory lacrimal glands responsible for reflex tear production

its blood supply is derived partly from vessels that supply the lids

its nerve supply includes the long ciliary nerve

Your answer was INCORRECT

Explanation
The conjunctiva contains parasympathetic nerves from the pterygopalatine ganglion. Its
blood supply is derived partly from the vessels supplying the superficial lids but also
from the anterior ciliary arteries, which are branches of the ophthalmic artery. The bulbar
conjunctiva is supplied with sensory innervation by the long ciliary nerve, a branch of the
nasociliary nerve. The palpebral conjunctiva is supplied by the supraorbital,
supratrochlear and lacrimal (all branches of the ophthalmic nerve) and by the infraorbital
nerve (a branch of the maxillary nerve). Accessory lacrimal glands exist in the
conjunctiva and these are responsible for baseline tear production. Reflex tearing is the
responsibility of the lacrimal gland.

Score: 23 %
Question 216 Part 3 – Tutor Mode
Which statement regarding the caliber of arterioles is NOT correct:

angiotensin II causes vasoconstriction

adrenaline causes vasodilatation in skeletal muscles

decreased oxygen tension causes vasodilatation

increased noradrenergic impulse causes vasodilatation

Your answer was INCORRECT

Explanation

Vasodilatation of the arterioles occurs in decreased oxygen tension and the presence of
adrenaline in skeletal muscle. Vasoconstriction occurs in the presence of angiotensin II,
increased noradrenergic discharge and locally released serotonin.

Score: 23 %
Question 217 Part 3 – Tutor Mode
Blunt ocular trauma with separation of the ciliary muscle attachment is likely to cause:

rubeotic glaucoma

phacomorphic glaucoma

phacolytic glaucoma

chronic open angle glaucoma

angle recession glaucoma

Your answer was INCORRECT


Explanation

Angle recession glaucoma occurs following blunt trauma. The ciliary muscle attachment
to the scleral spur is disrupted causing deepening of the angle and damage to the
trabecular meshwork causing rise in intra-ocular pressure.

Score: 23 %
Question 218 Part 3 – Tutor Mode
Tetracycline is associated with which ocular side-effect:

papilloedema

cataract

retinal toxicity

glaucoma

corneal opacification

Your answer was CORRECT

Explanation

Tetracycline is associated with papilloedema.

Score: 23 %
Question 219 Part 3 – Tutor Mode
The supraorbital nerve is derived from:

the lacrimal nerve

the zygomaticotemporal nerve

the trochlear nerve

the frontal nerve

Your answer was INCORRECT

Explanation

The supraorbital nerve is a terminal branch of the frontal nerve


Score: 23 %
Question 220 Part 3 – Tutor Mode
All of the following are true of the kidney EXCEPT:

it plays an important role in red blood cell production

over 99% of filtered water is reabsorbed by the kidney

antidiuretic hormone decreases the water permeability of the collecting duct

the blood flow accounts for 25% of the total cardiac output

Your answer was INCORRECT

Explanation

The kidneys receive approximately 25% of the total cardiac output. Over 99% of the
filtered water is reabsorbed by the kidney. Antidiuretic hormone increases the water
permeability of the collecting duct and hence increases water retention. The kidney
secrets erythropoietin which is essential for red blood cell production.

Score: 23 %
Question 221 Part 3 – Tutor Mode
If a father and a paternal grandfather of a boy with a congenital eye defect have the same
disease, what is the most likely mode of inheritance:

autosomal dominant

mitochondrial inheritance

X-linked recessive

autosomal recessive

Your answer was CORRECT

Explanation

Genetic diseases passing from one generation to the next are typical of autosomal
dominant inheritance. This case is not X-linked because X-linked recessive conditions
pass from mother to son. Mitochondrial inheritance passes from the mother to offspring
(male or female).

Score: 23 %
Question 222 Part 3 – Tutor Mode
Which is FALSE regarding neutralization of lenses:

the neutralizing lens is placed against the back surface of the spectacle lens

astigmatic neutralization requires each meridian to be neutralized separately

the spectacle lens is equal in power and sign to the neutralizing lens

neutralization occurs when there is no with or against movement

Your answer was INCORRECT

Explanation

The spectacle lens is equal in power and opposite in sign to the neutralizing lens. Other
statements are true.

Score: 23 %
Question 223 Part 3 – Tutor Mode
The numerator in the toric formula:

specifies the prismatic correction

specifies the axis and power of the meridian of maximal power

specifies the power of the spherical surface

specifies the axis and power of the base curve

Your answer was INCORRECT

Explanation

The numerator in the toric formula specifies the power of the spherical surface. The
denominator in the toric formula specifies the power and axis of the cylindrical element.

Score: 23 %
Question 224 Part 3 – Tutor Mode
Which statement is FALSE about the ECG:

the T wave indicates ventricular repolarisation

increased PR interval occurs in first degree heart block


the P wave indicates arterial repolarisation

the QRS complex indicates ventricular depolarisation

Your answer was INCORRECT

Explanation

In the ECG, the P wave indicates arterial depolarisation, while the QRS complex
indicates ventricular depolarisation. The T wave represents ventricular repolarisation. An
increased PR interval indicates first degree heart block.

Score: 23 %
Question 225 Part 3 – Tutor Mode
Which is FALSE regarding the RPE cells:

they are approximately 50 micrometers in height

they are connected by tight junctions

they possess apical microvilli to increase surface area

they contain large numbers of lysosomes and mitochondria

Your answer was CORRECT

Explanation

The RPE cells measure 14 micrometers tall in the centre of the retina and 10 micrometers
in the periphery. Other options are true.

Score: 23 %
Question 226 Part 3 – Tutor Mode
Regarding adrenaline and noradrenaline:

their release is under the control of the pituitary gland

both cause increased peripheral resistance of the blood vessels

both are produced by the adrenal medulla

both cause vasodilatation in the skeletal muscles

Your answer was INCORRECT


Explanation

Both adrenaline and noradrenaline are produced by the adrenal medulla. They are
secreted in response to stress. Adrenaline causes vasodilatation in the skeletal muscles
but noradrenaline has no such effect. While adrenaline decreases the peripheral resistance
of the blood vessels, noradrenaline increases it.

Score: 23 %
Question 227 Part 3 – Tutor Mode
Which of the following extraocular muscles can perform intortion of the globe:

the superior oblique

the medial rectus

the inferior oblique

the inferior rectus

Your answer was CORRECT

Explanation

The primary, secondary and tertiary actions of the recti are respectively:

 Medial rectus: Adduction (no secondary or tertiary actions)


 Superior rectus: Elevation, Intortion, Adduction
 Inferior rectus: Depression, Extorsion, Adduction
 Inferior oblique: Extorsion, Elevation, Abduction
 Superior oblique: Intorsion, Depression, Abduction
 Lateral rectus: Abduction (no secondary or tertiary actions)

Score: 23 %
Question 228 Part 3 – Tutor Mode
Which of the following statements about blood test results is FALSE:

macrocytic anaemia occurs in folate deficiency

hypersegmentation of the neutrophilic nucleus occurs in vitamin B12 deficiency

decreased erythropoietin production as in renal failure causes microcytic anaemia

normochromic normocytic anaemia is seen in chronic disease


microcytic hypochromic anaemia is seen in iron deficiency anaemia

Your answer was INCORRECT

Explanation

Iron deficiency anaemia causes microcytic hypochromic red blood cells. Folate and/or
vitamin B12 deficiency causes macrocytic hypochromic red blood cells. Both folate and
vitamin B12 deficiency are essential for the maturation of DNA so their deficiency can
cause hypersegmentation of the neutrophilic DNA. Chronic disease causes normocytic
normochromic anaemia. Erythropoietin is an essential hormone for red blood cell
production - its absence will cause normochromic normocytic anaemia.

Score: 23 %
Question 229 Part 3 – Tutor Mode
How thick is the choroid:

200-300 micrometers

300-500 micrometers

50-100 micrometers

100-200 micrometers

Your answer was INCORRECT

Explanation

The choroid is 100-200 micrometers thick, being thickest at the posterior pole and thinner
anteriorly.

Score: 23 %
Question 230 Part 3 – Tutor Mode
Which virus is NOT known to be associated with human cancer:

Epstein-Barr Virus

Human Immunodeficiency Virus

Herpes Zoster Virus

Human Papilloma Virus

Your answer was INCORRECT


Explanation

Viruses associated with human cancer:

 Epstein-Barr virus: Burkitt's lymphoma and nasopharyngeal carcinoma


 Human Papilloma virus: cervical cancer
 Hepatitis B and C: hepatic cancer
 Human Immunodeficiency Virus: Kaposi's sarcoma and non-Hodgkin's
lymphoma

Score: 23 %
Question 231 Part 3 – Tutor Mode
All of the following are TRUE of the neuronal action potential EXCEPT:

it is associated with an increased permeability of the membrane to sodium ions

its velocity is affected by the diameter of the nerve fibers

its velocity is faster in nerves that are myelinated than unmyelinated

it is an all or nothing phenomenon

its amplitude decreases as it travels along the nerve fibers

Your answer was INCORRECT

Explanation

The amplitude of an action potential does not change as it travels along the nerve fibers.
All other options are true.

Score: 23 %
Question 232 Part 3 – Tutor Mode
A 30-year-old patient is known to have hypogammaglobulinaemia. Which one of the
following organisms is the patient susceptible to?

Pneumococcus

E coli

Salmonella

Legionella
Bacteroides

Your answer was CORRECT

Explanation

IgG deficiency leads to susceptibility towards infection with encapsulated organisms


(staph, strep pneumoniae, haemophilus) and also pneumocystis carinii infection.

Score: 23 %
Question 233 Part 3 – Tutor Mode
Which of the following is TRUE of the third ventricle:

it is posterior to the pineal body

its roof is formed by the optic chiasm

it contains choroid plexus in its roof

it is connected to the fourth ventricle by the foramen of Monro

Your answer was INCORRECT

Explanation

The third ventricle is connected to the lateral ventricles via the interventricular foramen
of Monro. It is connected to the fourth ventricle by the aqueduct. The optic chiasm is
situated at the junction between the floor and the anterior wall of the third ventricle. The
pineal body is a midline body which is posterior to the third ventricle. The choroid plexus
in the roof of the third ventricle secretes cerebrospinal fluid.

Score: 23 %
Question 234 Part 3 – Tutor Mode
Which is FALSE about spectacle correction of aphakia:

the field of vision is decreased by a lenticular form lens

the ring scotoma is due to high induced prism of the lens periphery

it minimises objects

increasing the back vertex distance of an aphakic correction increases effective power
Your answer was INCORRECT

Explanation

Objects are magnified in aphakic correction. All other statements are correct.

Score: 23 %
Question 235 Part 3 – Tutor Mode
Which of the following drugs is a photosensitizer:

tetracycline

corticosteroids

amiodarone

isoniazid

Your answer was INCORRECT

Explanation

Photosensitising agents absorb visible and UV radiation. Examples include amiodarone,


phenothiazines and psoralens.

Score: 23 %
Question 236 Part 3 – Tutor Mode
Which mutation is MOST likely to cause premature termination of translation:

insertion of a single base into an intron of DNA

deletion of an entire gene

insertion of a single base change in a promoter

insertion of a single base into an exon of DNA

Your answer was INCORRECT

Explanation

A single base insertion in the exon or coding region would cause a frameshift mutation
and usually leads to premature termination of translation. A single base insertion in the
intron (or non-coding region) would be less likely to have this effect. Meanwhile a
mutation within the promoter region would most likely alter gene regulation. Deletion of
the entire gene would lead to complete lack of translation.

237-Which immunoglobulin has NOT been detected in tear samples?

IgA

IgG

IgD

IgE

IgM

Your answer was INCORRECT

Explanation

All major immunoglobulin classes except IgD have been detected in human tears. IgA
(secretory immunoglobulin) is the primary immunoglobulin in tears.

238-All of the following are true of dark adaptation EXCEPT:

rods are more sensitive than cones during dark adaptation

there is a shift in peak spectral sensitivity from 555 nm to 505 nm with dark adaptation

its takes about 12 minutes in humans

biphasic changes only occur in organisms where the retina processes both rods and cones

Your answer was INCORRECT

Explanation

Dark adaptation may take up to 30 minutes. The Perkinje effect describes the shift in
spectral sensitivity from 555nm to 505nm with dark adaptation. Rods are more sensitive
than cones. Organisms with only rods exhibit monophasic change.

239-The sphenoidal sinus drains into the:


the lesser meatus

the middle meatus

the inferior meatus

the superior meatus

Your answer was INCORRECT

Explanation

The nasal meatus are as follows:

 the superior meatus receives the opening of the sphenoid and posterior ethmoidal
sinuses
 the middle meatus receives the openings of the anterior ethmoidal sinuses, frontal
sinus and the maxillary sinus
 the inferior meatus receives the opening of the nasolacrimal duct

240-A sample of 50 men has a mean CD4 count of 1200 cells/mm3 with a standard
deviation of 80 cells/mm3. Which of the following is true?

the variance equals 160 cells/mm3

5% of men will have CD4 counts greater than 1360 cells/mm3

approximately 95% of men will have a CD4 count between 1040 and 1360 cells/mm3

a man with a CD4 count of 1120 cells/mm3 lies 2 standard deviations below the mean

approximately 68% of men will have a CD4 count between 1180 and 1220 cells/mm3

Your answer was INCORRECT

Explanation

This data is normally distributed so approximately 95% of the population will lie within
+-2SD of the mean which is 1200+2 (80) and 1200-2 (80) which is between 1040 and
1360.
241-Which is FALSE regarding Haemophilus influenzae:

it is a common cause of sinusitis

it is always encapsulated

it is a Gram negative coccobacilli

it is best grown on chocolate agar

Your answer was INCORRECT

Explanation

Haemophilus is not always encapsulated with a polysaccharide coat but when it is the
virulence is enhanced as it can avoid phagocytosis.

242-Which statement regarding the levator palpebrae superioris is FALSE:

it inserts into both the skin of the upper lid and the tarsal plate

its muscle belly passes forward below superior rectus

on its inferior aspect is Muller's muscle

it is supplied by the superior division of the oculomotor nerve

the medial horn of its aponeurosis inserts into the frontolacrimal suture

Your answer was INCORRECT

Explanation

The muscle belly of the levator palpebrae superioris passes forward above (not below)
the superior rectus close to the orbital roof. It inserts into both the skin of the upper lid
causing the horizontal palpebral sulcus and also onto the anterior surface of the tarsal
plate. The medial horn of its aponeurosis becomes the medial palpebral ligament and
inserts into the frontolacrimal suture.

243-The ciliary ganglion:


contains sensory nerve fibres

is found between the optic nerve and the medial rectus

is a sympathetic relay ganglion for fibers from the Edinger-Westphal nucleus

has a motor nerve that goes to the inferior oblique

Your answer was CORRECT

Explanation

The ciliary ganglon is found between the optic nerve and the lateral rectus muscle
contains. It contains:

 a parasympathetic ganglion which receives its parasympathetic preganglion fibres


from the Edinger-Westphal nucleus and which supplies the sphincter papillae
 sensory fibres leaving the posterior globe
 sympathetic postganglionic fibres from the superior cervical sympathetic ganglion
which supply the dilator pupillae

244-Which of the following statements regarding diagnostic tests is TRUE?

a high number of false negatives means the test has a low specificity

an imprecise test is of no clinical value

the validity of a test refers to its ability to give the same result when measured by
different observers

sensitivity is also known as the true negative rate

positive predictive value is dependent on the prevalence of the disease

Your answer was INCORRECT

Explanation

The positive predictive value is dependent on the prevalence of the disease, this is the
only true statement in the above question. It means that if a disease is very rare and you
test positive, it is still unlikely that you have it.

The validity of a test, which is also called accuracy, is the ability of the test to give a true
reading of what it is purporting to measure.

Sensitivity is also called the true positive rate. It is the probability that if you have the
disease, you will test positive. A high number of false negatives implies low sensitivity.

Specificity depends upon true negatives. It is the probability that if you do not have the
disease, you will test negative. A high number of false positives implies low specificity.

245-All of the following are involved in the pathway of taste sensation EXCEPT:

nucleus of solitary tract

greater petrosal nerve

chorda tympani

geniculate ganglion

Your answer was INCORRECT

Explanation

Taste sensation from the anterior two-thirds of the tongue passes via the chorda tympani
to the geniculate ganglion, where the cell body of the sensory neurone resides. The fibres
continue via the nervus intermedius of CN VII to the nucleus of the solitary tract.

Score: 0 %
Question 1 Part 3 – Tutor Mode
Which is FALSE regarding sterilization:

it can be achieved with ethylene oxide

it can be achieved at high pressures using an autoclave

it can be achieved with gamma radiation

dry heat is more effective than wet heat

it involves the destruction of all microbial life so that none can replicate
Your answer was INCORRECT

Explanation

Moist heat is more effective than dry heat, which requires higher temperatures and longer
exposure to achieve sterilization.

247-The
major histocompatibility antigen complex in humans (human leukocyte antigen
[HLA] system) is coded for by genes located on chromosome:

18

21

11

13

Your answer was INCORRECT

Explanation

The human leukocyte antigen (HLA) complex governs immune response and surveillance
and is encoded for by genes on chromosome 6.

248-The inferior orbital floor is formed by:

the lacrimal, maxillary and zygoma bones

the maxillary, zygoma and palatine bones

the frontal and maxillary bones

the palatine, sphenoid and maxillary bones

Your answer was INCORRECT

Explanation
The orbital floor is formed by the maxilla, zygoma and palatine bones.

249-Which statement is FALSE regarding prismatic corrections:

an 8 dioptre convex lens decentered 5 mm upwards has a prismatic power equivalent to a


4 prism dioptre base-up prism

a 6 dioptre concave lens decentered laterally by 2.5 mm has a prismatic power equivalent
to a 1.5 prism dioptre base-in prism

a 4 dioptre base-up prism for the right eye is equivalent to a 2 prism dioptre base-up
prism for the right eye with a 2 prism dioptre base-up prism for the left eye

a 3 prism dioptre base-out prism in front of the right eye is equivalent to a 1 dioptre base-
out prism in front of the right eye and 2 prism dioptre base-out prism in front of the left
eye

Your answer was INCORRECT

Explanation

A 4 dioptre base-up prism for the right eye is equivalent to a 2 prism dioptre base-up
prism for the right eye with a 2 prism dioptre base-down prism for the left eye. All other
options are correct.

250-The
following stages are involved in leukocyte movement to a site of inflammation
EXCEPT:

rolling

migration

division

adhesion

Your answer was INCORRECT

Explanation

Movement of leukocytes into the site of inflammation involve the following stages:
rolling, firm adhesion, tight adhesion, migration.
251-Which is FALSE regarding the ciliary body:

the posterior portion is termed the pars plicata

it is 5-6 mm wide

it extends posteriorly to the ora serrata

it extends anteriorly to the scleral spur

Your answer was CORRECT

Explanation

The anterior portion of the ciliary body is termed the pars plicata. It is 2mm wide and
composed of about 70 ciliary processes. The posterior portion of the ciliary body is
approximately 4mm wide and is termed the pars plana.

252-he lateral geniculate nucleus sends the large proportion of its fibres to:

the reticular formation

the inferior colliculus

the superior colliculus

the visual cortex

the hypothalamus

Your answer was INCORRECT

Explanation

The primary role of the lateral geniculate nucleus is a relay station for the conscious
pathway of visual perception. To this end, it sends the majority of its fibres to the visual
cortex, also called area 17. It also has connections to the superior colliculus, reticular
formation and hypothalamus.

253- The prismatic effect of a spherical lens:


is inversely proportional to the power of the lens

can prevent the jack-in-the-box phenomenon

increases toward the periphery of a lens

increases towards the centre of a lens

Your answer was INCORRECT

Explanation

The prismatic effect of a spherical lens increases towards the periphery of a lens. It can
cause intolerance of lenses, particularly high-powered lenses, if the lens is not properly
centred. It explains the jack-in-the-box effect for wearers of high-powered lenses and it is
also is the underlying mechanism for spherical aberration of lenses. The prismatic effect
produced is directly proportional to the spherical power of the lens and to the distance of
rays from the centre of the lens. The prismatic power is calculated by the formula:

P=FxD

Where P is the prismatic power in prism diopters, F is the lens power in diopters and D is
the distance from the centre of the lens in centimeters.

254-Chromosomes replicate during:

G0

interphase

G1

S phase

prophase

Your answer was INCORRECT

Explanation

Chromosomal replication occurs in the S phase as the cell prepares for mitosis.
255-Which extraocular muscle is inserted nearest to the cornea:

lateral rectus

inferior rectus

superior rectus

medial rectus

Your answer was INCORRECT

Explanation

The insertions of the recti muscles from the limbus are as follows:

 medial rectus 5.6mm


 inferior rectus 6.6mm
 lateral rectus 7.0mm
 superior rectus 7.8mm

256-All the statements below are true of the middle meningeal artery EXCEPT:

it does not supply the brain

it supplies the meninges

it lies within the inner layer of dura

it enters the skull through the foramen ovale

Your answer was INCORRECT

Explanation

The middle meningeal artery enters the skull through the foramen spinosum.

257-Which statement is FALSE regarding nerve supply to the cornea:

the nerve fibres are mainly A-delta and C-fibres which transmit pain and temperature
the corneal nerve supply originates from the ophthalmic division of the trigeminal nerve

nerve bundles in the peripheral cornea initially travel below the anterior one-third of the
stroma

nerves lose their perineurium and myelin sheaths 2mm from the limbus

Your answer was INCORRECT

Explanation

Corneal nerves lose their perineurium and myelin sheaths 1mm from the limbus; this
helps to maintain corneal clarity. Other options are true.

258-Which statement is TRUE of astigmatism:

oblique astigmatism indicates the principal meridians are not at 90 degrees to each other

it does not change through life

a stenopaeic slit can be used to assess the axis of astigmatism

when corrected with a minus cylinder at axis 90 degrees, it is called 'with the rule'

Your answer was INCORRECT

Explanation

A minus cylinder at axis 90 degrees represents 'against-the-rule' astigmatism. In oblique


astigmatism the principal meridians are at right angles but they do not lie within 20
degrees of the horizontal or vertical meridians. Astigmatism changes with age: with
increasing age the tendency is for the horizontal curvature of the cornea to become
greater than the vertical curvature

259-Normal stereoacuity is:

60 seconds of arc

300 seconds of arc


10 seconds of arc

150 seconds of arc

Your answer was CORRECT

Explanation

Normal stereoacuity is approximately 60 seconds of arc, though some individuals have


stereoacuity of 15 seconds of arc (the limit of the Frisby test) or better.

260-Which is TRUE regarding lymphatic drainage of the eye and adnexa:

no such drainage exist

drainage occurs directly into the cavernous sinus

only lymphatics from the eyelid drain to the cervical chain

half drains into the cavernous sinus and half into the cervical chain

Your answer was INCORRECT

Explanation

The eyelids have lymphatic drainage laterally to the preauricular and intraparotid lymph
nodes and medially to the submandibular and submental lymph nodes. These all drain
into the cervical chain. There are no lymphatics in the deep orbit.

261-urface ectoderm forms all of the following structures EXCEPT:

eyelid

sclera

cornea

conjunctiva

Your answer was INCORRECT

Explanation
The sclera is derived from the mesoderm. The conjunctiva, cornea and eyelid are all
derived from the surface ectoderm.

262-Hard exudates are found in which layer of the retina:

the inner plexiform layer

the outer plexiform layer

the nerve fibre layer

the internal limiting membrane

Bruch's membrane

Your answer was INCORRECT

Explanation

Hard exudates are caused by underperfusion leading to damage to the endothelium of


deep capillaries. This causes leakage of plasma into the outer plexiform layer.

263-Whih one of the following regarding malignant transformation in tissues is FALSE:

mutations in the p53 gene is the most common mutation seen in solid tumours

cancer usually results from the accumulation of mutations in multiple genes rather than
from single genetic defect

mutations in the p53 gene may make cancer cells resistant to radiotherapy

apoptosis does not occur in cells once they have undergone malignant
transformation

resistance to chemotherapy agents may occur due to mutant genes

Your answer was INCORRECT

Explanation
Cancer cells usually die from apoptosis. p53 mutations are the most common in solid
tumours. Cancer syndromes due to a single genetic defect do occur but are rare.

264-Which of the following is FALSE regarding hard exudates:

they are well circumscribed lesions

they are caused by leakage of plasma

they contain foamy macrophages with lipid in their cytoplasm

they occur in the inner plexiform layer

they are caused by endothelial damage to deep capillaries

Your answer was INCORRECT

Explanation

Hard exudates occur in the outer plexiform layer. Other options are true.

265-Which of the following enters the orbit through the annulus of Zinn:

the lacrimal nerve

the nasociliary nerve

the frontal nerve

the trochlear nerve

Your answer was INCORRECT

Explanation

The frontal, lacrimal and trochlear nerves enter the superior orbital fissure above the
tendinous ring. Structures that enter the orbit through the ring include: the nasociliary
nerve, the abducent nerve, the oculomotor nerve.

266-Which statement is FALSE about Descemet's membrane:


it occurs between the posterior stroma and the endothelium

it is rich in Type II collagen

it is 8-12 micrometers thick

it is continuous posteriorly with the trabecular meshwork

Your answer was INCORRECT

Explanation

Descemet's membrane is rich in basement membrane glycoproteins, laminin, and Type


IV collagen (but not Type II collagen).

267-Cephalosporins work by:

inhibiting protein synthesis

inhibiting DNA synthesis

inhibiting cell wall synthesis

inhibiting folate biosynthesis

Your answer was INCORRECT

Explanation

Cephalosporins prevent bacterial cell wall synthesis

268-Which is TRUE of sterilisation:

it can be achieved by wiping surgical instruments with alcohol swab

moist heat takes longer than dry heat to sterilize instruments

it involves the removal of all microorganisms except spores

it can be achieved by boiling surgical instruments in water for 10 minutes


ethylene oxide gas is used to sterilise instruments which are heat sensitive

Your answer was INCORRECT

Explanation

Sterilisation is the removal of all microorganisms including spores. Disinfection is the


removal of most but not all microorganisms. Sterilisation can be achieved with moist heat
in autoclave which takes shorter period than dry heat. Ethylene oxide gas is useful for
heat sensitive instruments. Irradiation and chemical germicides are other techniques of
sterilisation.

269-The seventh cranial nerve during its course traverses:

the foramen ovale

the stylomastoid foramen

the jugular foramen

the foramen rotundum

Your answer was INCORRECT

Explanation

The seventh cranial nerve traverses the internal auditory meatus of the petrous temporal
bone, the facial canal and emerges from the stylomastoid foramen to enter the parotid
gland.

270-The foramen ovale transmits all of the following EXCEPT:

accessory meningeal artery

maxillary nerve

mandibular nerve

the lesser petrosal nerve

Your answer was INCORRECT


Explanation

The foramen ovale transmits the mandibular nerve, accessory meningeal artery, the lesser
petrosal nerve and the emissary vein. The maxillary nerve is transmitted through the
foramen rotundum.

271-Which drug can cause macular toxicity in aphakic eyes:

epinephrine

acetazolamide

timolol

latanoprost

Your answer was CORRECT

Explanation

Epinephrine causes macular oedema in 20-30% of apakic eyes undergoing treatment for
glaucoma and is therefore contraindicated in these circumstances.

272-The foramen rotundum pierces:

the greater wing of sphenoid

the mandible

the maxilla

the lesser wing of sphenoid

Your answer was CORRECT

Explanation

The foramen rotundum pierces the greater wing of sphenoid.

273-Regarding transforming growth factor-beta (TGF-b):


it is synthesised and secreted only by a specific subset of Natural Killer cells

mutations in the receptor for TGF-b have been associated with hereditary
haemorrhagic telangiectasia

it binds to specific DNA sequences to activate transcription of DNA

it stimulates cell proliferation in normal tissues

it is a single polypeptide chain of 44 amino acids

Your answer was INCORRECT

Explanation

TGF-beta is a member of a family of dimeric polypeptide growth factors that includes


bone morphogenic proteins and activins. Virtually every cell in the body, including
epithelial, endothelial, haematopoietic, neuronal, and connective-tissue cells, produces
TGF-beta and has receptors for it. TGF-beta regulates the proliferation and differentiation
of cells, embryonic development, wound healing, and angiogenesis.

There are three isoforms of TGF-beta: TGF-beta1, TGF-beta2, and TGF-beta3. Each
isoform is encoded by a distinct gene and is expressed in both a tissue-specific and a
developmentally regulated fashion. During development, TGF-beta1 and TGF-beta3 are
expressed early in structures undergoing morphogenesis, and TGF-beta2 is expressed
later in mature and differentiating epithelium. TGF-beta regulates cellular processes by
binding to three high-affinity cell-surface receptors known as types I, II, and III.

Mutations in TGB-beta receptor are associated with hereditary haemorrhagic


telangiectasia.

TGF-beta is a potent inhibitor of cell proliferation in most epithelial, endothelial, and


haematopoietic cells. It arrests the cell cycle in the G1 phase. In cancer cells, mutations in
the TGF-beta pathway have been described that confers resistance to growth inhibition by
TGF-beta, thus allowing uncontrolled cellular proliferation.

274-Which statement regarding Cytomegalovirus is FALSE:

it is a double-stranded DNA virus

it rarely cause retinitis if the patient has a CD4 counts of more than 200 per cubic mm

it is the most common viral infection in HIV patients


in normal people it usually remains dormant in the nerve ganglia

Your answer was INCORRECT

Explanation

Cytomegalovirus is a double-stranded DNA virus. It rarely causes problems for


immunocompetent patients. In HIV patients, cytomegaloviral retinitis usually indicates
decreased CD4+ counts to less than 50 per mm3. Most cases of cytomegalovirus
infection in the immunocompromised are the result of reactivation. The precise site
where these viruses remain latent following initial infection of the immunocompetent
patient is not yet known.

275-Choose the incorrect statement. Tumour suppressor genes:

include the gene encoding the p53 protein

normally function to inhibit the cell cycle

were originally derived from retroviruses

act recessively at the cellular level

are involved in the development of neurofibromatosis

Your answer was INCORRECT

Explanation

Tumour suppressor genes are involved in the control of cell growth and differentiation.
Both genes need to be defective (two hits) for loss of regulation and tumorigenesis to
occur. Most of the genes responsible for the dominantly inherited cancer syndromes are
tumour suppressor genes. For example, in type I neurofibromatosis, loss of both NF1
genes (one by inheritance and the other by somatic mutation) results in
neurofibromatosis. The normal function of NF1 is as a GTPase-activating protein that
converts ras from its active, growth-promoting, GTP-bound form to the inactive GDP-
bound form. Other examples include: BRCA1 and 2 in familial breast/ovarian cancer,
FPC, familial retinoblastoma, familial melanoma, hereditary multiple exostoses, tuberous
sclerosis, Von Hippel-Lindau syndrome.

276-Which statement is FALSE about extraocular muscle:


type C fibres are required for tonic contractions

type A fibres are fast twitch and required for saccadic movements

extraocular muscle contains large numbers of specialized sensory and proprioceptive


nerve endings such as muscle spindles and Golgi tendon organs

type B fibres are slow twitch and needed for smooth pursuit

extraocular muscle is less vascular than skeletal muscle

Your answer was INCORRECT

Explanation

Extraocular muscle is more vascular than skeletal muscle. In fact, it is the most vascular
muscle in the body after the myocardium.

277-All of the following definitions are correct EXCEPT:

hypoplasia is failure of formation of an embryonic mass

aplasia is failure of differentiation of an embryonic cell mass to organ specific tissue

metaplasia is differentiation of a mature tissue into a different mature tissue

dysgenesis is failure of organization of tissue into an anatomically correct organ

Your answer was CORRECT

Explanation

Hypoplasia is failure of a developing organ to reach its full size. Agenesis is failure of
formation of an embryonic mass. All other definitions are correct.

278-All are true of adenovirus EXCEPT:

it can be spread by droplet infection

it contains double-stranded DNA


it contains an icosahedral capsid

serotypes 3,7,8 and 19 are responsible for pharnygoconjunctival fever

it is a common cause of upper respiratory tract symptoms

Your answer was INCORRECT

Explanation

Adenovirus has two ocular disease presentations: pharyngoconjunctival fever (in which
upper respiratory tract symptoms precede ocular features) and keratoconjunctivitis (in
which ocular involvement predominates). Serotypes 3, 7, 8 and 19 are responsible for
epidemic keratoconjunctivitis while serotypes 1, 2, 3, 5, 7 and 14 cause
pharyngoconjunctival fever.

279-The sixth cranial nerve on its path to the orbit passes through:

the pontine cistern

the cerebral aqueduct

the magna cistern

the spinal cistern

Your answer was CORRECT

Explanation

The sixth cranial nerve after exiting the brain stem near to the pons-medullary junction
ascends the pontine cistern. It passes forward through the cavernous sinus before entering
the orbit via the superior orbital fissure.

280-Which statement is FALSE regarding corneal shape:

it is more curved in the centre than in the periphery

the cornea is thicker in the periphery than in the centre


average central corneal thickness is approximately 570 micrometers

the corneal diameter is smaller horizontally than vertically

Your answer was INCORRECT

Explanation

The cornea is smaller in the vertical diameter (10.6mm) than in the horizontal diameter
(11.7mm). Other options are true.

281-In fluorescein angiography, pseudoautoflourescence occurs due to:

overlap in the spectral transmission of the excitation and barrier filters

use of a large bolus of fluorescein

ocular tissues which emit baseline fluorescence

overexposure of the angiography film

Your answer was CORRECT

Explanation

Fluorescein sodium is an orange dye which is excited by blue light of wavelength 465-
490nm. It then emits yellow-green light (520-530nm). For this reason, an angiography
camera has a blue excitation filter to illuminate the fundus with blue light for maximal
fluorescein absorbance. The camera then has a yellow-green barrier light which blocks
blue light and exposes the camera only to the yellow-green light from the fluorescein.

Pseudoautoflourescence occurs due to overlap in the spectral transmission of the


excitation and barrier filters, meaning that some blue reflected light does pass through the
barrier and appear as fluorescence.

282-The following are typical of Sjogren's syndrome EXCEPT:

increase in the number of conjunctival goblet cells

a lymphocytic cellular infiltrate

associated connective tissue disorder


involvement of the lacrimal gland and the oral mucosa

Your answer was CORRECT

Explanation

In Sjogren's syndrome there is a lymphocytic cellular infiltrate affecting the acinar glands
of the conjunctiva, oral mucosa and lacrimal gland. There is resulting dry eye and dry
mouth. It is commonly associated with other connective tissue autoimmune diseases such
as rheumatoid arthritis. The number of conjunctival goblet cells is reduces.

283-The superior sagittal sinus drains posteriorly into:

the straight sinus

the sigmoid sinus

the cavernous sinus

the transverse sinus

Your answer was INCORRECT

Explanation

The superior sagittal sinus drains posteriorly into the transverse sinus, which in turn joins
the sigmoid sinus.

284-Which is FALSE regarding central retinal vein occlusion:

it is predisposed by the physiological narrowing of the retinal vein in the lamina cribrosa

hypertension and diabetes are risk factors

on rare occasions the central retinal vein may recanalise

it is characterized by a hemorrhagic retinopathy

rubeotic glaucoma is common


Your answer was INCORRECT

Explanation

Examination of histological specimens reveals recanalization of the central retinal vein in


most cases. Other options are true.

285-Macrophages:

are involved in type III hypersensitivity response

are derived from blood monocytes

cannot produce cytokines

have shorter life span than neutrophils

Your answer was INCORRECT

Explanation

Macrophages are derived from blood monocytes. They have a longer life span than
neutrophils. They produce tumor necrosis factors and interleukins. They are involved in
type IV hypersensitivity response.

286-Which is FALSE regarding the refractive power of a curved surface:

it is proportional to the radius of curvature of the surface

it is proportional to the difference in refractive indices of the media

it is measured in diopters

it is determined by and obeys the principles of Snell 's law

Your answer was CORRECT

Explanation

The refractive power of a curved surface is determined by the principles of Snell's law. It
is measured in diopters and is equal to the difference in refractive indices of the media
divided by the radius of curvature. As such, the power is directly proportional to the
difference in refractive indices but indirectly proportional to the radius of curvature of the
surface

287-Which of the following statements regarding medical statistics is TRUE?

an odds value of 25% corresponds to a 1 in 4 risk of getting disease

a cohort study is an example of an interventional study

incidence is independent of the time period under study

the increase in female mortality from bronchial carcinoma is an example of a


secular trend

a case-control study is prospective

Your answer was INCORRECT

Explanation

A secular trend is a slow growing trend over time, as opposed to an epidemic or seasonal
trend. Incidence refers to the number of cases over a defined period of time. Odds value
is the ratio of the odds of getting a disease in one group to the other, and does not equate
to absolute risk.

288-Typically,
the antibody class produced to an antigen that has been previously exposed
to the immune system is:

IgM

IgE

IgG

IgD

IgA

Your answer was INCORRECT

Explanation
IgG is the major antibody formed following exposure to an antigen that has previously
been encountered.

289-Which bone is NOT a constituent of the orbital floor:

sphenoid bone

palatine bone

maxilla

zygoma

Your answer was CORRECT

Explanation

The orbital floor is composed of: the orbital plate of the maxilla, the orbital surface of the
zygoma and the orbital process of the palatine bone.

290-Which layer overlies the Bowman's layer:

the lamina fusca

the lamina densa

the lamina vitrea

the lamina lucida

Your answer was INCORRECT

Explanation

Overlying the Bowman's layer is the basal lamina of the corneal epithelium, which is
itself composed of two layers:

 the lamina lucida: which is more superficial


 the lamina densa: which is deeper

291-Which statement is TRUE about the nucleic acids:


histones mark the initiation site for transcription

introns are more common than exons on the DNA

in RNA thymine is replaced by uracil

they contain purine and pyrimidine which are bound together by hydrogen

Your answer was INCORRECT

Explanation

Purine and pyrimidine are bound by covalent bounds. Introns are the region of DNA
generating the precursor part of RNA which is excised during transcription and therefore
not a part of messenger RNA. The exon, by contrast, generates the RNA used in the
mRNA, which is transcribed into protein (remember: eXons are eXpressed). Histone is a
protein which is found in association with DNA in chromosomes and performs a
structural role.

292-How many cell layers are there in the corneal epithelium:

10-12

5-6

Your answer was INCORRECT

Explanation

There are five or six layers of cells comprising the stratified non-keratinised squamous
epithelium of the cornea.

293-All of the following pass through the common tendinous ring EXCEPT:

inferior division of oculomotor nerve


nasociliary nerve

frontal nerve

superior division of oculomotor nerve

Your answer was INCORRECT

Explanation

The frontal nerve passes superior to the common tendinous ring. Structures passing
within the ring include: the oculomotor nerve (superior and inferior divisions), aduncent
nerve, nasociliary nerve, sympathetic root of the ciliary ganglion and on occasion the
inferior ophthalmic vein.

294-The main glycosaminoglycan of the cornea is:

hyaluronate

chondroitin sulphate

keratan sulphate

dermatan sulphate

Your answer was INCORRECT

Explanation

The cornea comprises glycosaminoglycans which are important in maintaining regular


spacial arrangements of collagen fibrils. The main glycosaminoglycans of the corneal
stroma are keratan sulphate and dermatan sulphate. The corneal stroma is unusual in that
it contains no hyaluronate

295-Which statement is FALSE regarding the inferior oblique muscle:

it is inserted about 17mm behind the limbus

its posterior edge inserts behind the fovea

it arises from the posterior lacrimal crest of the infero-nasal orbital rim margin
its primary action is extortion

Your answer was INCORRECT

Explanation

The inferior rectus normally inserts 1 mm anterior and below the fovea. Other options are
true.

296-All are true of the Maddox rod EXCEPT:

it produces a straight line in the plane of the long axis of the lenses

it is used to determine the degree of latent deviation for distance

it is useful in the detection of macular disease

it is composed of cylindrical lenses

Your answer was CORRECT

Explanation

A Maddox rod forms an image perpendicular to the plane of the long axis of the lenses.
All other statements are true.

297-Which sinus lies below the orbital floor:

sphenoidal sinus

frontal sinus

maxillary sinus

ethmoidal sinus

Your answer was INCORRECT

Explanation
The maxillary sinus lies below the orbital floor. Tumour and infection can spread from
the maxillary sinus to the orbit through the floor.

298-Which of the following is NOT an example of apoptosis?

the killing of virus infected cells by cytotoxic T-cells

the formation of synapses in the brain

coagulative necrosis of myocytes following myocardial infarction

the sloughing off of the endometrium during menstruation

the formation of fingers and toes during embryogenesis

Your answer was INCORRECT

Explanation

Apoptosis is programmed cell death and does not trigger inflammation. It can be induced
by:

 Signals arising within the cell


 Death activators binding to the cell surface
 Reactive oxygen species

299-Which statement is false regarding natural killer (NK) cells:

they interact with MHC class I molecules

they have a role in recognizing and killing tumor cells

they secrete gamma interferon

they are a subset of T-lymphocytes

they lack the T-cell receptor

Your answer was INCORRECT


Explanation

NK cells are a subset of lymphocytes derived from bone marrow progenitors. They
function in cellular cytotoxicity and are not phagocytic. NK cell killing is especially
important in the control of the early phases of infection with intracellular pathogens such
as herpes viruses, Leishmania and Listeria monocytogenes. They also play a role in the
destruction of tumor cells. NK cells do not express T-cell receptors. They express a
number of non-specific adhesion molecules, which mediate the attachment to target cells.

Human NK cells express membrane proteins that recognize specific allelic forms of
MHC class I molecules bound to self-peptide. On binding their ligand, these receptors
inhibit NK cell killing. If the concentration of class I molecules on a cell is reduced, or
the range of peptides is markedly altered (as can occur in viral infection), NK cells are no
longer inhibited and cytolysis occurs. These cells release granules containing perforin and
granzymes, which lye target cells and then induce apoptotic cell death. NK cells also
express receptors for the Fc region of IgG and are important mediators of antibody-
dependent cellular cytotoxicity. The killing activity of NK cells is enhanced by IFN-
gamma, IFN-beta, and IL-12. They also secrete IL-1 and GM-CSF.

300-The nerve supply to the levator palpebrae superioris is derived from:

the supraorbital nerve

the supratrochlear nerve

the nerve to the medial rectus

the nerve to the superior oblique

the nerve to the superior rectus

Your answer was INCORRECT

Explanation

The nerve to the superior rectus supplies the levator palperae superioris which it pierces
from below.

301-The pretectal nucleus is involved in:

conscious vision

the pupillary light reflex


the arousal function

photoperiod regulation

the visual grasp reflex

Your answer was INCORRECT

Explanation

After the decussation of fibres in the optic chiasm, 90% of fibres pass along the lateral
root of the optic tract towards the lateral geniculate nucleus and on to the occipital cortex
for conscious vision. 10% of fibres travel along the medial root of the optic tract and are
responsible for unconscious stimuli. These fibres target:

 the superior colliculus: involved in visual grasp reflex, automatic scanning of


images

 the pretectal nucleus: involved in pupillary light reflex

 the parvocellular reticular formation: arousal function

 the retinohypothalamic tract : photoperiod regulation

302-All of the following substances are used in Gram staining EXCEPT:

iodine

Congo red

alcohol

safranin

Crystal violet

Your answer was INCORRECT

Explanation

Congo red is not used in Gram staining. The Gram staining process is as follows:
 Crystal violet (which is purple in colour) is applied and this attaches to negatively
charged groups in the cell wall, membrane and cytoplasm.
 The further addition of iodine strengthens this attachment.
 The addition of alcohol or acetone-alcohol decolorizes the crystal violet and
iodine causing the stain to diffuse out of the cell. This diffusion is lower in Gram-
positive cell walls because of the greater thickness and chemical composition
(particularly teichoic acids).
 Safranin or neutral red is then used as a counter-stain. This binds to free
negatively charged groups that are not already binding to crystal violet.

303-Keloid scars:

tend not to extend beyond the original incisional scar

are rare in the deltoid region

are caused by excess deposition of fibrin in the wound

are distinguished from hypertrophic scars by their extent.

Your answer was INCORRECT

Explanation

Keloid scars unlike hypertrophic scars extend beyond the original wound. They are
caused by excessive deposition of collagen. They are most common in the sternum and
deltoid regions.
304-Choose which statement is FALSE regarding trinucleotide DNA repeats:

they demonstrate genetic imprinting

they are the molecular basis for genetic anticipation

they are the cause of fragile X syndrome

they are the cause of Huntington's disease

they can produce a neurotoxic polyglutamine peptide sequence

Your answer was CORRECT


Explanation

Trinucleotide repeats occur to a variable degree in the coding and non-coding areas of
genes in the normal population (exhibit polymorphism). When the number of repeats
exceeds 30, the DNA becomes unstable and may expand up to several thousand copies
and interfere with gene expression leading to disease.

The number of trinucleotide repeats increases with successive generations leading to


more severe disease; this is called genetic anticipation. Genetic imprinting is the
phenomenon of differential gene expression depending on the sex of the parent who
transmitted it.

Diseases related to trinucleotide repeat expansion include: myotonic dystrophy,


spinocerebellar ataxia, Friedrich's ataxia and X-linked bulbospinal neuropathy
(Kennedy's syndrome

305-The ventilation to perfusion ratio is maximum at:

equal in both lungs

the apex of the lung

the middle lobe

the medial lobe

the base of the lung

Your answer was INCORRECT

Explanation

The apex of the lung is maximally ventilated and perfusion in this region is less; hence,
ratio of ventilation to perfusion is maximum in this part.

306-Which is the most common cause of human blindness globally:

Trachoma

Diabetic retinopathy

Glaucoma
Age-Related macular degeneration

Your answer was CORRECT

Explanation

Trachoma is the commonest cause of blindness globally.

307-Dysplasia:

is associated with increased cell staining with haematoxylin

involves the transformation of one mature cell type into another

is associated with a reduced cell number

is irreversible

Your answer was CORRECT

Explanation

Dysplasia is associated with increased cell number and nuclear abnormalities such as
hyperchromasia (increased cell staining with haematoxylin) and pleomorphism (altered
nuclear size and nuclear shape). Dysplasia may be caused by diverse cellular insults
including physical, chemical and viral insults. It may be reversible in its early stages.
Metaplasia is the transformation of one mature cell type into another.

308-All are true of the corrugator supercilii EXCEPT:

it is supplied by divisions of the temporal branch of the facial nerve

it draws the eyebrow upwards and medially

it assists in protecting the eyes in bright light

it produces vertical skin furrows on the forehead

Your answer was INCORRECT

Explanation
The corrugator supercilii draws the eyebrow downwards and medially to produce vertical
skin furrows on the forehead. Other options are true.

309-A glass prism of refracting angle 30 degrees is placed in the position of minimum
deviation. How much deviation will it produce:

60 degrees

30 degrees

45 degrees

15 degrees

Your answer was INCORRECT

Explanation

For a prism of refractive index n and refractive angle a, in the position of minimum
deviation:

angle of deviation = (n-1) x a

Therefore, for a glass prism: angle of deviation = (1.5-1) x a = a/2

310-The meridian of minimum curvature of a toric lens is called:

the spherical equivalent

principle axis

the optical cylinder

the base curve

Your answer was INCORRECT

Explanation

The principle meridian of minimum curvature of a toric lens is called the base curve.
311-Which statement about lignocaine is FALSE:

it inhibits the uptake of potassium into nerve cell membranes

it has a local vasodilatory effect

toxicity causes drowsiness and slurred speech

it is alkaline in solution

Your answer was CORRECT

Explanation

Lignocaine blocks the uptake of sodium into nerves (not potassium). Lignocaine toxicity
causes tingling around the mouth, followed by drowsiness and slurred speech which leads
to convulsions and then coma.

312-A point of light viewed through a vertical Maddox rod will produce:

a line perpendicular to the long axis of the rods

a line oriented in the direction of corneal astigmatism

a line perpendicular to the direction of corneal astigmatism

a line parallel to the long axis of the rods

Your answer was CORRECT

Explanation

A point of light viewed through a Maddox rod will produce a straight line perpendicular
to the long axis of the rods.

313-Which statement is FALSE about the parotid gland:

it receives its secretomotor fibres from the glossopharyngeal nerve

it is a serous gland
the parotid duct enter the oral cavity by piercing the buccinator

it is covered by a thick capsule derived from the deep cervical fascia

the facial nerve exits the gland anteromedially before dividing into 5 branches

Your answer was INCORRECT

Explanation

The facial nerve divides within the parotid gland before exiting it anteromedially. All
other options are true.

314-Which statement is FALSE regarding the tarsal plates:

they are 1mm thick

they measure 25-30mm from medial to lateral borders

they consist of dense fibrous connective tissue

the lower tarsal plate is greater in height than the upper tarsal plate

Your answer was INCORRECT

Explanation

The upper tarsal plate (10-12mm) is greater in height than the lower lid tarsal plate
(5mm). Other options are true.

315-Which is FALSE regarding Gullstrand's schematic eye:

it is a model of the human eye based on thick lens formulae

the axial length is 24.4 mm

the nodal points lie on either side of the cornea

the cornea is 0.5 mm thick


the refractive index of the cornea is less than that of the lens

Your answer was INCORRECT

Explanation

The nodal points lie either side of the posterior pole of the lens.

316-The choroid is composed of:

5 layers

3 layers

2 layers

4 layers

Your answer was CORRECT

Explanation

The choroid is composed of 5 layers:

 Bruch's membrane
 the choriocapillaris
 Haller's layer
 Sattler's layer
 the suprachoroid

317-Corneal endothelial density at birth is:

3-4000 cells per square millimeter

10,000 cells per square millimeter

6000 cells per squre millimeter

2500 cells per square millimeter

Your answer was CORRECT


Explanation

Corneal endothelial cell density at birth is 3-4000 cells per square millimeter. This falls to
2500cells per square millimeter in meddle age and 2000 cells per square millimeter in old
age. Density below 800 cells per square millimeter leads rapidly to oedema.

318-Which of the following is a recognized side-effect of systemic chloramphenicol


administration:

aplastic anaemia

thrombocytopaenia

haemolytic anaemia

idiopathic thrombocytopaenic purpura

Your answer was CORRECT

Explanation

Chloramphenicol is one of the commonest topical antibiotics in use in ophthalmology but


its systemic use is limited by its potential to cause a fatal aplastic anaemia.

319-All the following are true about the pre-aponeurotic fat EXCEPT:

it is located between the orbital septum and the levator

the lateral pad is larger than the medial pad

the medial pad has more fibrous tissue than the lateral pad

the medial pad has a yellowish appearance whereas the lateral pad is white

Your answer was INCORRECT

Explanation

The pre-aponeurotic fat pad is located between the orbital septum and the levator. It is
divided into medial and lateral portions by the trochlear. The lateral pad is larger and
more yellowish in colour whereas the medial pad is whitish. The difference is due to the
higher content of fibrous tissue in the whitish medial pad.
320-Bleeding time is increased in ALL of the following EXCEPT:

thrombocytopaenia

vitamin C deficiency

anaemia

aspirin ingestion

Your answer was INCORRECT

Explanation

Anaemia does not increase bleeding time.

Increased bleeding time occurs in:

 Thrombocytopaenia
 Disseminated Intravascular Coagulation (DIC)
 Platelet disorders
 Capillary wall abnormalities (such as in vitamin C deficiency)
 Von Willebrand's Disease
 Medications: including Aspirin, Warfarin, NSAIDs, Streptokinase, Urokinase

321-Damage to the oculomotor nerve would be expected to cause:

a dilated pupil unresponsive to light and accommodation

a dilated pupil unresponsive to light but responsive to accommodation

a constricted pupil unresponsive to light but responsive to accommodation

a constricted pupil unresponsive to light and accommodation

Your answer was CORRECT

Explanation
Damage to the parasympathetic fibres supplying the sphincter pupilae results in a dilated
pupil that does not respond to light or accommodation.

322-All are true of the lens capsule EXCEPT:

it is produced by the lens epithelium and lens fibres

it is approximately 50 micrometers thick at the anterior pole

it completely envelops the lens

it contains basement membrane glycoproteins

Your answer was INCORRECT

Explanation

The lens capsule varies in thickness being thinnest at the poles, measuring about 2
micrometers at the posterior and 15 micrometers at the anterior pole. Other options are
true.

323-The image formed by a thin concave lens is:

virtual, erect, magnified, further from the lens than object

virtual, erect, at infinity

real, inverted, outside F2

virtual, erect, diminished, inside F2

Your answer was INCORRECT

Explanation

The image formed by a thin concave lens is virtual, erect, diminished and inside F2. We
recommend being familiar with drawing ray diagrams by producing, from the top of any
object, 2 lines (a) one passing through the nodal point undeviated, (b) the other parallel to
the principle action passing through the second principle focus.

324-A Maddox rod is composed of:


concave cylindrical lenses

concave spherical lenses

convex cylindrical lenses

convex spherical lenses

Your answer was INCORRECT

Explanation

A Maddox rod is composed of strong convex cylindrical lenses oriented parallel to each
other mounted side by side.

325-Which statement about the lacrimal gland is FALSE:

it receives secretomotor innervation from the Edinger-Westphal nucleus

it receives sensory supply from the trigeminal nerve

it receives its blood supply principally from a branch of the ophthalmic artery

it is similar histologically to the salivary glands

Your answer was CORRECT

Explanation

The lacrimal gland receives its blood supply from the lacrimal artery, a branch of the
ophthalmic artery. Its sensory innervation is derived from the lacrimal nerve, an early
branch of the ophthalmic nerve (V1). Its parasympathetic innervation travels from the
superior salivatory nucleus along the seventh nerve, synapsing in the pterygopalatine
ganglion, before continuing to the lacrimal gland via the zygomatic nerves, branches of
the maxillary nerve (V2).

326-Which chromosome or gene defect is associated with retinoblastoma:

PAX 6
HOX

chromosome 11

chromosome 18

chromosome 13

Your answer was INCORRECT

Explanation

Retinoblastoma is caused by a defect in the Rb gene located on the long arm of


chromosome 13.

327-Which statement is FALSE regarding aminoglycosides:

they require aerobic transport mechanisms to enter the bacterial wall

they act on messenger RNA

they are effective against streptococci

they are not as effective against systemic infection if given orally

Your answer was INCORRECT

Explanation

Aminoglycosides require aerobic transport mechanisms to enter the bacterial cell and are
therefore not effective against Streptococcus and strict anaerobes which do not have
oxidative transport mechanisms. All other statements are true.

328-Regarding oedema all are true EXCEPT:

it is caused by increased escape of fluid from capillaries and venules

in oedema secondary to cardiac failure, the permeability of the vessel walls is unchanged

transudate contains more protein than exudates


the tissue pressure is important in controlling the amount of oedema

increased venous capillary hydrostatic pressure increases oedema

Your answer was INCORRECT

Explanation

All statements are true except that a transudate contains more protein than an exudate.

329-The sixth cranial nerve:

arises from a nucleus closely related to the facial nerve

passes through the medial wall of the cavernous sinus

is involved in the light reflex

innervates the lacrimal gland

Your answer was CORRECT

Explanation

The sixth cranial nerve nucleus is found medial to the facial nerve nucleus. It is a pure
motor nerve and supplies only the lateral rectus muscle. It enters the orbit by passing
through the lateral wall of the cavernous sinus. It is not involved in the light reflex

330-The trigeminal nerve is responsible for all of the following EXCEPT:

proprioception to the muscles of facial expression

tactile sensation to the external ear pinna

motor innervation of the muscles of mastication

nociceptive sensation to the anterior scalp

Your answer was INCORRECT

Explanation
The skin of the pinna is supplied by branches of the cervical nerves.

The trigeminal nerve is responsible for:

 tactile, nociceptive and thermal sensation to the face, and up to the vortex of the
scalp
 motor innervation of the muscles of mastication
 proprioceptive sensation from the muscles of mastication, muscles of facial
expression and the extraocular muscles

331-The hyaloid artery:

forms part of the vascular propria lentis

Bergmeister's papillae is the anterior remnant

regresses after birth

communicates freely with the choroidal circulation throughout development

Your answer was CORRECT

Explanation

The hyaloid artery supplies the lens and the inner layer of the optic cup. It forms the
vascular propria lentis which supplies the lens. It regresses before birth. Mitterdorf's dot
is the anterior remnant of the hyaloid artery whereas the Bergmeister's papillae is the
posterior remnant. The choroidal circulation arises from the long and short posterior
ciliary arteries and does not communicate with the hyaloid artery.

332-Which is TRUE of the optic nerve:

it contains more fibres in adult than in foetal life

it has its longest course within the cranium

it enters the middle cranial fossa lateral to the internal carotid artery

it has the ciliary ganglion on its lateral side

Your answer was INCORRECT


Explanation

The optic nerve enters the middle cranial fossa medial to the internal carotid artery. Its
longest course is within the orbit. It has more fibres in foetal life.

333-Which is the most common type of collagen in the sclera?

Type 5

Type 7

Type 1

Type 2

Type 3

Your answer was INCORRECT

Explanation

The sclera is composed mainly of Type 1 collagen.

334-Which statement regarding amyloid is FALSE:

it appears as extracellular eosinophilic hyaline material

it is a complication of chronic infection

it shows an apple green birefringence in polarised light

it occurs in patients with multiple myeloma

it is made up of calcified protein

Your answer was INCORRECT

Explanation

Amyloid is a fibrillar glycoprotein (not a calcified protein) which appears as extracellular


eosinophilic hyaline material. It shows an apple green birefringence in polarised light.
Amyloidosis may occur in multiple myeloma, in which case the amyloid is made up of
the light chain of immunoglobulin. Secondary amyloidosis occurs in chronic infection.

335-Ina sample of 100 patients on a cardiac rehabilitation programme, maximum exercise


tolerances on treadmill testing were normally distributed with a mean of 8 minutes and a
standard deviation of 2 minutes.

Which of the following conclusions is valid?

nearly all patients will have exercised for 8 minutes or more

the coefficient of variation equals 2.5%

around 5% of patients will have exercised for less than 4 minutes

the coefficient of variation equals 25%

all will have exercised for less than 14 minutes

Your answer was INCORRECT

Explanation

Variance is a measure of the spread of observations around the mean; and it equals to the
standard deviation squared which would be 4 minutes. The co-efficient of variation is the
standard deviation divided by the mean, which in this case is 2/8 or 25%.

336-In secondary lymphoid organs:

T-cells form primary follicles

B-cells in primary follicles express IgD and IgM on their surface

T-cells and B-cells form follicles in the red pulp of the spleen

CD8 T-cell responses are important for optimal activation of B-cells

most recirculating lymphocytes are memory B-cells

Your answer was INCORRECT


Explanation

Peripheral (secondary) lymphoid organs include spleen, lymph nodes, mucosal asociated
lymphoid tissue, the cutaneous immune system and liver. Lymph nodes are widely
distributed and encounter pathogens entering the body from all possible sites. In
lymphoid organs, T-cells and B-cells are segregated with B-cells aggregating to form
follicles. Primary follicles are formed by non-activated B-cells which have not undergone
class switching, i.e., they express IgM and IgD rather than IgG, E or A. CD8 T-cells are
cytotoxic T-cells. T-helper cells are important for optimal activation of B-cells in
secondary lymphoid organs.

The spleen is specialised to trap and mount an immune response against blood borne
antigens. It is comprised of the red pulp which contains an abundance of erythrocytes.
The white pulp forms a sleeve around penetrating arterioles and contains numerous T-
cells. An area called the marginal zone is closely associated with this periarteriolar
lymphoid sheath (PALS) and is rich in B-cell follicles. The marginal zone and PALS are
populated by interdigitating dendritic cells - specialised in the presentation of antigen to
T-cells. Macrophages located in the marginal zones of the spleen are very efficient at
trapping and retaining polysaccharide antigens. These are then presented to B-cells and
an appropriate immune response is initiated.

Lymph nodes are the first organised lymphoid structure to encounter antigens that enter
the tissues. Lymph nodes are divided into the cortex, the paracortex, and the medulla. The
outermost cortex contains lymphocytes (mostly B-cells), macrophages and follicular
dendritic cells arranged in primary follicles. Beneath the cortex is the paracortex, which
has large numbers of T-lymphocytes and interdigitating dendritic cells which have
migrated from peripheral tissue to the lymph node after endocytosis of antigenic material.
The medulla is the innermost layer of the lymph node, and consists of lymphocytes and
many antibody secreting plasma cells.

Lymphocytes recirculate throughout the body. Naive T-cells recirculate between blood
and lymphoid organs. They do not enter tissues due to a lack of expression of the
necessary adhesion molecules. Memory T-cells follow a different route. They also
recirculate between blood and lymphoid organs but enter peripheral tissues as well,
reflecting the possession of appropriate homing receptors. Therefore, memory T-cells can
encounter antigens in sites other than lymphoid organs. Memory B-cells recirculate
between blood and lymphoid organs, and retain the ability to migrate through germinal
centers. However, the majority of recirculating lymphocytes in secondary lymphoid
organs are T-lymphocytes.

337-Regarding the p53 protein, which one of the following statements is FALSE:

it suppresses cell division

it is a transcription factor
it is a regulator of apoptosis

a mutation of p53 is the primary defect in the Li Fraumeni syndrome

it is found only in cells that have undergone malignant transformation

Your answer was INCORRECT

Explanation

The normal function of p53 is to down-regulate the cell cycle by preventing the cell from
entering mitosis. p53 is known as the guardian of the genome. When there is DNA
damage, it influences transcription to facilitate DNA repair, or if the damage is too great,
initiates apoptosis. It is commonly mutated in many tumours (50%) and in the rare
autosomally inherited Li Fraumeni syndrome. In this syndrome, individuals inherit a
single mutated copy and are predisposed to a spectrum of chilhood and adult tumours-
sarcomas, breast tumours, brain tumours and leukaemia.

338-Cerebrospinal fluid is produced by the choroid plexus which is located in the:

I. Lateral ventricles
II. Third ventricle
III. Fourth ventricle

II and III

I only

I and II

I, II and III

Your answer was INCORRECT

Explanation

The CSF is produced by the choroid plexus of the lateral, third and fourth ventricles of
the brain.

339-What is the most common site for an ocular malignant melanoma:


ciliary body

iris

choroid

conjunctiva

Your answer was INCORRECT

Explanation

Malignant melanoma occurs most frequently in the choroid, where there is the largest
volume of pigmented tissue in the eye, followed by the ciliary body and iris respectively.

340-Which statement about the subarachnoid space is FALSE:

it is located between the pia mater and the arachnoid mater

it contains the cerebrospinal fluid

it contains the meningeal vessels

it extends to the lower border of the second sacral vertebra

Your answer was INCORRECT

Explanation

The subarachnoid space is located between the arachnoid mater and the pia mater. It
contains cerebral vessels and cerebrospinal fluid. The meningeal vessels are extradural.

341-Which structure is of ectodermal origin:

the corneal stroma

the ciliary muscle

iris stroma

the sclera
Your answer was CORRECT

Explanation

The following ocular structures are of ectodermal origin:

 retina
 retinal pigment epithelium
 epithelium of the ciliary body

Mesenchymal tissues include:

 the sclera
 the iris stroma
 the ciliary body

342-Which statement regarding the chorda tympani is FALSE:

it contains secretomotor nerves that supply the salivary glands

it contains parasympathetic nerves that supply the lacrimal nerve

it contains sensory neurons that supply the anterior two-third of the tongue

it passes between the internal jugular vein and the internal carotid artery as it passes
anteriorly

Your answer was INCORRECT

Explanation

The chorda tympani is a branch of the nervus intermedius which in turn is a sensory
component of the facial nerve. It contains: the parasympathetic nerve fibres that supply
the salivary glands and the sensory nerve fibres that innervate the anterior two-third of
the tongue. Anatomically, it passes between the internal jugular vein and internal carotid
artery as it travels anteriorly. The greater petrosal nerve that contains parasympathetic
nerve fibres to the lacrimal nerve exits the nervus intermedius at the geniculate body and
therefore is not part of the chorda tympani.

343-Which of the following is FALSE regarding the aminoglycosides:

they are useful against streptococci


drug monitoring is recommended

they inhibit protein synthesis

they can cause ototoxicity

Your answer was CORRECT

Explanation

Aminoglycosides such as gentamycin inhibit protein synthesis by interfering with the


reading of codons from messenger RNA. They require aerobic transport mechanisms to
enter cells and are therefore ineffective against anaerobes and streptococci, which do not
exhibit oxidative transport mechanisms. They can cause ototoxicity and nephrotoxicity
necessitating drug dose monitoring.

344-All are features of thyroid eye disease EXCEPT:

fibrosis between muscle fibres in chronic disease

perivascular lymphocytic infiltration of extraocular muscles

swelling of extraocular muscles on orbital imaging

raised T3 and T4

raised TSH

Your answer was INCORRECT

Explanation

Thyroid eye disease occurs in Grave's disease where the circulating T3 and T4 are
markedly elevated but the TSH is low due to negative feedback of thyroid hormones on
the pituitary. Other options above are true.

345-All of the following are true of Listing's reduced eye EXCEPT:

it treats the eye as a single refracting surface


the principal plane lies within the anterior chamber

the nodal point lies at the anterior surface of the lens

it has the same refractive power as Gullstrand's schematic eye

Your answer was INCORRECT

Explanation

The nodal point is at the posterior surface of the lens - all other options are true.

346-Which is FALSE of the levator palpebrae superioris:

its origin blends with the superior rectus

it is responsible for opening the eyelids

the lateral horn of its aponeurosis forms the lateral palpebral ligament

its origin is the greater wing of sphenoid

Your answer was INCORRECT

Explanation

The origin of the levator palpebrae superioris is the lesser wing of sphenoid above and in
front the optic canal. Its origin blends with the superior rectus. The lateral horn of its
aponeurosis forms the lateral palpebral ligament which inserts into the lateral orbital
tubercle.

347-The spinothalamic tracts transmit:

volitional motor pathways

vibration, touch and proprioception

pain and temperature

autonomic motor pathways


Your answer was INCORRECT

Explanation

The spinothalamic tracts of the spinal column transmit pain and temperature. First order
neurones synapse at the spinal level, second order neurones cross the midline at the spinal
level before ascending to synapse in the thalamus.

348-All of the following viruses are associated with human carcinoma EXCEPT:

Epstein-Barr virus

Human Papilloma virus

Hepatitis C

HIV

Hepatitis A

Your answer was INCORRECT

Explanation

Viruses that are associated with carcinoma include: Epstein-Barr virus (nasopharyngeal
carcinoma), hepatitis B and C (hepatocellular cancer), HIV virus (lymphoma, Kaposi's
sarcoma), human papilloma virus (cervical cancer).

349-Wessely's rings are a manifestation of what type of hypersensitivity?

Type II

Type I

Type V

Type IV

Type III

Your answer was INCORRECT


Explanation

Wessely's rings, also known as immune rings, are ring infiltrates of the corneal stroma,
parallel to the limbus. Some corneal rings are probably formed as antigen from a corneal
infiltrate encounters antibody from peripheral corneal blood vessels. The infiltrate
generally contains complement factors and polymorphonuclear neutrophils (PMNs).

350-Which statement regarding the sigmoid sinus is FALSE:

it is at risk of thrombosis if there is severe mastoid inflammation

it contains blood from the cavernous sinus

it grooves the temporal bone

it is a continuation of the transverse sinus

Your answer was INCORRECT

Explanation

The sigmoid is a continuation of the transverse sinus and it ends below the jugular
foramen. It grooves the occipital bone. It receives blood from the cavernous sinus via the
superior and inferior petrosal sinuses. Because of its close proximity to the mastoid air
cells, infection in this region can cause thrombosis.

351-How thick is the corneal epithelium?

100-120 micrometers

40 nanometers

10-15 micrometers

50-60 micrometers

Your answer was INCORRECT

Explanation

The corneal epithelium is 50-60 micrometers thick.


352-CD4+ cells:

are cytotoxic cells

regulate the immune functions by secreting interleukin-2

produce antibodies

are B lymphocytes

Your answer was INCORRECT

Explanation

CD4+ cells are known as T helper cells and account for 65% of circulating lymphocytes.
They are responsible for regulating the immune functions and secrete interleukin-2.
CD8+ cells are either cytotoxic T cells or suppressor T cells and have cytotoxic and
regulatory functions respectively
353-A 20 dioptre concave spectacle lens produces a prismatic effect of 60 prism dioptres.
How decentred is the lens:

3 meters

20 meteres

0.33 meters

3 centimeters

Your answer was INCORRECT

Explanation

The prismatic power is calculated by the formula:

P=FxD

Where P is the prismatic power in prism diopters, F is the lens power in diopters and D is
the distance from the centre of the lens in centimeters.

354-The sclera:
is thickest behind the insertions of the aponeurotic tendons of the extraocular muscles

consists of regularly arranged collagen

is composed mainly of collagen Type 1

has more proteoglycans and glycosaminoglycans in its matrix than the cornea

Your answer was INCORRECT

Explanation

The sclera consists of dense irregular connective tissue, unlike the cornea which is
regular in arrangement. The sclera also has less proteoglycans and glycosaminoglycans in
its matrix than the cornea. The sclera is thinnest behind the insertions of the aponeurotic
tendons and is composed mainly of Type 1 collagen.

355-The scarring of disciform macular degeneration is the result of:

hyalinization of RPE

hyperplasia of RPE

dysplasia of RPE

fibrous metaplasia of RPE

Your answer was INCORRECT

Explanation

Following choroidal neovascularization with haemorhage and leakage into the retina
there is fibrous metaplasia of RPE cells with deposition of collagen causing the disc-
shaped mass known as a disciform scar.

356-Which cellular component of the immune system has cytotoxic activity that is
antibody dependent?

natural killer (NK) cells


plasma cells

B-lymphocytes

T-lymphocytes

killer cells

Your answer was INCORRECT

Explanation

Killer, or K, cells require antibody to effect cell death through so-called antibody-
dependent cellular cytotoxicity (ADCC).

357-Which statement is FALSE of the pentose phosphate pathway:

it occurs in the cytoplasm

it is important in gluconeogenesis

it produces ribose which is important for nucleotide synthesis

it produces NADP for use in reductive biosynthetic reactions

Your answer was INCORRECT

Explanation

The pentose phosphate pathway converts glucose-6-phosphate to ribose with the


production of NADPH. Ribose is important for the synthesis of nucleic acids (DNA and
RNA) while NADPH is important for use in reductive biosynthetic reactions such as fatty
acid synthesis. The pathway also permits gluconeogenesis, the production of glucose
from unrelated precursors.

358-Which statement about topical eye drops is FALSE:

timolol is a non-selective beta-1 and beta-2 blocker

phenylephrine is a non-selective beta-agonist


betaxolol is a selective beta-1 blocker

apraclodine is an alpha-2 partial agonist

Your answer was INCORRECT

Explanation

Phenylephrine is a non-selective alpha-agonist. Other options are true.


359-Fibrotic retinal scarring after injury occurs due to proliferation of:

microglia

retinal pigment epithelial cells

astrocytes

Muller cells

Your answer was INCORRECT

Explanation

The astrocytes are responsible for retinal scar tissue formation.

The function of the various retinal neuroglial cells is summarized below:

 The Muller cells: are the principle supporting glial cells of the retina and are
analogous to central nervous system oligodendrocytes.

 The astrocytes: form a honeycomb scaffold perpendicular to the Muller cells.


They are responsible among other functions for laying down scar tissue in injured
or diseased retina.

 The microglia: are highly specialized mononuclear phagocytic cells

360-With regard to the lacrimal drainage system:

congenital blockage is due mainly to delay in development of the puntum.

the nasolacrimal duct is narrowest at the lowest end


the nasolacrimal duct drains into the superior meatus

the upper lacrimal punctum is medial to the lower punctum

Your answer was INCORRECT

Explanation

The upper lacrimal punctum is medial to the lower punctum. The nasolacrimal duct is
narrowest in the middle than either end. The duct runs drains via the inferior meatus.
Congenital blockage is usually due to failure of the lower end of the nasolacrimal duct
becoming patent.

361-Corneal compression can be suspected by comparing the following readings between


scans:

lens thickness

anterior chamber depth

corneal thickness

posterior chamber depth

Your answer was INCORRECT

Explanation

The A-scan produces measurements for anterior chamber depth, lens thickness, and total
eye length. The normal ACD reading is between 2.5 and 4 mm. An outlier AC depth
measurement much smaller than others taken for the same eye, suggests compression of
the cornea during that reading.

362-A loupe with a magnification of x8 has a lens power of:

4 dioptres

32 dioptres

2 dioptres
8 dioptres

Your answer was INCORRECT

Explanation

For the loupe, power of lens in diopters = magnification x 4

363-Which is FALSE of the stretch reflex:

it is inhibited by impulses from Golgi tendon organ

it originates in the muscle spindle

it is decreased in lesions of the motor cortex

it is a monosynaptic reflex with a response time of 1 msec

Your answer was INCORRECT

Explanation

The stretch reflex is increased (not inhibited) by upper motor neurone lesions. All other
statements are true.

364-Muscles involved in opening the jaw include:

the medial pterygoid

the temporalis

the masseter

the lateral pterygoid

Your answer was INCORRECT

Explanation

 Jaw opening muscles: lateral pterygoid and mylohyoid.


 Jaw closing muscles: medial pterygoid, temporalis and masseter.
365-The foramen rotundum transmits:

the maxillary nerve

the accessory meningeal artery

the lesser petrosal nerve

the mandibular nerve

Your answer was CORRECT

Explanation

The foramen rotundum transmits the maxillary nerve and the small veins of the
cavernous sinus. The other options supplied are transmitted through the foramen ovale.

366-Histamine causes:

negative chronotropic action

bronchodilatation

vasoconstriction

increased gastric acid secretion

Your answer was INCORRECT

Explanation

Histamine causes: bronchoconstriction, vasodilation and increased gastric acid secretion.


It has a positive chronotropic action resulting from a direct effect on heart receptors and
by baroreceptor reflex due to vasodilatation.

367-During a cataract operation, the intraocular lens is accidentally placed in the sulcus
rather than within the capsular bag.

This will result in:


myopic shift

hypermetropic shift

irregular astigmatism

with-the-rule astigmatism

Your answer was CORRECT

Explanation

The lens is placed more anteriorly and therefore results in a myopic shift compared to the
predicted outcome.

368-Which one of the following human leukocyte antigen (HLA) markers is associated
with the presumed ocular histoplasmosis syndrome (POHS)?

B8

B27

A29

B5

B7

Your answer was INCORRECT

Explanation

Human leukocyte antigens (HLA) associations include:

 A29: Birdshot chorioretinitis


 B7 and DR2: Presumed ocular histoplasmosis syndrome (POHS) and multiple
sclerosis (MS)
 B8: Sarcoidosis, intermediate uveitis
 B27: Idiopathic iritis, psoriatic arthritis, inflamm bowel disease, ankyl spondylitis,
Reiter's
 B44: Retinal vasculitis
 B51: Behcet's disease (in Asians only)
 DR4: juvenile rheumatoid arthritis (JRA), Vogt-Koyanagi-Harada (VKH)
syndrome, sympathetic ophthalmia (SO)

369-A pure H-1 histamine agonist would cause all EXCEPT:

increased gastric acid secretion

capillary dilatation

increased vascular permeability

bronchospasm

Your answer was CORRECT

Explanation

H1 receptors cause:

 increased vascular permeability


 arteriolar and capillary dilatation
 smooth muscle contraction
 bronchospasm
 CNS depressant

H2 receptors cause:

 increased pepsin and acid secretion


 increased myocardial stroke volume

370-All statements are true of the vitreous EXCEPT:

Cloquet's canal is a remnant of the hyaloid vein

the vitreous is attached to the inner limiting membrane of the retina especially near retinal
vessels

the cortical vitreous is attached to the pars plana via the vitreous base
vitreous cells known as halocytes bear similarities to macrophages

Your answer was CORRECT

Explanation

Cloquet's canal is a remnant of the hyaloid artery, which supplies the lens and vitreous in
the fetal eye.

371-Which is FALSE regarding the effects of contact lens wear on the cornea:

there is disturbance to the tear mucin layer

oxygen transfer from the tear film to the epithelium is reduced

there is a rise in lactate concentrations

the corneal stroma is thinned

Your answer was INCORRECT

Explanation

The corneal stroma is thickened in contact lens wear due to impairment of deturgescent
mechanisms. Other options are true.

372-Which statement is FALSE about the occipitofrontalis:

it causes vertical wrinkling of the forehead

it is supplied by temporal branches of the facial nerve

upon contraction it draws the scalp backwards and elevates the brows

it covers the dome of the skull from the eyebrows to the nuchal lines

Your answer was CORRECT

Explanation
The occipitofrontalis draws the scalp backwards and elevates the brows causing
horizontal or transverse wrinkles on the forehead and scalp.

373-The following extraocular muscles are correctly paired as yoked muscles EXCEPT:

right superior oblique and left inferior rectus

right medial rectus and left lateral rectus

right superior rectus and left inferior oblique

right inferior rectus and left superior rectus

Your answer was INCORRECT

Explanation

Yoked muscles receive equal and simultaneous innervation in accordance with the
Hering 's law. The right inferior rectus is yolked with the left superior oblique (as the
eyes look right and downwards). Other options supplied are true

374-Active transport occurs in all of the following EXCEPT:

glucose absorption from the gut

water reabsorption from the proximal convoluted tubule

potassium excretion in the distal convoluted tubule

movement of sodium ions across the cell membrane

water reabsorption from the collecting ducts of the kidneys

Your answer was INCORRECT

Explanation

Glucose absorption from the gut, potassium excretion in the distal convoluted tubule and
water reabsorption from the collecting ducts are all examples of active transport,
involving ATP. Water reabsorption from the proximal convoluted tubule occurs due to
the concentration gradient.
375-Which statement is FALSE of drug metabolism:
Phase I involves oxidation reactions

Phase II may involve the addition of glucuronate

Phase II involves reducing the water solubility

Cytochrome P450 enzymes carry out Phase I metabolism but not Phase II

Your answer was INCORRECT

Explanation

Phase I drug metabolism involves oxidation reactions and can be carried out by the
cytochrome P450 enzymes. Phase II metabolism involves conjugation reactions, such as
the addition of glucuronate, glutamine and acetate groups. This conjugation makes the
molecules more water soluble for excretion.

376-The infraorbital nerve is a terminal branch of:

the lacrimal nerve

the maxillary nerve

the nasociliary nerve

the frontal nerve

the mandibular nerve

Your answer was INCORRECT

Explanation

The infraorbital nerve is a terminal branch of the maxillary nerve .

377-Which statement is FALSE about the optic nerve:

the optic disc is approximately 1.5mm in diameter

the raised papilla of the optic disc is more prominent nasally than temporally
the maculopapillary bundle constitutes 90% of all axons leaving the eye

the intra-ocular portion of the optic nerve is unmyelinated

Your answer was INCORRECT

Explanation

The raised papilla of the optic disc is more prominent temporally than nasally owing to
the entry of the maculopapillary bundle temporally.

378-Which statement is FALSE regarding the thyroid gland:

it is supplied by branches of the internal carotid artery

it increases in size during pregnancy

it moves with swallowing because it is enclosed by the pretracheal fascia

it contains 4 parathyroid glands supplied by the inferior thyroid artery

Your answer was CORRECT

Explanation

The thyroid gland is supplied by the superior thyroid artery which is a branch of the
external carotid artery and the inferior thyroid artery which is a branch of the brachial
artery. The four parathyroid glands are supplied by the inferior thyroid artery. The
thyroid gland increases in size during pregnancy.

379-The following are true of the spleen EXCEPT:

it has a major haemopoietic function in foetus

it contains T cells in the white pulp

if removed in an adult this reduces susceptibility to infection

it contains B cells in the follicles


Your answer was INCORRECT

Explanation

The spleen forms the interface between the blood and the lymphatic system. It contains
the white pulps and the red pulps. The white pulps surround the arterioles and contain T
cells. B cells are found in the follicles. The red pulps remove old or damaged red blood
cells from the circulation. Splenectomy predisposes patients to certain infections such as
malaria and pneumoccal infections.

380-Which is FALSE regarding the composition of the aqueous humour compared to


plasma:

there is a higher concentration of ascorbate in aqueous

there is a higher concentration of lactate in aqueous

there is a lower concentration of albumin in the aqueous

there is a higher concentration of sodium in the aqueous

Your answer was INCORRECT

Explanation

The aqueous humour has a lower concentration of virtually all constituents compared to
the plasma, with the exception of lactate, ascorbate and chloride. It therefore has a lower
protein, glucose, sodium, potassium and bicarbonate concentrations.

381-Retinoblastoma:

is associated with loss of the short arm of chromosome 13

is autosomal dominant in the inherited form

is inherited in the majority of bilateral cases

is sporadic in the minority of unilateral cases

Your answer was INCORRECT

Explanation
Retinoblastoma is associated with loss of the long arm of chromosome 13.
When inherited it appears like an autosomal dominant condition but at the genetic levels
both alleles must be affected to produce the disease.
Both unilateral and bilateral cases are sporadic in the majority of cases.

382-Which statement is FALSE about the corneal epithelium:

it is a stratified non-keratinized squamous epithelium

its basement membrane is called the Bowman's layer

Dendritic cells decline sharply in density from the periphery towards the centre of the
cornea

MHC Class II cells are rare in the central corneal

Your answer was INCORRECT

Explanation

The corneal epithelial basement membrane is called the basal lamina and comprises the
lamina lucida and the lamina densa. The Bowman's layer is an acellular region of the
anterior stroma. Other options above are true

383-Two heterozygous parents for a recessive condition want to know the risk that their
child will be a carrier. What will you advise them:

75%

50%

33%

100%

25%

Your answer was INCORRECT

Explanation
There is a 1 in 2 chance of being a carrier. We recommend drawing a 2x2 box of
chromosomal probabilities.

384-What is the spherical equivalent of a toric lens with power +2.00DS/-2.00DC:

+1.00 DS

-2.00 DS

+0.00 DS

+4.00 DS

Your answer was CORRECT

Explanation

The spherical equivalent of a toric lens is calculated by addition of the spherical power
and half the cylindrical power.

385-Which statement about the human lens is FALSE:

it has a higher refractive index in the nucleus than the cortex

it contains a higher concentration of sorbitol in the diabetic

it contains nucleated cells mainly on its posterior surface

glucose is metabolised mainly by anaerobic glycolysis

Your answer was INCORRECT

Explanation

The lens contains nucleated cells mainly on its anterior surface. Other statements are true.

386-The glucose level in the normal cerebrospinal fluid is:

equal to the level in arterial blood

lower than in arterial blood


higher than in arterial blood

equal to the level in venous blood

Your answer was INCORRECT

Explanation

Almost all constituents of CSF are found in lower concentration than that of blood with
the exception of Chloride and H+, which are higher. Thus, the pH and the PO2 of the
CSF are lower than that of the arterial blood. Protein is lower in the CSF (in fact it is
almost absent in the normal CSF). The glucose in CSF is 2/3 that of the blood.

387-Which is TRUE of prostaglandin analogues:

they lower IOP mainly by increasing uveoscleral flow

they lower IOP mainly by decreasing aqueous production

they increase IOP mainly by increasing aqueous production

they increase IOP mainly by an effect on the central nervous system

Your answer was CORRECT

Explanation

Prostaglandin analogues lower IOP mainly by increasing uveoscleral flow

388-Atropine:

causes paralysis of the iris dilator muscle

causes ciliary body contraction

increases gastric emptying

has a sedative effect

increases sweating
Your answer was INCORRECT

Explanation

Atropine is a cholinergic antagonist. It causes:

 delayed gastric emptying


 reduced sweating
 sedation when administered systematically
 cycloplegia via ciliary body paralysis
 mydriasis via iris sphincter muscle paralysis

389-Which condition is associated with excessive sunlight exposure:

Fuch's endothelial dystrophy

Granular dystrophy

Labrador keratopathy

Lattice dystrophy

Reis-Buckler dystrophy

Your answer was INCORRECT

Explanation

Labrador keratopathy is associated with excessive sunlight exposure causing deposition


of keratinoid particles of protein in the stroma.

390-The trabecular meshwork arises from:

endoderm

neural crest mesenchyme

neuroectoderm

surface ectoderm
mesodermal mesenchyme

Your answer was INCORRECT

Explanation

The trabecular meshwork develops from neural crest derived mesenchyme.

391-Which of the following statements regarding the standard error of the mean is true?

the standard error of the mean has no units

the standard error of the mean is the standard deviation of the sampling
distribution of the mean

as sample size increases, standard error of the mean becomes greater

the standard error of the mean is proportional to the sample size

the standard error of the mean is usually greater than the standard deviation

Your answer was INCORRECT

Explanation

If one were to take several samples from the same population, all would result in normal
distribution curves, but they would be slightly different from each other. Each mean
would be scattered around the true population mean. The standard error of the mean is a
measure of this scatter, i.e. how accurate is the sample mean as an estimate of the true
population mean.

Note, SEM = Standard deviation/square root of sample size

This demonstrates that S.E.M. is affected by two factors:

 As sample size increases, S.E.M. decreases


 As standard deviation increases, S.E.M. increases

Note that S.E.M. is always smaller than standard deviation.

392-All are true of anaphylaxis EXCEPT:


it is produced by Ig E antibody

it causes degranulation of basophils and mast cells

it causes decreased eosinophil counts

it causes urticaria

Your answer was INCORRECT

Explanation

All statements are true except that anaphylaxis causes eosinophil counts to increase not
decrease.

393-All of the following are TRUE about antigen presenting cells (APCs) except:

They induce clonal T cell proliferation by producing interleukin-1

Neutrophils are antigen presenting cells

Langerhan's cells are the antigen presenting cells of the epidermis

Tumour necrosis factor alpha can turn endothelial cells into antigen presenting cells

CD8+ cells only recognise antigen presenting cells bearing MHC (major
histocompatibility complex) class I

Your answer was INCORRECT

Explanation

Langerhan's cells are the antigen presenting cells of the epidermis. CD8+ cells recognise
APCs bearing MHC class I molecules whereas CD4+ recognise APCs bearing MHC
class II molecules. Antigen presenting cells are B lymphocytes, mononuclear phagocytes,
tissue macrophages, dendritic cells, Langerhan's cells and activated endothelial cells but
not neutrophils.

394-All of the following muscles are supplied by the facial nerve EXCEPT:
temporalis

corrugator supercilii

fronalis

levator labii superioris

procerus

Your answer was CORRECT

Explanation

The muscles of facial expression are innervated by the facial nerve and include:

 procerus
 frontalis
 corrugator supercilii
 orbicularis oculi
 zygomaticus major and minor
 levator labii
 nasalis
 depressor supercilii
 occipitofrontalis.

The temporalis is a muscle of mastication supplied by the trigeminal nerve.

395-Which statement about the optic nerve is FALSE:

there are approximately 1.2 million ganglion cell axons in the nerve fibre layer of the eye

glaucoma damages the intra-orbital portion of the optic nerve

there is a high proportion of glial cells in the region of the lamina cribrosa

the optic nerve fibres pierce the lamina cribrosa which consists of irregular collagen
bundles in figure-of-eight patterns

Your answer was INCORRECT

Explanation
Glaucoma damages the intra-ocular portion of the optic nerve, that is, the portion of the
optic nerve between the optic disc and its exit from the sclera. Other options supplied are
true

396-The inner capillary plexus of the retina is contained in:

the inner nuclear layer

the inner plexiform layer

the inner segment

the ganglion cell layer

Your answer was INCORRECT

Explanation

The inner capillary plexus of the retina is contained in the ganglion cell layer.

397-Bowman's membrane:

terminates as the Schwalbe's line

is a basement membrane

is 50 microns thick

does not regenerate but heals by scaring

Your answer was INCORRECT

Explanation

Bowman's membrane is not a true basement membrane. It is 8-12 um thick and cannot
regenerate from injury but is replaced by scar tissue from stromal keratocytes. Descemet's
membrane terminates as Scwhalbe's line.

398-Which is FALSE regarding mitochondrial genetics:

males cannot pass a mitochondrial condition to their offspring


mitochondrial DNA is circular

mitochondria contain their own tRNA and rRNA necessary for protein synthesis

the sperm do not possess mitochondria

Your answer was INCORRECT

Explanation

Sperm contain many mitochondria necessary for the flagellate movements. However the
mitochondria do not enter the ova and therefore are not passed on to offspring.

399-All are true of mycobacteria EXCEPT:

they are aerobic

they are best cultured on Lowenstein-Jensen medium

they are usually Gram negative

they are acid fast

Your answer was INCORRECT

Explanation

Mycobacteria cannot be stained with Gram stain and require special staining techniques
such as the Ziehl-Neelsen stain.

400-The main source of image imperfection in a small pupil is:

coma aberration

spherical aberration

diffraction

chromatic aberration
Your answer was INCORRECT

Explanation

In a small pupil, abberations from refractive elements of the eye such as chromatic and
spherical abberations are reduced but diffraction, caused by light being disturbed as it
passes through a small opening, is increased.
401-Which is FALSE regarding indocyanine green:

it is absorbed by the RPE layer

it emits infrared light

it absorbs infrared light

it is better for viewing the choroidal circulation than fluorescein angiography

Your answer was CORRECT

Explanation

Indocyanine green (ICG) is not absorbed by the RPE cells, unlike fluorescein, and
therefore provides a better view of the choroidal circulation. It is useful in delineating
occult choroidal neovasculation such as polypoid CNV, which is not visible in
fluorescein. The ICG dye absorbs light of wavelength 805nm and reflects light of
wavelength 835nm, both being in the infrared spectrum.

402-A ray of light parallel to the principle focus of a thin lens will:

be refracted parallel to the principle plane of the lens

reflect off the lens by total internal reflection

be refracted through the focal point of the lens

pass through undeviated

Your answer was INCORRECT

Explanation
A ray of light passing through principle focus of a thin lens will be refracted through the
focal point of the lens. It is an important principle in the construction of ray diagrams, as
for any object, one can predict the image produced by drawing 2 lines (a) one passing
through the nodal point undeviated, (b) the other parallel to the principle action passing
through the second principle focus.

403-The outer capillary plexus of the retina is contained in:

the inner plexiform layer

the inner nuclear layer

the outer nuclear layer

the ganglion cell layer

Your answer was INCORRECT

Explanation

The outer capillary plexus of the retina is contained in the inner nuclear layer.

404-Regarding antiobiotic usage:

rifampicin is only effective against tuberculosis

intravenous vancomycin is useful in treating post-operative endophthalmitis

fusidic acid is ineffective against staphylococcus aureus infection

gentamicin is effective against Gram negative bacteria

Your answer was INCORRECT

Explanation

Fusidic acid is effective against Staphylococcus aureus but resistance can occur.
Vancomycin is ineffective against endophthalmitis when given intravenously as the drug
penetration into the vitreous cavity is poor. Gentamicin is effective against mainly Gram
negative bacteria (less so against Gram positives).

405-All of the following are likely to be found in a chalazion EXCEPT:


multinucleate giant cells

fat spaces

neutrophils

T lymphocytes

macrophages

Your answer was INCORRECT

Explanation

A chalazion is the definitive example of a lipogranuloma where the release of fat (sebum
from the meibomian glands) into tissue results in a chronic inflammatory response.
Neutrophils are not typical of a chronic lipogranuloma, which is normally populated by
macrophages, multinucleate giant cells and lymphocytes.

406-Microfilarialload was analysed in a cohort of children and found to be associated with


age. The regression coefficient was 0.024. Which of the following is TRUE?

the fact that the regression coefficient is so small means that it is unlikely to be
significant

the null hypothesis would state that the regression coefficient is zero

we can use the regression coefficient to calculate microfilarial load in elderly adults

regression coefficients can take any value between 0 and 1

the regression coefficient is an estimate of the strength of the association between age and
microfilarial load

Your answer was INCORRECT

Explanation

Regression coefficients are the gradient of the line of best fit and so can take any value. A
small regression coefficient may still be statistically significant (that is statistically
significantly different to zero) and may have a high correlation coefficient (r near to +1 or
-1). A regression coefficient of zero, however, implies no relation and therefore the null
hypothesis. In this case, we cannot use the regression coefficient to calculate microfilarial
load in elderly adults because we cannot assume the gradient stays the same outside our
sample. The correlation coefficient is an estimate of the strength of an association.

407-The accumulations of goblet cells on the tarsal conjunctiva are called:

Henle's crypts

Haller's layer

Sattler's layer

Manz's glands

Your answer was CORRECT

Explanation

The accumulation of goblet cells on the tarsal conjunctiva are Henle's crypts. Manz's
glands are the accumulation of goblet cells on the bulbar conjunctiva nasal to the limbus.

408-Regarding the herpes zoster virus:

shingles results from primary infection of herpes zoster virus

it can be inactivated by the administration of intravenous gentamycin

it is a RNA virus

it remains in the nerve ganglia for life in patients with previous infection

Your answer was INCORRECT

Explanation

Herpes zoster virus is a DNA virus. It can be inactivated by acyclovir. Following the
initial infection (chickenpox) it stays in the nerve ganglia for life and can become
reactivated in time of weakened immune system. The reactivation usually takes the form
of shingles.

409-Which is FALSE regarding the iris:


narrowing of the pupil increases the depth of focus

the dilator pupillae is a continuation of the pigmented ciliary epithelium

iris vessels are non-fenestrated

narrowing of the pupil reduces optical aberrations

Your answer was INCORRECT

Explanation

The dilator pupillae is a continuation of the non-pigmented ciliary epithelium. The


posterior pigmented iris epithelium is a continuation of the ciliary body pigmented cells.
Other statements above are true.

410-Which statement is NOT true of vitreous degeneration:

it usually commences in the late thirties

it is implicated in the pathophysiology of posterior vitreous detachment

it involves collagen fibrils coming out of suspension

it involves liquefaction of the collagen gel called syneresis

Your answer was CORRECT

Explanation

Vitreous degeneration commences in humans in the late teens. Other statements are true.

411-All of the following are true about the pituitary fossa EXCEPT:

it forms part of the anterior cranial fossa

it contains cavernous sinus laterally on either side

it is bounded by the anterior and posterior clinoid processes


its roof is formed by dura mater

it lies above the body of the sphenoid

Your answer was CORRECT

Explanation

The pituitary fossa is part of the middle cranial fossa. All other options supplied are true

412-Midget ganglion cells and midget bipolar cells are most common in:

the nasal periphery

the peri-papillary region

the fovea

the temporal periphery

Your answer was INCORRECT

Explanation

Midget ganglion and midget bipolar cells synapse with cones in a 1:1:1 ration and occur
most often in the fovea. This neural exclusivity helps to explain the high resolution of the
foveal region.

413-What is the pattern of inheritance of Best diseases?

autosomal dominant

X-linked recessive

autosomal recessive

mitochondrial

Your answer was CORRECT

Explanation
Best disease is an autosomal dominant hereditary macular degeneration

414-Zeil-Neilsen stain is best used to visualize:

Treponema pallidum

Mycobacterium tuberculosis

Acanthoemeba

Haemophilus

Your answer was INCORRECT

Explanation

Zeil-Neilsen stain is used for mycobacteria.

415-A21-year-old man with insulin dependent diabetes mellitus presents with


hypothyroidism. His HLA (human leukocyte antigen) haplotype is most likely to be:

HLA B51

HLA B27

HLA DR4 and DR1

HLA A1 B8 DR3

Your answer was INCORRECT

Explanation

HLA A1 B8 DR3 is commonly associated with organ specific autoimmune disease.


IDDM can also be associated with DR4. HLA B27 is strongly associated with
seronegative arthritis, and HLA B51 is associated with Behcet's disease. HLA DR1 and
DR4 (or a shared epitope in these class II proteins) is associated with RA (rheumatoid
arthritis).

416-The primary visual cortex is referred to as:


area 18

area 20

area 19

area 17

Your answer was INCORRECT

Explanation

The primary visual cortex is also known as area 17 in the map of the human cerebrum.

417-Which of the following does NOT apply in hypovolaemic shock:

the urine output decreases

the pulse rate increases

there is a fall in renin concentration

the blood flow to the brain and kidneys are initially maintained

there is a loss of more than 20% of the blood volume

Your answer was INCORRECT

Explanation

Renin secretion is increased during hypovolaemic shock to maintain the plasma volume
by conserving water. All other options are true.

418-All are true of Acanthoemeba EXCEPT:

it is a common cause of corneal ulcer in contact lens wearers

acanthoemeba keratitis is resistant to conventional antibiotic drops

it is a commensal and may be recovered from nasal and pharyngeal swabs


it is best grown on non-nutrient E.coli infused agar

Your answer was CORRECT

Explanation

Acanthoemeba is an uncommon cause of contact lens related corneal ulceration, which is


usually caused by pseudomonas. However, it is more likely in contact lens wearers. It is
grown on non-nutrient e.coli agar and is difficult to culture. It responds poorly to
conventional first-line antibiotic drops.

419-The Bruch's membrane is composed of:

4 layers

2 layers

5 layers

3 layer

Your answer was INCORRECT

Explanation

The Bruch's membrane is composed of 5 layers:

 the RPE basal lamina


 an inner collagenous zone
 a middle elastic layer
 an outer collagenous zone
 the basement membrane of endothelial cells in the choriocapillaris

420-All of the following antibiotics inhibit cell wall synthesis EXCEPT:

gentamicin

cephalosporins

vancomycin
penicillins

Your answer was CORRECT

Explanation

Penicillins, cephalosporins and vancomycin all inhibit cell wall synthesis.


Aminoglycosides such as gentamicin inhibit bacterial protein synthesis.

421-Which of the following describes retinal oedema following blunt trauma:

commotio retinae

retinoschisis

cystoid macular oedema

serous retinal detachment

retinal macroangiopathy

Your answer was CORRECT

Explanation

Commotio retinae describes diffuse retinal oedema following blunt trauma likely due to
transient ischaemia and disruption of axoplasmic flow.

422-Which antibiotic does NOT inhibit bacterial cell wall synthesis:

benzylpenicillin

vancomycin

cephalosporins

ciprofloxacin

Your answer was INCORRECT

Explanation
Ciprofloxacin is a quinolone antibiotic, which blocks DNA synthesis by inhibiting DNA
gyrase. Meanwhile penicillins, cephalosporins and vancomycin all inhibit cell wall
synthesis.

423-The following are true about the Fc regions of immunoglobulins EXCEPT:

they are responsible for antibody binding

they are involved in mast cell binding

they are involved in the activation of the complement cascade

they determine the ability of the immunoglobulins for transplacental transfer

Your answer was CORRECT

Explanation

Antibody binding is carried out by the Fab (antibody) fragments. Other options are true.

424-Which statement is FASLE regarding glucagon:

it inhibits gluconeogenesis in the liver

it is secreted by alpha-islet cells of the pancreas

it is a polypeptide hormone

it has a positive cardiac inotropic effect

Your answer was CORRECT

Explanation

Glucagon stimulates gluconeogenesis. All other options are true.

425-All of the following are useful stains for identifying fungi EXCEPT:

Gram stain

Gomori methenamine silver


PAS (periodic acid-Schiff)

Haematoxylin and eosin (H & E)

Your answer was CORRECT

Explanation

Fungi stain:

 pink with H & E


 purple with PAS
 black with Gomori methenamine silver

426-Regarding CO2 in venous blood:

it is carried mainly by the haemoglobin

CO2 does not cross the blood brain barrier

it is transported mainly in the form of bicarbonate ions

oxygenated haemoglobin binds CO2 more readily than deoxygenated haemoglobin

it is converted to bicarbonate ions by carbonic anhydrase in the plasma

Your answer was INCORRECT

Explanation

CO2 is converted to hydrogen ions and bicarbonate ions by the carbonic anhydrase in the
red blood cells. Most bicarbonate ions leave the red blood cells and are carried in the
plasma. Only 5% of CO2 is bound to the haemoglobin. Deoxygenated haemoglobin binds
CO2 better than oxygenated haemoglobin. CO2 crosses the blood brain barrier readily.

427-Which of the following genetic conditions is NOT associated with increased risk of
cancer:

Chromosome 13q deletion


Neurofibromatosis Type 1

Turner's syndrome

Neurofibromatosis Type 2

Down's syndrome

Your answer was INCORRECT

Explanation

Turner's syndrome is not associated with an increased risk of cancer. Chromosome 13q
deletion is associated with retinoblastoma. Neurofibromatosis Type 1 is associated with
meningioma, glioma and phaeochromocytoma. Neurofibromatosis Type 2 is associated
with bilateral acoustic neuroma. Down's syndrome is associated with acute leukemia.

428-Which of the following is integral to the structure of cellular basement membranes?

laminin

fibrillin

Type II collagen

elastin

Your answer was CORRECT

Explanation

Laminin is an extracellular matrix protein integral to the structure of basement


membranes.

429-Arandomised controlled trial for a new drug compared to placebo, has been
undertaken in a cohort of 100 patients. The trial follows the principle of intention to treat.

Which of the following must occur in an intention-to-treat analysis?

exclusion of the patients who did not take the new drug
exclusion of the patients who dropped out due to side effects

inclusion of patients who were still living after the trial

in both groups patients who drop out are included in analysis at the end of the trial

exlusion of the patients who were not followed up

Your answer was INCORRECT

Explanation

An intention-to-treat analysis requires that patients who drop out (e.g. due to side effects)
should still be included in statistical analysis at the end of the trial.

430-A reduction in the number of chromosomes occurs in:

Fragile-X syndrome

Retinoblastoma

Klinefelter's syndrome

Turner syndrome

Down's syndrome

Your answer was INCORRECT

Explanation

Turner syndrome has 1 less sex chromosome XO. Down's syndrome and Klinefelter's
have an extra chromosome (trisomy 21, and xxY respectively). Fragile X and
retinoblastoma have the normal number of chromosomes.

431-Which is FALSE regarding the central retinal artery:

its large arterial branches travel in the nerve fibre layer

its large arterial branches contain a thin internal elastic lamina


it pierces the optic nerve about 1cm behind the eye

it is a branch of the ophthalmic artery

Your answer was INCORRECT

Explanation

The arterial branches of the central retinal artery contain no internal elastic lamina, which
is lost at the optic disc. This means they are not involved in temporal arteritis. The other
statements above are true.

432-Which of the following statements is TRUE regarding the thyroid hormones:

T4 is more active than T3

in the serum most of the thyroid hormones are bound to albumin

they are steroid hormones

thyroid hormones sensitize the myocardium to noradrenaline

Your answer was INCORRECT

Explanation

Thyroid hormones are peptides not steroids. T3 is the more active form compared to T4.
Over 99% of thyroid hormones are bound to the protein in serum and the most common
protein that binds these hormones is the globulin thyroxine-binding globulin (TBG).
Thyroxine sensitizes myocardium to noradrenaline.

433-Which is true of endotoxin:

it is derived from the cell wall of Gram negative bacteria

it elicits an antibody response which may protect the host from future attack

it cannot survive autoclaving

it is involved in botulism
Your answer was CORRECT

Explanation

Endotoxins are lipopolysaccharides derived from the cell wall of gram negative bacteria.
They are heat stable, non-antigenic, cause septicaemia and fever and activate the
alternative complement pathway.

434-Which statement is FALSE regarding the visual pathway:

the optic tract leaves the chiasm and winds around the cerebral peduncles

the lateral root of the optic tract terminates in the medial geniculate body

the optic tracts divide into a large lateral root and a smaller medial root

the fibres from the nasal hemiretina decussate at the chiasm

Your answer was INCORRECT

Explanation

The lateral root of the optic tract terminates in the lateral geniculate body and is
concerned with conscious vision.

435-The posterior pituitary gland secretes:

thyrotrophin

oxytocin

adrenocorticotrophic hormone (ACTH)

prolactin

Your answer was INCORRECT

Explanation

The anterior pituitary gland secretes:

 prolactin
 ACTH
 TSH
 LH and FSH
 GH

The posterior pituitary gland secretes:

 oxytocin
 ADH

436-Which of the following promotes proliferation of cells during development:

collagen

proteoglycans

hyalaronan

fibronectin

Your answer was INCORRECT

Explanation

Fibronectin is an extracellular matrix protein that binds to surface proteins called


integrins on cell membranes and also binds to other extracellular matrix proteins. It has
an important role in cell growth, differentiation, migration and wound healing among
other roles.

437-Which statement is NOT true regarding corneal healing:

stromal keratocytes transform into fibroblasts to heal stromal wounds

the Bowman's layer regenerates more slowly than the epithelium

the corneal epithelium heals from the limbus centrally

Descemet's membrane does not regenerate

Your answer was INCORRECT


Explanation

Neither the Bowman's layer nor Descemet's membrane can regenerate. Endothelial cells
cannot replicate either and must slide to fill defects. Other statements above are true.

438-The retinal pigment epithelium is continuous anteriorly with:

the pigmented ciliary epithelium

the non-pigmented ciliary epithelium

the scleral spur

the pigmented trabecular meshwork

Your answer was CORRECT

Explanation

The RPE is continuous anteriorly with the pigmented ciliary epithelium.

439-Which statement regarding hypermetropia is FALSE:

it is present in most children between the ages of 2 and 6 years

it should be fully corrected in esotropic children

contact lens correction reduces the demand for accommodation

it delays the onset of presbyopia

Your answer was INCORRECT

Explanation

In hypmertropia presbyopia develops earlier. All other options are true.

440-Which is FALSE regarding the corneal stroma:

it contains glycosaminglyans including hyaluronate


it has a higher water content than the sclera

the main collagen is Type I

collagen fibril diameter is highly regular

Your answer was CORRECT

Explanation

The cornea comprises glycosaminoglycans which are important in maintaining regular


spacial arrangements of collagen fibrils. The main GAG's of the corneal stroma are
keratan sulphate and dermatan sulphate. The corneal stroma is unusual in that it contains
no hyaluronate.

Note that the corneal stroma has a higher hydration than most tissues including the sclera
with a water content of 80%, in spite of the endothelium pump. This is because of the
high water-imbibing properties of the cornea, due to its high glycosaminoglycan content.

441-HLA B27 is most strongly associated with:

Behcet syndrome

ankylosing spondylitis

sympathetic ophthalmitis

Birdshot chorioretinopathy

Your answer was INCORRECT

Explanation

The following HLA associations are recognized:

 ankylosing spondylitis: B27


 Behcet syndrome: B51
 Birdshot chorioretinopathy: A29

442-A 4-month-old baby receives his third vaccination against diptheria, tetanus and
pertussis.
Which of the following is most true of his antibody response:

genetic recombination produces heavy and light chains from the same gene

it is unimportant in his subsequent response to infection with diptheria

the diversity of antibody specificities is identical to the number of different B-cell clones

his B-cells cannot switch antibody production from IgG to IgM

Your answer was INCORRECT

Explanation

Immunoglobulin genes coding for heavy and light chains are located on different
chromosomes. The genes encoding the heavy chain are found on chromosome 14 in
humans. Genes encoding the kappa light chain are found on chromosome 2 and the
human lambda light chain gene cluster is located on chromosome 22. Each locus has a
similar organisation - the variable region genes are composed of multiple gene segments
(+/- 100 - 200) separated by non-coding DNA. At the 3' end of these genes are additional
coding sequences which comprise the joining (J) and diversity (D) segments. Only the
heavy chain gene locus has diversity segments. J and D segments code for the carboxyl
terminal ends of the variable domains. Different V, J and D genes recombine to give rise
to contiguous DNA that codes for the variable domain of the protein. The constant region
genes are located at varying distances from the 3' end of the V genes. Splicing of the
primary RNA transcript aligns the constant genes adjacent to the recombined VDJ DNA
segments. Random recombination of different V, D, and J gene segments generates
antibody diversity. Diversity is also increased as there is imprecise joining between the 3'
end of a V gene and the 5' end of a J segment in a light chain or the ends of V, J, and D
segments in a heavy chain. These ends can recombine at any of several nucleotides in the
germline sequence. Additional diversity is provided by the action of TdT - an enzyme
which adds extra nucleotides to the rearranged VJ or VDJ segments. It must also be
remembered that the combination of different light chains with different heavy chains
contributes to the range of antibody produced. After the mature B-cell has left the bone
marrow and is activated by specific antigen, the cell enters the germinal centre of lymph
nodes and spleen where it undergoes somatic hypermutation. This process introduces
point mutations into the variable region exons of both heavy and light chains, particularly
in the hypervariable region. These mutations serve to increase the affinity of the BCR for
the specific antigen. Follicular dendritic cells present specific antigen to the rapidly
mutating B-cell population, and only those cells baring surface antibody with high
affinity can 'outcompete' other cells, receive a survival signal, and avoid apoptosis. Thus,
the diversity of antibody specificities far exceeds the number of different B-cell clones.
RAG1/2 are necessary to recombine immunoglobulin and T-cell receptor genes. These
proteins recognise a signal sequence adjacent to the points at which recombination will
take place; the RAG proteins introduce a single strand DNA break, and initiate somatic
recombination. In their absence, there is no rearrangement of the Ig or TCR genes which
remain in the germline configuration. This impairs B-cell and T-cell development and
results in the SCID phenotype. Mature B-cells leaving the bone marrow express IgM and
IgD as the B-cell receptor. After activation in the periphery these cells can switch
antibody production to IgG, IgE, or IgA by a process termed switch recombination. They
are however unable to switch back to production of IgM after isotype switch has occured
as the process involves a looping out and deletion of intervening DNA encoding the Cm
and Cd region. The antibody response (blocking antibodies) is important in the provision
of protective immunity against diptheria toxin.

443-Which mediator of acute inflammation is NOT derived from cells:

cytokines

leukotriene

complement

histamine

Your answer was INCORRECT

Explanation

Most mediators are derived from cells with the exception of: complement, kinin, clotting
factors, and fibrinogen, which are all derived from the plasma.

444-The fourth cranial nerve nucleus resides in:

the medulla

the upper pons

the caudal pons

the midbrain

Your answer was INCORRECT

Explanation
The fourth cranial nerve nucleus resides in the midbrain.

445-The depth of the anterior chamber depth is:

1.0 mm

5.1 mm

3.1 mm

1.5 mm

Your answer was INCORRECT

Explanation

The average adult eye has an anterior chamber depth of 3.1mm.

446-The internal carotid artery:

gives rise to the posterior cerebral arteries

traverses the foramina transversaria of the cervical vertebra 1-6

forms the basilar artery

enters the cranium through the carotid canal

Your answer was INCORRECT

Explanation

The internal carotid artery enters the cranium through the carotid canal and terminates as
the middle and anterior cerebral arteries. The vertebral artery transverses the foramina
transversaria of the cervical vertebrae 1-6. The basilar artery is formed by the vertebral
artery.

447-The functions of the following sensory receptors are correct EXCEPT:

free nerve endings for nociception


Ruffini's corpuscles for proprioception

Meissner's corpuscles for touch

Pacinian corpuscles for nociception

Your answer was INCORRECT

Explanation

All statements are correct except Pacinian corpuscles are for vibration and
proprioception.

448-Which is FALSE regarding retinopathy of prematurity:

the medial retina is usually the last to be perfused in a premature infant

it can result in abnormal vascular proliferation

it can be treated by laser photocoagulation

it may lead to retinal detachment

Your answer was CORRECT

Explanation

The temporal retina is usually the last to be perfused in a premature infant, and it is from
this area of ischemic retina that growth factors are produced that drive new abnormal
vessel formation. If left untreated vasoproliferation can lead to retinal detachment and
blindness.

449-The orbital wall which is most likely to be involved in a blow-out fracture is:

the lateral wall

the floor

the roof

the medial wall


Your answer was INCORRECT

Explanation

The medial orbital wall is the thinnest orbital wall but the orbital floor is the most likely
to be involved in blow-out fractures

450-The cells of the retinal pigment epithelium:

contain melanin granules

are shorter at the fovea than elsewhere in the retina

are of mesenchymal origin

form the inner blood-retina barrier

Your answer was CORRECT

Explanation

The cells of the retinal pigment epithelium are of ectodermal origin. They are taller in the
fovea and flatten towards the periphery. They contain melanin granules which absorb
excess light and prevent radical damage to the cell. They phagocytose the photoreceptors
segment and store vitamin A and therefore play an important part in the regeneration of
photoreceptors. They form the outer blood-retina barrier.

451-A 36 dioptre convex lens used as a loupe will produce a magnification of:

12x

3x

6x

9x

Your answer was INCORRECT

Explanation

For the loupe, power of lens in diopters = magnification x 4


452-All statements are true about the oculomotor nerve and its branches EXCEPT:

the superior division of the oculomotor nerve supplies the superior rectus and the levator
palpebrae superioris

pre-ganglionic parasympathetic fibres from the oculomotor nerve pass to the ciliary
ganglion via the branch of the inferior division that supplies the inferior oblique

the inferior division of the oculomotor nerve supplies the medial rectus, inferior rectus
and inferior oblique

post-ganglionic parasympathetic fibres reach the eye via the long ciliary nerves

Your answer was INCORRECT

Explanation

Post-ganglionic parasympathetic fibres reach the eye after leaving the ciliary ganglion via
the short ciliary nerves, to supply the choroid, the sphincter pupillae of the iris and the
ciliary muscle.

A 16-year-old with eczema has several anaphylactic attacks to an unknown trigger.


453-
Which of the following is most true of his illness:

attacks are unlikely to be triggered by inhaled or ingested antigen

histamine antagonists have no role in his management

he is likely to have higher IgE levels than his unaffected sibling

exposure to the triggering antigen is likely to occur several hours before the attack

IgM levels reflect disease activity

Your answer was INCORRECT

Explanation

Anaphylaxis and eczema are examples of IgE-mediated (immediate) type I


hypersensitivity reactions. This type of reaction tends to be triggered by inhaled or
ingested antigen. IgE antibody produced in response to these antigens binds with high
affinity to receptors on mast cells and basophils (sensitisation). Later exposure to the
same allergen (usually at low dose) cross-links these receptors and results in
degranulation of mast cells with release of vasoactive substances - histamine, 5HT,
heparin, prostaglandins and leukotrienes. Histamine is the major effector molecule of the
immediate phase. The reaction occurs rapidly after antigen exposure within 2 to 30
minutes and results in increased vascular permeability and smooth muscle constriction.
As the immediate hypersensitivity reaction subsides (+/- 4-6 hrs later), the late phase
reaction develops. This is characterised by infiltration of inflammatory cells including
eosinophils. Other clinical manifestations of type I hypersensitivity include allergic
rhinitis, bronchial asthma, hay fever and food allergies. Atopic individuals tends to have
higher levels of circulating IgE and are therefore prone to this type of allergic reaction.
Histamine antagonists are useful to counteract mast cell released histamine.

Score: 24 %
Question 209 Part 3 – Tutor Mode
The retinal pigment epithelial cells are connected by:

desmosomes

gap junctions

tight junctions

hemidesmosomes

Your answer was INCORRECT

Explanation

The retinal pigment epithelial are connected by tight junctions which play a vital role in
maintaining the integrity of the blood-retina barrier.

455-The
inferior branch of the oculomotor nerve supplies the following structures
EXCEPT:

medial rectus

dilator pupillae of the iris

inferior oblique muscle

ciliary body
Your answer was INCORRECT

Explanation

The inferior branch of the oculomotor nerve, via its parasympathetic fibres, supplies the
ciliary muscle and the sphincter pupillae muscle of the iris. It does not, however, supply
the dilator pupillae, which is supplied by sympathetic fibres travelling via the nasociliary
nerve. The inferior branch also provides motor innervation to the inferior rectus, medial
rectus and inferior oblique.

456-All of the following are TRUE of the autonomic nervous system EXCEPT:

botulin toxin blocks acetylcholine receptors

preganglionic sympathetic fibres are usually shorter than preganglionic parasympathetic

acetylcholine is the neurotransmitter at both sympathetic and ganglia

pre-ganglionic sympathetic cell bodies are found in the lateral horn of the spinal chord

Your answer was CORRECT

Explanation

Botulin toxin inhibits the exocytosis of acetylcholine in the preganglionic fibers. The
other options are true.

457-The largest cranial nerve is:

the oculomotor

the vagus

the facial

the trigeminal

Your answer was INCORRECT

Explanation
The trigeminal nerve is the largest cranial nerve leaving the brainstem, as it supplies
sensation to the entire face.

458-The Bruch's membrane is also known as:

the lamina fusca

the lamina densa

the lamina cribrosa

the lamina vitrea

Your answer was INCORRECT

Explanation

Bruch's membrane is otherwise known as the lamina vitrea. The lamina fusca is the outer
layer of the sclera. The lamina cribrosa transmits the optic nerve. The lamina densa is
part of the basement membrane of the corneal epithelium.

459-Graves' disease is a manifestation of what type of hypersensitivity?

Type IV

Type V

Type III

Type II

Type I

Your answer was INCORRECT

Explanation

Graves' disease and myasthenia gravis are examples of Type V hypersensitivity. In these
conditions, antibodies react with cell surface receptors and either stimulate or depress
cellular function.
460-Which statement is FALSE about the corneal epithelium:

it is 50-60 micrometers thick

its anterior surface has numerous microvilli

it is a stratified non-keratinized squamous epithelium

epithelial cells are held together by numerous hemi-desmosomes

Your answer was INCORRECT

Explanation

Corneal epithelial cells are held together by numerous desmosomes. Hemi-desmosomes


anchor the epithelial cells to the underlying basal lamina. Other options are true.

461-Regarding X-linked recessive disease:

the fathers pass the abnormal gene to their sons

the maternal age is a factor in passing the abnormal gene to the offsprings

the mother is always the carrier

female offspring of a carrier female have a 50:50 chance of being a carrier

Your answer was INCORRECT

Explanation

X-linked recessive disease is always manifested phenotypically in the male due to the
absence of a paired X-chromosome. However, some females may also exhibit the signs
and symptoms of the disease despite being a carrier (variable penetrance). This may
occur if the normal X-chromosome is inactivated. The mother is not always the carrier

462-Which statement is INCORRECT regarding the wave theory of light:

light waves are coherent if they have the same frequency and amplitude
destructive interference contributes to the clarity of the cornea

destructive interference results from the interaction of light waves of opposite phase

wavelength of light is inversely proportional to its frequency

low reflection coatings depend upon the property of destructive interference

Your answer was CORRECT

Explanation

Light waves are coherent if they are in phase with one another. All other options are true.

Score: 24 %
Question 218 Part 3 – Tutor Mode
Which one of the following cytokines is commonly secreted by the T-helper 2 cell:

IL-2

IFN-gamma

IL-4

TNF-alpha

IL-1

Your answer was INCORRECT

Explanation

TH1 cells commonly secrete IFN gamma and IL2, leading to B cell, natural killer and
macrophage activation. TH2 cells secrete IL3, 4,5 and 6, leading to mast cells and
eosinophil activation.

464-All of the following receive blood draining from the cavernous sinus EXCEPT:

sigmoid sinus

superior petrosal sinus

superficial middle cerebral vein


inferior petrosal sinus

Your answer was INCORRECT

Explanation

The superficial middle cerebral vein drains into the cavernous sinus. Other options
supplied are true.

465-Which is FALSE regarding bifocal glasses:

the optical centre lies midway between the distance and reading segments

decentration of the reading segment can be used to treat diplopia for near

the executive type may be used in the treatment of accommodative spasm

they are used in the management of convergence excess esotropia

Your answer was CORRECT

Explanation

The distance and reading segments of bifocals have separate optical centres - so the
statement is false. All other statements are true.

466-The variable regions of the immunoglobulin molecule correspond to:

the carboxy termini of the light chains

the amino termini of both the light and heavy chains

the amino termini of the light chains

the carboxy termini of both the light and heavy chains

Your answer was INCORRECT

Explanation
The variable regions of the immunoglobulin molecule are the amino termini of both the
light and heavy chains.

467-Zonuluae occludantes occur:

between retinal pigment epithelial cells

in the choriocapillaris

between corneal epithelial cells and the Descemet's membrane

between pigmented ciliary epithelial cells

Your answer was CORRECT

Explanation

Zonulae occludantes refer to tight junctions between cells. Tight junctions occur between
retinal pigment epithelial cells of the retina which is vital to the integrity of the blood-
retinal barrier. They also occur between non-pigmented ciliary epithelium which is
important in the blood aqueous barrier.

468-All of the statements below are true about chromatic aberration EXCEPT:

the degree of chromatic aberration in the human eye is 2-3 dioptres

blue light is more deviated by refraction than red

it cannot be corrected by optical means

perspex produces greater dispersion of light than crown glass

it is the basis of the duochrome test

Your answer was INCORRECT

Explanation

Achromatic lenses can overcome chromatic aberration

469-Which is the FALSE statement about mitochondrial DNA:


a defect in mtDNA can give rise to Leber's hereditary optic atrophy

a defect in mtDNA can cause myoclonic epilepsy and ragged red fibres (MERRF)

the genome lacks introns

it is inherited exclusively through the paternal line

it is heterogeneous from cell to cell in the same individual

Your answer was INCORRECT

Explanation

Mitochondria possess genomes that are distinct from nuclear DNA. Mitochondrial DNA
is composed of a circular loop of double-stranded DNA. Spermatozoa contain abundant
mitochondria but these are confined to the tail region and do not penetrate the ovum on
fertilization. Thus, the mitochondrial genome is inherited from the mother. In mammals,
there are no introns and genes overlap.

The mitochondrial genome encodes 37 genes that produce structural RNA (22 tRNA, and
2 rRNA), and proteins that form part of the oxidative phosphorylation electron transport
chain (complex I, III, IV and V). Complex II consists entirely of nuclear DNA encoded
subunits. Each cell has thousands of mitochondria; therefore, the mtDNA content within
an individual is heterogeneous.

Defects in mtDNA give rise to a heterogeneous group of disorders characterized by the


presence of abnormal muscle fibrils called red ragged fibers. This appearance isdue to the
accumulation of abnormal mitochondria. Mutations of mitochondrial DNA have been
implicated in Leber's hereditary optic atrophy (LHON) - painless subacute visual loss,
with central field defects, abnormal color vision and optic atrophy. Myoclonic epilepsy
and ragged red fibers (MERRF) is characterized by myoclonus, epilepsy and ataxia with
the frequent occurence of dementia, optic atrophy and variable degrees of deafness. Other
diseases include; Leigh disease, Kearns-Sayre syndrome, familial progressive external
opthalmoplegia (PEO), MELAS (mitochondrial myopathy, encephalopathy, lactic
acidosis and stroke-like episodes).

470-Which is TRUE of the back vertex distance:

it is used in determining the power of a reading addition

it is greater in a trial frame than in a phoropter


it is measured from the anterior orbital rim

increased back vertex distance decreases the effective power of a concave lens

Your answer was INCORRECT

Explanation

Back vertex distance is measured from the anterior corneal surface. It is greater in a
phoropter than in a trial frame. The back vertex distance is not used in determining the
power of the reading addition.

471-Which is FALSE regarding Giant cell arteritis:

biopsy reveals patchy vessel involvement

there is often obliteration of the artery lumen

Giant cells are found mostly in the internal elastic lamina

it can affect the central retinal arteries and retinal arterioles

Your answer was INCORRECT

Explanation

Giant cell arteritis can affect the cerebral arteries, the ophthalmic artery and the central
retinal artery but it will not affect retinal arterioles because they do not possess an elastic
layer.

472-All of the pairs below are correctly matched EXCEPT:

Type IV hypersensitivity: giant cell arteritis

Type II hypersensitivity: Arthus reaction

Type IV hypersensitivity: contact dermatitis

Type I hypersensitivity: angioedema


Your answer was INCORRECT

Explanation

All are correctly matched except Arthus reaction is a Type III (not Type II)
hypersensitivity.

 Type I hypersensitivity is provoked by re-exposure to a specific antigen. These


exposures may be by ingestion, inhalation, injection, or direct contact. It is
mediated by IgE antibodies and produced by the immediate release of histamine,
arachidonate and derivatives of basophils and mast cells. The reaction may be
local or systemic.

 In Type II hypersensitivity, the antibodies produced by the immune response bind


to antigens on the patient's own cell surfaces. IgG and IgM antibodies bind to
these antigens to form complexes that activate the classical pathway of
complement for eliminating cells presenting with foreign antigen. The reaction
takes hours to a day. Hemolytic anemia is an example.

 In Type III hypersensitivity, soluble immune complexes form in the blood and are
deposited in various tissues where they may trigger an immune response
according to the classical pathway of complement activation. The reaction takes
hours to days to develop. Serum sickness and Arthus reaction are classical
examples.

 Type 4 hypersensitivity is often called delayed type as it takes two to three days to
develop. Unlike the other hypersensitivities which are antibody mediated it is a
type of cell-mediated response.

473-Which of the following statements regarding complement is true:

the classical complement pathway can be activated by IgG and IgM

the only function of complement activation is the lysis of pathogens

the alternative pathway involves activation by lectin

compliment proteins are produced by most epithelial cells

Your answer was CORRECT

Explanation
Complement proteins are synthesised by macrophages and hepatocytes. There are 9
plasma proteins (heat labile) circulating in an inactive form.

There are three pathways of complement activation: the classical and alternative
complement pathway and the lectin pathway. All share a common terminal set of events
which lead to the formation of the membrane attack complex which lyses pathogens.

 The classical pathway is activated by immune complexes or by antibody bound to


antigen on e.g., the bacterial cell wall. IgM (most efficiently), IgG1, IgG2, IgG3,
but not IgA can activate the classical pathway.

 The alternative complement pathway is activated in the absence of antibody by


bacterial cell wall constituents e.g., LPS, techoic acid, some viruses, dextran, and
occasionally IgA.

 The lectin pathway is initiated by binding of serum lectin to mannose-containing


proteins or to CHO on the bacteria or viruses.

The membrane attack complex (MAC) formed by complement activation is capable of


lysing a broad spectrum of microorganisms - particularly enveloped viruses and gram-
negative bacteria. However, gram-positive bacteria are generally resistant to
complement-mediated lysis due to the thick layer of peptidoglycan in their cell wall or
due to capsular sialic acid. Other microorganisms can evade lysis by the MAC by various
mechanisms. Deficiencies in terminal complement components result in infections with
Neisseria spp, but generalised susceptibility to infection is not noted. Therefore, lysis of
organisms by the MAC is only one of the multitude of effects of the complement
pathway:

 its most important function is performed by intermediates which facilitate the


uptake and destruction of pathogens and immune complexes. Complement
components (e.g., C3b, C5a and C4b) are recognised by specific receptors on
phagocytes and enhance phagocytosis of the coated pathogen.

 C3a, C4a and C5a act as anaphylatoxins - they induce smooth muscle contraction
and increase vascular permeability.

 C3a and C5a mediate degranulation of mast cells with histamine release and
vasodilatation resulting in the recruitment of antibody, complement and
phagocytic cells to the site of infection.

 Complement plays an important part in viral neutralisation by coating the virus


and blocking their attachment to cellular receptors.

 The complement system also promotes the phagocytic clearance of immune


complexes.
 Complement derivatives promote humoral immunity by aiding in antigen
presentation to B cells in germinal centres, and by binding to a receptor on B cells
which acts as a co-stimulator.

474-Which type of collagen is important in the formation of anchoring fibrils connecting


the corneal epithelium to the underlying Bowman's layer:

Type VII

Type V

Type II

Type I

Your answer was CORRECT

Explanation

Type VII collagen forms an integral part of hemidesmosomes and the anchoring fibril
architecture of the corneal epithelium to the Bowman's.

475-All are true of the oculomotor nerve parasympathetic fibres EXCEPT:

they follow the inferior division of the third nerve after its bifurcation in the cavernous
sinus

they synapse in the ciliary ganglion

they originate from the Edinger-Westphal nuclei in the midbrain

they are transmitted via the long ciliary nerve to the iris sphincter

Your answer was INCORRECT

Explanation

The parasympathetic fibres are transmitted via the short ciliary nerve to the iris sphincter.

476-Whichof the following processes most commonly leads to Duchenne, rather than
Becker, Muscular Dystrophy?
lyonization

chromosomal non-disjunction

out of frame mutation

DNA mismatch repair

X-autosome translocation

Your answer was INCORRECT

Explanation

Muscular dystrophy is caused by an abnormality of the dystrophin gene and its protein
product. If an out-of-frame mutation occurs, the gene will not be transcribed properly and
the dystrophin protein will not be produced at all. This leads to Duchenne Muscular
Dystrophy (DMD), with its severe symptoms and limited lifespan. In contrast, Becker
Muscular Dystrophy is not as severe. This is because the mutation in the gene in this
disease is in-frame, in other words the C and N-terminus binding regions are preserved, it
is only the middle part of the protein that is produced wrongly. This leads to less
limitation of function and a milder form of the disease compatible with a normal lifespan
and prolonged ambulation. X-autosome translocation and lyonization explain the
occasional appearance of this disease in females.

477-The left recurrent laryngeal nerve:

supplies the cricothyroid muscle

arises from the vagus nerve as it crosses the first part of the subclavian artery

is a pure motor nerve

courses superiorly in the groove between the esophagus and trachea

Your answer was INCORRECT

Explanation

The left recurrent laryngeal nerve arises from the vagus nerve as it crosses the arch of the
aorta. It ascends in the groove between the trachea and esophagus and lies anterior to the
sympathetic trunk. It also has a sensory component that supplies part of the mucous
membrane of the larynx and trachea. It supplies all the laryngeal muscles except the
cricothyroid muscle which is supplied by the external laryngeal nerve.

478-All statements are true of the abducens nerve EXCEPT:

it courses upwards in the pontine cistern

it is bound down to the brainstem close to its origin by the anterior superior
cerebellar artery

it enters the orbit through the superior orbital fissure and within the tendinous ring

it changes from a vertical to horizontal course on reaching the apex of the petrous
temporal bone

Your answer was INCORRECT

Explanation

The abducens nerve is bound down to the brainstem close to its origin by the anterior
inferior cerebellar artery

479-Which of the following is FALSE regarding the identification of lenses:

measurement of plastic lenses with a Geneva lens measure requires a correction factor

it is best to place a neutralising lens against the back surface of the spectacle lens

prismatic power is best measured with a focimeter

the Geneva lens measure determines surface curvature

Your answer was INCORRECT

Explanation

Prismatic power cannot be measured with a focimeter.

480-Which statement is TRUE regarding refraction:


a convex spherical curved surface causes parallel light to diverge

the refracting power of a spherical curved surface in air is equal to its refractive
index divided by its radius of curvature

on passing into a medium of greater refractive index, light is deviated away from the
normal to the interface

Snell's law states that the angle of incidence divided by the angle of refraction is equal to
the change in refractive index

Your answer was INCORRECT

Explanation

Light is deviated towards the normal to the interface on passing into a medium of greater
refractive index. Snell's law slates that the sine of the angle of incidence divided by the
sine of the angle of refraction is equal to the change in refractive index. A convex
spherical curved surface causes parallel light to converge.

481-Ina survey of 200 patients, 61 have a slow-rising pulse, 40 of whom have aortic
stenosis on echo. A further 4 also have aortic stenosis on echo, who are not found to have
a slow-rising pulse.

Which of the following is correct?

the specificity of a slow-rising pulse for AS equals 21/156

the positive predictive value of a slow-rising pulse for diagnosing AS equals 40/44

the pretest probability of aortic stenosis is 44/200

we cannot be sure of the prevalence of aortic stenosis in this sample

the sensitivity of a slow-rising pulse for AS equals 40/61

Your answer was INCORRECT

Explanation
For this and similar questions you should plot a standard sensitivity-specificity table and
plot the values. Pretest probability of this test is same as disease prevalence, that is 40 + 4
/ 200 = 44/200 and this is the correct option.

482-The following are true of oncogenes EXCEPT:

gene translocation can transform proto-oncogenes

mutated proto-oncogenes are associated with cancer

proto-ongenes cause abnormal cell division

ras oncogenes are the most commonly observed oncogenes in human tumours

Your answer was INCORRECT

Explanation

Proto-oncogenes are genes that are necessary for normal cell division and proliferation.
Mutations in proto-oncogenes by viral insertion, gene translocation or other means can
produce oncogenes, which are un-regulated. ras oncogenes are the most commonly
observed oncogenes in human tumours.

483-In which part of a cell is the G protein located?

mitochondrion

cell membrane

nucleus

nuclear membrane

cytoplasm

Your answer was INCORRECT

Explanation
G proteins are so-called because they bind the guanine nucleotides GDP and GTP. They
are heterotrimers (i.e., made of three different subunits) associated with the inner surface
of the plasma membrane and transmembrane receptors. These are called G-protein-
coupled receptors (GPCRs).

484-The following are true about inositol triphosphate (IP3) EXCEPT:

it increases the intracellular concentration of calcium

it acts as a second messenger

it causes smooth muscle relaxation

it is increased by activation of a1-adrenoreceptors

Your answer was INCORRECT

Explanation

IP3 is a secondary messenger increased by activation of alpha-adrenoreceptors. It binds


to the endoplasmic reticulum membrane and increases the intracellular calcium
concentration and this in turn results in smooth muscle contraction, increased secretion
from exocrine glands and the force and rate of cardiac contraction.

485-The posteriorly directed fibres of the optic radiation are supplied by:

lateral striate artery

anterior cerebral artery

anterior choroidal artery

posteror choroidal artery

Your answer was CORRECT

Explanation

The posteriorly directed fibres of the optic radiation are supplied by the lateral striate
artery which is a branch of the middle cerebral artery.

486-The image formed by an object outside the focal point F1 of a thin covex lens is:
virtual, erect, magnified, further from the lens than object

virtual, erect, at infinity

real, inverted, outside F2

virtual, erect, diminished, inside F2

Your answer was INCORRECT

Explanation

The image formed by an object outside the focal point F1 of a thin convex lens is real,
inverted and outside F2. We recommend being familiar with drawing ray diagrams by
producing, from the top of any object, 2 lines (a) one passing through the nodal point
undeviated, (b) the other parallel to the principle action passing through the second
principle focus.

487-Which is TRUE about Staphylococcus epidermidis:

the majority are sensitive to penicillin

they are coagulase positive

they are destroyed by povidone iodine

they appear as Gram positive cocci in chains

Your answer was INCORRECT

Explanation

Staphylococcus epidermidis are coagulase negative Gram +ve cocci which occur in
bunches. They are resistant to penicillin. Although they are skin commensals, they can
give rise to infection. They are destroyed by povidone iodine.

488-Which statement is FALSE regarding the lens:

the adult lens measures approximately 10mm in diameter


it is supplied with nourishment from both the aqueous and vitreous humours

the anterior radius of curvature is greater than the posterior curvature

the lens becomes less round with age

Your answer was INCORRECT

Explanation

The lens becomes more round with age. Other options are true.

489-Which of the following is NOT associated with increased risk of tumours:

Turner's syndrome

Type II neurofibromatosis

Down's syndrome

Type I neurofibromatosis

Chromosome 13q deletion

Your answer was CORRECT

Explanation

Turner syndrome is not related to an increased risk of cancer.

Type I neurofibromatosis is associated with meningioma, optic nerve glioma and


phaeochromocytoma.

Type II neurofibromatosis is associated with bilateral acoustic neuroma.

Down's syndrome is associated with an increased risk of leukemia.

Chromosome 13q deletion is associated with the development of retinoblastoma.

490-Which is FALSE regarding Chlamydia trachomatis:

it is an obligate intracellular parasite


immunofluorescence can be used to identify infection

it is highly sensitive to erythromycin

it grows in McCoy culture media

subtypes D to K cause trachoma

Your answer was INCORRECT

Explanation

Subtypes A to C cause trachoma (eye to eye contact) while subtypes D to K cause


paratrachoma (sexually transmitted).

491-The dorsal root of the spinal column transmits:

post-ganglionic sympathetic fibres

pre-ganglionic sympathetic fibres

sensory neurones

motor neurones

Your answer was INCORRECT

Explanation

The dorsal root transmits sensory neurones whose cell bodies reside in the dorsal root
ganglion. Motor neurones are transmitted through the ventral roots, including somatic
(skeletal motor) and visceral (sympathetic and parasympathetic) components.

492-The foramen ovale pierces:

the maxilla

the ethmoid
the lesser wing of sphenoid

the greater wing of sphenoid

Your answer was INCORRECT

Explanation

The foramen ovale pierces the greater wing of sphenoid.

493-Which is FALSE regarding the cross-cylinder:

the handle is mounted at 45 degrees to the axes of the cylinders

the markings on the lens correspond to the axes of power of the cylinders

it can be used to check the power of the trial cylinder prescribed

it can be used to check the axis of the trial cylinder prescribed

Your answer was INCORRECT

Explanation

The markings on a cross-cylinder correspond to the axes of no power of the individual


cylinders. As such, the power of each cylinder lies at 90 degrees to the marked axis,
which corresponds to the marked axis of the other cylinder, of opposite sign.

494-Mean cholesterol level in 300 men and women was measured to be 4.9 and 5.3
mmol/l, respectively.

Which of the following does NOT apply?

the t-test would be a good way of testing whether the difference were due to chance

the alternative hypothesis would state that this difference is real

this difference may be due to chance

a Type I error would occur if the null hypothesis were rejected when it were true
a Type II error would occur if a difference were found where none existed

Your answer was INCORRECT

Explanation

The t-test is used in order to determine whether the means of two normal distributions are
the same or not. A Type I error is a false positive (falsly rejecting the null hypothesis) and
a Type II error is a false negative (falsly retaining the null hypothesis).

495-Which of the following is FALSE regarding sympathetic ophthalmitis:

it can lead to phthisis bulbi

it can occur after traumatic injury to one eye

it is a bilateral granulomatous pan-uveitis

Dalen-Fuchs nodules are an accumulation of macrophages in the retinal neurons

Your answer was INCORRECT

Explanation

Sympathetic ophthalmitis is a bilateral granulomatous pan-uveitis which can occur after


traumatic injury to one eye, usually where uveal tissue is incarcerated in sclera. Dalen-
Fuch's nodules occur which are an accumulation of macrophages in the retinal pigment
epithelium. The condition can lead to exudative retinal detachment secondary to severe
inflammation, with consequent hypotony and phthisis bulbi.

496-Leber's hereditary optic neuropathy is an example of:

mitochondrial inheritance

autosomal dominance inheritance

X-Linked recessive inheritance

autosomal recessive inheritance

Your answer was CORRECT


Explanation

Leber's hereditary optic neuropathy is a mitochondrially inherited condition.

497-Which is FALSE regarding corneal epithelial wound healing:

mitotic activity takes place at the limbus

full ultrastructural integrity is only restored after 2-3 weeks

healing depends on intercellular signaling mediated by matrix components and integrins

epithelial cells initially slide as a sheet of single cells to cover defects

Your answer was INCORRECT

Explanation

While most corneal defects are closed within 24-48 hours, full ultrastructural integrity
with the formation of hemidesmosomes and Type VII collagen anchoring fibrils takes
many more months. This explains the recurrent corneal erosion syndrome in many
patients with a previous epithelial defect.

498-The apices of the retinal pigment epithelium are in closest approximation to:

the inner plexiform layer

the retinal nerve fibre layer

the photoreceptor outer segment

the photoreceptor inner segment

Your answer was INCORRECT

Explanation

The apices of the RPE are in close approximation with the photoreceptor outer segments.

499-The following are true about cholesterol EXCEPT:


cholesterol is the most abundant sterol found in human tissue

it is essential for the production of thyroxine

HMG CoA reductase is the main regulator enzyme of cholesterol synthesis

all the carbon atoms of cholesterol are derived from acetyl CoA

Your answer was INCORRECT

Explanation

Thyroxine is not a steroid hormone (it is a peptide) and therefore it is not formed from
cholesterol. Other options are true.

500-A10 dioptre concave lens is decentred by 30 mm. What is the prismatic effect
produced:

10 dioptres

100 dioptres

300 dioptres

30 dioptres

Your answer was INCORRECT

Explanation

The prismatic power is calculated by the formula:

P=FxD

Where P is the prismatic power in prism diopters, F is the lens power in diopters and D is
the distance from the centre of the lens in centimeters.
501-Which statement is FALSE regarding ciprofloxacin:

it acts on DNA polymerase

it is a fluoroquinolone antibiotic
it is effective against Pseudomonas

it is effective against Staphylococcus

Your answer was CORRECT

Explanation

Ciprofloxacin is a fluoroquinolone antibiotic. It inhibits bacterial DNA synthesis by its


action on bacterial DNA gyrase. It is effective against many species of Pseudomonas,
Staphylococcus and Haemophilus.

502-Choose the CORRECT statement. Phagocytic cells:

release histamine which affects the adaptive immune system

initiate firm attachment by binding to MHC class I on the endothelium

have receptors that recognise viral membrane constituents

require clonal expansion to mount an effective response

are important for the resolution of inflammation

Your answer was INCORRECT

Explanation

The major phagocytic cells of the immune response are macrophages and neutrophils.
Phagocytes of the innate immune system recognise common motifs on invading
pathogens using cell surface pattern recognition receptors. They are part of the innate
immune system and mount a rapid response that does not require clonal expansion. These
cells phagocytose foreign particles and microbes. They are also important for the clearing
of apoptotic cells and debris in the resolution phase.

Phagocytic cells have surface receptors that recognise the Fc portion of immunoglobulin
as well as complement components. Binding of these receptors to their ligands enhances
phagocytosis of antigen; a process termed opsonisation.

Internalised microbes are killed by phagocytes that secrete reactive oxygen intermediates
and possess potent microbicidal properties. These cells initiate and amplify the adaptive
immune response by producing pro-inflammatory cytokines such as IL1, IL6 and TNF.
Phagocytic cells enter tissues by rolling on selectins and binding via integrins to ICAM
(and similar molecules). They transmigrate by binding to PECAM and similar adhesion
molecules. Mast cells release histamine when antigen binds and cross-links cell surface
IgE.

503-Which statement is FALSE about the corneal stroma:

it is composed of 200-250 layers of flattened collagenous lamellae

the main glycosaminoglycans in the cornea are keratin sulphate and chondroitin

the corneal stroma contains blood and lymphatic vessels only in the anterior stromal
layers

keratocytes are stellate in shape

Your answer was INCORRECT

Explanation

The corneal stroma normally contains no blood or lymphatic vessels. Other options are
true.

504-Which is TRUE of contact lenses:

fluorescein should be used to check the fit of soft contact lenses

contact lenses can be used in the management of ptosis

only preservative-free ophthalmic medications should be used in hard lenses

scleral contact lenses are not fenestrated

Your answer was INCORRECT

Explanation

Contact lenses can be used to treat ptosis - a ptosis prop can be made by moulding a ledge
onto a contact lens. Soft contact lenses become impregnated with preservatives but there
is no necessity to use preservative-free for hard lenses. Flourescein stains soft contact
lenses and therefore is not used to check fit. Scleral contact lenses are fenestrated.
505-A 4-year-old boy presents with recurrent abscesses, which respond poorly to
antibiotics.

What cell is most likely to be defective:

Macrophage

Neutrophil

B-cell

T-cell

Eosinophil

Your answer was INCORRECT

Explanation

Neutrophils are granulocytes with multilobed nuclei. They are phagocytes and are an
important cellular components of the innate immune response. They circulate in the
blood stream and only enter tissues if there is inflammation (unlike macrophages) to
which they are principally directed by IL8. These cells possess receptors for
immunoglobulin G and for complement components. They phagocytose opsonised
antigen and release toxic oxygen compounds (respiratory burst), nitric oxide,
phospholipases and proteases from granules to kill internalised organisms. Patients with
neutrophil defects suffer from recurrent chest infections with bacteria or fungi, recurrent
skin abscesses often caused by Staphylococcus aureus and poor wound healing. Primary
neutrophil deficiencies are rare and are usually due to an inherited abnormality of the
neutrophil itself. The problem can affect phagocytosis or the metabolic pathways
involved in intracellular killing

506-Acute inflammation:

is always harmful

causes increased vascular permeability coupled with infiltration of inflammatory


cells

causes the release of C8a, a potent chemokine


causes the release of CRP, the half-life of which is several days

Your answer was INCORRECT

Explanation

The combination of increased vascular permeability and cellular influx gives rise to the
calor, rubor and dolor of the inflammatory process. CRP is an acute phase protein which
is synthesised by hepatocytes as a result of the acute inflammatory process. It is secreted
in increased amounts within 6 hours of an acute inflammatory stimulus. CRP has a half-
life of about 12 hours. C-reactive protein binds to phosphocholine moieties found on
microbial polysaccharides. It acts as an opsonin-enhancing phagocytosis, and also
activates the classical complement pathway. Acute infection results in activation of
complement. C5a is a complement intermediate which acts as a potent chemokine.

507-A letter published in a journal suggests that an established antiepileptic drug may
cause gum hyperplasia. The manufacturer wishes to set up a study to determine rapidly
and efficiently whether this is true.

Which one of following techniques is most appropriate?

double-blind, randomized, placebo controlled study

dose-ranging study

sequential trial

case-control study

meta-analysis

Your answer was INCORRECT

Explanation

An established drug should have many trials published on data. Meta-analysis


incorporates the data in order to identify effects which some trials may be too small to
detect.

508-The maxillary sinus drains into:

the inferior meatus


the middle meatus

the superior meatus

the lateral meatus

Your answer was INCORRECT

Explanation

The maxillary sinus drains via the middle meatus.

509-The image formed by an object inside the focal point F1 of a thin convex lens is:

real, inverted, outside F2

virtual, erect, diminished, inside F2

virtual, erect, at infinity

virtual, erect, magnified, further from the lens than object

Your answer was INCORRECT

Explanation

The image formed by an object inside the focal point F1 of a thin convex lens is virtual,
erect, magnified and further from the lens than the object. We recommend being familiar
with drawing ray diagrams by producing, from the top of any object, 2 lines (a) one
passing through the nodal point undeviated, (b) the other parallel to the principle action
passing through the second principle focus.

510-The parasympathetic nerve supply to the lacrimal gland travels through which of the
following structures:

cavernous sinus

lesser petrosal nerve

otic ganglion
pterygopalatine ganglion

Your answer was INCORRECT

Explanation

The parasympathetic nerve supply to the lacrimal gland involves the superior salivatory
nucleus, the nervus intermedius of the seventh cranial nerve, the greater petrosal nerve,
the pterygopalatine ganglion where it synapses, before travelling to the lacrimal gland.

511-Which is FALSE regarding Staphylococcus aureas:

it is the main commensal of the outer eye

it grows well on blood agar

it produces exotoxin

it is a Gram positive bacteria

it is aerobic and non-motile

Your answer was INCORRECT

Explanation

Staphylococcus produces numerous virulence factors including lipase, proteinase and


hyaluronidase but it does not produce exotoxins.

512-How long is a rod photoreceptor:

12 micrometers

15 micrometers

100 micrometers

50 micrometers

Your answer was INCORRECT


Explanation

A rod photoreceptor is 100-120 micrometers long while a cone is 60-75 micrometers.

513-The principle of diffraction is used in:

anti-glare spectacle coatings

the Placido disc

the stromal cornea to aid clarity

multi-focal intra-ocular lenses

Your answer was INCORRECT

Explanation

Some multi-focal intra-ocular lenses use the principle of diffraction, which is the
interference of waves of light as they encounter an obstruction or opening, in order to
create multi-focality.

514-The Geneva lens measure assesses:

the surface curvature of a lens

the axes of an astigmatic lens

the vertex power of a lens

the power of prism

Your answer was CORRECT

Explanation

The Geneva lens measure assesses the surface curvature of a lens. The total power of the
lens can be determined by the sum of the surface powers. The Geneva lens is calibrated
for Crown glass and requires a correction factor for any other material.

515-A 14-year-old girl presents with high fever, rigors and myalgia. Nasal aspirate is
positive for influenza virions.
Which of the following is true of her B cell response?

deficiency of either CD40 or CD40L still results in the production of IgG

affinity maturation takes place in the blood stream

the antibody response to the virus does not require T cell help

memory cells are not formed as repeated infections with influenza often occur

her B cells express immunoglobulin on their surface

Your answer was INCORRECT

Explanation

B cells usually require T cell help for full activation. B cells are activated in the primary
immune response which initially produce IgM. With continuing T cell help, B cells then
undergo heavy chain class switching and enter the germinal centres in secondary
lymphoid organs. The germinal centres are the sites of immunoglobulin affinity
maturation and memory B cell formation. Various factors including nature of T cell help,
antigen exposure site and cytokine profile determine the isotype of the heavy chain
produced. CD40 and CD40L are required for co-stimulation by T cells. Deficiency of
either CD40 or CD40L impairs class switching. Certain antigens can activate B cells in
the absence of T cell help, called thymus-independent antigen. T cell-independent B cell
responses are mainly due to carbohydrate antigen e.g., pneumococcal polysaccharide.
These antigens are not processed and presented in association with MHC molecules, and
therefore cannot activate T helper cells. Most TI antigens have highly repetitive epitopes
(e.g., LPS/endotoxin), which are able to cross-link the B cell surface immunoglobulin and
activate these cells. Some T cell-independent antigens can cause the proliferation of B
cells regardless of their specificity, polyclonal B cell activation. Antibody responses to T-
independent antigens consist mainly of IgM antibodies of low affinity without the
production of memory cells. The influenza virus will activate T- and B cells, and result in
memory cell production. Genetic mutation in the virus is responsible for immune evasion
and repeated infections.

516-Astatistician wants to compare the significance of HbA1c glucose levels in a group of


100 patients before and after treatment with a new diabetic drug.

What is the most appropriate statistical test?


likelihood ratios

chi square test

Kaplan-Meier curve

paired t test

logistic regression analysis

Your answer was INCORRECT

Explanation

In a clinical trial where the input variable is type of treatment - a nominal variable - and
the outcome may be some clinical measure which is normally distributed, the required
test is then the T-test. The paired t-test and the Wilcoxon signed-rank test are often
conducted to compare two continuous outcomes from paired observations. An
assumption underlying these tests is that the responses from pair to pair are mutually
independent.

517-Which statement is FALSE regarding the lacrimal gland:

the orbital portion is larger than the palpebral portion

destruction of the palpebral portion removes 50-70% of tear production

the orbital portion is lodged in the lacrimal fossa

the orbital and palpebral portions are connected via an isthmus

Your answer was INCORRECT

Explanation

The lacrimal gland is composed of two portions: a larger orbital portion and a smaller
palpebral portion (about one-quarter the size). The three to five main excretory ducts of
the orbital portion must traverse the palpebral portion, which contributes a further five to
seven ducts to the total of about 12 excretory ducts. As all ducts pass through it,
destruction of the palpebral portion renders the entire gland non-functional.

518-In
a survey of 100 men admitted to a hospital 30 have disordered liver function tests
(LFTs), 21 of which have palmar erythema. Six of those without disordered LFTs are
also judged to have palmar erythema.

Which of the following statements is correct?

we cannot be sure of the pre-test probability of liver disease in this population

the predictive value of a positive test will be the same across all populations

palmar erythema has a specificity of 64/70 as a marker of liver disease

the sensitivity of palmar erythema as a marker for liver disease is 21/27

if a patient has palmar erythema, the probability that he has liver disease is equal to 21/30

Your answer was INCORRECT

Explanation

True positives: 21, false positives: 6, true negatives: 64; false negatives:9. Sensitivity of
palmer erythema would be 21/(6+64); the probability of liver disease with palmer
erythema would be 21/(21+6).

519-All of the following are TRUE about the chemoreceptor in the carotid body EXCEPT:

it is involved in reflex peripheral vasoconstriction

it is stimulated by a decrease in PO2 of arterial blood

it is stimulated by an increase in PCO2

it is inhibited by a decrease in pH of arterial blood

Your answer was INCORRECT

Explanation

The carotid body is stimulated by increase in PCO2, decrease in PO2 and pH. When
stimulated it causes reflex peripheral vasoconstriction.

520-Which is TRUE of acid-base balance in the body:


increased hydrogen ions concentration inhibits respiration

increased bicarbonate ions concentration increases respiration

the normal venous plasma is less acidic than the arterial plasma

the anion gap is increased in ketoacidosis

Your answer was INCORRECT

Explanation

The normal arterial pH is 7.4 and that of the venous is slightly lower. Acidosis (increased
hydrogen ions concentration) increases respiration and alkalosis (increased bicarbonate
ions concentration) decreases respiration. The anion gap is defined as the difference
between the concentration of cations other than sodium and the concentration of anions
other than chloride and bicarbonate ions. It is increased in ketoacidosis and lactic
acidosis.

521-Which statement is FALSE about the foramena:

the jugular foramen transmit the vagus nerve

the foramen spinosum transmits the accessory meningeal artery.

the foramen ovale transmits the mandibular nerve

the stylomastoid foramen transmit the facial nerve

Your answer was INCORRECT

Explanation

The foramen spinosum transmits the middle meningeal artery. The accessory meningeal
travels through the foramen ovale. Other statements are true.

522-All of the following muscles have their origin in the tendinous ring EXCEPT:

superior oblique
lateral rectus

inferior rectus

medial rectus

Your answer was CORRECT

Explanation

The superior oblique originates superomedial to the optic canal. The inferior oblique
originates behind the orital margin lateral to the nasolacrimal duct. All other extraocular
muscles (that is all four recti) originate from the tendinous ring.

523-Uveal tissue is most strongly attached to the sclera at:

the scleral spur

the optic disc

the macula

the ora serrata

Your answer was INCORRECT

Explanation

Uveal tissue is most strongly attached to the sclera at the optic disc.

524-Choose the one correct statement. Regarding antibodies:

IgE can bind to receptors on mast cells, basophils and eosinophils

a specific B-cell always produces the same class of antibody

the IgG subclasses are not transported across the placenta

IgG3 and IgG1 are the most effective complement-fixing antibodies

IgM and IgA are largely confined to the blood


Your answer was CORRECT

Explanation

An immunoglobulin molecule is composed of two identical heavy chains and two


identical light chains. Both heavy and light chains are made up of a variable domain (the
amino acid sequence of the domain varies among different antibodies) attached to one
constant domain (light chain) or many constant domains (in the case of the heavy chain).
The variable domain of antibodies results from the rearrangement of the gene segments
encoding this domain. Light chains are the products of successful rearrangements at
either the kappa or lambda gene loci. Heavy chains are derived from a successful variable
gene rearrangement spliced to one of a number of constant-region genes. The choice of
constant-region gene determines the class or isotype of the immunoglobulin. There are
important functional and structural differences between the isotypes. The primary
immune response results in the production of IgM. Subsequent activation of antibody
results in class switching - the B-cell which originally produced IgM switches to the
production of antibody with the same specificity, but of a different isotype.

There are five main isotypes of immunoglobulin in humans: IgM, IgG, IgA, IgE, and
IgD. IgG antibodies have 4 subclasses-IgG1 to 4, and IgA has two subclasses-IgA1 and
IgA2.

Characteristics of the different immunoglobulon isotypes:

 IgG (80% of total immunoglobulin): activates the classical complement pathway;


transferred across the placenta; monomeric; important immunoglobulin in the
secondary immune response.

 IgM (6% of total Ig): activates the classical complement pathway +++; IgM fixes
complement most effectively; pentameric structure; present on the membranes of
B cells; mostly confined to the intravascular space; important in the primary
immune response.

 IgA (10-15 % of total serum Ig): is the predominant immunoglobulin in the


external secretions; activates the alternative complement pathway; secretory IgA
dimeric form with J chain; occasionally exists as a tetramer; IgA is very important
for mucosal immunity.

 IgD (<1% of total serum Ig): B cell surface Ig; possible role in B/T cell activation.

 IgE (0.001% of total serum Ig): binds with high affinity to mast cells and are
involved in active immunity against helminth parasites.

525-Which statement is FALSE with respect to lens development:


the foetal lens is more spherical than the adult

the lens first appears as vesicles with a single layer of epithelial cells

the lens is formed from neural crest cells

the lens first appears at 27 days of gestation

Your answer was INCORRECT

Explanation

The lens arises from the ectoderm. Other statements are true.

526-Reis-Buckler dystrophy affects which layer of the cornea:

Descemet's membrane

corneal epithelium

Bowman's layer

corneal endothelium

stroma

Your answer was INCORRECT

Explanation

Reis-Buckler dystrophy is an autosomal dominant dystrophy affecting the Bowman's


layer. Clinically there are fine reticular opacities in the superficial cornea in early adult
life.

527-Which is FALSE regarding the skin tumours:

squamous cell carcinoma is more common in the immunocompromised

basal cell carcinoma is more common than squamous cell carcinoma


sebaceous gland carcinoma has a poorer prognosis than squamous cell carcinoma

squamous cell carcinoma spreads haemotogenously

Your answer was INCORRECT

Explanation

Squamous cell carcinoma spreads by the lymphatics. Other statements are true.

528-The image formed by a prism:

is always undeviated

is always deviated towards the apex

is always deviated towards the base

may deviate towards the apex or towards the base dependent on the angle of incidence

Your answer was INCORRECT

Explanation

The image formed by a prism is erect, virtual and displaced towards the apex.

529-Which of the following is FALSE regarding the complement system:

C5b-9 attacks the cell membrane

activation causes recruitment of inflammatory cells

the classic pathway is activated by the cell wall of bacteria

deposition of C3b on the bacterial cell wall causes phagocytosis by leucocytes

Your answer was INCORRECT

Explanation

The complement system is a complex system of serum proteins that interact in a cascade.
There are two pathways by which complement activation is initiated
530-All of the following are caused by defects in mitochondrial DNA except:

Leber's hereditary optic neuropathy

Kearn-Sayer's syndrome

Leigh disease

Leber's congenital amaurosis

Your answer was INCORRECT

Explanation

Leber's congenital amaurosis is an autosomal recessive condition.

531-All of the following are muscles of facial expression EXCEPT:

nasalis

levator labii superioris

buccinator

zygomaticus major

Your answer was INCORRECT

Explanation

The muscles of facial expression are innervated by the facial nerve and include:

 procerus
 frontalis
 corrugator supercilii
 orbicularis oculi
 zygomaticus major and minor
 levator labii
 nasalis
 depressor supercilii
 occipitofrontalis
The buccinator is a muscle of mastication.

532-Which statement about the sella turcica is FALSE:

it is positioned at the central part of the ethmoid body

the anterior and posterior clinoid processes give attachment to the tentorium cerebelli

it occurs between the two cavernous sinuses

the central hollow of the sella turcica is called the hypophyseal fossa

Your answer was CORRECT

Explanation

The sella turcica is located at the central part of the sphenoid body. Other options are
true.

533-Hyalinization of vessel walls is associated with all EXCEPT:

diabetes

hyperlipidaemia

ageing

hypertension

Your answer was INCORRECT

Explanation

Hyalinization of vessel walls occurs when the walls become thickened due to deposition
of collagen. It occurs with ageing, diabetes and hypertension. Hyperlipidaemia
predisposes to atheroma but it is not a cause of hyalinization.

534-All of the following are features of hypertensive retinopathy EXCEPT:

hyalinization
retinal hemorrhages

lymphocytic infiltration of vessel walls

fibrinoid necrosis

hard exudation

Your answer was INCORRECT

Explanation

Hypertensive retinopathy results in hyalinization of blood vessel walls. Further vessel


narrowing and ischaemia lead to retinal hemorrhages, fibrinoid necrosis of the vessels
with exudation and papilloedema. Lymphocytic infiltration is not a feature of
hypertensive retinopathy.

535-A Jackson cross-cylinder with axis markings of +0.50 and -0.50 is named a:

-0.50 D cross-cylinder

plano cross-cylinder

+1.00 D cross-cylinder

+0.50 D cross-cylinder

Your answer was INCORRECT

Explanation

The cross-cylinder is named by the power of the cylinder in the toric formula. A lens with
markings of +0.50D and -0.50D has a toric equation of: -0.50DS/+1.0DC. Thus, the
cylinder is named a 1.00D cross-cylinder, which is marked on the handle.

536-A local anaesthetic would be expected to penetrate tissues best:

when administered with a vasodilator

when the solution is warmed


when in an alkaline solution

when the tissue is inflamed

Your answer was INCORRECT

Explanation

Local anaesthetics are weak bases and penetrate tissues best when they are in an alkaline
solution where a greater proportion is non-ionised (lipophilic). Anaesthetics penetrate
inflamed tissue poorly because it is an acidic environment. Warming an anaesthetic
reduces the pain involved in instilling it but does not affect its penetrating ability.
Anaesthetics are often administered together with vasoconstrictors to retard vascular
absorption.

537-Which statement is FALSE regarding the lens:

it contains lens epithelium under the anterior but not the posterior lens capsule

it contains an anterior inverted Y-shaped suture and a posterior Y-shaped suture

its anterior lens capsule is thicker than its posterior lens capsule

it has a lower refractive power than the cornea

Your answer was INCORRECT

Explanation

The anterior Y-shaped suture of the lens is upright and the posterior oneis inverted. Other
options are true.

538-Which corneal dystrophy is a localized form of mucopolysaccharidosis:

Lattice dystrophy

Macular dystrophy

Meesman's dystrophy

Granular dystrophy
Avellino dystrophy

Your answer was INCORRECT

Explanation

Macular dystrophy is an autosomal recessive corneal stromal dystrophy. It is a localised


form of mucopolysaccharidosis with mucopolysaccharide granules in the cytoplasm of
keratocytes.

539-One of the following statements regarding interferons is incorrect:

IFN-beta is of therapeutic benefit in multiple sclerosis

IFN-alpha and IFN-beta have an anti-viral action

IFN-gamma is produced by T lymphocytes and NK cells in response to viral infection

IFN-gamma upregulates the expression of MHC class I and II molecule

All the interferons are structurally similar and bind to the same receptor on the
surface of target cells

Your answer was INCORRECT

Explanation

Interferons are cytokines. There are three types of interferons - IFN-alpha is produced
primarily by mononuclear phagocytes, INF-beta is produced by fibroblasts and
interferon-gamma is produced by T cells and NK cells. IFN-alpha and IFN-beta are
termed type I interferons. The type I interferons bind to the same cell surface receptor.

The interferons are anti-viral proteins produced by cells in response to viral infection.
Their functions include:

 inhibition of protein synthesis and DNA replication in virus-infected cells


 increase in MHC class I expression and antigen presentation in all cells
 activation of NK cells to kill virus-infected cells.

Interferon alpha has been shown to inhibit cell proliferation in vitro. This cytokine is an
endogenous pyrogen and is one of the causes of fever and myalgia in infection.
Indications for alpha-IFN therapy include hairy cell leukemia (90% response rate), renal
cell carcinoma, chronic active hepatitis B or C, juvenile laryngeal papillomatosis (HPV
induced) and Kaposi's sarcoma in AIDS.

Gamma-IFN acts as an immune regulator and augments phagocytic activity; it has its
own receptor. It is used clinically in patients with chronic granulomatous disease,
although its precise mechanism of action is unclear. It may increase production of
superoxide ion in neutrophils in these patients.

IFN-beta is used in the treatment of patients with multiple sclerosis.

540-The optic canal transmits all of the following EXCEPT:

post-ganglionic parasympathetic fibres

the ophthalmic artery

the optic nerve

meningeal coverings

Your answer was CORRECT

Explanation

The optic canal transmits sympathetic nerves in association with the ophthalmic artery
but does not transmit parasympathetic fibres. All other options are true.

541-Following major surgery, the body shows ALL of the following responses EXCEPT:

decreased lipolysis

decreased peripheral glucose uptake

decreased protein synthesis

increased protein breakdown

Your answer was CORRECT

Explanation
The body following surgery initiates the stress response under cortisol and catecholamine
control resulting in: increased protein breakdown, decreased protein synthesis, increased
lipolysis, increased glycogenolysis and reduced peripheral glucose uptake.

542-Structures visible on gonioscopy from anterior to posterior are:

Schwalbe's line, pigmented trabecular meshwork, non-pigmented trabecular meshwork,


scleral spur, ciliary body

Schwalbe's, pigmented trabecular meshwork, non-pigmented trabecular meshwork,


ciliary body, scleral spur

Schwalbe's line, non-pigmented trabecular meshwork, pigmented trabecular meshwork,


ciliary body, scleral spur,

Schwalbe's line, non-pigmented trabecular meshwork, pigmented trabecular


meshwork, scleral spur, ciliary body

Your answer was INCORRECT

Explanation

The order of visible structures on gonioscopy from anterior to posterior are: Schwalbe's
line, non-pigmented trabecular meshwork, pigmented trabecular meshwork, scleral spur,
ciliary body

543-During wound healing all are true EXCEPT:

macrophages are common features at the late stage of healing

healing by first intention occurs in a clean and closely approximated wound

the tensile strength of the scar is determined by the amount of collagen in the wound

granulation tissue is the first phase of healing

Your answer was CORRECT

Explanation

All statements are true except that giant cells, macrophages and lymphocytes are features
of chronic inflammation and are not common features in wound healing.
544-The maxillary division of the trigeminal nerve is a derivative of:

the second pharyngeal arch

the fourth pharyngeal arch

the first pharyngeal arch

the third pharyngeal arch

Your answer was INCORRECT

Explanation

The maxillary and mandibular branches of the trigeminal nerve are derivatives of the
superior and inferior components of the first pharyngeal arch respectively.

545-Venous drainage of the choroid occurs via:

the posterior choroidal vein

the vortex veins

the anterior choroidal vein

the central retinal vein

Your answer was INCORRECT

Explanation

The venous drainage of the choroid occurs via the large vortex veins. There are 4-6
vortex veins in the eye which pierce the sclera via emissary canals.

546-When there is unilateral damage to the cervical sympathetic ganglia, ocular findings
include all of the following EXCEPT:

failure of the pupil to react to light

paralysis of Muller 's muscle


low intraocular pressure

updrawing of the lower eyelid

Your answer was CORRECT

Explanation

Horner 's syndrome causes miosis but the pupil can still react to light which is mediated
by the parasympathetic autonomic nervous system.

547-All of the following are features of Horner's syndrome EXCEPT:

conjunctival injection

a small pupil that does not react to light

up-drawing of the lower eyelid

partial ptosis that disappears with topical phenylephrine

decreased sweating of the skin around the eye

Your answer was INCORRECT

Explanation

Horner's syndrome occurs when there is interruption of the cervical sympathetic neurons
that supply the orbit. Features of Horner's syndrome include: miosed eye that reacts
normally to light, partial ptosis that disappears with topical phenylephrine, conjunctival
injection due to loss of the sympathetic tone of the blood vessels, up drawing of the lower
eyelid with resultant apparent enophthalmos and decreased sweating of the face.

548-The chromosomes line up at the equator of the cell at which phase of mitosis:

interphase

metaphase

prophase
anaphase

Your answer was INCORRECT

Explanation

The stages of mitosis are:

 interphase: chromosomes prepare to divide


 prophase: the chromosomes condense
 metaphase: the chromosomes line up on the equator of the cell
 anaphase: the chromosomes begin to separate
 telophase: chromosomes have segregated to the poles and the nuclear membrane
reforms

549-The subretinal space occurs between:

the internal limiting membrane and the neural retina

the RPE and its basement membrane

the RPE and the neural retina

the RPE basement membrane and Bruch's membrane

Your answer was INCORRECT

Explanation

The subretinal space is a potential space between the RPE and the neural retina which
only becomes evident in pathologic states, for example retinal detachment.

550-The retina:

contains an optic disc which is lateral to the macula

has more cones than rods

receives all its blood supply from the central retinal artery
has cones and rods on its inner surface

Your answer was INCORRECT

Explanation

There are 20 times more rods than cones. Both cone and rod photoreceptors lie on the
outer surface of the retina. The optic disc is medial to the macula. The central retinal
artery is the only artery that supplies the retina.

551-Acyclovir:

is active against all viruses within the herpes family

inhibits DNA polymerase

should be dosed twice daily as maximum

is more effective against herpes simplex keratitis if given orally than topically

Your answer was INCORRECT

Explanation

Acyclovir acts by inhibiting DNA polymerase. It is not effective against


cytomegalovirus, a member of the herpes family. Both topical and oral acyclovir are
effective against herpes simplex keratitis. It is usually dosed 5x per day

552-Sorbitol is produced from glucose by the action of:

catalase

aldose reductase

pyruvate kinase

glutathione reductase

Your answer was INCORRECT

Explanation
In states of excess glucose, as occurs in diabetes, glucose is diverted by aldose reductase
to the production of sorbitol. Sorbitol cannot be transported out of cells easily, thereby
disrupting the osmotic and metabolic balance of the cell. It is believed that the sorbitol
pathway plays an important role in the development of cataract by its effect on lens fibre
cells. Indeed, aldose reductase inhibitors have been shown to prevent the development of
cataract.

553-The principle supporting glial cells of the retina are:

the Muller cells

the amacrine cells

the microglia

the astrocytes

Your answer was CORRECT

Explanation

The principle retinal neuroglial cells are as follows:

 The Muller cells: are the principle supporting glial cells of the retina and are
analogous to central nervous system oligodendrocytes.

 The astrocytes: form a honeycomb scaffold perpendicular to the Muller cells.


They are responsible among other functions for laying down scar tissue in injured
or diseased retina.

 The microglia: are highly specialized mononuclear phagocytic cells

Note: amacrine cells are an association neurone involved in signal modulation.

554-The corneal reflex involves all of the following EXCEPT:

the ophthalmic nerve

the short ciliary nerve

the facial nerve


the nasociliary nerve

Your answer was INCORRECT

Explanation

The corneal reflex refers to the reflex blink when the cornea is touched. The afferent
pathway of the corneal reflex involves: the long ciliary nerve (not the short ciliary nerve),
the nasociliary nerve, the ophthalmic nerve and the trigeminal nerve. The first synapse is
in the spinal nucleus of the trigeminal nerve and the second synapse is in the facial
nucleus. The efferent stimulus travels along the facial nerve causing contraction of the
orbicularis oculi resulting in a blink response.

555-The volume of the globe is:

15ml

6.5ml

3 ml

12ml

Your answer was INCORRECT

Explanation

The volume of the eye is 6.5ml.

556-Which statement is FALSE regarding the corneal endothelium:

it is a stratified squamous epithelium

its basement membrane is the Descemet's membrane

it has a critical role in maintaining the state of corneal hydration

the cells are polygonal in shape

Your answer was CORRECT

Explanation
The corneal endothelium is a simple squamous epithelium. Other options are true

557-All statements below are true of the trochlear nerve EXCEPT:

its nucleus lies in the anterior part of the periaqueductal grey matter in the inferior
colliculus

its fibres decussate immediately after leaving the brainstem

it emerges from the posterior aspect of the brain stem

it supplies only somatic efferent innervation to the superior oblique

Your answer was INCORRECT

Explanation

The trochlear nerve is unusual in that it decussates before leaving the brainstem. It is also
unusual in that it is the only somatic efferent nerve to emerge from the posterior aspect of
the central nervous system.

558-Which statement is true of the aqueous humour:

its rate of formation is about 2.5ul/minute

its glucose levels are higher than that of the plasma

it has a lower lactic acid concentration than in the plasma

it contains the same concentration of protein as in the plasma

Your answer was CORRECT

Explanation

The aqueous humour is produced at between 2 to 3 ul/minute. It has a very low protein
concentration compared with the blood. It has a higher lactic acid concentration and
slightly lower glucose. The ascorbate concentration is between 10 to 50 times that of the
plasma.

559-A34-year-old smoker presents with glomerulonephritis and pulmonary haemorrhage.


Blood tests reveal anti glomerular basement membrane antibodies.
Which type of hypersensitivity reaction is likely to be causing the tissue damage:

Type I

Type III

Type III and IV

Type IV

Type II

Your answer was INCORRECT

Explanation

The patient has Goodpastures Syndrome - a clinical manifestation of a type II


hypersensitivity reaction. Autoantibodies are produced against the glomerular basement
membrane and the alveolus. Anti-GBM disease commonly presents with haematuria,
nephritic urinary sediment, sub-nephrotic proteinuria and rapidly progressive renal failure
over weeks, with or without pulmonary haemorrhage. When pulmonary haemorrhage
occurs, it usually predates nephritis by weeks or months. Pulmonary changes can vary
from fluffy pulmonary infiltrates on CXR to catastrophic pulmonary haemorrhage. The
gold standard for diagnosis is renal biopsy which shows diffuse proliferative GN with
focal necrotizing lesions and crescent formation. Immunofluorescence shows linear
depositon of IgG and complement. Anti-GBM antibodies can be detected by serology. In
type II hypersensitivity reactions, antibodies are directed against cell-surface or matrix-
associated antigen. IgG or IgM bound to the cell surface activates the complement
cascade or promotes phagocytosis of antibody-coated cells. Antibody dependent cellular
cytotoxicity may also result. In this process, leukocytes such as NK cells bind to antibody
via Fc receptors and release cytotoxic granules to destroy the antibody-coated cells.
Transfusion reactions, autoimmune haemolytic anaemia and Goodpasture's disease are
examples of type II hypersensitivity reactions. In type II reactions, bound antibody can
interfere with the normal function of cellular receptors, e.g., myasthenia gravis. In this
disease, autoantibodies bind to acetylcholine receptors at the neuromuscular junction.
This interferes with normal motor end plate function and also results in a reduction in
receptor numbers. Occasionally, bound antibody can activate receptors, e.g., in Graves
disease, stimulatory antibodies directed against the TSH receptor result in
hyperthyroidism.

560-The following are all clinical uses of prisms EXCEPT:


objective measurement of squint angle by prism cover test

assessment of simulated blindness

measurement of fusional reserve by using increasingly powerful prisms

detection of microtropia by using a four-dioptre base-in prism

Your answer was INCORRECT

Explanation

Microtropia describes a very small esotropia and it can be detected clinically by using a 4
dioptre base-out prism. When put before the deviating eye there is no movement as the
image remains within the suppression scotoma. However, when placed before the normal
fixing eye, movement occurs.

561-Which statement is TRUE regarding diagnostic lenses:

the panfundoscope lens is a combination of 2 concave lenses

the 78 dioptre lens produces an erect image which is then inverted by the slit lamp

in indirect ophthalmoscopy a 20D lens must be held nearer to the patient than a 30D lens

the Hruby lens produces an erect diminished image

Your answer was INCORRECT

Explanation

The image produced by the 78 dioptre lens is inverted. The panfundoscope consists of 2
convex lenses. A 20 dioptre lens must be held further from the patient than a 30 dioptre
lens in indirect ophthalmoscopy.

562-The ciliary ganglion lies between the optic nerve and:

the lateral rectus

the superior rectus


the medial rectus

the inferior rectus

Your answer was CORRECT

Explanation

The ciliary ganglion lies between the optic nerve and the lateral rectus.

563-Which statement is FALSE regarding cocaine:

it induces miosis of the pupil

it has an anaesthetic effect when applied locally

it increases body temperature

it causes tachycardia and hypertension

it inhibits the reuptake of monoamines at the synaptic space

Your answer was CORRECT

Explanation

Cocaine causes pupil dilation not constriction. Other options are true.

564-During accommodation for near:

the amount of light entering the eye increases

the field of vision decreases

the ciliary muscle relaxes

the thickness of the lens decreases

Your answer was INCORRECT

Explanation
During accommodation the ciliary muscle contracts and thus the lens becomes thicker.
The pupils constrict and therefore both the field of vision and the amount of light entering
the eye are reduced.

565-Which nerve runs along the length of the inferior orbital fissure towards the lateral
orbital wall:

the zygomatic nerve

the zygomaticofacial nerve

the infraorbital nerve

the zygomaticotemporal nerve

Your answer was CORRECT

Explanation

The zygomatic nerve runs the length of the inferior orbital fissure towards the lateral
orbital wall, where it pierces the zygoma as 2 branches: the zygomaticotemporal and
zygomaticofacial nerves.

566-Asit winds around the crus of the midbrain, the trochlear nerve runs immediately
beneath which vessel:

the posterior cerebral artery

the anterior cerebral artery

the inferior cerebellar artery

the superior cerebellar artery

the basilar artery

Your answer was CORRECT

Explanation
The trochlear nerve runs immediately beneath the posterior cerebral artery and above the
superior cerebellar artery.

567-Maximal number of axon in the optic nerve is achieved during:

5 weeks gestation

40 weeks gestation

2 weeks after delivery

15 weeks gestation

Your answer was INCORRECT

Explanation

The maximal number of axons in the optic nerve is achieved at 14-15 weeks gestation
when they number nearly 6 million. The number declines steadily thereafter so that at
birth there are approximately 1.3million.
Flag:

568-Which corneal dystrophy shows the deposition of amyloid microscopically:

Granular dystrophy

Meesman's dystrophy

Fuch's dystrophy

Lattice dystrophy

Your answer was INCORRECT

Explanation

Lattice dystrophy is an autosomal dominant stromal dystrophy characterized by fine


criss-crossing lines in the stroma. Microscopy reveals these deposits consist of amyloid.
Amyloid deposition is also present in Avellino dystrophy.
569-Zonulae occludens are also known as:
tight junctions

gap junctions

hemidesmosomes

desmosomes

Your answer was CORRECT

Explanation

Zonulae occludens are tight junctions, which are specialized junctions between cells that
help to form a barrier against paracellular diffusion of all molecules, including water and
ions. They are important in the blood-aqueous and blood-retinal barriers.

570-Which of the following best describes the mode of action of tetracycline:

inhibits protein synthesis

inhibits cell wall synthesis

inhibits cell membrane synthesis

inhibits nucleic acid synthesis

Your answer was CORRECT

Explanation

Tetracyclines inhibit protein synthesis by their action on aminoacyltransferase RNA and


the messanger RNA-ribosome complex. They have a broad spectrum of activity against
Gram-positive and Gram-negative bacteria and Chlamydia.

571-The foramen spinosum transmits:

the maxillary nerve

the lesser petrosal nerve


the accessory meningeal artery

the middle meningeal artery

Your answer was INCORRECT

Explanation

The foramen spinosum transmits the middle meningeal artery and vein and the meningeal
branch of the mandibular nerve.

572-Which is FALSE regarding the sclera:

it is more hydrated than the cornea

it contains mainly Type I collagen

it is mostly acellular with only sparse numbers of fibroblasts

its collagen fibres are less regular than the cornea

Your answer was CORRECT

Explanation

The cornea is more hydrated (80% water) than the sclera (70% water). Other options
supplied are true.

573-Which of the following immunoglobulin classes is/are important in antigen reception


at the lymphocyte cell surfaces for anamnestic immune responses?

1. IgD
2. IgG
3. IgM
4. IgE

4 only

1, 2, and 3

1, 2, 3, and 4
2 and 4

1 and 3

Your answer was INCORRECT

Explanation

Following initial exposure to an antigen, IgG will be produced in great abundance and
may thereafter participate in the reidentification of the antigen, as well as response to it.
IgE, after initial sensitization, binds to mast cell surfaces. Cross-linking by antigen of
bound IgE molecules leads to histamine release (Type I hypersensitivity).

574-Which is FALSE regarding endotoxins:

they cause fever

they are heat stable

they are produced by Gram negative bacteria

they are proteins

Your answer was INCORRECT

Explanation

Endotoxins are lipopolysaccharides derived from the cell wall of lysed or dead Gram-
negative bacteria. They are heat stable and non-antigenic. They produce:

 fever
 septic shock
 induction of complement

575-Afather suffers from an autosomal dominant condition. What is the chance that his
daughter will be affected:

75%

50%
100%

25%

33%

Your answer was INCORRECT

Explanation

There is a 1 in 2 chance of passing his gene and therefore a 50% chance of an affected
daughter. The chance is obviously the same for his son, as this is an autosomal condition.
We recommend drawing a 2x2 box of chromosomal probabilities.

576-The following are true about the cavernous sinus EXCEPT:

it is traversed by the optic nerve

it is traversed by the maxillary branch of the trigeminal nerve

it is traversed by the trochlear nerve

it lies on the body of the sphenoid bone

Your answer was CORRECT

Explanation

The optic nerve does not traverse the cavernous sinus. The chiasm lies superior and
medial to each cavernous sinus. Other statements are true.

577-The primary muscle responsible for lid closure is:

Muller's muscle

the orbicularis oculi

the frontalis

the corrugator supercilii


the levator palpebrae superioris

Your answer was INCORRECT

Explanation

The main muscle responsible for lid closure is the orbicularis oculi. The levator palpebrae
superioris is responsible for lid opening. The corrugator supercilii and frontalis are
muscles of facial expression. Muller's muscle assists in lid elevation under sympathetic
innervation.

578-Which is FALSE regarding the ophthalmic artery:

the ophthalmic artery lies superolateral to the optic nerve on entering the orbit

the anterior ciliary arteries are derived from muscular branches of the ophthalmic artery
which course along the recti muscles

the ophthalmic artery is a branch of the internal carotid artery

the ophthalmic artery enters the orbit via the orbital canal

Your answer was CORRECT

Explanation

The ophthalmic artery is positioned inferolateral to the optic nerve at the orbital apex. It
then winds around the optic nerve to pass above the nerve as it proceeds anteriorly in the
orbit above the medial rectus and under the superior oblique.

579-Choose the one correct statement. The thymus:

is required for T-lymphocyte maturation

is involved in affinity maturation

contains myeloid stem cells

is large in adult life


is involved in deletion of autoreactive B-cells

Your answer was CORRECT

Explanation

The thymus is a primary lymphoid organ required for the maturation of T-lymphocytes. It
has two lobes divided into multiple lobules by septa. Each lobule consists of an outer
cortex and an inner medulla. The cortex is mainly composed of immature thymocytes
(developing T-cells) enveloped by branched cortical epithelial cells and scattered
macrophages. The medulla consists of mature thymocytes and medullary epithelial cells.
There are also macrophages, bone marrow derived dendritic cells and Hassall's
corpuscles.

T-cell precursors arise in the bone marrow and then migrate to the thymus where they
undergo maturation to mature T-cells. This organ provides a specialised environment in
which these developing T-cells undergo positive and negative selection to ensure MHC
restriction and to eliminate autoreactive T-cells. The cells that first populate the thymus
have not rearranged their TCR genes and are TCR-/CD4- CD8-. Rearrangement of the
beta chain of the TCR and surface expression of this chain as part of the pre-T-cell
receptor induces proliferation and the expression of both CD4 and CD8. These cells are
now referred to as double positive thymocytes (DP). TCR alpha chain rearrangement
results in the expression of the mature TCR on the cell surface. These DP thymocyte now
undergo positive and negative selection to ensure that their TCR recognises foreign
antigen bound to self MHC, but is not autoreactive - i.e. it does not have a high affinity
for self peptide /MHC complexes. 99% of all thymocytes fail these selective processes.
The thymic cortical epithelial cells mediate positive selection of developing T-cells, and
negative selection is predominately mediated by bone-marrow derived dendritic cells and
macrophages. Those thymocytes that are positively selected down-regulate either CD4 or
CD8 depending on which class of MHC is recognised by their TCR. The mechanisms of
this process are controversial. The single positive (SP) thymocytes migrate to the
periphery and are now termed mature peripheral T-cells.

The human thymus is fully developed before birth, and the rate of T-cell production is
greatest before puberty. The thymus is largest in childhood, and the organ decreases in
size after puberty. A thymoma may cause enlargement in adulthood. Affinity maturation
of B-lymphocytes is a process resulting in the production of high affinity antibody post
B-cell activation. It takes place in the germinal centres of secondary lymphoid organs and
does not involve the thymus.

580-During phototransduction:

11-cis-retinal molecules are converted to all-trans-retinal

hyperpolarisation occurs due to closure of potassium channels


transducin converts GTP to GDP

dark current only occurs in the presence of photoreceptors stimulation

Your answer was CORRECT

Explanation

Transducin converts GDP to GTP. Dark current is present in the dark. In the presence of
light, the sodium channels (not potassium) close and give rise to hyperpolarization. 11
cis-retinal molecules are converted into trans-retinal.

581-Ethambutol is associated with which ocular side-effect:

retinal toxicity

corneal deposits

nystagmus

optic neuritis

Your answer was INCORRECT

Explanation

Ethambutol and isoniazid, both anti-tuberculous medications, are known to be associated


with optic neuritis.

582-Which is a constituent of the lateral orbital wall:

ethmoid

palatine bone

lesser wing of sphenoid

greater wing of sphenoid

Your answer was INCORRECT


Explanation

The lateral wall of the orbit is composed of the greater wing of sphenoid posteriorly and
the zygomatic bone anteriorly.

583-Which of the following components is NOT necessary for a successful polymerase


chain reaction (PCR)?

ribosomes

a primer sequence

a piece of DNA

nucleotides

Taq polymerase

Your answer was CORRECT

Explanation

The polymerase chain reaction has revolutionised molecular biology. It allows the
amplification of one sequence of DNA (or RNA) from a very small amount of the
original molecule.

A PCR vial will contain several components; a piece of DNA (RNA), large quantities of
the necessary nucleotides, large quantities of the primer sequence and the DNA
polymerase. This last element is called Taq polymerase.

Initially, the mixture is simply heated to 90 degress centigrade to unwind the DNA
strands. It then has to be cooled to 55 degrees centigrade to allow complementary primer
sequences to bind to the orginal DNA strands. Following this the temperature is raised
again to 75 degrees C, which is the optimal temperature for Taq polymerase. It begins to
add nucleotides to the primer sequence and eventually the original DNA template strands
are replicated. This process is repeated severeal hundreds of times, each time doubling
the amount of DNA material.

A ribosome is not needed.

584-Which is FALSE regarding Borrelia burgdoferi:

it cannot be detected by ELISA


it can cause Lyme disease

it is a spirochaete

it is best visualized by dark-ground microscopy

Your answer was CORRECT

Explanation

Borrelia is a spirochaete and in common with this family it is helical in structure,


flagellate and is best visualized by dark-ground microscopy. It causes Lyme disease with
transmission via ticks. It can be detected by ELISA to measure specific IgM and IgG
antibodies.

585-Which is FALSE regarding catoptric images:

the first is the basis of Hirschberg's test

the fourth image can be used to analyse accommodation

the second image is utilized by the keratometer

the third image is utilized in measuring anterior chamber depth

Your answer was INCORRECT

Explanation

The first image is utilised by the keratometer.

586-Which statement is FALSE regarding accommodation:

the AC/A is greater than normal in convergence excess esotropia

cyclopentolate reduces accommodation

there is a strict relationship between accommodation and convergence

it declines rapidly at the age of 60


Your answer was INCORRECT

Explanation

Accommodation is absent by the age of 60. All other options true.

587-A15-year-old girl presents with a fever and a sore throat. On examination her
pharynx and tonsils were red and inflamed and the tonsils were studded with white
exudates.

Which cells are most likely to be the first to respond to this infection:

plasma cells

CD8 positive T-cells

natural killer cells

macrophages

neutrophils

Your answer was INCORRECT

Explanation

This is likely to be a bacterial infection (most likely beta haemolytic streptococci). The
first cells to respond to a bacterial infection are the neutrophils (components of the innate
immune response) followed later by macrophages and cells of the adaptive immune
system. Natural killer cells are important in the response to viral infection.

588-Which blood test is positive in over 90% of patients with Wegener's granulomatosis:

p-ANCA

anti-rho

anti-ds DNA

c-ANCA
rheumatoid factor

Your answer was INCORRECT

Explanation

c-ANCA which is antineutrophil cytoplasmic antibody is positive in over 90% of patients


with systemic Wegener's.

589-Which is TRUE regarding the eyelids:

the meibomian glands secrete a mucinous solution

the lower lid is supplied by the maxillary division of the trigeminal nerve exclusively

there are more meibomian glands in the upper than lower lids

the orifices of the meibomian glands are anterior to the root of eyelashes

Your answer was INCORRECT

Explanation

The meibomian gland secretion is lipid rich and is an important component of the tear
film. There are about 30 meibomian glands in the upper tarsal plate and slightly fewer in
the lower tarsal plate. The orifices of the meibomain glands are found posterior to the
root of the eyelashes. The lower eyelid is supplied by branches of the infraorbital nerve (a
division of maxillary nerve) and infratrochlear nerve (branch of the ophthalmic nerve)

590-The outer layer of the tear film is composed of:

aqueous

blood

lipid

mucus

Your answer was INCORRECT

Explanation
The tear film is composed of three layers:

 an outer oily or lipid layer containing secretions from the meibomian glands

 a middle aqueous layer containing protein, electrolytes and water secreted from
the lacrimal glands

 an inner mucin layer derived from goblet cells and conjunctival epithelial cells

591-The seventh cranial nerve exits the brainstem:

from the dorsal surface of the pons

at the cerebellopontine angle

from the ventral midbrain

at the border of the midbrain and pons laterally

Your answer was INCORRECT

Explanation

The seventh cranial nerve exits the brainstem at the cerebellopontine angle.

592-Which condition is characterized by cyst formation due to an unstable corneal


epithelium:

Cogan's dystrophy

Thiel-Behnke dystrophy

Reis-Buckler dystrophy

Lattice dystrophy

Meesman's dystrophy

Your answer was CORRECT

Explanation
Microcyst formation due to an unstable corneal epithelium is observed in Cogan's
microcystic dystrophy.

593-The lacrimal sac fossa is made up of:

lacrimal bone only

ethmoid bone and lacrimal bone

palatine bone and lacrimal bone

maxillary bone and lacrimal bone

Your answer was INCORRECT

Explanation

The lacrimal fossa is made up of the frontal process of the maxillary bone and the
lacrimal bone

594-Which is FALSE regarding Thromboxane A2:

it is produced mainly by the endothelium of the capillaries

it causes platelet aggregation

it acts through cyclic adenosine monophosphate (cAMP) mediation

it causes vasoconstriction

Your answer was CORRECT

Explanation

Thromboxane A2 is produced by platelets. It plays an important role in platelet


aggregation and its action is mediated via cAMP. It also causes vasoconstriction.
Prostacyclin opposes the action of Thromboxane A2 and inhibits platelet aggregation.

595-All of the following are TRUE about aneurysm EXCEPT:

a true aneurysm contains all three layers of the vessel wall


a saccular aneurysm involves the whole circumference of the artery

a berry aneurysm typically occurs at the circle of Willis

a dissecting aneurysm occurs when the arterial media is deficient

Your answer was INCORRECT

Explanation

An aneurysm is an abnormal localised dilatation of a blood vessel. True aneurysms


contain all three layers of the vessel wall. A false aneurysm or pseudoaneurysm is a
haematoma that forms outside the vessel wall, and is contained by surrounding tissues, as
a result of a leaking artery. Berry aneurysm is found in the Circle of Willis and is caused
by vessel wall deficiency. Saccular aneurysm involves only part of the circumference of
the vessel wall whereas fusiform aneurysm involves the entire circumference. Dissecting
aneurysm occurs in arterial media deficiency as in Marfan's syndrome.

596-Northern blotting:

allows rapid detection of an infective organism

involves the polymerase chain reaction

involves electrophoresis

is used to detect DNA molecules

Your answer was INCORRECT

Explanation

A northern blot is very similar to a Southern blot except that it is RNA rather than DNA
which is extracted, run on a gel and transferred to a filter membrane. It is mRNA which is
isolated and hybridised in northern blots. It is used to determine if RNA of a particular
sequence is present.

597-The retinal pigment epithelium is:

a stratified non-keratinised squamous epithelium


a monolayer of myoepithelial cells

a monolayer of columnar epithelial cells

a stratified keratinized squamous epithelium

Your answer was INCORRECT

Explanation

The RPE is a monolayer of columnar/cuboidal epithelial cells.

598-Which technique is best for detecting chromosomal gains, losses and translocations:

FISH

ELISA

RFLP

PCR

Your answer was CORRECT

Explanation

FISH (fluorescent in situ hybridization) involves attaching a complementary strand of


DNA which is fluorescently labeled to specific sequences on the chromosome. It is useful
in detecting chromosomal gains, losses and translocations.

599-Typically, the first class of antibody produced against a newly encountered antigen is:

IgG

IgM

IgE

IgD

IgA
Your answer was INCORRECT

Explanation

IgM peaks earlier and disappears earlier than IgG during the primary immune response.

600-Epstein-Barr virus is implicated in all of the following EXCEPT:

non-Hodgkin's lymphoma

infectious mononucleosis

Burkitt's lymphoma

nasopharyngeal carcinoma

Your answer was CORRECT

Explanation

EBV is implicated in:

 Burkitt's lymphoma
 nasopharyngeal carcinoma
 infectious mononucleosis

601-Which statement is true of the EOG:

it gives an absolute value in microvolts

it depends on a normal retinal pigment epithelium

it can be easily produced in anaesthetized patients

the maximal dark response exceeds the light response in normal people

Your answer was INCORRECT

Explanation
The EOG is an indirect measure of the standing potential of the eye. It depends on a
normal retinal pigment epithelium. It requires co-operation of the patient who moves the
eyes back and forth over a specific distance. The light response is higher than dark
response. The result is given as a ratio between light to dark response. The normal ratio is
more than 180%

602-Which of the following is commonly added to penicillin to prevent its inactivation by


beta-lactamase enzymes:

hyaluronic acid

clavulanic acid

glucuronic acid

fucidic acid

Your answer was INCORRECT

Explanation

Clavulanic acid is a beta-lactamase inhibitor commonly added to amoxicillin (co-


amoxiclav) to prevent its degradation by bacterial beta-lactamase enzymes.

603-All of the following are muscles of facial expression EXCEPT:

Orbicularis oculi

Platysma

Buccinator

Frontalis

Temporalis

Your answer was INCORRECT

Explanation

All the choices above are muscles of facial expression (arising from the second
pharyngeal arch, and supplied by the facial nerve) except the temporalis, which (together
with the masseter, lateral and medial pterygoid) is a muscle of mastication. These
muscles arise from the first pharyngeal arch and are supplied by the mandibular branch of
the trigeminal nerve.

604-Sterilization at 121 degrees Celsius at 100 kPa requires:

15 minutes

3 minutes

60 minutes

100 minutes

Your answer was CORRECT

Explanation

Sterilisation at 120 degrees Celsius requires at least 15 minutes. At 132 degrees Celsius
sterilization can be achieved in 3 minutes.

605-Which is FALSE regarding acyclovir:

it is a guanine derivative

it can be used long-term for prophylaxis

it inhibits DNA polymerase

it acts against all herpes viruses

it can be administered both topically and orally

Your answer was INCORRECT

Explanation

Acyclovir acts against the herpes group of viruses but it is not effective against all herpes
viruses; it has minimal action against cytomegalovirus.

606-Regarding mitochondrial DNA:


it accounts for 10% of total human DNA

sperm do not contain mitochondrial DNA

it encodes protein needed for oxidative phosphorylation

the inheritance of mitochondrial disorders is similar to X-linked recessive disorders

Your answer was INCORRECT

Explanation

Mitochondrial DNA accounts for 0.5% of total human DNA.


The sperm is rich in mitochondrial DNA but it is destroyed soon after fertilization.
Mitochondrial DNA inheritance is maternal and is distinct from X-linked disorders.

607-The ophthalmic artery runs:

superolateral to the optic nerve

inferomedial to the optic nerve

superomedial to the optic nerve

inferolateral to the optic nerve

Your answer was INCORRECT

Explanation

The ophthalmic artery runs inferolateral to the optic nerve.

608-100men have their serum uric acid measured. The mean is 5.6 mg per 100 ml with a
standard deviation of 1.0 mg per 100 ml.

Which of the following is NOT true?

the standard error of the mean is equal to 0.1 mg per 100 ml


the standard error of the mean allows us to estimate how close our sample mean is likely
to be to the population mean

we would have a better estimate of the true population mean if our sample size were
greater

the standard error of the mean would be halved if we doubled sample size

we would have a better estimate of the true population mean if the standard deviation of
our sample were less

Your answer was INCORRECT

Explanation

If we were to take several samples from the same population, all would result in normal
distribution curves, but would be slightly different from each other. Each mean would be
scattered around the true population mean. The standard error of the mean is a measure of
this scatter, i.e. how accurate is the sample mean as an estimate of the true population
mean.

Note, SEM = Standard deviation/square root of sample size

This demonstrates that S.E.M. is affected by two factors:

 As sample size increases, S.E.M. decreases


 As standard deviation increases, S.E.M. increases

Note that S.E.M. is always smaller than standard deviation.

609-The pterygopalatine fossa is spanned by:

the mandibular nerve

the frontal nerve

the lacrimal nerve

the maxillary nerve

the zygomaticotemporal nerve

Your answer was INCORRECT


Explanation

The pterygopalatine fossa is spanned by the maxillary nerve before it enters the orbit via
the inferior orbital fissure as the infraorbital nerve.

610-The lipid-lowering effect of a new drug is compared to atorvastatin. The significance


level is set at 0.01 and a difference is found (p = 0.001).

What is the best conclusion to draw?

the alternative hypothesis should be accepted

the probability of a type I error occuring is 1%

the p-value associated with this statin means that it is a clinically significant new drug

a larger sample should have been used

the trial would have been better run against placebo

Your answer was CORRECT

Explanation

Where the p-value is less than the critical value, there is good evidence to accept the
alternative hypothesis. The p-value is the probability of committing a Type I error, which
in this case is 0.1%. Clinical significance (the effect in the real world) is not related to
statistical significance.

611-Which is FALSE regarding the detection of lens types with a cross drawn on a piece
of paper:

a concave lens causes with movement

spherical lenses cause no distortion of the cross

a prism displaces one line of the cross regardless of its position with respect to the cross

astigmatic lenses cause distortion of the cross when it is lined with the principle
meridia
Your answer was INCORRECT

Explanation

Astigmatic lenses cause distortion of the cross except when the arms of the cross are
aligned with the principle meridia of the lens. As such, the toric lens can be rotated about
the cross and once the cross is undistorted (arms at 90 degrees) the principle meridia are
aligned and can be marked out on the lens.

612-The corneal endothelium utilizes most of its glucose:

for ATP production anaerobically with conversion to lactate

for ATP production aerobically via the Krebb's cycle

for the production of sorbitol via the aldose reductase pathway

for the production of antioxidants via the pentose phosphate pathway

Your answer was CORRECT

Explanation

The corneal endothelium has a high energy requirement to maintain its pump mechanism.
As such, the major metabolic pathway for glucose is in the production of ATP, which it
accomplishes via anaerobic glycolysis. A smaller proportion of glucose is metabolized
aerobically via the Krebb's cycle. Some glucose is also diverted to the pentose phosphate
pathway, which is important for the production of antioxidants such as glutathione and
ascorbic acid. The production of sorbitol from glucose is a minor pathway and only
occurs to a significant degree in hyperglycemic states

613-Which statement is FALSE regarding the orbicularis oculi:

the palpebral fibres pass within the eyelid anterior to the orbital septum and tarsal plate

the orbital fibres run circumferentially in an elliptical fashion around the orbital margin

the palpebral portion originates from the medial palpebral ligament

the lacrimal portion is attached to the anterior lacrimal crest

the orbital portion arises from the medial palpebral ligament


Your answer was INCORRECT

Explanation

The lacrimal portion is attached to the posterior lacrimal crest. Other options are true.

614-In which condition do Hassall-Henle warts form?

Fuch's dystrophy

T cell lymphoma

Reis-Buckler

Human papilloma virus infection

Your answer was CORRECT

Explanation

Hassall-Henle warts refer to the nodular excrescences on the posterior surface of


Descemet's membrane found in Fuch's dystrophy.

615-Myelin of the peripheral nerves is produced by:

Muller cells

Schwann cells

oligodendrocytes

fibroblasts

macrophages

Your answer was INCORRECT

Explanation

Myelin of the peripheral nerve is produced by Schwann cells whereas that of the central
nervous system is produced by oligodendrocytes.
616-Which statement is FALSE regarding aqueous outflow:
contraction of the anterior ciliary muscle increases aqueous outflow by its mechanical
action on the trabecular meshwork

the trabecular meshwork terminates anteriorly at Schwalbe's line

the trabecular meshwork cells maintain the state of hydration of the trabecular
connective tissue but have no phagocytic capacity

the cribriform trabecular meshwork occurs immediately beneath Schlemm's canal and has
the highest resistance of all meshwork layers

Your answer was INCORRECT

Explanation

The trabecular meshwork can be subdivided into three anatomical zones: the innermost
uveal meshwork, the corneoscleral meshwork and the outermost cribriform meshwork,
which is adjacent to Schlemm's canal and has the highest resistance to flow. The anterior
ciliary muscle fibres are attached to the trabecular meshwork and on contraction they
open the meshwork, increasing aqueous flow. Note that trabecular cells have phagocytic
capacity in addition to maintaining the health and hydration of the connective tissues.

617-All of the following are true of the retinal pigment epithelium EXCEPT:

the cells have a low regenerative capacity

its cells are more flattened in the centre than in the periphery

it is derived from neuro-ectoderm

the cells are hexagonal and highly organized

Your answer was INCORRECT

Explanation

RPE cells are more flattened in the periphery than in the centre. Other options are true.

618-The abducens nerve:

lies medial to the internal carotid artery in the cavernous sinus


contains sympathetic nerve fibres

enters the orbit within the tendinous ring

is the most slender cranial nerve

Your answer was INCORRECT

Explanation

The abducens nucleus is located in the floor of the fourth ventricle. It is a pure motor
nerve and lies lateral to the internal carotid artery in the cavernous sinus. The trochlear
nerve is the most slender of all cranial nerves.

619-In
the rod photoreceptor, discs or lamellae migrate from the base to the tip of the outer
segment over:

24 hours

2 hours

30 days

10 days

3 days

Your answer was INCORRECT

Explanation

Migration of a disc from the base of the rod outer segment where it is formed to the tip
where it is shed takes 10 days.

620-Regarding Type II Diabetes Mellitus:

there is a substantial loss of beta-cells in the islets of Langerhans

there is an increased risk amongst first-degree relatives


the tissue response to insulin is normal

it is less common than type I diabetes mellitus

it is associated with certain HLA groups

Your answer was INCORRECT

Explanation

Type II diabetes mellitus (DM) is more common than Type I DM. Unlike Type I DM, the
beta-cells in the islets of Langerhans are usually not lost in Type II DM. There is,
however, resistance of cellular response to insulin in Type II DM. There is an increased
risk of Type II DM amongst the first degree relatives of sufferers. Type I DM is
associated with certain HLA groups but not Type II DM.

621-How long is a cone photoreceptor:

120 micrometers

12 micrometers

70 micrometers

15 micrometers

Your answer was INCORRECT

Explanation

A rod photoreceptor is 100-120 micrometers long while a cone is 60-75 micrometers.

622-The following are true about the lacrimal gland EXCEPT:

the palpebral portion drains into the superior conjunctival fornix through 12 ducts

the lymphatic drainage is to the parotid gland

it receives secretomotor innervation from the third cranial nerve

the palpebral portion of the gland is 1/4 the size of the orbital portion
Your answer was INCORRECT

Explanation

The lacrimal gland contains two parts: palpebral and orbital. The palpebral portion is
about 1/4 the size of the orbital portion. The palpebral portion contains 12 ducts that
originate from the orbital part and pass into the palpebral part before opening into the
superior conjunctival fornix. Surgical excision of the palpebral part will cause scarring of
the ducts whereas removal of the orbital part will remove the nerve innervation -
therefore excision of either part will affect tear secretion. The lacrimal gland receives its
secretomotor innervation from the facial nerve. Its lymphatics drain into the parotid
nodes.

623-Both Descemet 's membrane and the lens capsule are composed mainly of which type
of collagen:

Type III

Type I

Type II

Type IV

Your answer was INCORRECT

Explanation

Both Descemet 's membrane and the lens capsule are composed mainly of Type IV
collagen, which is a non-fibrillar structural collagen important to the formation of
basement membranes.

624-All of the following are corneal stromal dystrophy's EXCEPT:

Lattice dystrophy

Labrador keratopathy

Avellino dystrophy

Meesman's dystrophy
macular dystrophy

Your answer was INCORRECT

Explanation

Meesman's dystrophy is a corneal epithelial dystrophy while the other options are all
stromal

625-Which statement is FALSE regarding slit-lamp illumination:

specular reflection is used to examine the corneal endothelium

retro-illumination is optimised by co-axial illumination

sclerotic scatter requires co-axial illumination

sclerotic scatter depends upon internal reflection within the cornea

Your answer was INCORRECT

Explanation

Sclerotic scatter requires off-axis illumination, not co-axial. Other statements are true.

626-Which of the following is TRUE about the pituitary gland:

the posterior lobe secretion affects thyroid hormone secretion

a portal system exists between the posterior pituitary gland and the hypothalamus

it is drained by the cavernous sinus

it is situated above the optic chiasm

Your answer was INCORRECT

Explanation

The pituitary gland is located below the optic chiasm.


The portal system exists between the anterior pituitary gland and the hypothalamus.
TSH is secreted by the anterior lobe.
627-All of the following are examples of metastatic calcification EXCEPT:

hyperparathyroidism

atheroma

skeletal metastasis

hypervitaminosis

Your answer was INCORRECT

Explanation

Metastatic calcification occurs where there is deposition of calcium in normal tissue due
to hypercalcaemic states. Atheroma, where calcium is deposited as a consequence of
hyalinised and necrotic tissue, is an example of dystrophic calcification.

628-All of the following are true of a pterygium EXCEPT:

it is predisposed in hot dry environments

it occurs most commonly at the limbus inferiorly

the conjunctival stroma is hyalinised

there is degeneration of elastin histologically

Your answer was INCORRECT

Explanation

Pterygium is a condition characterized by thickening and hyalinization of the


conjunctival stroma with deposition of degenerating elastin. It is most common at the
limbus in the interpalpebral fissure. It is more common in hot, dusty environments

629-Tear film mucins are secreted mainly by:

the acinar cells of the accessory lacrimal glands


the acinar cells of the lacrimal gland

corneal epithelial cells

conjunctival goblet cells

Your answer was INCORRECT

Explanation

The mucin component of the tear film is the innermost layer. It provides a link between
the hydrophobic cornea surface and the aqueous secretion of the lacrimal glands. It is
produced by the goblet cells of the conjunctival epithelium.

630-Which is FALSE regarding the genetics of retinitis pigmentosa:

the rhodopsin gene is implicated in autosomal dominant RP

it is a heterogenous group of cone-rod dystrophies

X-linked RP usually presents in childhood

approximately 80% of RP is autosomal dominant

Your answer was INCORRECT

Explanation

Figures vary but RP is heterogenous with about 20% autosomal recessive, 20%
autosomal dominant, up to 25% X linked and 25% appear isolated.

631-During DNA replication, where can Okazaki fragments be found?

attached to RNA molecules

the replication fork

the leading strand

the lagging strand


the parent strand

Your answer was INCORRECT

Explanation

DNA replication is a semiconservative process, in other words one strand of the original
DNA molecule is incorporated into each of the two daughter strands. The DNA helix
(parent strand) is initially unwound by a helicase. The double helix is opened up to form
two single strands that act as templates. This occurs at the replication fork. DNA
polymerase adds nucleotides to these templates to allow a new DNA strand (leading
strand) to grow in a 5' to 3' direction. DNA polymerase can ONLY do this in the 5' to 3'
direction. So how can a further strand be made in the 3' to 5' direction using the second
template (the lagging strand)? The answer is that synthesis of this strand is discontinuous
and involves the synthesis of short nucleotide strands called Okazaki fragments by a
different DNA polymerase. These are then joined together by DNA ligase.

632-Approximately how many meibomian glands are there on the upper lid:

120

40

30

80

60

Your answer was INCORRECT

Explanation

There are approximately 30 meibomian glands on the upper lid and slightly fewer on the
lower lid.

633-Serum amylase and gamma-gutamyl transferase were compared within a population


of 900 inpatients. Which of the following is true?

the same methods for determining the correlation coefficient can be applied irrespective
of whether the data are normally or non-normally distributed
an r-value of 0.96 implies significant cause and effect

the null hypothesis would state that r equals zero

correlation coefficients take a value between 0 and 1

Your answer was INCORRECT

Explanation

Correlation coeffecients range from -1 to +1. A high degree of association does not imply
cause and effect. Statistical methods for determining association depend on whether the
population is normally distributed or not.

634-Phthisis bulbi is an example of:

dystrophic calcification

metastatic calcification

senile calcification

metamorphic calcification

Your answer was CORRECT

Explanation

Phthisis bulbi is an example of dystrophic calcification where calcium is deposited in


diseased or necrotic tissue. In phthisis bulbi, calcification usually occurs in the
metaplastic fibrous tissue derived from the proliferation of RPE cells in a hypotonic eye.

635-Which nerve is transmitted through the temporal bone:

olfactory nerve

facial nerve

mandibular nerve

vagus nerve
Your answer was INCORRECT

Explanation

Only the facial nerve and the vestibulocochlear nerves are transmitted through the
temporal bone.

636-The cerebrospinal fluid (CSF):

is normally produced at a rate of 100ml per day

is formed mainly in the choroid plexus

flows from the lateral ventricles through the aqueduct into the third ventricle

is found in greater volume in the skull than in the spine

Your answer was INCORRECT

Explanation

The normal rate for CSF production is 400- 500ml per day. The normal volume is 150ml.
There is more CSF in the spine than in the skull. CSF is formed mainly by choroid
plexus. It flows from the lateral ventricles into the third ventricle through the
interventricular foramen of Munro and then through the aqueduct into the fourth
ventricle.

637-In
response to acute haemorrhage, the following compensatory events occur
EXCEPT:

vasoconstriction of the renal efferent arterioles

cerebral vasoconstriction

raised circulating angiotensin II concentration

increased chemoreceptor discharge

Your answer was INCORRECT

Explanation
A drop in blood volume reduces baroreceptor stretch which in turn increases sympathetic
output. This results in generalised vasoconstriction, except for the brain and heart vessels.
The renal efferent arterioles constrict more than the afferent arterioles. Hypoxia from a
reduced volume increases chemoreceptor discharge.

638-Vagal stimulation causes ALL of the following EXCEPT:

delayed A-V conduction

a fall in heart rate

decreased stroke volume

increased ventricular contractility

Your answer was INCORRECT

Explanation

Vagal stimulation increases the parasympathetic activities resulting in: slowing of the
heart rate, delayed A-V conduction, decreased stroke volume, decreased atrial and
ventricular contractility

639-Where is drusen deposited in age-related macular degeneration:

between the RPE cell and its basement membrane

between the RPE basement membrane and Bruch's membrane

between Bruch's and the choriocapillaris

between the RPE cell and the outer photoreceptor layer

Your answer was INCORRECT

Explanation

Drusen is situated between the basement membrane of the RPE and the Bruch's
membrane.

640-Which statement is FALSE about the corneal stroma:


sensory nerve fibres are present in the posterior stromal layers en route to the
epithelium

is composed of 200-250 layers of flattened collagenous lamellae

collagenous lamellae are oriented parallel to the corneal surface

is a dense connective tissue of remarkable regularity

Your answer was CORRECT

Explanation

Sensory nerve fibres are present in the anterior stromal layers en route to the epithelium

641-The first cell that an antigen typically contacts in the cascade of immune response is
the:

macrophage

plasma cell

natural killer (NK) cell

T-lymphocyte

B-lymphocyte

Your answer was CORRECT

Explanation

Macrophages can initiate the immune cascade by phagocytosing antigen and presenting it
to T cells. Macrophages are also known as antigen-presenting cells.
Flag:

642-Which condition involves deposition of a light-chain-derived amyloid:

Alzheimer's disease
rheumatoid arthritis

familial Mediterranean fever

Waldenstrom macroglobulinaemia

Your answer was INCORRECT

Explanation

Amyloid can be derived from proliferation of light chain fragments of immunoglobulin,


as in Waldenstrom macroglobulinaemia and multiple myeloma.

643-Six prism diopters of left esotropia are best corrected by:

3 dioptres base-in left eye, 3 dioptres base-in right eye

3 dioptres base-out left eye, 3 dioptres base-out right eye

6 dioptres base-out left eye

3 dioptres base-out left eye, 3 dioptres base-in right eye

Your answer was INCORRECT

Explanation

Generally when prescribing prisms the correction is split between the two eyes.

644-The pterygopalatine ganglion:

is a sympathetic ganglion

receives preganglionic parasympathetic fibers from the lesser petrosal nerve

contains preganglionic fibers that arise in the lacrimal nucleus of the facial nerve

contains secretomotor nerve to the parotid gland

Your answer was INCORRECT


Explanation

The pterygopalatine ganglion is a parasympathetic ganglion. Its preganglionic fibers arise


from the lacrimal nucleus of the facial nerve and they run in the greater petrosal nerve
before synapsing in the ganglion. The ganglion contains secretomotor nerve fibres to the
lacrimal gland.

645-Which is FALSE regarding the accommodation mechanism:

fibres mediating accommodation reach the ciliary muscle by the long ciliary nerves

contraction of the ciliary muscle induces relaxation of the zonule

accommodation is mediated via fibres from the third cranial nerve

contraction of the ciliary muscle moves the ciliary muscle forward

Your answer was CORRECT

Explanation

Parasympathetic innervation of the ciliary muscle is derived from the third cranial nerve
via the Edinger-Westphal nucleus. The pre-ganglionic fibres synapse in the ciliary
ganglion and reach the ciliary muscle by the short (not long) ciliary nerves.

646-Which statement is FALSE regarding a cross-cylinder:

it can determine both the axis and the power of the cylindrical correction

the handle is positioned between the principal axes

the power and sign of the 2 cylinders in the cross are equal

it is equivalent to a pair of cylindrical lenses with their axes at right angles to one another

Your answer was INCORRECT

Explanation

The power of the cylinders is equal but the signs are opposite.

647-What is the mode of action of the imidazole antimicrobials:


inhibits folate synthesis

inhibits protein synthesis

inhibits cell membrane synthesis

inhibits mitochondrial oxidation

Your answer was INCORRECT

Explanation

The imidazoles are antimycotic agents which inhibit the synthesis of sterols which are an
important constituent in fungal cell membranes.

647-All are true of E.coli EXCEPT:

it is a facultative anaerobe

it ferments glucose

it is a Gram negative rod

it is non-motile

it grows well on MacConkey agar

Your answer was INCORRECT

Explanation

E. coli is motile with polar flagella. Other options are true.

648-How thick is the tear film?

4 micrometers

2 micrometers

9 micrometers
13 micrometers

Your answer was INCORRECT

Explanation

The tear film measures 7-9 micrometers.

649-Which statement is true of the ERG:

the b wave is produced by the photoreceptors

it cannot be performed on anaesthetized patients

dark adaptation increase the amplitude of the a and b waves as well as the latency

it is affected by optic neuritis

Your answer was INCORRECT

Explanation

The ERG records retinal activity. It can be performed on anesthetized patients. It is not
affected by the ganglion cells and therefore not affected by optic neuritis. It contains an
upgoing a wave produced by photoreceptors and downgoing b wave produced by Muller
and bipolar cells. Dark adaptation increases the amplitude the a and b waves as well as
the latency.

650-The systemic amyloid associated with rheumatoid arthritis is derived from:

serum AA protein

immunoglobulin light chains

prealbumin

polypeptide hormones

Your answer was CORRECT

Explanation
Amyloid deposition may be derived from a number of sources depending on the
pathology. It may occur secondary to systemic disease such as rheumatoid, in which case
it is derived from serum AA protein, an acute phase reactant. It may occur from light
chain immunoglobulins as in myeloma and macroglobulinaemia. It may occur from
polypeptoid hormones as in carcinoma of the thyroid. In Alzheimers it is derived from
prealbumin.

651-Which statement is FALSE regarding oncogenes:

mutated proto-oncogenes are associated with cancer

viral insertion can disrupt normal proto-oncogenes

proto-oncogenes regulate the normal cell division

oncogenes are associated with a reduced risk of cancer

Your answer was INCORRECT

Explanation

Oncogenes are derived from normal proto-oncogenes by mutation. The mutation can be
brought about by point mutation, viral insertion, gene translocation or gene amplification.
Proto-oncogenes regulate normal cell division while oncogenes increase the risk of
malignant transformation of a cell.

652-All of the following are produced from arachadonic acid EXCEPT:

thromboxane

prostaglandins

fibrinogen

leukotrienes

Your answer was INCORRECT

Explanation
Fibrinogen is a plasma protein produced in the liver. Prostaglandins, thromboxane and
leukotrienes are all eiconsanoids, produced from arachadonic acid. Prostaglandins and
thromboxane require cyclo-oxygenase, while leukotrienes require lipoxygenase.

653-A teratoma is a tumour deriving from:

striated muscle

meningeal tissue

cartilage

germ cells

Your answer was INCORRECT

Explanation

A teratoma is a tumour arising from germ cells.

654-The cell bodies of the ganglion cells are located in which retinal layer:

ganglion cell layer

nerve fibre layer

inner nuclear layer

outer nuclear layer

Your answer was CORRECT

Explanation

The ganglion cell bodies are located in the ganglion cell layer, which is the innermost
nucleated layer of the retina.

655-Which statement is FALSE about the tarsal plate:

the tarsal plates are modified thickenings of the orbital septum


the muco-cutaneous junction occurs at the level of the meibomian gland openings

the upper lid tarsal plate contains about 30 meibomian glands

the grey line marks the posterior boundary of the tarsal plate

Your answer was INCORRECT

Explanation

The grey line marks the anterior boundary of the tarsal plate and is a useful surgical
landmark dividing the lid skin and contents from the tarsal plate and orbital contents. The
muco-cutaneous junction between the skin of the lid and the conjunctiva occurs at the
level of the meibomian gland openings.

656-All are true of Botulinum toxin EXCEPT:

it can be used as treatment for essential blepharospasm

it is produced by Clostridium perfringens

it can be inactivated by heating

it prevents the release of acetylcholine from the nerve terminal

Your answer was INCORRECT

Explanation

Botulinum toxin is produced by Clostridium botulinum. It is an exotoxin and is therefore


denatured by heat. It produces its effect by inhibiting exocytosis of acetylcholine from
cholinergic terminals.

657-The subarachnoid space terminates at the level of:

the fourth lumbar vertebra

the third lumbar vertebra

the second sacral vertebra


the coccyx

Your answer was INCORRECT

Explanation

The subarachnoid space terminates at the second sacral vertebra

658-Which is FALSE regarding exotoxins:

they are heat-resistant

they are proteins

tetanus toxoid is an example

they produce an antibody response

Your answer was CORRECT

Explanation

Exotoxins are proteins released by Gram-positive bacteria and produce specific effects at
sites distant to their release. They are highly toxic, antigenic and are readily destroyed by
heat.

659-Regarding mitochondrial DNA all are true EXCEPT:

a male with a mitochondrial disorder can pass it on to his daughter only

it is expressed in muscle cells

it is responsible for coding enzymes used in oxidation

it is circular and double-stranded

Your answer was CORRECT

Explanation

Mitochondrial DNA are transmitted via the maternal line only. Both sperm and ova have
mitochondria but the sperm mitochondria do not enter the ova. Mitochondrial DNA is
expressed in mitochondria rich tissue such as the muscles, the neural tissues and the
myocardium. Patients with mitochondrial disorders are therefore susceptible to myopathy
and cardiomyopathy. DNA in the mitochondria code for enzymes involved in oxidation
and energy production.

660-Which statement regarding the foramen is correct:

The carotid canal is in the maxillary bone

The foramen ovale occurs in the frontal bone

The jugular foramen occurs between the temporal bone and the occipital bone

The stylomastoid foramen is in the occipital bone

Your answer was INCORRECT

Explanation

The sytlomastoid foramen is found in the temporal bone between the mastoid process and
the styloid. The carotid canal is on the inferior surface of the petrous part of the temporal
bone. The jugular foramen is formed between the temporal and occipital bones. The
foramen ovale is found in the greater wing of the sphenoid

661-The inferior orbital fissure transmits all of the following EXCEPT:

the zygomatic nerve

the lacrimal nerve

branches from the pterygopalatine ganglion

the infraorbital nerve

Your answer was INCORRECT

Explanation

The lacrimal nerve passes through the superior (not inferior) orbital fissure. Other options
supplied are true.
662-Raised intracranial pressure:
is associated with a decrease in blood pressure

is associated with a decrease in pulse rate

is associated with an increase in the respiratory rate

may be reduced by hyperventilation

Your answer was INCORRECT

Explanation

Raised intracranial pressure is associated with:

 raised blood pressure


 raised pulse rate
 periodic respiration.

Hyperventilation decreases the PCO2 and can be used to reduce the intracranial pressure
by decreasing the cerebral blood flow to the arachnoid granulations. However, the
reduction in cerebral blood flow may exacerbate cerebral ischemia.

663-Stimulation of the sympathetic nervous system results in all of the following


EXCEPT:

contraction of the bladder detrusor muscle

pupillary dilatation

vasoconstriction of the skin and mucous membrane

decreased gastrointestinal motility

Your answer was CORRECT

Explanation

Sympathetic nervous system stimulation causes pupil dilatation, reduced gastrointestinal


motility and causes bronchodilatation. It diverts blood away from the skin and mucous
membrane by vasoconstriction and relaxes the detrusor muscle of the bladder

664--2.00 DS/ -1.00 DC x 170 is equivalent to:


-3.00 DS/ + 1.00 DC x 80

+8.00 DS/-2.50 DC x 135

+ 11.50 DS/+2.75 DC x 12

+ 3.75 DS/-6.25 DC x 75

Your answer was CORRECT

Explanation

-2.00 DS/ -1.00 DC x 170 is equivalent to -3.00DS/+1.00DC x 80

665-Which is FALSE regarding acute retinal necrosis:

it can be caused by herpes zoster virus

viral inclusion bodies cannot be detected in the retina histologically

specimens show hemorrhagic necrosis

it can be caused by herpes simplex virus

Your answer was INCORRECT

Explanation

Viral inclusion bodies are visible on light microscopy within the retinal neurones and the
RPE.

666-The anterior ethmoid foramen is found:

between the frontal and ethmoid bones

within the sphenoid bone

between the ethmoid bone and lacrimal bone

within the frontal bone


Your answer was CORRECT

Explanation

The anterior ethmoid foramen occurs at the junction of the frontal and ethmoid bones in
the medial orbital wall. It transmits the anterior ethmoidal vessels and the anterior
ethmoidal nerve.

667-Which orbital communication occurs between the greater and lesser wings of the
sphenoid:

inferior orbital fissure

foramen ovale

foramen rotundum

superior orbital fissure

Your answer was INCORRECT

Explanation

The superior orbital fissure occurs between the lesser and greater wings of sphenoid.

668-Which is FALSE regarding stereoacuity:

the normal stereoacuity is 60 seconds of arc

it requires images to fall on Panum's area

it cannot be measured without polarizing spectacles

it is measured in seconds of arc

Your answer was INCORRECT

Explanation

Stereoacuity can be measured by several tests. The Titmus test requires polarizing
spectacles but not the Frisby test.
669-Which of the following is MOST likely in Sturge-Weber syndrome:
retinal haemangiomata

choroidal haemangioma

astrocytic hamartomas

Lisch nodules

Your answer was INCORRECT

Explanation

Sturge-Weber syndrome is characterized by naevus flammeus of the facial skin with


angioma of the meninges. It is associated intra-ocularly with:

 choroidal haemangioma
 glaucoma due to vascular proliferation in the angle

670-Which is FALSE regarding retinal capillaries:

they are surrounded by pericytes and astrocyte foot processes

they are absent entirely from the foveola

they are surrounded by a thin basal lamina

their endothelial cells are joined by tight junctions

Your answer was INCORRECT

Explanation

The retinal capillary endothelial cells are joined by tight junctions. They are surrounded
by a thick (not thin) basal lamina. They are also surrounded by pericytes and astrocyte
foot processes, all of which play an important role in the integrity of the blood-retinal
barrier.

671-All of the following are TRUE of keloid scars EXCEPT:

it is otherwise known as a hypertrophic scar


steroid therapy can reduce the formation of keloid

it is more likely to develop if a wound becomes infected

Patients of Afro-Caribbean origin with dark complexion are at higher risk

Your answer was CORRECT

Explanation

A hypertrophic scar is raised but does not go beyond the boundary of the original wound,
unlike keloid. All other statements are true.

672-All of the following are muscles of mastication EXCEPT:

lateral pterygoid

medial pterygoid

buccinator

masseter

Your answer was INCORRECT

Explanation

All the choices above are muscles of mastication except the buccinator. Muscles of
mastication (masseter, lateral and medial pterygoid) arise from the first pharyngeal arch
and are supplied by the mandibular branch of the trigeminal nerve. The buccinator is a
muscle of facial expression and together with the other muscles of facial expression, it
arises from the second pharyngeal arch, and is supplied by the facial nerve.

673-Which of the following statements regarding intention-to-treat analyses is correct?

they are preferable because they require a smaller sample size

they are preferable because they assess the efficacy of a new treatment

they are preferable because they remove potential biases in the trial
they are preferable because they do not require us to blind patients to the treatment they
are receiving

they are preferable because they allow us to randomise patients to different treatment
arms where this would otherwise be unethical

Your answer was INCORRECT

Explanation

Intention-to-treat analyses assess outcomes by the groups to which patients were


originally assigned, irrespective of whether they actually received the intervention. It is
done for two reasons:

 Statistically: patients dropping out or transferring to other arms may not do so


randomly, if we did not do an ITT analysis we would introduce bias
 Clinically: ITT analyses assess an intervention's likely effect in day-to-day
practice (effectiveness) rather than under ideal conditions (efficacy).

674-The cribriform plate is part of:

the maxilla

the ethmoid bone

the frontal bone

the sphenoid bone

Your answer was INCORRECT

Explanation

The cribriform plate is part of the ethmoid bone and it transmits the olfactory nerves.

675-Which of the following diseases does NOT involve abnormal G protein activity?

myasthenia gravis

cholera
pseudohypoparathyroidism

pituitary adenomas

McCune-Albright syndrome

Your answer was CORRECT

Explanation

G proteins can be inhibitory or stimulatory. In general they consist of three subunits,


alpha, beta and gamma. When the alpha unit is activated it binds GTP and separates from
the rest of the complex to cause effects downstream. As it does so, GTP is converted to
GDP. The alpha subunit, with GDP bound, then rejoins the rest of the complex. Cholera
toxin causes a stimulatory G protein (Gs) to be continually switched on, leading to severe
diarrhoea. Similiar G protein abnormalities have been found in all the other diseases
listed bar myasthenia gravis, which is an autoimmune disorder targeting post-synaptic
ACh receptors.

676-Regarding pilocarpine which statement is FALSE:

it facilitates aqueous drainage

it causes pupillary contraction

it draws on the scleral spur and opens the trabecular meshwork

it has no effect on aqueous secretion

Your answer was INCORRECT

Explanation

Pilocarpine facilitates aqueous drainage by constricting the iris and by opening the
meshwork from its action on the ciliary muscle. It also reduces aqueous secretion by
reducing blood flow to the ciliary body.

677-Which rectus muscle inserts 7.0 mm from the cornea:

superior rectus
inferior rectus

lateral rectus

medial rectus

Your answer was INCORRECT

Explanation

The insertion distances of the four recti muscles from the limbus are as follows:

 medial rectus 5.6mm


 inferior rectus 6.6mm
 lateral rectus 7.0 mm
 superior rectus 7.8mm

678-Compared to blood the CSF has:

lower glucose

more lymphocytes

higher immunoglobulins

higher protein

Your answer was CORRECT

Explanation

Most constituents exist in lower concentration in the CSF compared to blood, including
glucose, which is 60% of its blood concentration. Only chloride, magnesium and
hydrogen ions are in higher concentration in the CSF.

679-Which statement is FALSE regarding the geniculocalcarine tracts, or optic radiations:

the fibres from the inferior portion of the retina take a wider course

the fibres initially pass anteriorly to form a loop called the loop of Miller
the axons are destined for the occipital cortex

the tract passes through the temporal lobe

the axons have their cell bodies in the lateral geniculate nucleus

Your answer was INCORRECT

Explanation

The geniculocalcarine tracts initially pass anteriorly to form a loop called the loop of
Meyer (not Miller) which passes through the parietal and temporal lobes. Fibres from the
inferior retina (corresponding to the superior visual field) take a wider course passing
deeper into the temporal lobe than fibres from the superior retina. Fibres from the macula
tend to have the straightest course.

680-Which of the following is TRUE about smooth muscle:

it shows visible striation

it is composed of single nucleated cells that act as a synctium

it contains troponin

it acts under somatic nervous control

Your answer was INCORRECT

Explanation

Smooth muscle shows no visible striation. It is innervated by the autonomic nervous


system. It contains myosin, actin and tropomyosin but not troponin. It is made up of
single nucleated cells that act as a syncytium.
681-Fibres from the optic chiasm to the superior colliculus are involved in all EXCEPT:

the visual grasp reflex

automatic scanning of images

visual association pathways


the pupillary light reflex

Your answer was INCORRECT

Explanation

After the decussation of fibres in the optic chiasm, 90% of fibres pass along the lateral
root of the optic tract towards the lateral geniculate nucleus and on to the occipital cortex
for conscious vision. 10% of fibres travel along the medial root of the optic tract and are
responsible for unconscious stimuli. These fibres target:

 the superior colliculus: involved in visual grasp reflex, automatic scanning of


images

 the pretectal nucleus: involved in pupillary light reflex

 the parvocellular reticular formation: arousal function

 the retinohypothalamic tract : photoperiod regulation

682-The following are true about the trigeminal nerve EXCEPT:

it innervates the paranasal sinuses

it emerges from the brain stem between the pons and the medulla

it supplies the muscles of mastication

it emerges from the brain stem as separate sensory and motor roots

Your answer was INCORRECT

Explanation

The trigeminal nerve emerges from the brain stem at the junction between the pons and
the middle cerebellar peduncle.

683-All are true of the sympathetic trunk EXCEPT:

in the thorax it is covered by the pleura

in the neck it lies alongside the bodies of the cervical vertebrae


it extends from the base of the skull to the lower lumbar region

it enters the thorax anterior to the neck of the first rib

Your answer was INCORRECT

Explanation

The sympathetic trunk extends from the base of the skull to the coccyx. In the neck, it is
covered by the prevertebral fascia and lies alongside the bodies of the cervical vertebrae.
It enters the thorax through the neck of the first rib. In the thorax it is covered by the
pleura.

684-Ina sample of 200 gastroenterology inpatients, 25 were felt to have ascites on clinical
examination. Ultrasound on all revealed 100 to have ascites, of which 21 had been
detected clinically.

Which of the following interpretations is NOT correct:

a high number of false negatives would mean a low sensitivity

positive predictive values of clinical exam for ascites is 21/25

sensitivity on clinical exam for ascites is 21/25

prevalence of ascites in this sample was 50%

the high proportion of true negatives means the test has a high specificity

Your answer was INCORRECT

Explanation

The incorrect option above would be true if it was 21/(21+79).

685-Which human tissue has the highest oxygen consumption per weight:

myocardium

renal cortex
retina

choroid

Your answer was INCORRECT

Explanation

The retina is a highly active metabolic tissue, with the highest consumption of oxygen per
weight in the human.

686-Which statement is FALSE regarding cytomegalovirus retinitis:

it is typically seen in immunocompromised patients

infected cells are enlarged with an owl-eye appearance

retinal hemorrhaging is characteristic

it is characterized by areas of retinal necrosis

Your answer was INCORRECT

Explanation

CMV retinitis demonstrates areas of retinal necrosis usually without hemorrhage.

687-Which layer occurs immediately beneath the nerve fibre layer of the retina:

internal limiting membrane

outer segment

inner plexiform layer

inner nuclear layer

ganglion cell

Your answer was INCORRECT


Explanation

The eight layers of the neurosensory retina from outer to inner, that is from the RPE
upwards are:

 outer segment
 inner segment
 outer nuclear
 outer plexiform
 inner nuclear
 inner plexiform
 ganglion cell
 nerve fibre layer

688-The crystalline lens strongly absorbs which wavelength of light:

UV-B

IR-B

UV-C

UV-A

Your answer was INCORRECT

Explanation

The crystalline lens strongly absorbs UV-A, which is the nearest UV (315nm-400nm)
light to the visible spectrum. UV-B and UV-C are usually absorbed by the cornea and
sclera.

689-Which of the following structures on its cell surface allows a hepatocyte to alert the
immune system to a virus that has infected it:

T-cell receptor

Immunoglobulin D

Class I major histocompatibility complex


Fc receptor

Class II major histocompatibility complex

Your answer was INCORRECT

Explanation

Every nucleated cell in the body expresses class I major histocompatibility complex
(MHC) molecules, which present intracellular material on the cell surface. Degraded
virus is presented to circulating CD8+ (cytotoxic) T-cells which are then activated. The
result is destruction of the infected cell. MHC class II molecules are only expressed on
specialised antigen presenting cells but are inducible on endothelial/epithelial cells.
Peptides presented on MHC class II molecules are primarily derived from the
extracellular environment and are recognised by CD4+(T-helper) cells. Immunoglobulin
is produced by B-cells and is there are 5 different classes (G, A, M, D, E). IgD is
immunoglobulin bound to the surface of B-cells. It acts as the B-cell receptor. Fc
receptors are expressed on various cells and allow the Fc portion of an immunoglobulin
molecule to bind to these cells.

690-Which is FALSE regarding alpha-adrenergic agonists:

apraclonidine is an alpha-1 agonist

alpha-2 receptors are located mostly on the presynaptic nerve endings

alpha-1 receptors mediate excitatory responses

alpha-1 agonists cause a rise in IOP

Your answer was CORRECT

Explanation

Alpha-1 agonists are excitatory, principally causing smooth muscle contraction, as in the
contraction of the dilator pupillae muscle. This dilatation causes an early rise in IOP.
Alpha-2 agonists occur mainly on the presynaptic nerve endings and are inhibitory as
they prevent further release of neurotransmitter. Alpha-2 agonists such as clonidine cause
a reduction in IOP and this is thought to be mediated centrally by the stimulation of the
medullopontine sympathetic nerve and by reducing aqueous production locally.
Apraclonidine is a partial alpha-2 agonist.

691-Immunoglobulin G:
is the main immunoglobulin in normal conjunctiva

contains two antigen-binding sites

is the least common immunoglobulin in plasma

cannot cross the placenta

Your answer was INCORRECT

Explanation

IgG is the most common immunoglobulin in plasma, accounting for about 75% of the
circulating immunoglobulins in healthy individuals. IgG antibodies predominate during a
secondary antibody response. Thus, the appearance of specific IgG antibodies generally
corresponds with the 'maturation' of the antibody response, which is switched on upon
repeated contact with an antigen. It crosses the placenta freely. It has two antigen binding
sites. The main immunoglobulin in conjunctiva is IgA.

692-Which is FALSE regarding the rod lamellae:

they are shed predominantly in the evening

they number up to 1000 in each outer segment

they are phagocytosed by the RPE

they are contained within a membrane

Your answer was CORRECT

Explanation

Rod lamellae are shed in a circadian rhythm predominantly in the early morning.

693-Which statement about the Bowman's layer is FALSE:

it is a modified acellular region of stroma

it is 8-12 micrometers thick


it terminates abruptly at the limbus

it is composed mainly of Type II collagen

Your answer was INCORRECT

Explanation

The Bowman's layer consists of fine, randomly arranged collagen fibrils which are
mainly Types I, III, V and VI.

694-The image formed by an object outside the centre of curvature of a concave mirror is:

real, inverted and diminished

virtual, inverted and enlarged

real, inverted and enlarged

erect, virtual and enlarged

Your answer was CORRECT

Explanation

For an object outside the centre of curvature of a concave mirror, the image formed is
real, inverted and diminished; and it lies between the centre of curvature and the principle
focus.

695-During A-scan biometry, corneal compression from excessive pressure from the probe
will:

cause an artificial shortening of the axial length

have no affect on the measurement

damage the probe

cause an artificial lengthening of the axial length


Your answer was CORRECT

Explanation

Corneal compression from excessive pressure during A-scan biometry causes an artificial
shortening of the axial length.

696-Which vessels are typically affected in Takayasu's disease:

small and medium sized arteries

any large vessel with an elastic lamina

vessels arising from the aortic arch

renal arterioles

Your answer was INCORRECT

Explanation

Takayasu's disease affects major vessels arising from the arch of the aorta.

697-Acquired optic nerve disease tends to cause:

red-green defects

deuteranopia

blue-yellow defects

symmetric depreciation of colours

Your answer was CORRECT

Explanation

 red-green defects: acquired optic nerve disease (except glaucoma and autosomal
dominant optic atrophy)

 blue-yellow defects: acquired retinal disease (except cone dystrophy and


Stargardt's)
698-Which artery supplies the lateral root of the optic tract:

anterior choroidal artery

anterior cerebral artery

posteror choroidal artery

lateral striate artery

posterior communicating artery

Your answer was CORRECT

Explanation

The lateral root of the optic tract is supplied by the anterior choroidal artery.

699-The corneal endothelium obtains its oxygen from:

the air

the conjunctival

the precorneal tear film

the aqueous

Your answer was INCORRECT

Explanation

The corneal epithelium obtains its oxygen from the precorneal tear film. The endothelium
and the keratocytes in the deep stroma by contrast obtain their oxygen supply from the
aqueous humour.

700-The most common corneal dystrophy is:

Reis-Buckler dystrophy
Labrador keratopathy

Fuch's endothelial dystrophy

Granular dystrophy

Lattice dystrophy

Your answer was INCORRECT

Explanation

Fuch's endothelial dystrophy is the most common corneal dystrophy, occurring in the
elderly with a female preponderance.

701-The frontal eye field is primarily concerned with:

fixation

saccades

dolls-head reflex

visual memory

ductions

Your answer was INCORRECT

Explanation

The frontal eye field is concerned with voluntary control of eye movements (saccades).

702-Which is FALSE regarding mitosis and the cell cycle:

biosynthesis occurs during G1

G0 is the rest phase

chromosomes align in the centre of the nucleus during metaphase


chromosomal separation begins in telophase

Your answer was INCORRECT

Explanation

Chromosomal separation begins in anaphase with completion of separation during


telophase

703-Which is FALSE regarding Human Immunodeficiency Virus (HIV):

it gains entry by binding to CD4 antigen on the cell surface

the earliest sign of infection is p24 protein in the blood

Type II is primarily homosexually transmitted

it affects CD4 positive T cells

Your answer was INCORRECT

Explanation

Type I HIV occurs primarily in urban centres in the USA and Europe and is more
common in homosexuals and intravenous drug abusers. Type 2 is more common in
Africa and is mainly heterosexually transmitted.

704-Which of the following lymphocytes has cytotoxic activity without a specific antigen
receptor and is not antigen specific?

natural killer (NK) cells

B-lymphocytes

macrophages

T-lymphocytes

plasma cells

Your answer was CORRECT


Explanation

Natural killer (NK) cells are a distinct class of lymphocytes that have the ability to lyse a
wide variety of cell types. It is felt that they represent the front-line defense against viral
infections and neoplasia. NK cells may be involved in ocular protection against
cytomegalovirus (CMV) retinitis or herpes simplex ocular infections.

705-The visible wavelengths of light are:

600nm-900nm

500nm-820nm

400nm-780nm

380nm-650nm

Your answer was INCORRECT

Explanation

The visible spectrum of light is 400nm-780nm.

706-Which statement about keratocytes is FALSE:

keratocytes are stellate in shape

keratocytes are connected to neighbouring cells by tight junctions

keratocytes are modified fibroblasts

keratocytes are flattened cells and occur between collagenous lamellae

Your answer was INCORRECT

Explanation

Keratocytes are connected to neighbouring cells by gap junctions

707-Regarding interferon:
it activates macrophages

it acts by neutralizing exotoxin

it is a virus specific molecule

it can be produced by endothelial cells

Your answer was CORRECT

Explanation

Interferon is a glycoprotein produced by T lymphocytes and macrophages. There are


three types: alpha, beta and gamma. It has non-specific antiviral properties. Its effects
include:

 direct anti-proliferative action on tumour and virus infected cells


 activation of natural killer cells and macrophages
 increased expression of class I HLA antigen on tumour or infected cells.

708-Which are the myelin-forming cells of the central nervous system:

microglia

oligodendrocytes

Schwann cells

astrocytes

Your answer was INCORRECT

Explanation

The glial cells of the central nervous system include:

 Oligodendrocytes: the myelin-forming cells of the CNS.

 Astrocytes: perform a variety of structural and functional roles including


metabolic, healing, blood-brain and blood-retinal barriers.

 Microglia: are monocytes and perform phagocytosis


709-The abducens nerve near its exit from the brain stem is pinned down by:

the posterior inferior cerebellar artery

the anterior inferior cerebellar artery

the superior cerebellar artery

the posterior cerebral artery

Your answer was INCORRECT

Explanation

The sixth cranial nerve is pinned down near to its exit from the pons by the anterior
inferior cerebellar artery.

710-Which of the following is true regarding the normal distribution?

it would be unusual for the mean, mode and median to have the same value

approximately 68% of values will fall between limits with a value of -2z and 2z

a value with a z-score of -2 lies two variances below the mean

approximately 5% of values will be above 2 standard deviations from the mean

95% of values will fall within 1.96 standard deviations either side of the mean

Your answer was INCORRECT

Explanation

A normal distribution is a theoretical frequency distribution to which many biological


data approximate. It is defined as a symmetrical bell-shaped distribution in which mean =
mode = median. It is sufficient to describe a normal distribution by mean and SD and by
no other data.

Variance is a measure of the spread of observations about the mean and it is equal to the
SD (Standard deviation) squared.
The units of z-score are in standard deviation such that a value with a z-score of +1 lies 1
standard deviation above the mean.

In a normal distribution:

 About 68% of the observations fall within 1 s.d. on the either side of the mean
 About 95% of the observations fall within 2 s.d. on the either side of the mean
(more exactly, 95% observations fall within 1.96 s.d. either side)
 About 99% of the observations fall within 3 s.d. on the either side of the mean

711-The cornea:

contains Descemet's membrane which is not a true basement membrane

is thicker centrally than peripherally

contains 4,000 endothelium cells per square mm at birth

has a completely acellular collagenous stroma

Your answer was INCORRECT

Explanation

The cornea is thicker peripherally (about 1.2 mm) than centrally (average 0.5 to 0.6 mm).
It contains about about 3-4000 cells per square mm at birth, the number decreases with
age. It contains Descemet's membrane, which is produced by the endothelial cells and
forms its basement membrane. It contains flattened keratocytes within the stroma. Other
cells such as macrophages, lymphocytes or neutrophils may be seen.

712-Whichstructure passes through the superior orbital fissure, usually inferior to the
common tendinous ring:

nasociliary nerve

abducent nerve

inferior division of the oculomotor nerve

inferior ophthalmic vein


Your answer was INCORRECT

Explanation

The inferior ophthalmic vein passes through the superior orbital fissure but usually
outside and inferior to the common tendinous ring. Other options supplied always pass
through the common tendinous ring.

713-Which margin of the orbital rim is thickest:

the lateral margin

the inferior margin

the medial margin

the superior margin

Your answer was CORRECT

Explanation

The lateral margin of the orbital margin is thickest, probably because it is the most
exposed and therefore most prone to trauma.

714-Which statement is FALSE about the ciliary body:

it is supplied with parasympathetic innervation via the short ciliary nerves

contraction of the ciliary body reduces the flow of aqueous though the trabecular
meshwork

it is supplied with blood from both the long posterior ciliary arteries and the anterior
ciliary arteries

it is supplied with sympathetic innervation which synapses in the superior cervical


ganglion

Your answer was INCORRECT

Explanation
Contraction of the ciliary body increases aqueous flow by the action of the anterior fibres
on the scleral spur, which moves inward and posteriorly, thereby opening up the
neighbouring trabecular meshwork.

Score: 25 %
Question 471 Part 3 – Tutor Mode
All of the following structures pass through the superior orbital fissure outside the
common tendinous ring EXCEPT:

the superior division of the oculomotor nerve

the frontal nerve

the trochlear nerve

the lacrimal nerve

Your answer was CORRECT

Explanation

The superior division of the oculomotor nerve passes through the superior orbital fissure
and within the tendinous ring. The other options pass through the superior orbital fissure
but above the tendinous ring.

716-The following are ocular associations of rheumatoid arthritis EXCEPT:

scleromalacia perforans

iritis

scleritis

peripheral corneal ulceration

Your answer was INCORRECT

Explanation

Rheumatoid arthritis due to its associated immune-complex vasculitis impairs the


circulation to the anterior segment of the eye leading to: corneal ulceration, scleritis and
thinning of the sclera (scleromalacia perforans).

717-The basilar artery is a branch of:


the spinal artery

the posterior cerebral artery

the vertebral artery

the posterior communicating artery

Your answer was INCORRECT

Explanation

The basilar artery is derived from the junction of the two vertebral arteries.

718-Choose the one correct statement. Regarding antibodies:

idiotype is determined by genetic variation in the heavy and light chains

they recognise only the peptide antigen

the class is determined by the presence of either kappa or lambda light chains

they consist of one heavy and one light chain

there are always 3 constant domains on each heavy chain

Your answer was CORRECT

Explanation

All immunoglobulins consist of 4 polypeptide chains - 2 identical heavy chains bound to


2 identical light chains. Each light chain consists of a varible domain (the amino acid
sequence varies among light chains of different antibodies) and a constant domain. There
are two types of light chains - lambda and kappa chains. No functional differences have
been found between antibodies composed of lambda or kappa light chains. Each heavy
chain consists of one variable domain and a number of constant domains - IgM and IgE
have 4 heavy chain constant domains, whereas IgG, IgD and IgA have 3 heavy chain
constant domains. The variable domains of both the heavy and the light chains combine
to form the antigen-binding site, and confer specificity on the antibody. The unique
sequence of the variable regions of the antigen-binding site of individual antibody
molecules is termed the idiotype. The heavy chain constant region of different antibodies
differs in structure, and it is this portion of the antibody that determines the class
(isotype) and thus its different properties and effector functions. There are five
immunoglobulin isotypes in humans- IgG, IgM, IgA, IgE and IgD. Antibodies bind to
conformational epitopes i.e., sites on the antigen surface brought together by folding -
they recognise tertiary and quarternary structures of proteins and carbohydrates. Some
antibodies recognise and bind to linear segments of polypeptide. The enormous antibody
repertoire in humans is mostly the result of combinatorial rearrangement of germline
immunoglobulin genes encoding the variable regions of the heavy and light chains.

719-Which of the following best describes the mode of action of gentamicin:

inhibits cell wall synthesis

inhibits protein synthesis

inhibits nucleic acid synthesis

inhibits cell membrane synthesis

Your answer was INCORRECT

Explanation

Gentamicin is an aminoglycoside and it prevents protein synthesis by preventing the


reading of codons from messenger RNA.

720-The most appropriate culture medium for acanthoemaba is:

nutrient-deficient agar with E.coli overlay

McConkey agar

chocolate agar

Lowenstein-Jensen medium

Your answer was CORRECT

Explanation

Acanthoemeba is notoriously difficult to culture. The best medium is nutrient-deficient


agar with E.coli overlay.
721-The termination of Descemet's membrane gives rise to:

the scleral spur

Schlemm's canal

Schwalbe's line

the pigmented trabecular meshwork

Your answer was INCORRECT

Explanation

The termination of Descemet's membrane gives rise to Schwalbe's line, which is an


important landmark in gonioscopy.

722-Which test requires the use of polarized spectacles:

Frisby test

TNO test

Titmus test

Sheridan-Gardiner test

Your answer was INCORRECT

Explanation

The Titmus, TNO and Frisby are all tests of stereoacuity. The Titmus test requires
polarized spectacles to be worn while the TNO test requires red-green spectacles. The
Frisby test is taken without spectacles. The Sheridan-Gardiner is a test of visual acuity
and does not require spectacles or aids.

723-The effect of a new technique of retinal surgery on the length of hospital stay
following retinal detachment is compared to current best practice. Critical level is set at
0.01. A difference is found where p=0.004.

Which of the following is TRUE?


a Type II error would occur if we rejected the null hypothesis when it were true

the probability of a Type I error occurring would be 0.004

the power of this trial may have been increased by looking at a smaller number of
individuals

acting on the basis of a false positive finding is the same as committing a Type II error

the null hypothesis should be retained

Your answer was INCORRECT

Explanation

The null hypothesis states that there is no difference between two populations being
compared and that the difference is due to chance. The p value is the probability of a
Type I error (false positive) which occurs when the null hypothesis is rejected when it is
true. A Type 2 error (false negative) occurs when the null hypothesis is accepted when it
should have been rejected (there is a difference).

724-Regarding MHC molecules:

they are not required for T-cells to recognise antigen

class I molecules consist of a homodimer of an alpha, beta or gamma chain

class II molecules are expressed by all cells

they can determine the responsiveness of an individual to certain peptides

genes encoding MHC molecules are scattered throughout the genome

Your answer was INCORRECT

Explanation

The MHC loci in humans is located on the short arm of chromosome 6. It includes genes
encoding class I, class II and class III molecules.
 Class I antigens (HLA A, B, and C) are present on all nucleated cells. They
consist of a single transmembrane polypeptide chain associated with beta-2-
microglobulin.

 MHC class I molecules present antigenic peptides derived from within the cytosol
(endogenously e.g. viral antigens) to CD8+ T-cells. MHC class II antigens e.g.
(HLA DR, DP, DQ) consist of two polypeptide chains and are found on antigen-
presenting cells (dendritic cell, Langerhans cells, macrophages, B-cells) and
occasionally activated T-cells. These molecules present antigenic peptides derived
from outside cells (exogenously) such as bacterial antigens to CD4+ T-cells.

 MHC class III genes encode complement components including C4 (A and B), C2
and factor B. Other proteins that are encoded within this region include TNF-
alpha, lymphotoxins (LTA, LTB), and 21-hydroxylase.

 Non-classical class I-like genes or class IB molecules (e.g., HLA-E, F, G) are also
encoded by the MHC. These are nonpolymorphic proteins expressed with Beta-2-
microglobulin. The precise role of the non-classical MHC class I molecules is not
clear but may be important in inhibiting killing by NK cells.

T-cells only recognise peptide antigen in combination with MHC.

The MHC haplotype may determine responsiveness to particular peptides by determining


whether the peptide is presented by the MHC molecules that are encoded by the
represented alleles.

725-Regarding T lymphocytes:

Helper T cells enhance the immune response

T lymphocytes contain Fc receptors

Suppressor T cells are the primary effector T cells

T lymphocytes produce antibodies when activated

Your answer was CORRECT

Explanation

B cells, not T cells contain Fc receptors. Cytotoxic T cells are the primary effector T
cells. Helper T cells produce interleukins and together with suppressor (or regulatory) T
cells regulate the immune response.
726-Antibodies perform all of the following functions EXCEPT:

facilitate differentiation of B cells

trigger the complement cascade

neutralization of toxins

opsonisation

Your answer was CORRECT

Explanation

Antibody binding:

 facilitates phagocytosis (a process termed opsonisation)


 neutralizes bacterial toxins
 triggers the complement cascade

Antibody binding plays no role in the differentiation of B cells.

727-Bipolar cell bodies lie in which layer of the retina:

outer nuclear layer

inner plexiform layer

outer plexiform layer

inner nuclear layer

Your answer was INCORRECT

Explanation

Bipolar cell bodies are found in the inner nuclear layer.

728-How long is the intraorbital segment of the optic nerve:


10mm

5 mm

1mm

25 mm

Your answer was INCORRECT

Explanation

The optic nerve segments are as follows:

 intraocular: 1mm
 orbital: 25-30 mm
 intracanalicular: 4-10mm
 intracranial: 10mm

729- All statements are true of the prostaglandins EXCEPT:

they cause pain when applied directly to the nerve endings

non-steroidal anti-inflammatory drugs reduce prostaglandins production by inhibiting


cyclooxygenase

leukotrienes produce increased vascular permeability

thromboxane A2 causes both vasoconstriction and platelet aggregation

Your answer was CORRECT

Explanation

Prostaglandins sensitise the nerve endings to pain but do not themselves produce pain.
Cyclooxygenase is the rate determining enzyme in their production and is inhibited by
non-steroidal anti-inflammatory drugs. Thromboxane A2 causes both vasoconstriction
and platelet aggregation. Leukotrienes are involved in allergic reaction and inflammation.
They produce bronchoconstriction, vasoconstriction and increased vascular permeability

730-All of the following are important to the corneal endothelial pump EXCEPT:
Na/K ATPase

Na/HCO3 ATPase

Na/H antiport

GLUT3 transporter

Your answer was INCORRECT

Explanation

The endothelium pump mechanism is a fairly complex process and involves:

 At the aqueous interface: Na/HCO3 co-transporter and HCO3/Cl exchanger


 At the stromal interface: Na/K ATPase and Na/H antiporter

GLUT is an active glucose transporter and has no role in the endothelium pump.

731-A recent multi-centre study reports that the 10-yr risk of death due to cardiac causes is
12% on placebo and 8% on drug X. The authors conclude that 'a 33% reduction in
cardiac deaths is seen with X'.

The figure '33%' represents which of the following?

number needed to treat

relative risk reduction

control event rate

experimental event rate

absolute risk reduction

Your answer was INCORRECT

Explanation

If a drug reduces the incidence of heart attacks from 10% to 5% then, the control event
rate (CER) is 10% and the experimental event rate (EER) is 5%.
The relative risk reduction (RRR) is 50%.

The absolute risk reduction (ARR) is 5%.

The number needed to treat (NNT) is 100/5 = 20.

732-Which of the following is TRUE about exotoxins:

it is resistant to inactivation by heat

it is produced by both Gram positive and Gram negative bacteria

it is produced by Clostridium botulinum

it can be detected using the Limulus test

Your answer was INCORRECT

Explanation

Exotoxin is produced by Gram positive bacteria. It is made up of protein and is


inactivated by heat. When injected into the body it causes the formation of antibodies.
Botulinum toxin is an exotoxin produced by Clostridium botulinum. Limulus Tests is a
sensitive method for detection of bacterial endotoxins.

733-Which statement about ras oncogenes is false:

they are mutated proto-oncogenes

they encode for proteins which act as signal transducers

they are only expressed in malignant tissues

they are highly conserved in evolution

they encode for proteins with sequence homology to G proteins

Your answer was INCORRECT

Explanation
Oncogenes were originally found as genes carried by cancer-inducing viruses. They are
the mutated form of normal eukaryote genes called proto-oncogenes, which are highly
conserved in evolution because of their central role in cell growth and differentiation.
Proto-oncogenes/oncogenes encode for growth factors, growth factor receptors, enzymes,
and transcription factors and in the case of ras oncogenes, intracellular signal transducers,
low molecular weight G-proteins. Under normal circumstances, G-proteins bind GTP
when activated, and are inactivated by their own GTPase activity. A single amino acid
mutation is sufficient to change the ras proto-oncogene into an oncogene. Ras oncogenes
inhibit their GTPase activity resulting in ongoing activation. Ras is the commonest
oncogene in human tumors, others include myc, rel, fos, and jun.

734-Which of the following cells is typically found in a granuloma:

Epithelioid cells

B lymphocytes

Langerhan's cells

Neutrophils

Your answer was CORRECT

Explanation

A typical granuloma contains multinucleated giant cells (called Langhan's cells)


surrounded by epitheliod cells, which are activated macrophages. Langhan's cells must
not be confused with Langerhan's cells, which are dendritic cells in the epidermis.

735-Which statement is FALSE about the light reflex of the pupil:

it is impaired in patients with unilateral cataract

it involves the superior colliculus

it is impaired in destructive lesion of the pretectal nuclei

it is impaired if the ciliary ganglion is damaged

Your answer was CORRECT

Explanation
The afferent pathway of the pupillary light reflex involves in order: the optic nerves,
optic chiasm, optic tracts, the pretectal nuclei in the midbrain, and finally (bilateral
projection to) the Edinger-Westphal nuclei. The efferent (parasympathetic) pathway
involves: the Edinger-Westphal cell group of the oculomotor nucleus, the ciliary ganglion
(synapse), and the short ciliary nerves reaching the iris. Cataract does not impair the
pupillary light reflex.

736-Which best describes the course of the ophthalmic artery in the orbit in relation to the
optic nerve:

inferior to lateral

inferior to medial

medial to superior

superior to lateral

Your answer was INCORRECT

Explanation

The ophthalmic artery enters the orbit infero-lateral to the optic nerve, with which it is in
close association. In the posterior third of the orbit the ophthalmic artery travels
superiorly and medially across the optic nerve to travel superior and medially to the optic
nerve.

737-The ration of cones: bipolar cells : ganglion cells in the fovea is usually:

2:1:1

1:1:1

1:1:3

3:1:1

1:2:2

Your answer was INCORRECT


Explanation

In the fovea the ratio is 1:1:1, while in the periphery the bipolar cell may receive input
from 50-100 rods.

738-Which modality of treatment uses infrared light:

photodynamic therapy

NdYAG capsulotomy

pan retinal photocoagulation

transpupillary thermotherapy

Your answer was INCORRECT

Explanation

Transpupillary thermotherapy (TTT) uses infrared light to heat the treated area (up to 40
degrees Celsius). It is used in the treatment of choroidal melanoma.

739-The anteroposterior diameter of the adult eye is:

25-27mm

23-25mm

20-23mm

27-30mm

Your answer was INCORRECT

Explanation

The normal anteroposterior diameter of the adult eye is 24.5mm though this varies, being
longer in myopes and shorter in hypermetropes.

740-Which is false regarding translation:


aminoacyl-tRNA synthetase is an important enzyme

it occurs in the cytoplasm

it occurs in a 5 to 3 direction

anticodons occur on mRNA

Your answer was INCORRECT

Explanation

Anti-codons refer to the complementary sequences of nucleotides on the tRNA molecules


as they attach onto the mRNA molecule at the ribosome.

741-All are true of acetazolamide EXCEPT:

it reduces intra-ocular pressure by reducing aqueous production

its site of action is the non-pigmented ciliary epithelium

it can cause renal stones

it can cause hyperkalaemia

it is a carbonic anhydrase inhibitor

Your answer was INCORRECT

Explanation

Acetazolamide is a carbonic anhydrase inhibitor which reduces IOP by its effect on the
non-pigmented ciliary epithelium secretory function. Acetazolamide has a number of side
effects including:

 hypokalaemia
 acidosis
 fatigue
 paraesthesia
 dermatitis
 renal stones

742-Class II MHC proteins are found on the surface of all of the following EXCEPT:
Langerhans' cells

dendritic cells

neutrophils

macrophages

Your answer was INCORRECT

Explanation

Class II MHC proteins are found only on surface of certain cells: macrophages, B cells,
dendritic cells of the spleen, Langerhans cells of the skin.

743-Which is FALSE regarding a focimeter:

the target is usually a ring of dots

the collimating lens is fixed

prismatic power can be measured

determination of bifocal add requires back vertex power measurement

Your answer was INCORRECT

Explanation

Spectacles are designated by their back vertex power, so this is what is measured for the
distance element of spectacles in a focimeter. However, to determine the near segment
add in bifocal lenses, it is necessary to measure the front vertex power (as the near add is
at the front of the spectacle) and subtract the front vertex power of the distance from the
front vertex power of the near to determine the near add.

744-The first cell in the eye to produce melanin is:

retinal pigment epithelium

iris melanocyte
uveal melanocyte

conjunctival melanocyte

Your answer was CORRECT

Explanation

The retinal pigment epithelium cells are the first in the eye to produce melanin.

745-The most comprehensive method to test colour vision is:

D-15

Lanthony New Colour Test

Ishihara plates

Farnsworth-Munsell hue 100

Your answer was INCORRECT

Explanation

The most comprehensive test for colour vision is the Farnsworth-Munsell hue 100 which
comprises 84 coloured discs in four groups of 21. The D-15 test is shorter and cannot
distinguish mild defects. The Ishihara plates are designed for discerning red-green defects
(the most common inherited) though they are often used as a screen for reduced colour
vision. The Lanthony New Colour Test is designed for children.

746-Which statement about the primary visual cortex is FALSE:

the area above the calcarine fissure is known as the cuneus gyrus

it extends as far posteriorly as the occipital pole

it occurs on the lateral surface of the occipital cortex

it is supplied by the posterior cerebral artery


Your answer was INCORRECT

Explanation

The primary visual cortex occurs on the medial surface of the occipital cortex. The area
above the calcarine fissure is known as the cuneus gyrus and that below the fissure
known as the lingual gyrus.

747-B lymphocytes:

differentiate into either plasma cells or memory cells on antigen exposure

play an important part in cell-mediated immunity

constitute 70% of the circulating pool of small lymphocytes

can be easily differentiated from T lymphocytes using the light microscope

Your answer was CORRECT

Explanation

B cells play an important part in humoral immunity while T cells are involved in cell-
mediated immunity. On encountering antigens, B cells may differentiate into plasma cells
which produce antibodies or memory cells. B cells constitute 30% of circulating
lymphocytes while T cells constitute 70%. B and T cells cannot be distinguished using
light microscope.

748-The main collagen of the corneal stroma is:

Type V

Type II

Type VII

Type I

Your answer was INCORRECT

Explanation
Type I collagen comprises 50-55% of the collagen in the corneal stroma. The regular
arrangement and regular fibril diameter of the Type 1 collagen in the corneal stroma is
believed to be important in maintaining its clarity. Type VI collagen is also important in
the stroma, while Types III and V are less abundant but may play important roles in
maintaining regularity of spacing.

749-Which statement regarding the rate of diffusion of a drug across a membrane is


FALSE:

it is inversely proportional to the thickness of the membrane

it is directly proportional to the difference in concentration across the membrane

it is increased the more hydrophilic the drug

it is decreased the more ionized the drug

Your answer was INCORRECT

Explanation

According to Fick's Law:

Rate of diffusion = diffusion coefficient x concentration gradient / membrane thickness

The diffusion coefficient itself is a function of lipid solubility, ionization and molecular
size.

750-Which is TRUE regarding the pinhole:

it fully corrects all refractive errors

the stenopaeic slit acts as an elongated pinhole

it can help differentiate between macular and optic nerve disorders

multiple pinholes have more effect than a single aperture

Your answer was INCORRECT

Explanation
The pinhole overcomes only low to moderate refractive error. Pinhole exam cannot
differentiate macular from optic nerve disorders. Single aperture pinholes are as effective
as multiple aperture pinholes.

751-Which statement is FALSE regarding G proteins:

they bind GTP but not GDP

they are found in all types of cells

they are reduced in activity in pseudohypoparathyroidism

they are commonly mutated in patients with acromegaly

they link cell surface receptors to secondary messenger pathways

Your answer was CORRECT

Explanation

G proteins link an external stimulus or first messenger with secondary messenger


pathways that ultimately lead to a change in the state of the cell. This is a fundamental
and ubiquitous process in biology. Activation of a receptor by the first messenger causes
the G protein to release GDP and bind GTP enabling interaction with a secondary
messenger system such as cyclic-GMP. Increased G protein activity is seen in 40% of
patients with pituitary somatotroph tumors and also following cholera infection.
Decreased G protein activity is seen in pseudohypoparathyroidism.

752-Which statement about Clostridium tetani is FALSE:

it produces an exotoxin

it causes gas gangrene

it is an obligatory anaerobe

it is motile

Your answer was INCORRECT

Explanation
Clostridium is a genus of Gram positive obligate anaerobes capable of producing
endospores. Gangrene is caused by Clostridium perfringins while tetanus is produced by
the tetanus toxoid of Clostridium tetani.

653-What is the most common lacrimal gland tumour:

adenocarcinoma

adenoid cystic carcinoma

pleomorphic adenoma

mucoepidermoid carcinoma

Your answer was INCORRECT

Explanation

Pleomorphic adenoma, otherwise known as benign mixed tumour, is the most common
tumour of the lacrimal gland. It is a benign, slow growing lesion and has a high
recurrence rate following excision. Excision is recommended as there is a risk of
malignant conversion to pleomorphic carcinoma.

754-Which statement is FALSE about the corneal stroma:

the collagenous lamellae are oriented parallel to the corneal surface

collagenous lamellae are orientated at right angles to each other

it is a dense connective tissue of high regularity

it is composed of 20-40 layers of flattened collagenous lamellae

Your answer was INCORRECT

Explanation

The corneal stroma is composed of 200-250 layers of flattened collagenous lamellae.


Other options are true.

755-Which is TRUE regarding interferon:


it acts by neutralizing exotoxin

it binds to specific viral sequences

it integrates with the DNA of virus infected cells

it increases HLA expression on the cell surface

Your answer was INCORRECT

Explanation

Interferon is a glycoprotein produced by T lymphocytes and macrophages. There are


three types: alpha, beta and gamma interferon. It has has non-specific antiviral properties
including: direct antiproliferative action on tumour and virus infected cells, activation of
natural killer cells and macrophages, increased expression of class I HLA antigen on
tumour or infected cells.

756-The oculomotor nerve nucleus lies in:

the medulla

the midbrain

the hypothalamus

the pons

Your answer was INCORRECT

Explanation

The oculomotor nerve nucleus is found in the periaqueductal grey matter in the midbrain.

757-Regarding DNA analysis which statement is FALSE:

Northern blotting is used to detect DNA sequences

Southern blotting is primarily used to detect DNA sequences


Western blotting is used to detect specific proteins in a mixture of protein

Gel electrophoresis is used in both Southern and Northern blotting

Your answer was CORRECT

Explanation

Northern blotting is used for detecting RNA sequences. Other statements are true.

758-A25-year-old patient is known to have C3 deficiency. This is likely to predispose to


which form of infection?

Mycoplasma

Neisseria

Legionella

Pneumococcus

Staphylococcus

Your answer was INCORRECT

Explanation

C3 deficiency predisposes to life threatening infections by encapsulated organisms such


as pneumococcus.

759-All of the following agents are active against Pseudomonas EXCEPT:

gentamicin

neomycin

ciprofloxacin

benzylpenicillin

Your answer was INCORRECT


Explanation

Pseudomonas is naturally resistant to penicillins and most beta-lactam derived antibiotics.


Pseudomonas is sensitive to aminoglycoside and quinolone antibiotics.

760-The suspensory ligament of Lockwood arises from all the following structures except:

the sheath of inferior rectus muscle

check ligaments of lateral rectus muscle

the sheath of superior rectus muscle

check ligaments of medial rectus muscle

Your answer was INCORRECT

Explanation

The suspensory ligament of Lockwood forms a hammock-like support for the globe. It is
composed of:

 the sheath of inferior rectus


 the medial check ligament
 the lateral check ligament
 sheath of inferior oblique

Score: 25 %
Question 517 Part 3 – Tutor Mode
A post-marketing surveillance study of a new drug therapy for a cardiac condition was
carried out on 20,000 subjects who had completed clinical trials.

Which one of the following accurately reflects information generated from such a study?

cost benefit analysis

cost effectiveness

adverse events profile

comparative therapeutic benefit


Your answer was INCORRECT

Explanation

Phase 0 trials are also known as human microdosing studies.

Phase I trials are designed to assess the safety (pharmacovigilance), tolerability,


pharmacokinetics, and pharmacodynamics of a drug.

Phase II studies are sometimes divided into Phase IIA and Phase IIB.

 Phase IIA is specifically designed to assess dosing requirements (how much drug
should be given).
 Phase IIB is specifically designed to study efficacy (how well the drug works at
the prescribed doses).

Phase III studies are randomized controlled multicenter trials on large patient groups
(300-3,000 or more depending upon the disease/medical condition studied) and are aimed
at being the definitive assessment of how effective the drug is, in comparison with
current 'gold standard' treatment.

Post-marketing surveillance (phase IV study) assesses potential adverse effects of newly


marketed drugs.

762-Which is FALSE regarding Best disease:

a yellow disc is visible over the fovea on fundoscopoy

there is accumulation of lipofuscin in the retinal outer plexus

it involves degeneration of the macula

it is autosomal dominant

Your answer was INCORRECT

Explanation

Best disease is a hereditary autosomal dominant macular degeneration characterized


histologically by massive accumulation of lipofuscin in the RPE cells with atrophy of the
overlying photoreceptor layer.

763-Which is FALSE regarding the rods:


their outer segment is thinner than the cone photoreceptor

the lamellae are enclosed by a cell membrane

maximum spectral sensitivity is 555nm

the outer segment contains rhodopsin

Your answer was INCORRECT

Explanation

Maximal spectral sensitivity of the rods is 496 nm.

764-The central corneal thickness is:

0.4 mm

0.7 mm

0.5 mm

0.6 mm

0.3 mm

Your answer was INCORRECT

Explanation

Average central corneal thickness is 0.52mm.

765-Which is FALSE regarding the cross-cylinder:

the patient is asked to look at the smallest line he can see on the chart

the 0.50D cylinder is used to check the power in patients with good VA

the 1.00D cylinder is used to check the power in patients with poor VA
the 1.00D cylinder is used to check the axis of the cylinder in all patients

Your answer was CORRECT

Explanation

A patient is usually asked to look at two lines above the smallest he can see, allowing him
to discriminate the sharpness of images with the cross. Other options are true.

765-Which of the following organelles contains a double stranded circular DNA?

peroxisome

Golgi body

nucleus

ribosome

mitochondrion

Your answer was INCORRECT

Explanation

Mitochondrial DNA (mtDNA) is a small, closed-circular molecule of double stranded


DNA that exhibits a strict maternal transmission and is present within cells in multiple
copies.

766-Which statement is FALSE regarding the vascular and nerve supply of the lids:

the lids are a site of anastomoses between the internal and external carotids

the infraorbital nerve supplies the lower lid medially

the zygomaticofacial nerve supplies the upper lid laterally

the pretarsal portion derives its blood supply from branches of the external carotid

Your answer was INCORRECT

Explanation
The zygomaticofacial nerve supplies the lower lid laterally, while the lateral palpebral
branch of the lacrimal nerve supplies the upper lid laterally. The infraorbital nerve
supplies the medial two-thirds of the lower lids, while the supraorbital and supratrochlear
nerves supply the medial two-thirds of the upper lid. Regarding vascular supply, the
pretarsal portion derives its arterial supply from the superficial temporal and facial
arteries, which are branches of the external carotids, while the post-tarsal portion is
supplied by branches of the anterior ciliary arteries which are branches of the ophthalmic
artery (internal carotid).

767-The blood supply to the laminar part of the optic disc is from:

the internal carotid artery

the short posterior arteries

the long posterior arteries

the central retinal artery

Your answer was INCORRECT

Explanation

The blood supply to the optic nerve head is as follows:

 superficial nerve fibre layer: branches of the central retinal artery

 prelaminar: peri-papillary choroidal arterioles

 laminar: short posterior ciliary arteries

 post-laminar: small branches of central retinal artery and pial branches from
choroidal arterioles, central retinal artery and ophthalmic artery.

768-Which statement is FALSE about the oculomotor nerve:

it can be affected by an anterior communicating artery aneurysm

its nucleus resides in the midbrain

it traverses the cavernous sinus before entering the orbit


it divides into superior and inferior divisions near the superior orbital fissure

Your answer was CORRECT

Explanation

The oculomotor nerve runs alongside the posterior communicating artery on its journey
to the orbit and can be compromised by a PCA aneurysm.

769-The corneal endothelial cell density is:

1500 cell/mm2

3000 cell/mm2

1000 cell/mm2

5000 cell/mm2

Your answer was INCORRECT

Explanation

Average corneal endothelial density in adulthood is between 2500 and 3000 cells/mm2.

770-Which complement component is present in the highest serum concentrations?

C9

C1q

C3

C4

C5

Your answer was INCORRECT

Explanation
C3 is present in serum at the highest concentration.

771-How thick is the corneal epithelium:

100 micrometers

5 micrometers

50-60 micrometers

25 micrometers

Your answer was INCORRECT

Explanation

The corneal epithelium is 50-60 micrometers thick.

772-Which statement is FALSE about the corneal epithelium:

it is a stratified keratinized squamous epithelium

cells are held together by numerous desmosomes

it is 50-60 micrometers thick

cells are held to the underlying basal lamina by hemidesmisomes

Your answer was CORRECT

Explanation

The cornea is a stratified non-keratinized squamous epithelium. Other options are true.

773-Which structure drains into the middle meatus:

the posterior ethmoidal sinus

the maxillary sinus

the sphenoid sinus


the nasolacrimal duct

Your answer was INCORRECT

Explanation

The nasal meatus are as follows:

 the superior meatus receives the opening of the posterior ethmoidal sinuses
 the middle meatus receives the openings of the anterior ethmoidal sinuses, frontal
sinus and the maxillary sinus
 the inferior meatus receives the opening of the nasolacrimal duct

774-Which is FALSE regarding topical steroid eye drops:

they must enter the cell in order to have an effect

they can cause increased intraocular pressure

prednisolone phosphate has better corneal penetration than prednisolone acetate

they can cause cataract

Your answer was INCORRECT

Explanation

Prednisolone acetate is more lipophilic and therefore penetrates the cornea better than
prednisolone phosphate. Other options are true.

775-All of the following are Gram negative rods EXCEPT:

Pseudomonas

Haemophilus

Neisseria

E. coli
Brucella

Your answer was INCORRECT

Explanation

Neisseria are Gram negative cocci. All other options are Gram negative rods.

776-Zonulae adherens are otherwise known as:

gap junctions

tight junctions

hemidesmosomes

desmosomes

Your answer was INCORRECT

Explanation

Zonulae adherens are desmosmes, which are specialized adherens junctions between
cells. There main role is in mechanical adhesion.

777-Which statement is FALSE about the ciliary muscle:

its inner circular muscle fibres are also known as Muller's muscle

its outer longitudinal fibres are attached to the scleral spur

its middle radial fibres are continuous with the corneoscleral trabeculae

it comprises the anterior two-thirds of the ciliary body

during accommodation the ciliary body moves backward and inward

Your answer was INCORRECT

Explanation
During accommodation, it is believed that the ciliary body moves forward and inward,
which lessens the tension on the zonules, allowing increased curvature of the lens. Other
options supplied are true.

778-All of the following nerves pass through the cavernous sinus EXCEPT:

oculomotor nerve

mandibular nerve

trochlear nerve

maxillary nerve

ophthalmic nerve

Your answer was INCORRECT

Explanation

The cavernous sinus receives the third, fourth, sixth, V1 and V2 but not the mandibular
nerve (V3).

779-Which is FALSE of glycolysis:

it produces two molecules of ATP and two molecules of NADH

it occurs in the mitochondria

it does not require oxygen

it results in glucose splitting into two molecules of pyruvate

Your answer was INCORRECT

Explanation

Glycolysis is the conversion of glucose to two molecules of pyruvate with the production
of two NADH and two ATP molecules. It occurs in the cytoplasm and does not require
oxygen.

780-Which cells play the role of wandering phagocytes in the retina:


Muller cells

microglia

retinal pigment epithelial cells

astrocytes

Your answer was INCORRECT

Explanation

The microglia are responsible for phagocytosis and immune defense in the retina.

The function of the various retinal neuroglial cells is summarized below:

 The Muller cells: are the principle supporting glial cells of the retina and are
analogous to central nervous system oligodendrocytes.

 The astrocytes: form a honeycomb scaffold perpendicular to the Muller cells.


They are responsible among other functions for laying down scar tissue in injured
or diseased retina.

 The microglia: are highly specialized mononuclear phagocytic cells

781-Insulin:

is made up of two chains joined by three disulphide bonds

has an elimination half-life of about 60 minutes after subcutaneous injection

is synthesised by the alpha cells of the islets of Langerhans

circulates in the blood mostly bound to globulin

Your answer was CORRECT

Explanation
Insulin contains two chains, A and B, joined by three disulphide bonds. Insulin is
secreted by the beta cells of the islets of Langerhans as proinsulin. It circulates in the
blood mainly as free hormones. Despite its short half-life of 10 minutes, its
pharmacological effect is prolonged due to bonding with the tissue receptors.

782-Functions of the retinal pigment epithelium include all EXCEPT:

phagocytosis of rod and cone outer segments

transport and storage of metabolites and vitamins

reflection of excess light

selectively permeable barrier between the choroid and neuroretina

Your answer was INCORRECT

Explanation

The melanin of the RPE assists in the absorption (not reflection) of light within the eye,
thus reducing scatter. Other options are true.

783-Which statement is FALSE about the iris:

the anterior border layer is composed of modified stroma which is deficient in areas

the iris stroma is a loose connective tissue in free communication with the aqueous

the dilator pupillae is composed of myoepithelial cells which are lightly pigmented and
innervated by sympathetic fibres

the apices of the posterior pigment epithelium oppose the basal processes of the
overlying myoepithelial cells

Your answer was INCORRECT

Explanation

The apices of the heavily pigmented posterior pigment epithelium oppose the apices of
the myoepithelial cells and this doule layer of epithelium constitutes the major site of the
blood-aqueous barrier.
784-All of the following cause granulomatous inflammation EXCEPT:

mycobacterium

Bartonella

Borrelia bergdorferi

Treponema pallidum

Moraxella

Your answer was INCORRECT

Explanation

Granulomatous reactions are caused by intra-cellular organisms, which initiate a cell


mediated response with marcrophages, lymphocytes and plasma cells. Moraxella is a
Gram negative bacilli and would not typically cause a granulomatous response.

785-Which of the following arteries and its branches supplies the lateral geniculate body:

anterior cerebral

lateral striate

posterior cerebral artery

middle cerebral artery

Your answer was INCORRECT

Explanation

The posterior cerebral artery supplies the LGN via its branches called the posterior
choroidal arteries. The anterior choroidal artery, a branch of the internal carotid, also
helps to supply the LGN.

786-The corneal endothelium is held in to its underlying basal lamina by:

gap junctions
tight junctions

hemidesmosomes

desmosomes

Your answer was INCORRECT

Explanation

The corneal endothelium is held to its underlying basal lamina by hemidesmosomes.


Endothelial cells are held together to each other by desmosomes.

787-The following are true of G-protein cell signaling EXCEPT:

the a-subunit of G protein activates adenylate cyclase

G-proteins are made up of three subunits

protein kinase is inactivated by cAMP

activation of a membrane receptor causes an exchange of GDP for GTP

Your answer was INCORRECT

Explanation

Protein kinase is activated by cAMP. All other statements are true.

788-The following are major functions of the complement system EXCEPT:

opsonization

cytolysis of micro-organisms

removal of antibody-antigen complexes

production of inflammatory mediators

Your answer was INCORRECT


Explanation

The complement system has three main functions: opsonisation, cytolysis of pathogenic
organisms, and the production of inflammatory mediators. Opsonisation refers to the
process in which C3 is attached to immune complexes, bacteria or bacterial products.
Removal of antibody-antigen complexes is NOT a role of complement.

789-Which statement is FALSE about the corneal endothelium:

it is a simple squamous epithelium

it has a critical role in maintaining the state of corneal hydration

the cells are polygonal in shape

the Descemet's membrane is not the true basement membrane of the endothelium

Your answer was INCORRECT

Explanation

The Descemet's membrane is indeed the modified basement membrane of the corneal
endothelium. Other options supplied are true.

790-Which is FALSE regarding the optic chiasm:

upper retinal fibres lie superiorly in the chiasm

superior nasal retinal fibres form Wilbrand 's knee

macula fibres primarily decussate posteriorly and superiorly

inferior nasal fibres decussate anteriorly and inferiorly

Your answer was INCORRECT

Explanation

The layout of the optic chiasm fibres are as follows:

 temporal hemiretinal fibres continue ipsilaterally


 nasal hemiretinal fibres decussate
 inferior nasal fibres in decussating loop forward into the opposite optic nerve for a
short distance called Wilbrand 's knee (junctional scotoma)
 fibres from upper retinal quadrants lie superior
 fibres from the lower retinal quadrants lie inferior
 inferior nasal fibres decussate anteriorly and inferiorly
 superior nasal fibres decussate posteriorly and superiorly
 macula fibres decussate primarily in the posterior superior portion of the chiasm

791-Mean systolic blood pressure (SBP) of a sample of 100 consultants is 162 mmHg with
a standard deviation of 10 mmHg.

Which of the following can be said?

approximately 68% of consultants will have an SBP between 152 and 172 mmHg

z-score of a consultant with an SBP of 182 mmHg is +0.5

z-score of a consultant with an SBP of 132 mmHg is +3

variance equals 20 mmHg

approximately 68% of consultants will have an SBP between 157 and 167 mmHg

Your answer was CORRECT

Explanation

68% of observations fall within +- 1SD of the mean. Accordingly, about 68% of the
observations will fall between 152 mmHg and 162 mmHg. Note variance is equal to the
standard deviation squared. A z-score of +1 indicates that the value is 1 standard
deviation above the mean.

792-With regard to cellular signal transduction which of the following statements is


FALSE:

secondary messengers cannot be activated other than by a G protein

the acetylcholine receptor acts as a sodium channel

steroid hormones affect gene expression by indirectly binding to DNA


reduced G protein activity is seen in Albright's hereditary osteodystrophy

G proteins typically interact with first and secondary messengers

Your answer was CORRECT

Explanation

The steroid hormone receptor complex binds to DNA. Steroid hormones bypass the need
for G-proteins; and some membrane receptors activate secondary messenger systems via
direct enzymatic action, usually in a cascade mechanism. Albright's hereditary
osteodystrophy is pseudohypoparathyroidism. This syndrome is associated with
mutations in the gene encoding a subunit of a G protein (Gsa). Second messenger
systems include: adenylate cyclase/cyclic AMP, phosphatidylinositol/diacylglycerol, and
receptor-linked ion channels.

793-The arachnoid villi drain CSF into:

the straight sinus

the sigmoid sinus

the superior sagittal sinus

the cavernous sinus

Your answer was INCORRECT

Explanation

The superior sagittal sinus drains the cerebral bridging veins and, through the arachnoid
villi, the cerebrospinal fluid.

794-Which statement is FALSE about Descemet's membrane:

it is rich in laminin

it is continuous posteriorly with the ciliary body

it is rich in basement membrane glycoproteins


it is the modified basement membrane of the corneal endothelium

Your answer was INCORRECT

Explanation

It is continuous posteriorly with the trabecular meshwork. Other options are true.

795-Which is FALSE regarding the refractive index:

the refractive index of a material is related to its optical density

the refractive index from medium 1 to medium 2 is the refractive index of medium 2
divided by the refractive index of medium 1

the absolute refractive index of air is approximately zero

the refractive index from medium 1 to medium 2 is equal to the angle of incidence
divided by the angle of refraction

Your answer was INCORRECT

Explanation

The refractive index from medium 1 to medium 2 is equal to the sin of the angle of
incidence divided by the sin of the angle of reflection.

796-Which of the following nerves is positioned most medially in the cavernous sinus:

oculomotor nerve

abducent nerve

trochlear nerve

maxillary nerve

Your answer was INCORRECT

Explanation
The abducent nerve is located most medially in the cavernous sinus. It is more likely to
be involved in cavernous sinus thrombosis because it is less protected than the other
nerves III, IV, V1 and V2 which are against and protected by the lateral wall of the sinus.

797-Chloroquine causes which ocular side effect:

cataract

retinal degeneration

glaucoma

optic neuritis

Steven's Johnson syndrome

Your answer was INCORRECT

Explanation

Chloroquine binds to melanin in the RPE cells leading to long-term retinal toxicity.

798-The vertebral artery is derived from:

the spinal artery

the ascending aorta

the basilar artery

the subclavian artery

Your answer was INCORRECT

Explanation

The vertebral arteries are derived from the subclavian artery. They give rise to the
anterior spinal arteries and the posterior inferior cerebellar arteries. They then join
together to form the basilar artery.

799-A group of experienced ophthalmic surgeons report on a randomised placebo-


controlled trial comparing a particular retinal surgery technique as compared to a sham
operation. Their study concludes that 'using this advanced surgical technique reduces the
risk of vision loss from 4.3% to 3.8% (p<0.05)'.

What has this study proved about the surgical procedure:

its efficacy

its acceptability

its usefulness

its effectiveness

its safety

Your answer was CORRECT

Explanation

This is an experienced group of retinal surgeons working in ideal conditions and


demonstrates procedural efficacy. It is often difficult to generalise the findings in a study
group to everyday practice.

Efficacy = the effect of something under ideal or laboratory conditions.

Effectiveness = the effect of something in the real world.

800-Which statement about necrotizing fasciitis is FALSE:

high doses of benzylpenicillin given intravenously are indicated

streptococci are most commonly implicated

excision of the necrotic tissue should be carried out within 48 hours

hyperbaric oxygen may limit the spread

Your answer was INCORRECT

Explanation
Wide surgical excision should be carried out immediately without delay. Other options
are true.

801-The lateral geniculate nucleus consists of how many laminae or cell layers:

15

Your answer was INCORRECT

Explanation

The LGN consists of six laminae or cell layers.

802-In the orbit:

the ciliary ganglion is medial to the optic nerve

the ophthalmic artery is the first branch of the internal carotid artery

the ophthalmic artery emerges from the optic canal

the superior ophthalmic vein passes through the common tendinous ring

Your answer was INCORRECT

Explanation

The superior ophthalmic vein passes backward through the superior orbital fissure
outside the common tendinous ring. The ophthalmic artery is NOT the first branch of the
internal carotid artery. It enters the orbit through the optic canal. The ciliary ganglion is
situated lateral to the optic nerve.

803-Which ocular condition is predisposed in a patient with homocystinuria:


macular degeneration

glaucoma

dislocation of the lens

retinal macroaneurysm

angle recession

Your answer was INCORRECT

Explanation

Patients with homocystinuria are more likely to suffer lens dislocation due to weakened
zonular fibres.

804-A prism which produces a linear apparent deviation of 2cm of an object 1 meter away
has a power of:

2 dioptres

0.5 dioptres

20 dioptres

1 dioptre

Your answer was CORRECT

Explanation

1 prism diopter of power is defined as that which produces a linear apparent displacement
of 1cm of an object 1 m away. A displacement of 2cm therefore represents a power of 2
dioptres.

805-The image formed by an object lying between the centre of curvature and the principle
focus of a concave mirror is:

erect, virtual and enlarged


real, inverted and diminished

real, inverted and enlarged

virtual, inverted and enlarged

Your answer was INCORRECT

Explanation

For an object lying between the centre of curvature and the principle focus of a concave
mirror, the image formed is real, inverted and enlarged; and it lies behind the centre of
curvature

806-Regarding the orbit:

the lateral walls are approximately parallel

The volume of the orbit is 30ml

the medial walls are set approximately 45 degrees from the sagittal plane

the orbit is widest at the orbital margin

the orbit is roughly pyramidal in shape with base at the optic canal and apex at the orbital
margin

Your answer was INCORRECT

Explanation

The orbit is pyramidal with apex at the optic canal and base at the orbital margin. The
medial walls are approximately parallel, while the lateral walls are set approximately 45
degrees from the sagittal plane. The orbit is widest 1.5cm behind the orbital margin. The
volume of the orbits is approximately 30ml.

807-A subdural haematoma is most likely to result from:

rupture of a Berry aneurysm


laceration of the middle meningeal artery

embolus of the anterior cerebral artery

laceration of the superior cerebral bridging veins

Your answer was INCORRECT

Explanation

A subdural haematoma is most likely to occur from laceration to the superior cerebral
bridging veins. An epidural haematoma occurs from rupture of the middle meningeal
artery, while a ruptured Berry aneurysm would cause a sub-arachnoid haemmorhage.

808-Which of the following are typical histological changes seen in benign hypertension:

hyalinization and intimal proliferation of the muscular media of the medium sized
arteries and arterioles

loss of endothelial cells of arterioles

formation of new vessels

fibrinoid necrosis of small arteries and arterioles

Your answer was CORRECT

Explanation

Intimal proliferation and hyalinization of the muscular media are the two histological
features commonly seen in benign hypertension especially in the medium-sized renal
arteries and renal arterioles. Fibrinoid necrosis of small arteries and arterioles are seen in
malignant hypertension. Loss of endothelial cells of arterioles occur in diabetes mellitus.

809-The cavernous sinuses:

extend from the inferior orbital fissure to the apex of the temporal bone

contain the internal carotid artery on their lateral wall

open posteriorly into the petrosal sinuses


contain the oculomotor nerve on their medial wall

Your answer was INCORRECT

Explanation

The cavernous sinuses extend from the superior orbital fissure to the apex of the temporal
bone. They open posteriorly into the petrosal sinuses. They contain the internal carotid
artery on their medial wall and the oculomotor nerve on their lateral wall.

810-The following arteries form the circle of Willis EXCEPT:

the posterior cerebral arteries

the posterior communicating arteries

the middle cerebral arteries

the anterior cerebral arteries

Your answer was INCORRECT

Explanation

The following arteries make up the circle of Willis:

 Anterior cerebral arteries


 Anterior communicating artery
 Internal carotid arteries
 Posterior cerebral arteries
 Posterior communicating arteries

The basilar artery and middle cerebral arteries, though they supply the brain, are not
considered part of the circle.

811-Cotton wool spots in the retina occur at the level of:

the outer segments

the outer plexiform layer


the retinal pigment epithelium

the nerve fibre layer

the internal limiting membrane

Your answer was INCORRECT

Explanation

Cotton wool spots are fluffy white swellings in the nerve fibre layer of the retina due to
ischaemia and interruption of axoplasmic flow.

812-The inability to visualize the iridocorneal structures of the eye on a slit lamp without
the assistance of special lenses is an example of the optical principle of:

chromatic aberration

diffraction

total internal reflection

spherical aberration

Your answer was INCORRECT

Explanation

Total internal reflection occurs when light moving from a denser medium to a less dense
medium hits the surface at an angle greater than the critical angle. At this point, light is
reflected back rather than passing into the second medium. It explains why the structures
of the iridocorneal cannot be viewed without the assistance of special mirrored devices.

813-A lens with a focal length of 10 meters has a power of:

10 dioptres

0.01 dioptres

0.1 dioptres
1 dioptres

Your answer was INCORRECT

Explanation

The dioptric power of a lens is equal to the reciprocal of the focal length measured in
meters.

814-Which of the following is FALSE regarding central retinal artery occlusion:

it often leads to rubeotic glaucoma

it results in a cherry red spot on fundoscopy

it is more commonly due to embolus than thrombosis

it is associated with hypertension

Your answer was CORRECT

Explanation

Central retinal artery occlusion leads to total infarction of the inner retinal tissue. This
means there are no vasoactive factors released to drive neovascularisation. As a result
rubeotic glaucoma is rare occurring in fewer than 5% of cases, unlike CRVO where up to
50% are affected with rubeosis.

815-Which statement is FALSE regarding leakage of lens protein into the anterior
chamber:

it can induce a giant cell granulomatous reaction

it can physically block trabecular outflow called phakolytic glaucoma

it can occur spontaneously

it can cause a rise in pressure by macrophages obstructing the trabecular meshwork

Your answer was INCORRECT

Explanation
Lens protein leaking into the anterior chamber can occur spontaneously in a cataractous
lens or secondary to trauma. The lens protein can induce a massive giant cell
granulomatous reaction or it can cause a more low-grade macrophage response. In the
latter, intra-ocular pressure may rise due to blockage of the trabecular meshwork by
macrophages (called phakolytic glaucoma) or due to blockage of the meshwork by lens
matter (called lens particle glaucoma

816-Which statement is FALSE regarding the visual pathway:

the lateral root of the optic tract is attached to the outer wall of the third ventricle

the medial root of the optic tract carries about 30% of optic nerve fibres

the lateral geniculate nucleus is a part of the thalamus

the medial root of the optic tract connects to the superior colliculus

the posterior cerebral artery runs below and parallel to the optic tract

Your answer was INCORRECT

Explanation

The medial root of the optic tract carries approximately 10% of optic nerve fibres. It
connects to both the superior colliculus and the pretectal area.

817-Which of the following passes through the foramen rotundum:

the mandibular nerve

the meningeal artery

the facial nerve

the maxillary nerve

Your answer was INCORRECT

Explanation
Only the maxillary nerve passes through the foramen rotundum. The mandibular nerve
and the meningeal artery pass through the foramen ovale. The facial nerve passes through
the styloid foramen.

818-The optic canal is located in the:

maxillary bone

lesser wing of the sphenoid bone

palatine bone

greater wing of the sphenoid bone

Your answer was INCORRECT

Explanation

The optic canal is located in the lesser wing of the sphenoid bone.

819-Which is FALSE regarding the choroid:

it has a non-fenestrated capillary bed

it is responsible for nourishing the outer retina

it has a high vascular flow-rate

its inner surface forms Bruch's membrane

Your answer was CORRECT

Explanation

The capillaries of the choriocapillaris are fenestrated. Other options are true.

820-Retinal capillary endothelial cells are surrounded by all EXCEPT:

pericytes

astrocyte foot processes


Muller cells

basal lamina

Your answer was INCORRECT

Explanation

Retinal capillary endothelial cells are surrounded by:

 a thick basal lamina


 pericytes
 astrocyte foot processes

821-The Bowman's layer:

is 50-60 micrometers thick

is a modified corneal epithelial basement membrane

is a modified acellular region of stroma

extends approximately 1mm posterior to the limbus

Your answer was INCORRECT

Explanation

The Bowman's layer is a modified acellular region of stroma. It is 8-12 micrometers thick
and terminates abruptly at the limbus.

822-All are true of Neurofibromatosis Type 1 EXCEPT:

Lisch nodules occur which are melanocytic proliferations on the iris

it is associated with opic nerve glioma

it is characterized by cafe-au-lait spots


it is associated with acoustic neuroma

Your answer was INCORRECT

Explanation

NF Type 1 occurs from a mutation in the NF-1 gene on chromosome 17. It is


characterized by:

 cafe-au-lait spots
 Lisch nodules
 goniodysgenesis
 retinal hamartomas
 optic nerve glioma

NOTE: acoustic neuroma is associated with NF Type 2.

823-Phase II drug metabolism:

involves hydrolysis of drugs

decreases drug water solubility

involves conjugation reactions

prevents drug excretion by the kidneys or liver

Your answer was INCORRECT

Explanation

Phase II drug metabolism involves a conjugation reaction and the addition of glucuronic
acid is the most common. The process increases drug water solubility and facilitates drug
excretion by the kidneys or liver.

824-Which are the phagocytic cells of the central nervous system:

oligodendrocytes

Schwann cells
astrocytes

microglia

Your answer was INCORRECT

Explanation

The glial cells of the central nervous system include:

 Oligodendrocytes: the myelin-forming cells of the CNS.

 Astrocytes: perform a variety of structural and functional roles including


metabolic, healing, blood-brain and blood-retinal barriers.

 Microglia: are monocytes and perform phagocytosis

825-Which immunoglobulin class is considered the oldest phylogenetically?

IgA

IgE

IgM

IgD

IgG

Your answer was INCORRECT

Explanation

IgM or IgM-like immunoglobulins tend to be the only type present in organisms with the
most rudimentary immune systems.

826-Mean systolic blood pressure (SBP) of a sample of 100 consultants is 162 mmHg and
follows a normal distribution with a standard deviation of 10 mmHg.

Which of the following can be said?


approximately 68% of consultants will have an SBP between 157 and 167 mmHg

a consultant with an SBP of 132 mmHg lies 3 standard deviations above the mean

variance equals 20 mmHg

a consultant with an SBP of 182 mmHg lies half a standard deviation above the mean

approximately 68% of consultants will have an SBP between 152 and 172 mmH

827-The Prentice position of a prism:

does not obey Snell's law

is usually specified for plastic ophthalmic prisms

occurs when either surface of the prism is normal to light

occurs when the angle of incidence equals the angle of emergence

Your answer was INCORRECT

Explanation

The Prentice position of a prism occurs when one surface of the prism is normal to the
ray of light so that all deviation occurs at the other surface. The strength of a prism in the
Prentice position is greater than the position of minimum deviation, which occurs when
the angle of incidence equals the angle of emergence. The Prentice position is usually
specified for glass ophthalmic prisms while the position of minimum deviation is
specified for plastic ophthalmic prisms.

828-The superior orbital fissure transmits all of the following EXCEPT:

the infraorbital nerve

the abducent nerve

the trochlear nerve

the lacrimal nerve


the frontal nerve

Your answer was CORRECT

Explanation

The following structures pass through the superior orbital fissure:

 Lazy: the lacrimal nerve


 French: the frontal nerve
 Tarts: the trochlear nerve
 Sit: superior division of oculomotor nerve
 Nakedly: nasociliary nerve
 In: inferior division of oculomotor nerve
 Anticipation: abducens nerve nerve and the nasociliary nerve

829-Systemic side-effects of beta-adrenergic blockade include all EXCEPT:

syncope

central nervous system depression

bronchodilation

bradycardia

Your answer was INCORRECT

Explanation

Beta-blockers cause bronchospasm not bronchodilation. Other options are true.

830-Which is FALSE regarding retinoblastoma:

inherited cases are due to a defect on chromosome 6

its prevalence is 1 in 20,000 live births

it is a tumour of photoreceptor cells


approximately 40% of cases are inherited

Your answer was CORRECT

Explanation

The Rb tumour suppressor gene implicated in retinoblastoma resides on the long arm of
chromosome 13.

831-Which condition is associated with astrocytic hamartomas within the retina:

Tuberous sclerosis

Von-Hippel Lindau syndrome

Neurofibromatosis Type 2

Neurofibromatosis Type 1

Your answer was CORRECT

Explanation

Tuberous sclerosis is an autosomal dominant condition characterized by multiple


astrocytic hamartomas across many tissues including CNS, muscle and kidney. In the eye
it causes astrocytic hamartomas within the retina.

832-Which statement is FALSE about the origins of the oculomotor nerve:

its nuclei originate in the inferior colliculus

the nerve leaving the brainstem initially passes forward, laterally and downward in the
interpeducular fossa

its nerve fibres emerge from the brainstem at the border of the midbrain and the pons

it contains parasympathetic fibres which originate in the Edinger-Westphal nucleus

Your answer was CORRECT

Explanation
The oculomotor nuclei originate in the superior (not inferior) colliculus of the midbrain in
the ventral region of the periaqueductal grey matter. The oculomotor nerve nuclei are of 2
main types: motor nuclei which innervate the extraocular muscles and the Edinger-
Westphal nucleus which supplies parasympathetic fibres to the choroid, iris and ciliary
body.

833-All are true of the tear film EXCEPT:

it contains secretions from the meibomian glands

it contributes to the refractive function of the eye

it is composed of an outer lipid layer

it is increased with topical atropine

Your answer was INCORRECT

Explanation

The tear film is decreased by atropine, which is an anti-muscurinic and reduces


parasympathetic input to the lacrimal gland.

834-Which is FALSE regarding the visual pathway:

the mammillary bodies are posterior to the chiasm

the optic nerves travel within the subarachnoid space after leaving the orbits

the optic chiasm occurs at the floor of the fourth ventricle

the anterior communicating artery lies above the optic chiasm

Your answer was INCORRECT

Explanation

The optic chiasm is situated at the junction of the anterior wall and floor of the third
ventricle. Other statements are true.

835-Which of the following statements is FALSE about the eye:


it is situated in the anterior portion of the orbit

large eyes tend to be myopic

it is nearer the floor of the orbit than the roof

it has a diameter of approximately 23mm

Your answer was INCORRECT

Explanation

The eye is situated nearer the roof of the orbit than the floor. Other options are true.

836-Which is FALSE of the abducens nerve:

it passes through the cavernous sinus lying medial to the ascending internal carotid

its nucleus lie in the midpons

it has the longest intracranial course of any cranial nerve

it emerges from the brainstem at the border of the pons and medulla near the midline

Your answer was CORRECT

Explanation

The abducens nerve passes through the cavernous sinus and lies lateral to the ascending
portion of the internal carotid artery.

837-All of the following are true about elastin EXCEPT:

it is an insoluble protein

it is an integral component of deformable tissues such as arterial walls

it is integral to the structure of the lens zonule

it has a very regular tertiary structure


Your answer was INCORRECT

Explanation

Elastin has a very irregular coil-like tertiary structure, unlike collagen which is a regular
triple helix. It does not occur in all tissues but is an important component of deformable
tissues such as arterial cell walls and the lens zonules.

838-Which is FALSE regarding glycosaminoglycans:

they perform an important structural role in the vitreous

they are important in maintaining transparency of the cornea

they are comprised of long chains of repeating dipeptides

the most common glycosaminoglycan in the cornea is keratan sulphate

Your answer was INCORRECT

Explanation

Glycosaminoglycans are long chains of repeating disaccharides. They can occur on their
own, or attached to proteins when they are known as proteoglycans. They have important
structural and regulatory roles in tissues. In the eye, GAG's are important to the resilience
of the vitreous (mainly hyaluronic acid) and they also maintain corneal transparency by
regulating collagen fibril diameter and spacing in the corneal stroma (keratan sulphate).

839-An ischaemic event in the left temporal lobe would most likely produce:

a contralateral congruous homonymous hemianopia

a bitemporal heminanopia

a contralateral incongruous homonymous hemianopia

a contralateral lower quadrantanopia

a contralateral upper quadrantanopia

Your answer was INCORRECT


Explanation

The temporal lobe receives those fibres of the geniculocalcarine tract (optic radiation)
corresponding to the inferior retina, which is the upper half of the visual field. It will
therefore result in a contralateral upper quadrantanopia.

840-Which statement is FALSE regarding the inferior rectus muscle:

it is innervated from its superior surface

it is inserted 6.5 cm behind the limbus

it is the main depressor of the globe when the eye is adducted

it lies inferior to the inferior oblique muscle

Your answer was INCORRECT

Explanation

The inferior rectus actions are: depression, medial rotation of the eye ball and extorsion.
The effect of depression is greatest when the eyeball is abducted. Other options above are
true.

841-Which is FALSE regarding the photoreceptors:

rods are longer than cones

the photoreceptor outer segments are in closest proximity to the RPE microvilli

axons from the photoreceptors synapse in the inner plexiform layer

the outer limiting membrane occurs at the junction of the inner and outer segments

Your answer was INCORRECT

Explanation

Axons from the photoreceptors synapse in the outer plexiform layer where they
communicate with bipolar cells and horizontal cells.
842-Which antibiotic inhibits protein synthesis by binding to the 30S ribosome subunit:

erythromycin

tetracycline

rifampicin

sulphonamide

chloramphenicol

Your answer was INCORRECT

Explanation

 Macrolides (such as erythromycin) and chloramphenicol inhibit protein synthesis


by binding to the 50S subunit of the bacterial ribosome.

 Tetracyclines and aminoglycosides inhibit protein synthesis by binding to the 30S


subunit of bacterial ribosomes.

 Sulphonamide inhibits biosynthesis of folic acid.

 Rifampin inhibits RNA synthesis by inhibiting DNA-dependent RNA


polymerase.

843-All of the following are ocular manifestations of Wegener's EXCEPT:

scleritis

orbital mass

iritis

corneoscleral ulceration

Your answer was INCORRECT

Explanation
Wegener's can cause corneascleral ulceration, scleritis and an orbital mass. However,
iritis is not specifically associated with Wegener's granulomatosis.

844-Which statement is FALSE regarding basal cell carcinoma:

it is associated with UV exposure

it spreads via the lymphatics

it is the most common malignant tumour in clinical ophthalmology

it is characterized by a central ulcer with rolled edges

Your answer was INCORRECT

Explanation

Basal cell carcinoma spreads by local invasion. Other statements are true.

845-Infectionwith which pathogen is characteristically associated with an owl-eye


inclusion body:

Chlamydia trachomatis

cytomegalovirus

herpes zoster virus

Borrelia bergdoferi

Your answer was INCORRECT

Explanation

Owl-eye inclusion bodies are characteristic of cytomegalovirus infection as seen for


example in cytomegalovirus retinitis in immunocompromised patients.

846-All are true of the pterygopalatine ganglion EXCEPT:

it supplies the sphincter pupillae of the iris


it supplies secretomotor fibres to the glands of the nose

it supplies parasympathetic fibres to the lacrimal gland

it is located in the pterygopalatine fossa

Your answer was CORRECT

Explanation

The pterygopalatine ganglion does not supply the iris, which is supplied with
parasympathetic innervation via the oculomotor nerve synapsing in the ciliary ganglion.
Other statements are true.

847-Which statement is FALSE regarding Toxocara infection:

it can cause a rapid inflammation in the retinal mid-periphery

eggs are passed in dog faeces

it is associated with an increase in eosinophil count

it can cause a low-grade fibrous reaction in the retina

larvae when active produce an intense inflammatory response

Your answer was INCORRECT

Explanation

Toxocara laravae do not elicit an inflammatory response but when they die, the immune
system becomes activated which can cause one of the following in the eye:

 low grade fibrous retinal involvement


 rapid retinal inflammation with eosinophilia
 vitritis and pars planitis

848-Which statement about the ciliary ganglion is FALSE:

the motor root of the ciliary ganglion emerges from the branch of the oculomotor nerve
supplying the inferior oblique
the ciliary ganglion lies between the lateral rectus and the optic nerve

the sensory root of the ciliary ganglion emerges from the nasociliary nerve

the long ciliary nerves emerge from the ciliary ganglion to supply the orbit

Your answer was INCORRECT

Explanation

Short ciliary nerves emerge from the ciliary ganglion to supply the orbit. Other options
are true

849-The layer of choroid comprising large arteries and veins is:

Haller's layer

Sattler's layer

the choriocapillaris

Bruch's membrane

Your answer was CORRECT

Explanation

Haller's layer lies within the choroid and comprises large arteries and veins. Sattler's
layer, which lies above Haller's layer, is composed of arterioles and venules which supply
the choriocapillaris directly above.

850-Which of the following trial designs is most appropriate for the situation described:

a one-tailed analysis to compare the rate of side effects in a new treatment compared to
current best treatment.

a randomised-controlled trial to look at the association between sleeping position and


sudden infant death syndrome.
a case-control study to determine whether a toxic chemical which leaked briefly into a
city's water supply is related to lung cancer.

a cohort study to investigate the association between smoking and the risk of choroidal
melanoma.

a case-control study to investigate the association between smoking and the risk of
choroidal melanoma.

Your answer was INCORRECT

Explanation

Case-control studies are most appropriate for rare outcomes and common exposures;
cohort studies for rare exposures and common outcomes. RCTs using human subjects are
not always ethical. Two-tailed tests look for a difference in any direction while a one-
tailed test looks only for an increase or only for a decrease (and is consequently rarely
used in clinical trials).
851-What is the second most common method of human immunodeficiency virus (HIV)
transmission in Western countries?

blood transfusions

sexual intercourse

intravenous drug use

transplacental transmission

Your answer was INCORRECT

Explanation

Intravenous drug use accounts for approximately 25% of cases of human


immunodeficiency virus (HIV) transmission. Sexual intercourse accounts for 70% of
cases.
852-From anterior to posterior the medial orbital wall is composed of:

lacrimal bone, maxilla, ethmoid, sphenoid

lacrimal bone, maxilla, sphenoid, ethmoid


maxilla, lacrimal bone, ethmoid, sphenoid

maxilla, ethmoid, lacrimal, sphenoid

Your answer was INCORRECT

Explanation

The medial orbital wall is composed (anterior to posterior) of: the frontal process of the
maxilla, the lacrimal bone, the orbital plate of the ethmoid and the body of the sphenoid.

853-The commonest defect of colour vision is due to:

deuteranomaly

deuteranopia

protanopia

protanomaly

Your answer was CORRECT

Explanation

The commonest defect of colour vision is deuteranomaly which occurs in 5% of men and
0.3% of women. It is due to a shift in the spectral sensitivity of green cones.
Deuteranopia indicates complete absence of green cones and is rare. The genes for red
and green pigment are on the X chromosome while chromosome 7 carries the blue
pigment gene. Thus deuteranomaly and protanomaly are X-linked.

854-Which of the following statements is FALSE regarding the CSF:

blockage of the arachnoid granulation causes communicating hydrocephalus

it is produced by the modified ependymal cells of the choroid plexus

it flows from the ventricles to the subarachnoid space via the roof of the 3rd
ventricle
it is absorbed mainly through the arachnoid granulations in the superior sagittal sinus

Your answer was INCORRECT

Explanation

The ventricular system communicates with the subarachnoid space via the roof of the
fourth ventricle.

855-Diabetic retinopathy is characterized histologically by all EXCEPT:

loss of pericytes

cotton wool spots due to nerve fibre ischemia

thinning of the capillary basement membrane

degeneration of capillary endothelial cells

hyalinization of vessels

Your answer was INCORRECT

Explanation

In diabetes there is thickening and multilayering of the basement membrane of small


vessels. Other statements are true.

856-Which is FALSE regarding polyarteritis nodosa:

it can affect retinal and choroidal vessels

it causes coagulative necrosis of vessel walls

it affects medium and small sized arteries

it leads to ischaemia of tissues including heart, lung and kidney

Your answer was INCORRECT

Explanation
Polyarteritis causes fibrinoid necrosis of vessel walls leading to thrombosis and
ischaemia.

857-Which is TRUE of parasympathetic antagonists administered topically to the eye:

they decrease lacrimal secretions

they activate muscarinic receptors

they produce miosis

they cause ciliary muscle contraction

Your answer was CORRECT

Explanation

Parasympathetic antagonists inhibit acetylcholine muscarinic receptors by definition.


They produce mydriasis, cycloplegia and they reduce lacrimal secretions.

858-The image formed by an object within the principle focus of a concave mirror is:

real, inverted and diminished

virtual, inverted and enlarged

erect, virtual and enlarged

real, inverted and enlarged

Your answer was INCORRECT

Explanation

For an object within the principle focus of a concave mirror, the image formed is erect,
virtual and enlarged; and it lies behind the mirror.

859-Henle's fibre is formed by:


outer nuclear layer

outer plexiform layer

inner plexiform layer

inner nuclear layer

Your answer was INCORRECT

Explanation

In the macular region the outer plexiform layer is known as Henle's fibre layer.

860-The maxillary sinus drains into the:

the inferior meatus

the superior meatus

the lesser meatus

the middle meatus

Your answer was INCORRECT

Explanation

The nasal meatus are as follows:

 the superior meatus receives the opening of the sphenoid and posterior ethmoidal
sinuses
 the middle meatus receives the openings of the anterior ethmoidal sinuses, frontal
sinus and the maxillary sinus
 the inferior meatus receives the opening of the nasolacrimal duct

861-The frontal eye field lies in the vascular territory of:

the middle cerebral artery

the anterior cerebral artery


the spinal artery

the posterior cerebral artery

Your answer was CORRECT

Explanation

The frontal eye field is in the territory of the middle cerebral artery. This might seem
counter-intuitve as the frontal eye field is in the frontal lobe, but remember that the
middle cerebral artery supplies the lateral convexity of the brain while the anterior
cerebral artery supplies the medial surface of the hemisphere.

862-Which of the following immunoglobulins are pentameric:

Ig G

Ig M

Ig E

Ig D

Ig A

Your answer was INCORRECT

Explanation

Monomeric immunoglobulins contain the basic structure of two identical light chains and
two identical heavy chains linked by disulphide bonds

863-Which cells connect the photoreceptors to the ganglion cells:

cones

bipolar cells

amacrine cells
horizontal cells

Your answer was INCORRECT

Explanation

Bipolar cells are primarily responsible for transmission from photoreceptors to ganglion
cells

864-Which is FALSE regarding the prognosis of uveal melanoma:

monosomy 3 is associated with metastatic spread

the prognosis is worse in the elderly

ciliary body melanoma carries a worse prognosis than choroid melanoma

spindle cells carry a poorer prognosis than epithelioid cells

Your answer was INCORRECT

Explanation

Epithelioid cells carry a poorer prognosis than spindle cells. Other options are true.

865-All of the following ophthalmic materials are effectively inert EXCEPT:

silicone encircling bands

PMMA intra-ocular lenses

hyaluronic acid

silicon oil

Molteno drainage tubes

Your answer was INCORRECT

Explanation
Silicon oil produces a low-grade macrophage reaction after it is emulsified. All other
materials mentioned do not typically cause an inflammatory response; though they may
become covered with fibrous capsules.

866-What wavelength of light is emitted by fluorescein sodium:

835nm

465nm

805nm

525nm

Your answer was INCORRECT

Explanation

Fluorescein:

 absorbs blue light of 465-490nm wavelength


 emits yellow-green light of wavelength 520-530nm

Indocyanine green:

 absorbs 805nm infrared light


 emits 835nm infrared light

867-Which statement is FALSE regarding progressive outer retinal necrosis:

it occurs in immunosuppressed patients

it can be caused by herpes simplex virus

it can be caused by herpes zoster infection

it usually involves vitritis and retinal vasculitis


Your answer was INCORRECT

Explanation

Progressive outer retinal necrosis (PORN) involves destruction of the outer retina without
the accompanying vitritis and vasculitis associated with acute retinal necrosis.

868-Fluorescein dye absorbs light of which wavelength:

yellow

blue

red

green

Your answer was INCORRECT

Explanation

Fluorescein absorbs and is excited by blue light (465-490nm). It is for this reason that
white light from the flash of a fluorescein angiography camera passes through a blue
'excitation' filter.

869-The oculomotor nerve supplies:

the lateral rectus muscle

the cornea

the superior oblique muscle

the ciliary muscle

Your answer was INCORRECT

Explanation

The oculomotor nerve supplies all the extraocular muscles with motor innervation except
the superior oblique muscle (trochlear nerve) and the lateral rectus (abducent nerve).It
also supplies parasympathetic innervation to the eye, which emanates from the Edinger-
Westphal nucleus and synapses in the ciliary ganglion on its way to the globe via the
short ciliary nerves. The ciliary muscle and the sphincter muscle are supplied by these
parasympathetic fibres. The cornea is supplied by the trigeminal nerve.

870-A
10 degree refractive angle glass prism in the position of minimum deviation has a
power of approximately:

20

100

10

Your answer was INCORRECT

Explanation

A 10 degree refractive angle glass prism in the position of minimum deviation deviates
light through 5 degrees (deviation= angle/2) and has a power of 10 prism diopters (1
prism diopter = half an angle of apparent deviation) assuming refractive index of 1.5.

871-Which statement is TRUE about the ABO blood groups:

patients with blood group O are regarded as universal recipients

anti-A and anti-B antibodies are found in patients with blood group AB

blood group O is the most common type

patients with blood group B are regarded as universal donors

blood group A is the least common type

Your answer was INCORRECT

Explanation

Blood group O is the most common and blood group AB is the least common. Blood
group O is regarded as universal donors and blood group AB as universal recipients.
Anti-A and anti-B antibodies are found in patients with blood group O. Blood group AB
contains no antibodies against any ABO group.

872-Which statement is TRUE about the ABO blood groups:

patients with blood group O are regarded as universal recipients

anti-A and anti-B antibodies are found in patients with blood group AB

blood group O is the most common type

patients with blood group B are regarded as universal donors

blood group A is the least common type

Your answer was INCORRECT

Explanation

Blood group O is the most common and blood group AB is the least common. Blood
group O is regarded as universal donors and blood group AB as universal recipients.
Anti-A and anti-B antibodies are found in patients with blood group O. Blood group AB
contains no antibodies against any ABO group.

873-Which statement is FALSE about the corneal stroma:

transparency of the cornea depends on regular spacing of collagen fibrils

the main glycosaminoglycans in the cornea are keratan sulphate and chondroitin

collagenous lamellae are composed principally of Type III collagen

collagenous lamellae are oriented parallel to the corneal surface

Your answer was INCORRECT

Explanation

The lamellae are composed principally of Type I collagen. Other options are true.

874-Theangular magnification produced by an image positioned at the focal point F1 of a


convex lens is equal to:
infinity

zero

one

the dioptric power of the lens divided by four

Your answer was INCORRECT

Explanation

The angular magnification produced by an image positioned at the focal point F1 of a


convex lens is equal to one. This is because the image lies at infinity and therefore the
angle subtended by the object and the image are equal, producing an angular
magnification of one.

Remember:
angular magnification = apparent image size/apparent object size
or, angular magnification = tan(angle image)/ tan (angle object)

875-A study examined the effect of a new drug on stroke prevention over 10 years. 10% of
patients in the placebo group had stroke in comparison to 5% of patients in the treatment
group.

What is the relative risk reduction?

10%

20%

15%

50%

5%

Your answer was INCORRECT

Explanation
The relative risk reduction (RRR) = (5/10) x 100 = 50%

The absolute risk reduction (ARR) = 10% - 5% = 5%

876-Which process is used to separate RNA molecules?

Eastern Blot

Gel Blot

Northern Blot

Western Blot

Southern Blot

Your answer was INCORRECT

Explanation

DNA molecules can be separated by a process called Southern Blotting, named after the
man who invented the technique. Therefore when a technique to separate RNA molecules
became available, it was given the name of a Northern Blot. A Western Blot is used for
proteins. There is no Eastern blot. The term Gel Blot is not generally used.

877-Sterilisation can be achieved by all of the following EXCEPT:

a dry fan-assisted oven to a temperature of 160 degrees C for 1 hour

an autoclave with steam to a temperature of 121 degrees C for 120 minutes

Gamma-radiation

ethylene oxide

Your answer was CORRECT

Explanation

Dry heat takes much longer than steam and a temperature of 160 degrees in a fan-assisted
oven will take several hours to achieve sterilization.
878-Which is FALSE regarding topical eye drops:
cocaine prevents the reuptake of noradrenaline at nerve endings

brimonidine is a selective alpha-1 agonist

epinephrine stimulates both alpha and beta receptors

hydroxyamphetamine increases the release of noradrenaline from nerve terminals

phenylephrine stimulates alpha receptors

Your answer was INCORRECT

Explanation

Brimonidine is a selective alpha-2 agonists. The other options are true.

879-Local anaesthetics work by:

blocking potassium channels

exciting potassium channels

blocking sodium channels

exciting sodium channels

Your answer was INCORRECT

Explanation

Local anaesthetics work by blocking sodium channels and therefore preventing the
propagation of an action potential across the nerve.

880-Which of the following hormones does NOT utilise cAMP as its second messenger?

adrenaline

TSH

PTH
glucagon

insulin

Your answer was INCORRECT

Explanation

Currently, four second messenger systems are recognised in cells. They are used to
amplify the small signal received at the cell membrane when a ligand binds to a receptor.
A number of important hormones lead to a rise in intracellular cAMP, which increases
the activity of protein kinase A. These include adrenaline and noradrenaline, PTH, TSH,
Glucagon, FSH, LH, calcitonin and ADH. The receptor for insulin is a tyrosine kinase.
Following binding of the hormone, the receptor phosphorylates itself, then a variety of
other targets. Other second messengers include calcium, cyclic GMP and
phosphoinositides.

881-Which of the following statements regarding bias is correct?

bias can be corrected at the analysis stage of a trial

the value of RCTs is that they remove all sources of bias

case-control studies are least susceptible to bias

blinding is an attempt to remove bias

bias is due to random error

Your answer was INCORRECT

Explanation

Bias refers to systematic (as opposed to random) error. RCTs remove all sources of
confounding, not bias. Case-control studies are retrospective, and therefore particularly
susceptible to bias.

882-Exotoxins are liberated from:

fungi
Gram positive bacteria

Gram negative bacteria

mycobacteria

Your answer was INCORRECT

Explanation

Exotoxins are proteins released by Gram-positive bacteria and produce specific effects at
sites distant to their release. They are antigenic and are readily destroyed by heat.

883-Translation is controlled by:

peptidyl transferase

RNA polymerase

DNA polymerase

DNA gyrase

Your answer was CORRECT

Explanation

Translation is the production of an amino acid sequence from mRNA in the cytoplasm. It
is controlled by 2 main enzymes: aminoacyl-tRNA synthetase (which helps join tRNA to
its specific amino acid) and by peptidyl transferase which involves the uncoupling of the
tRNA from the amino acid and joining of the amino acid to the growing polypeptide
chain.

884-Which is FALSE regarding a Maddox rod:

to test for horizontal muscle imbalance the rod should be vertical

light parallel to the axis of the cylinders passes through the cylinder undeviated

a horizontal line is produced on the retina perpendicular to the rod axis


it uses convex cylindrical lenses mounted side by side

Your answer was CORRECT

Explanation

The Maddox rod uses strong convex cylindrical lenses mounted side by side. Light that is
parallel to the axis of the cylinders passes through the cylinders undeviated and this light
is focused by the eye to produce a straight line, which is perpendicular to the rod axis.
The Maddox is used to test muscle imbalance. To test for horizontal muscle imbalance
the Maddox rod should be horizontal to give a vertical line (and vice versa for vertical
muscle imbalance).

885-Which is FALSE of the image formed by a convex mirror:

it lies between the mirror and the principle focus

virtual

inverted

diminished

Your answer was INCORRECT

Explanation

The image formed by a convex mirror, irrespective of the distance of the object from the
mirror, is virtual, erect and diminished; and it is located behind the mirror, between the
mirror and the principle focus.

886-Which of the following is TRUE regarding mitochondrial DNA:

disease involving mitochondrial DNA show an autosomal dominant inheritance

it is a double-stranded straight DNA

mutation of mitochondrial DNA is more common than nuclear DNA

diseases involving the mitochondrial DNA affect only the muscles

Your answer was INCORRECT


Explanation

Mitochondrial DNA has a double stranded round DNA. It has no intron or repair system
and therefore more susceptible to mutation than the nuclear DNA. Mitochondrial DNA
disorders commonly involve the muscle but also the nerves such as in Leber's optic
neuropathy. They are inherited maternally.

887-Which statement is FALSE about Descemet's membrane:

it is not the true basement membrane of the corneal endothelium

it occurs between the posterior stroma and the endothelium

it is 8-12 micrometers thick

it is continuous posteriorly with the trabecular meshwork

Your answer was CORRECT

Explanation

Descemet's is the modified basement membrane of the corneal endothelium. All other
options are true.

888-The supratrochlear nerve is derived from:

the nasociliary nerve

the frontal nerve

the trochlear nerve

the lacrimal nerve

the supraorbital nerve

Your answer was INCORRECT

Explanation

The supratrochlear nerve supplies the skin of the medial brow and forehead and is a
terminal branch of the frontal nerve, itself a branch of the ophthalmic nerve.
889-All are true of Toxoplasma EXCEPT:

adult chorioretinal involvement implies secondary reactivation

it is an obligate intracellular protozoon

infection can be acquired from inadequately cooked meet

congenital toxoplasma may result in intracranial calcification and mental retardation

Your answer was CORRECT

Explanation

Adult disease may be from reactivation of prior infection in utero or it may arise from
primary infection in adulthood.

890-Which of the following is NOT contained in the pterygo-palatine fossa:

the pterygopalatine ganglion

the maxillary nerve

the maxillary artery

the lesser superficial petrosal nerve

Your answer was INCORRECT

Explanation

The pterygopalatine fossa is the distribution centre for the main vessels and nerves of the
middle third of the face. The greater and deep petrosal nerves join together before
entering the pterygo-palatine fossa. The lesser superficial petrosal nerve does not pass
through the pterygo-palatine fossa, it synapses in the otic ganglion and supplies the
parotid gland.

891-All are true of acanthoemeba EXCEPT:

it is sensitive to 3rd generation cephalosporins


it is very difficult to culture

it is more common in soft contact lens wearers

it is a free-living protozoon

Your answer was CORRECT

Explanation

Acanthoemeba corneal involvement is poorly responsive to conventional antibiotics,


including the latest generation cephalosporins. Treatment requires propamidine
isethionate, chrohexidine and polyhexamthyl biguanide.

892-Which of the following hypersensitivity reaction types involve the participation of


antibodies?

1. Type I
2. Type II
3. Type III
4. Type IV

4 only

1, 2, and 3

1 and 3

1, 2, 3, and 4

2 and 4

Your answer was INCORRECT

Explanation

Type I reactions involve cross-linking of IgE bound to mast cells and basophils. Type II
reactions frequently feature IgG or IgM interaction with either cytotoxic cells or
circulating immunoglobulins and antigens and the subsequent deposition of those
complexes. (Note that some type II processes are not strictly antibody dependent.) Type
III reactions result from antigen-antibody complex formation.
893-Which is FALSE regarding dispersive power:

light of shorter wavelength is deviated more than light of longer wavelength

it is measured by the angle formed between the red and blue light around yellow

it is related to the refractive index of the material

dispersion occurs in the formation of a rainbow

Your answer was INCORRECT

Explanation

Dispersive power is not related to refractive index. It is measured by the angle formed
between the red and blue light around yellow. Light of shorter wavelength (violet) is
deviated more than light of longer wavelength (red). Dispersion of wavelengths occurs
through water droplets in the formation of a rainbow.

894-Which structure drains into the superior meatus:

the frontal sinus

the maxillary sinus

the nasolacrimal duct

the posterior ethmoidal sinus

Your answer was INCORRECT

Explanation

The nasal meatus are as follows:

 the superior meatus receives the opening of the posterior ethmoidal sinuses
 the middle meatus receives the openings of the anterior ethmoidal sinuses, frontal
sinus and the maxillary sinus
 the inferior meatus receives the opening of the nasolacrimal duct
895-The analgesic effect of a new drug is compared to morphine. The significance level is
set at 0.05 and a difference is found (p = 0.09).

What is the best conclusion to draw?

a larger sample should have been used

the trial would have been better run against placebo

the drug should not be recommended for clinical practice

the probability that the difference found arose by chance is greater than we are
prepared to accept

the alternative hypothesis should be accepted

Your answer was INCORRECT

Explanation

Where the p-value is less than the critical value, there is good evidence to accept the
alternative hypothesis. The p-value is the probability of committing a Type I error (false
positive), that is, rejecting the null hypothesis when in fact the difference occurred by
chance.

896-Two heterozygous parents for a recessive condition want to know their chances of
having a child NOT affected with the disease. What will you advise them:

75%

50%

100%

25%

33%

Your answer was CORRECT

Explanation
There is a 1 in 4 chance of being affected (homozygous) and therefore a 75% chance of
not having the disease. We recommend drawing a 2x2 box of chromosomal probabilities.

897-Regarding anti-sense oligonucleotides, which statement is FALSE:

they are a normal constituent of eukaryotic cells.

they are effective against viral RNA

they could be used to target the translation of oncogenes

they bind to target RNA and block its processing

they are composed of DNA

Your answer was INCORRECT

Explanation

Anti-sense oligonucleotides are short sequences of DNA, which are complementary to


the target RNA. They have to be delivered into the cell by cationic liposomes,
microinjection or viral vectors. The sequences also have to be modified to prevent
degradation by nucleases. This technique could be used to prevent the processing of
activated oncogene products or could be used to target viral RNA.

898-Which is TRUE of ionizing irradiation:

mitotic cells are more sensitive to radiation than non-mitotic cells

the main cellular target for radiation is the cell membrane

it cannot be used to sterilise surgical instruments

x-rays have deeper penetration than gamma rays

genetic alteration from radiation is usually due to gene deletion

Your answer was CORRECT

Explanation
The main cellular target for radiation is the DNA and the principal action is strand
breakage. Gamma rays are more penetrative than X-rays. Dividing cells are more
radiosensitive than non-dividing cells. Radiation can be used to sterilize surgical
instruments.

Score: 24 %
Question 656 Part 3 – Tutor Mode
In which condition are you LEAST likely to find a cotton wool spot:

ischaemic central retinal vein occlusion

pre-proliferative diabetic retinopathy

central retinal artery occlusion

malignant hypertension

Your answer was INCORRECT

Explanation

Cotton wool spots are a sign of microvascular ischaemia and are found commonly in
hypertension and diabetes. They can also occur in HIV and vasculitic diseases. They may
also be seen after the evolution of an ischaemic CRVO leaves areas of retina
underperfused. They are not a characteristic of CRAO.

900-Endotoxins:

are found in the bacterial cytoplasm

can often survive autoclaving

are produced mainly by Gram positive bacteria

elicit the classical pathway of the complement system

Your answer was INCORRECT

Explanation

Endotoxins are lipopolysaccharides derived from the cell wall of gram negative bacteria.
They are heat stable unlike exotoxins. They are non-antigenic as against exotoxin. They
cause septicemia and fever and activate the alternative pathway (non-antibody mediated)
of the complement system.

901-All of the following are true regarding nuclear DNA except:

nuclear DNA exists as nucleoprotein complex by wrapping around histones

exons code for messenger DNA

ionic bonds exist between the paired bases

introns exist between coding sequences of the DNA

Your answer was INCORRECT

Explanation

Hydrogen bonds exist between the paired bases. Other options supplied are true.

902-Which of the following is a monosynaptic reflex:

corneal reflex

blink reflex

knee jerk reflex

withdrawal reflex

Your answer was INCORRECT

Explanation

Monosynaptic reflexes usually originate and terminate in the same muscle with a very
fast response of about 20 milliseconds. Apart from the knee jerk reflex, the other options
supplied above are polysynaptic reflexes.

903-Which of the following forms part of the lacrimal sac fossa:

frontal process of maxilla


ethmoid bone

palatine bone

frontal bone

Your answer was CORRECT

Explanation

The lacrimal sac fossa is formed by the lacrimal bone and the frontal process of the
maxilla

904-The following are true about blood coagulation EXCEPT:

vitamin K is responsible for the production of factors II, VII, IX and X

thrombin converts fibrinogen to fibrin

platelets are essential for blood clotting

heparin inhibits blood coagulation through its interference with vitamin K


metabolism

Your answer was INCORRECT

Explanation

Heparin increases the activity of anti-thrombin III which inhibits the clotting cascade. All
other statements are true.

905-Which is FALSE regarding systemic lupus erythematosus:

it is a multisystem autoimmune condition

anti-double stranded DNA bodies are raised

phospholipid antibodies are raised

ocular involvement is common


a small vessel vasculitis may occur

Your answer was INCORRECT

Explanation

Ocular involvement with SLE is rare. When present it can produce:

 retinal microinfarcts
 central retinal artery occlusion
 central retinal vein occlusion
 choroidopathy

906-Which antibiotic is NOT a good choice for chlamydia infection:

rifampicin

erythromycin

tetracycline

cephalosporin

Your answer was INCORRECT

Explanation

Chlamydia is an obligate intracellular organism and antibiotics such as cephalosporins


that target bacterial cell walls will have little effect.

907-The following are true about entopic phenomenon EXCEPT:

asteroid hyalosis causes significant visual disturbance due to entopic phenomenon

the size of one's pupil can be observed with a pinhole

it can be produced by cells in the vitreous

it can be produced by palpation of the eyeballs

Your answer was CORRECT


Explanation

Entopic phenomenon are visual perceptions that are produced by the structures of one's
own eyes e.g. cells in the vitreous or through eyeball palpation. The edge of one's pupil
can be seen when a uniformly illuminated background is viewed through a pinhole.

908-Which statement is FALSE about the ciliary processes:

the pigmented epithelium actively secretes aqueous humour

they contain an outer pigmented and an inner non-pigmented epithelium

the epithelium is derived from neuroectoderm

the apices of the pigmented and non-pigmented epithelium face one another

Your answer was CORRECT

Explanation

The ciliary processes form an intergral part of the blood-aqueous barrier.


Macromolecules escape the highly fenestrated capillaries in the stroma of the ciliary
processes, and then cross the pigmented epithelial cells, held together by permeable
junctions. At the inner non-pigmented epithelium, there are numerous tight junctions
controlling the passage of molecules and the cells actively secrete aqueous humour with
high numbers of mitochondria.

909-Which of the following drugs is NOT used in the treatment of HIV/AIDS virus:

etanercept

lamivudine

T-20

efavirenz

combivir

Your answer was CORRECT

Explanation
All of the above are used in the treatment of HIV except etanercept which is used in the
treatment of rheumatoid arthritis, psoriasis, psoriatic arthritis and ankylosing spondylitis.
It is an anti-TNF alpha drug which suppresses the immune system and therefore would
not be used in a patient with HIV.

 Lamivudine is a nucleoside reverse transcriptase inhibitor


 Efavirenz is a non-nucleoside reverse transcriptase inhibitor
 Zidovudine and Lamivudine are packaged together as combibir
 T-20 is a fusion inhibitor; it sticks to gp41 to prevent HIV entering the cell

910-Whichof the following aminoglycosides is most likely to induce allergic reaction


when applied to the eye:

gentamicin

tobramycin

neomycin

amikacin

Your answer was INCORRECT

Explanation

Allergic conjunctivitis is most common with topical neomycin, occurring in up to 8% of


patients.

911-How many rods are there in the retina:

1.2 million

6.6 million

12 million

9 million

115 million

Your answer was INCORRECT


Explanation

There are 115 million rods in the human eye.

912-The image formed by a prism is:

erect, real, displaced towards the apex

erect, virtual, displaced towards the apex

inverted, virtual, displaced towards the base

erect, real, displaced towards the base

Your answer was INCORRECT

Explanation

The image formed by a prism is erect, virtual and displaced towards the apex.

913-Which of the following branches of the ophthalmic division of the trigeminal nerve
enters the orbit outside the circle of Zinn:

frontal and nasociliary nerves

frontal, lacrimal and nasociliary nerves

frontal and lacrimal nerves

lacrimal and nasociliary nerves

Your answer was INCORRECT

Explanation

There are three main branches of the first division of the trigeminal nerve: frontal,
lacrimal and nasociliary. The frontal and lacrimal nerves travel outside the circle of Zinn
while the nasociliary nerve travels within the circle of Zinn.

914-All of the following are true of IOP-lowering agents EXCEPT:


prostaglandin analogues reduce IOP by increasing trabecular outflow

pilocarpine increases aquous outlflow by direct mechanical effect on the ciliary body
muscle

apraclonidine decreases aqueous formation

beta-blockers decrease aqueous production by their effect on the ciliary body

Your answer was CORRECT

Explanation

Prostaglandin analogues reduce IOP by increasing uveoscleral outflow. Other options are
true.

915-Regarding genetic tests:

restriction endonucleases cut the DNA into lengths of about 20 nucleotide base sequences

mRNA can be detected with Southern blotting

polymerase chain reaction requires the use of DNA polymerase

DNA can be detected with Northern blotting

Your answer was INCORRECT

Explanation

Southern blotting is used to detect DNA whereas Northern blotting is for mRNA.
Restriction endonucleases cut the DNA into lengths of 4 to 6 nucleotide base sequences.

916-Protein degradation takes place in which one of the following areas?

peroxisome

mitochondria

Golgi body
ribosome

proteasome

Your answer was INCORRECT

Explanation

There are two major intracellular organelles to digest damaged or unneeded proteins:

 lysosomes: for extracellular proteins from endocytosis, receptor mediated


endocytosis & phagocytosis

 proteasomes: for endogenous proteins (proteins synthesised within cell)


transcription factors, cell cycle cyclins, virus coded proteins, improperly folded
proteins due to translation errors and proteins damaged by cytosol molecules.

917-Which corneal dystrophy is characterized histologically by hyaline bodies:

Lattice dystrophy

Granular dystrophy

Meesman's dystrophy

Macular dystrophy

Avellino dystrophy

Your answer was INCORRECT

Explanation

Granular dystrophy is an autosomal dominant corneal stromal dystrophy in which opaque


granules of non-birefringent hyaline bodies form in the anterior stroma.

918-Which immunoglobulin class(es) may exist in polymer form?

1. IgA
2. IgD
3. IgM
4. IgG

1, 2, and 3

2 and 4

4 only

1, 2, 3, and 4

1 and 3

Your answer was INCORRECT

Explanation

IgA may exist in a dimeric form (two subunits), especially when secreted. IgM is
produced as a pentamer (five subunits).

919-Human T cell lymphotrophic viruses (HTLV) are implicated in all of the following
EXCEPT:

nasopharyngeal carcinoma

Sezary syndrome

T cell lymphomas

mycosis fungoides

Your answer was CORRECT

Explanation

HTLV is implicated in T cell lymphomas including mycosis fungoides and Sezary


syndrome. It is endemic in Japan, Africa and the Caribbean. EBV is associated with
nasopharyngeal carcinoma.

920-Regarding T cells:
CD8 T cells are activated by T helper cells

cytotoxic T cells recognise antigen bound by MHC class II molecules

these cells tend to reside in a particular lymph node

CD4 T cells can lyse virally infected cells

only CD4 T cells proliferate in the mixed lymphocyte reaction

Your answer was CORRECT

Explanation

Lymphocytes are specialised cells which mediate the immune response. T lymphocytes
are subdivided into T-helper cells (CD4+) and cytotoxic T-cells (CD8+). T helper cells
augment cell-mediated immunity by activating macrophages and CD8 cytotoxic T cells
and also stimulate the humoral (B cell mediated) immune response. The main function of
cytotoxic T cells is to lyse infected target cells.

T lymphocytes bear receptors on their surface that recognise specific foreign peptide
antigen presented in the groove of cell surface proteins termed MHC molecules. CD8+
cytotoxic T lymphocytes recognise peptide presented by MHC class I molecules and
CD4+ T helper cells recognise peptide presented by MHC class II molecules. Whereas,
class II MHC proteins are only expressed on certain cell types termed professional
antigen presenting cells (dendritic cell, macrophages, B cells); MHC class I proteins are
present on all nucleated cells.

Lymphoid organs are the sites where antigens are concentrated and naive T lymphocytes
continually recirculate from the blood through these organs. This increases the chances of
immune cells encountering their specific antigen. After activation, effector T
lymphocytes acquire the ability to extravasate to sites of inflammation where they
accumulate. Memory T lymphocytes follow a different route to naive T lymphocytes.
They preferentially patrol sites of inflammation.

The mixed lymphocyte reaction occurs when donor and recipient lymphocytes are mixed.
The foreign HLA molecules are recognised by CD4+ T cells. These cells proliferate and
stimulate CD8+ T cells to proliferate and lyse the foreign cells.

921-The anterior portion of the optic radiation is supplied by:

anterior choroidal artery


posterior cerebral

lateral striate artery

posteror choroidal artery

Your answer was CORRECT

Explanation

The anterior portion of the optic radiation is supplied by the anterior choroidal artery.

922-Regarding lens fibres:

the lens bow is posteriorly oriented

deep lens fibres actively synthesise crystallins

lens fibres may measure up to 12mm in length

lens fibres are rich in microfilaments and microtubules oriented perpendicular to the long
axis of the cell

Your answer was INCORRECT

Explanation

Lens fibres may measure up to 12mm in length. Superficial lens fibres actively synthesise
lens proteins called crystallins. Lens fibres are rich in microfilaments and microtubules
which are oriented parallel to the long axis of the cell. The lens bow, created by the
elongation of the lens fibres coupled with the anterior migration of the nuclei as the
maturing lens fibres sink deeper into the lens, is anteriorly oriented.

923-Regarding head injury:

countercoup lesions occur at site perpendicular to impact

diffuse axonal injury only occurs when the brain moves within an intact skull

the brain heals by gliosis


diffuse axonal injury is caused by secondary brain damage

Your answer was INCORRECT

Explanation

Diffuse axonal injury is caused by primary brain damage during impact. It can occur in
both opened and closed head injury. Countercoup lesions occur at the site diagonal to the
site of impact.

924-All of the following are TRUE about the chromosomes EXCEPT:

mitosis results in 2 daughter cells each with a full complement of chromosomes

23 chromosomes are found in germinal cells

Klinefelter's syndrome results from an extra Y chromosome in the male

the Barr body is due to an inactivated X chromosome

Your answer was INCORRECT

Explanation

In Klinefelter's syndrome, the male cell has an extra X chromosome. Other statements are
true.

925-The concentration of adrenaline at the adrenergic synapse is:

increased by noradrenaline receptor blockers

decreased by cocaine

decreased by mono-amineoxidase-inhibitors

controlled mainly by the activity of the enzyme COMT

Your answer was CORRECT

Explanation
The concentration of adrenaline in the synaptic cleft is increased by:

 cocaine which inhibits adrenaline uptake


 MAO inhibitors which block the metabolism of adrenaline
 noradrenaline receptor blockers and partial agonists of noradrenaline (as they
prevent the binding and uptake of adrenaline at the post-synaptic receptor).

Note that COMT is involved in the metabolism of adrenaline intracellularly not at the
neuronal synapse.

926-All of the following are true of necrosis EXCEPT:

enzymatic necrosis occurs in acute pancreatitis

liquefaction necrosis occurs in brain infarction

chalk necrosis occurs in tuberculosis

coagulative necrosis occurs in coronary infarction

Your answer was INCORRECT

Explanation

There are several types of necrosis (cell death). All of the associations above are correct
with the exception of tuberculosis which involves casseous necrosis (not chalk necrosis,
which is a non-entity).

927-Toxacara canis:

is a intracellular protozoan

is a nematode

is a fungus

is a helminth

Your answer was INCORRECT

Explanation
Toxocara canis is a helminth. The adult forms are found in the intestines of dogs and cats
where they lay eggs that are secreted in the faeces. Treatment is with oral thiabendazole.

928-Which of the following attaches cells to the extracellular matrix and to ligands on
other cells:

peripheral membrane protein

glycoproteins

microtubules

integrins

microfilaments

Your answer was INCORRECT

Explanation

Integrins are heterodimeric transmembrane proteins that contain alpha and beta subunits.
They need magnesium and calcium to work.

929-Which is FALSE of the optic chiasm:

it has the internal carotid artery as its immediate lateral relation

it forms the floor of the recess of the third ventricle

it is inferior to the olfactory tract

it is in close relation to the oculomotor nerve superiorly

Your answer was INCORRECT

Explanation

The oculomotor nerves lie on the lateral side of the optic chiasm. Other statements are
true.

930-Which statement is FALSE regarding trachoma:


it responds to tetracycline

Stage III is characterized by fibrous replacement of inflammatory tissue

it can be spread by direct contact

Stage I involves the cornea

Your answer was INCORRECT

Explanation

Trachoma is characterized by the following recognized stages:

 Stage I: involvement of the conjunctival stroma


 Stage II: involvement of the cornea with pannus formation
 Stage III: fibrous replacement of inflamed tissue
 Stage IV: contraction with entropion and trichiasis.

931-Which of the following structures contains fifth cranial nerve fibers:

ciliary ganglion

geniculate ganglion

trigeminal ganglion

otic ganglion

Your answer was INCORRECT

Explanation

The otic ganglion is a parasympathetic ganglion located immediately below the foramen
ovale. It communicates with the glossopharyngeal and facial nerves.

The geniculate ganglion contains the facial nerve.


The trigeminal ganglion contains the fifth cranial nerve.

The ciliary ganglion contains the third cranial nerve.

The pterygopalatine ganglion (or sphenopalatine ganglion) is a parasympathetic ganglion


found in the pterygopalatine fossa. It receives a sensory, a motor, and a sympathetic root.
Its sensory root is derived from two sphenopalatine branches of the maxillary nerve. Its
motor root is derived from the nervus intermedius through the greater superficial petrosal
nerve. Its sympathetic root is derived from the carotid plexus through the deep petrosal
nerve.

932-Which statement is FALSE regarding the zonules:

they are radially inserted on the lens capsule ranging from 2mm anterior to 1mm
posterior to the equator

their site of synthesis is believed to be the non-pigment epithelial cells of the ciliary body

they arise from the pars plana ciliary epithelium

they contain a high collagen content

Your answer was INCORRECT

Explanation

The zonules are composed of a unique fibrous non-collagenous protein similar in


properties to elastin. Other options are true.

933-The nasolacrimal duct drains via:

the inferior meatus

the middle meatus

the medial meatus

the superior meatus

the lesser meatus

Your answer was CORRECT


Explanation

The nasolacrimal duct drains into the inferior meatus of the nasal cavity.

934-Which of the following bones is NOT a component of the medial orbital wall:

maxilla

lacrimal bone

frontal bone

ethmoid

Your answer was INCORRECT

Explanation

The medial orbital wall is composed of: the frontal process of the maxilla, the lacrimal
bone, the orbital pate of the ethmoid and the body of the sphenoid.

935-MRSA (methicillin-resistant Staphylococcus aureus):

is more likely to cause deeper infection than ordinary Staphylococcus aureus strains

is not usually present in healthy immunocompetent adults

is sensitive to vancomycin

is resistant to iodine solution

Your answer was INCORRECT

Explanation

Vancomycin is effective therapy for MRSA. The infection caused by MRSA is no more
deeper than the ordinary strain. It is an important cause of nosocomial pathogen. It is
transmitted between patients via the hands of health workers. Some of whom may be
healthy carriers. MRSA is found to colonize the nose, axilla and groins. It is killed by
exposure to iodine.

936-Which is FALSE regarding the organization of the primary visual cortex:


fibres from the superior retina pass to the upper lip of the calcarine sulcus

the cells in Layers II and III project to the lateral geniculate nucleus

the line of Gennari is formed by myelinated fibres projecting to Layer IV

Layer IV is more cellular and receives input from the macula

Your answer was INCORRECT

Explanation

There are six basic layers of the primary visual cortex. The layers have well established
connections including:

 Layer IV: receives fibres from the macula


 Layer II and III: project to the secondary visual cortex
 Layer V: projects to the superior colliculus
 Layer VI: projects to the LGN

937-Which statement is FALSE about the intra-cranial course of the oculomotor nerve:

it runs forward in the lateral wall of the cavernous sinus

it divides into superior and inferior branches before entering the orbit

it passes between the posterior cerebral artery and the superior cerebellar artery

it grooves the posterior clinoid process

it enters the orbit through the superior orbital fissure within the tendinous ring

Your answer was INCORRECT

Explanation

The oculomotor nerve divides into superior and inferior branches after it enters the orbit
through the superior orbital fissure. Other statements supplied are true.

938-Which is TRUE of the arterial supply of the orbit:


the inferior rectus unlike other recti receives only one muscular artery

the central retinal artery enters the optic nerve on its superior surface

the ophthalmic artery arises from the internal carotid within the cavernous sinus

the eyelids are supplied mainly by branches of the external carotid artery

Your answer was INCORRECT

Explanation

The central retinal artery enters the optic nerve on its inferior surface.
The lateral rectus and not the inferior rectus receives only one muscular artery.
The eyelids are supplied mainly by the medial and the lateral palpebral arteries, branches
of the internal carotid artery.

939-Which is the only immunoglobulin class to readily cross the human placenta?

IgA

IgE

IgM

IgD

IgG

Your answer was INCORRECT

Explanation

IgG transfer occurs both passively and by active transport across the placenta. Minimal
amounts of IgA also may cross by passive diffusion.

940-Amother is a carrier of an X linked recessive condition. What is the chance that her
daughter will also be a carrier:
50%

100%

0%

33%

75%

Your answer was CORRECT

Explanation

The answer is 50%. We recommend drawing a 2 x 2 box of chromosomal probabilities.

941-Which statement is FALSE regarding the ribosomes:

they are involved in transcription of protein

they consist of granules made up of RNA and proteins

they are found mainly in the cytoplasm

their functions are inhibited by chloramphenicol

Your answer was CORRECT

Explanation

Ribosomes are involved in the translation of protein not transcription, which is the
process whereby messanger RNA is produced from the DNA sequence. All other
statements are true.

942-Secretory Ig A:

is responsible for mucosal immunity

is transferred across placenta to the foetus


is found in the plasma

is assembled in the lymph nodes

is pentameric

Your answer was CORRECT

Explanation

IgA is a dimer. It is responsible for mucosal immunity. It is transported across the


mucosal epithelium bound to a receptor complex. It is not found in plasma and does not
cross the placenta.

943-Which is FALSE regarding the choriocapillaris:

its lobules are fed by arterioles in Haller's layer

its density is greatest at the macula

its capillaries are wide-bore

its capillaries are fenestrated

Your answer was CORRECT

Explanation

The hexagonal lobules or divisions of the choriocapillaris are each fed from arterioles
arising in Sattler's layer. Haller's layer lies beneath Sattler's layer and contains major
arteries and veins.

944-All
of the following are examples of non-granulomatous inflammatory reactions
EXCEPT:

multiple sclerosis

Bechet's disease

syphilis
thyroid eye disease

Your answer was INCORRECT

Explanation

Syphilis is characterized by a granulomatous inflammatory process.

945-In the classic wheal-and-flare reaction, which of the following is NOT involved:

mast cells

eosinophils

IgE

basophils

macrophages

Your answer was INCORRECT

Explanation

Macrophages are NOT a part of the acute-phase hypersensitivity Type I reaction.

946-Which antibody class(es) fix(es) complement?

1. IgA
2. IgM
3. IgD
4. IgG

1 and 3

4 only

1, 2, and 3

2 and 4
1, 2, 3, and 4

Your answer was INCORRECT

Explanation

IgA and IgE may play a role in activating the alternative pathway, but neither directly
binds complement components nor initiates the complement cascade like IgG and IgM.

947-The following are true about the autonomic nervous system EXCEPT:

parasympathetic fibres enter the orbit through the inferior branch of the oculomotor nerve

Horner's syndrome can occur due to an aneurysm of the internal carotid artery

the parasympathetic post-ganglionic fibres supplying the lacrimal gland arise from
the ciliary ganglion

the sympathetic system originates in the posterolateral hypothalamus

Your answer was INCORRECT

Explanation

The parasympathetic fibres to the lacrimal gland arise from the pterygopalatine ganglion.
Other statements are true.

948-All of the following are true of cerebrospinal fluid EXCEPT:

the chloride level is higher than that of arterial blood

the protein level is lower than that of arterial blood

the glucose level is lower than that of venous blood

the PO2 is the same as that of arterial blood

the pH is lower than that of arterial blood

Your answer was INCORRECT

Explanation
Almost all constituents of CSF are found in lower concentration than that of blood with
the exception of Chloride and H+, which are higher. Thus, the pH and the PO2 of the
CSF are lower than that of the arterial blood. Protein is lower in the CSF (in fact it is
almost absent in the normal CSF). The glucose in CSF is 2/3 that of the blood.

949-The corneal epithelium utilizes most of its glucose:

for ATP production aerobically via the Krebb's cycle

for ATP production anaerobically with conversion to lactate

for the production of sorbitol via the aldose reductase pathway

for the production of antioxidants via the pentose phosphate pathway

Your answer was INCORRECT

Explanation

The corneal epithelium utilizes most of its glucose (85%) for the production of ATP via
glycolysis anaerobically with lactate production. A small proportion of glucose is
metabolized aerobically via the Krebb's cycle. Some glucose is also diverted to the
pentose phosphate pathway, which is important for the production of antioxidants such as
glutathione and ascorbic acid. The production of sorbitol from glucose is a minor
pathway and only occurs to a significant degree in hyperglycemic states.

950-Four prism diopters of right hyperphoria are best corrected by:

2 dioptres base-out right eye, 2 dioptres base out left eye

4 dioptres base-down right eye

2 dioptre base-down right eye, 2 diopter base-up left eye

4 dioptres base-up left eye

Your answer was INCORRECT

Explanation

Generally, when prescribing prisms the correction is split between the two eyes.
951-All of the following are true of the optic canal EXCEPT:

it is narrowest posteriorly

it is formed by the two roots of the lesser wing of the sphenoid

it passes anteriorly, inferiorly and laterally from the middle cranial fossa to the orbit

it transmits the ophthalmic artery

Your answer was CORRECT

Explanation

The optic canal is narrowest anteriorly. Other options supplied are true.

952-Which of the following is associated with Wegener's granulomatosis:

multinucleate giant cells in the internal elastic lamina

small vessel vasculitis with granulomatous infiltration

medium sized vasculitis with fibrinoid necrosis

a linear deposition of IgG along the basement membrane

Your answer was INCORRECT

Explanation

Wegener's is characterized histologically by small vessel vasculitis with necrosis and


granulomatous inflammation.

953-The orbicularis oculi is supplied with motor innervation by:

the fourth cranial nerve

the sixth cranial nerve

the seventh cranial nerve


the third cranial nerve

the fifth cranial nerve

Your answer was INCORRECT

Explanation

The orbicularis is a muscle of facial expression and is supplied by the facial nerve
(seventh cranial nerve). It is the primary muscle responsible for lid closure.

954-The angle of minimum deviation of a prism occurs when:

the angle of incidence divided by the angle of emergence equals the refractive index

the angle of emergence is normal to the surface of the prism

the angle of incidence is normal to the surface of the prism

the angle of incidence equals the angle of emergence

Your answer was INCORRECT

Explanation

The angle of minimum deviation of a prism occurs when the angle of incidence equals to
the angle of emergence.

955-A cross-cylinder uses:

a sphero-cylindrical lens with power of cylinder twice the sphere and opposite in
sign

two cylindrical lenses of equal power and sign at 90 degrees

two prismatic lenses of equal and opposite sign at 45 degrees

two cylindrical lenses of equal power and opposite sign at 45 degrees

Your answer was CORRECT


Explanation

A cross-cylinder is a sphero-cylindrical lens with the power of the cylinder twice the
sphere and opposite in sign. This is the same as superimposing two cylindrical lenses of
equal power but opposite sign with axes at right angle.

956-All are true of Hepatitis C except:

intravenous drug users are more at risk

it is a single stranded RNA virus

it is less likely to lead to liver cirrhosis than Hepatitis B

it is implicated in malignant tranformation

Your answer was INCORRECT

Explanation

Hepatitis C is more likely to lead to carrier status and chronic hepatitis (up to 60% of
infections) that Hep B.

957-Which of the following is a Gram-positive cocci:

Streptococcus sp.

Haemophilus sp.

Moraxella sp.

Meningococcus sp.

E. coli sp.

Your answer was CORRECT

Explanation

Streptococcus species are Gram positive cocci. Other options supplied are all Gram
negative species
958-Which is FALSE regarding Human Immunodeficiency Virus (HIV):
the diagnosis of AIDS cannot be made on the basis of T lymphocyte counts

the earliest sign of infection is p24 core protein in the blood

infection can be diagnosed by detection of antibodies using ELISA

antibodies to HIV have no real protective or neutralizing function

Your answer was CORRECT

Explanation

The diagnosis of AIDS can be made both on clinical grounds by the manifestation of
indicator diseases or it can be made on the basis of T cell count where CD4 T cell count
falls below 200 per microliter.

959-Which statement about keratocytes is FALSE:

keratocytes are flattened cells and occur between lamellae

keratocytes are ovoid in shape

keratocytes are connected to neighbouring cells by gap junctions

keratocytes are modified fibroblasts

Your answer was INCORRECT

Explanation

Keratocytes are stellate in shape. Other options are true.

960-All are true of mucous membrane pemphigoid EXCEPT:

it can cause blistering of mucosa

it is an autoimmune response against basement membrane components

pathologically there is linear deposition of IgM and C3


it can cause a fibrinous reaction with scarring and symblepharon of the lids

it can be diagnosed by immunofluorescence

Your answer was INCORRECT

Explanation

Immunoflourescence study demonstrates linear deposition of IgG (not IgM) and C3 along
the basement membrane.

961-A prism with an apparent angle of deviation of 30 degrees is equivalent to:

30 dioptres

90 diptores

60 dioptres

15 dioptres

Your answer was INCORRECT

Explanation

Under standard conditions of ophthalmic usage:

1 prism diopter = half a degree angle of apparent deviation

962-All are true of the blood supply to the retina EXCEPT:

the retinal circulation has a high oxygen exchange

the retinal circulation has a high flow rate

the tight junctions of the RPE are important in the integrity of the blood-retina barrier

the inner two-thirds of the retina is nourished by branches of the central retinal artery

Your answer was INCORRECT

Explanation
The retinal circulation has a low flow rate (25mm/s) and a high oxygen exchange. By
contrast, the choroidal circulation has a high flow-rate and a low oxygen exchange.

963-Which of the following is true about body potassium:

potassium leaves the cells in the presence of insulin

the concentration of the plasma potassium increases at time of metabolic acidosis

aldosterone decreases urinary excretion of potassium

90% of the total body potassium is found in the serum

Your answer was INCORRECT

Explanation

Most of the body potassium is found within the cells. The concentration of plasma
potassium rises in metabolic acidosis. Potassium enters the cells in the presence of
insulin. Aldosterone causes sodium retention and potassium loss.

964-Choose the correct statement regarding co-stimulation of T-cells:

it only occurs in the thymus

it is always required for T-cell activation

it can activate T-cells without antigen binding

it triggers the production of IL2

Your answer was INCORRECT

Explanation

Superantigens such as TSST-1 can activate T-cells without co-stimulation. These


bacterial toxins comprise proteins which exert their effect by forming a bridge between
the MHC complex of antigen-presenting cells and T-cell receptors. Because binding to T-
cell receptors is indiscriminate, excess stimulation of T-cell proliferation occurs. Co-
stimulation of T-cells by professional antigen presenting cells is mediated by the
B7/CD28 ligand complex. This co-stimulation triggers the production of IL2, which
drives T-cell proliferation. Co-stimulation without antigen signaling is ineffective.
Interaction of the TCR with MHC on an antigen presenting cell in the absence of co-
stimulation can result in T-cell anergy. CD40/CD40L binding mediates T-cell and B-cell
interactions and isotype switching. This interaction is also necessary for optimal T-cell
function

965-Which of the following occurs locally in acute inflammation:

histamine concentration is increased

monocytes are the first cells seen in the extracellular space

reduced vascular permeability

constriction of the small blood vessels

Your answer was CORRECT

Explanation

In acute inflammation, there is vasodilatation, stasis of blood and retraction of the


endothelial cells making the blood vessels leaky. Neutrophils are the first cells seen in the
extracellular space. Histamine is an important mediator that causes vasodilatation and
increased vascular permeability.

966-When comparing the means of two large samples, which of the following statements
is TRUE?

the null hypothesis is that the population means are different

the null hypothesis is that the population means are the same

the alternative hypothesis is that the sample is different

the null hypothesis is that the sample means are the same

there is no null hypothesis

Your answer was INCORRECT

Explanation
The null hypothesis is that the population means are the same. Note that although we
perform tests and draw our conclusions from samples, null and alternative hypotheses
refer to the truth, i.e. they refer to our populations.

967-Karyotype can be obtained from all of the following EXCEPT:

retinoblastoma cells

erythrocytes

chorionic villi

neutrophils

Your answer was INCORRECT

Explanation

Normal erythrocytes do not contain nuclei and therefore it is not possible to obtain a
karyotype. All other cells listed will contain nuclei with a full complement of
chromososmes.

968-Amiodarone is commonly associated with which ocular side-effect:

corneal opacification

glaucoma

herpetic keratitis

nystagmus

Your answer was CORRECT

Explanation

Amiodarone can commonly cause corneal opacification known as vortex keratopathy

969-Which statement is TRUE about the offspring of a female carrier of an X-linked


recessive disorder and a normal male:
half of their sons will be symptomatic

half of their daughters will be symptomatic

half of their children will symptomatic

half of their sons will be asymptomatic carriers

Your answer was CORRECT

Explanation

Half of the sons will have the condition as they have a 50% chance of inheriting the X
chromosome from the mother. The other half will have a normal X chromosome.

970-ALL
of the following hormones counteract the hypoglycemic effect of insulin
EXCEPT:

growth hormone

thyroxine

cortisol

adrenaline

glucagon

Your answer was INCORRECT

Explanation

There are five hormones which counteract the hypoglycemic effect of insulin:

 glucagon
 cortisol
 growth hormone
 adrenaline
 noradrenaline

971-Which branch of the external carotid artery contributes to the blood supply of the
medial eyelids:
lacrimal artery

transverse facial artery

ophthalmic artery

angular artery

Your answer was INCORRECT

Explanation

The eyelids are supplied by the palpebral arcades (marginal and peripheral arcades)
which are themselves supplied from both medial and palpebral ends by feeder arteries as
follows:

 medial palpebral artery: supplied by the angular artery (branch of facial artery,
branch of external carotid) and the dorsalis nasi (branch of ophthalmic artery,
branch of internal carotid)

 the lateral palpebral artery: supplied by the transverse facial artery (branch of
external carotid) and the lacrimal artery (branch of ophthalmic artery, branch of
internal carotid)

972-All of the following will influence the diffusion coefficient of a drug EXCEPT:

the thickness of the cell membrane

the degree of ionization of the drug

the molecular size of the drug

the lipid solubility of the drug

Your answer was CORRECT

Explanation

The diffusion coefficient of a drug is determined by its lipid solubility, ionization and
molecular size. The thickness of the cell membrane will affect the rate of diffusion as per
Fick's law, but it has no influence on an individual drug's diffusion coefficient.
973-Which is FALSE about the superior orbital fissure:

is approximately 22mm long

it is bound by the lesser and greater wings of sphenoid

it is comma-shaped, being wider at its lateral than medial end

it is the largest communication between the orbit and the cranial cavities

Your answer was INCORRECT

Explanation

The superior orbital fissure is comma-shaped, being wider at its medial end than laterally.
Other options supplied are true.

974-Wheredoes the image formed by an object outside the centre of curvature of a


concave mirror lie:

behind the mirror

between the principle focus and the mirror

outside the centre of curvature

between the centre of curvature and the principle focus

Your answer was INCORRECT

Explanation

For an object outside the centre of curvature of a concave mirror, the image formed is
real, inverted and diminished; and it lies between the centre of curvature and the principle
focus.

975-The lacrimal fossa is formed by:

the frontal bone and the lacrimal bone


the frontal process of the maxillary bone and the ethmoid bone

the frontal process of maxillary bone and the lacrimal bone

the lacrimal bone and the ethmoid bone

Your answer was INCORRECT

Explanation

The lacrimal fossa is formed by the lacrimal bone and the frontal process of the maxilla.
It houses the lacrimal sac.

976-Psammoma bodies are seen histologically in:

optic nerve glioma

choroidal melanoma

retinoblastoma

optic nerve meningioma

Your answer was INCORRECT

Explanation

Psammoma bodies are seen histologically in mengiomas

977-Which is true of local anaesthetics:

their effects are weakened by administration with sodium bicarbonate

they are weak acids

their action is shortened by concurrent adrenaline administration

they inhibit the sodium ion pump

Your answer was INCORRECT


Explanation

Local anaesthetics inhibit the sodium pump, preventing initiation of an action potential.
They are weak bases. Local anaesthetic activity can be enhanced by altering the pH of a
drug preparation to maximise the amount of drug in the unionised (unprotonated) form.
Local acidosis such as caused by wound infection greatly reduces the action of local
anesthetics. Sodium bicarbonate on the other hand makes local anaesthetic more
hydrophobic thereby increasing its cellular penetration. This increases its potency. The
use of adrenaline causes vasoconstriction and decreases the rate of removal of LA by the
blood.

978-A 23 year old male complains of reduced vision. Systemic enquiry reveals he is under
investigation for progressive muscle weakness. On examination he has cataracts, an
expressionless face, frontal balding and gonadal atrophy.

How is this condition inherited:

mitochondrial

autosomal recessive

autosomal dominant

X-linked recessive

Your answer was INCORRECT

Explanation

This is a case of myotonic dystrophy which is an autosomal dominant condition due to a


mutation in chromosome 19.

979-Which of the following muscles is supplied by the contralateral oculomotor nucleus:

levator superioris

superior rectus

inferior oblique

inferior rectus
Your answer was INCORRECT

Explanation

The axons for most of the extraocular muscles are uncrossed from the nucleus to the eye
(in other words, ipsilateral innervation). However there are 2 exceptions:

 the levator palpebrae come from both sides of the central caudal subnucleus via
crossed and uncrossed pathways

 the superior rectus muscle innervation comes from the superior rectus subnucleus
on the contralateral side (contralateral innervation)

980-Which of the following anti-fungal agents act by interfering with the cell wall of the
fungi:

Griseofluvin

5-fluorocytosine

Cyclohexidine

Amphotericin B

Your answer was INCORRECT

Explanation

 Polyene antifungal drugs (Amphotericin, nystatin, and pimaricin) interact with


sterols in the cell membrane to form channels through which small molecules leak
from the inside of the fungal cell to the outside.
 Azole antifungal drugs (Fluconazole, itraconazole, and ketoconazole) inhibit
cytochrome P450-dependent enzymes (particularly C14-demethylase) involved in
the biosynthesis of ergosterol, which is required for fungal cell membrane
structure and function.
 Grisan (Griseofluvin) binds to tubulin, preventing microtubule assembly.
 Glutaramide (Cycloheximide) inhibits protein synthesis at ribosomal level.
 Antimetabolite antifungal drugs (5-Fluorocytosine) acts as an inhibitor of both
DNA and RNA synthesis via the intracytoplasmic conversion of 5-fluorocytosine
to 5-fluorouracil.

981-With regard to immunoglobulin A:


it is secreted by the lacrimal gland

it is the heaviest immunoglobulin

it is effective against gonococcus

it is the first immunoglobulin to be produced when the body is invaded by viruses

Your answer was CORRECT

Explanation

IgA is dimeric and therefore lighter than IgM, which is a pentamer. It is excreted in the
tears and milk and is an important line of defense of mucous membranes. It activates the
alternative complement pathway (unlike many antibodies that activate the classical
pathway). It is ineffective against gonococcus, which produces a protease against IgA.
IgM is the first antibody to appear in infection.

982-The thinnest orbital wall is:

the medial wall

the orbital floor

the orbital roof

the lateral wall

Your answer was CORRECT

Explanation

The medial wall is the thinnest of the orbital walls and is largely transparent in the dried
skull.

Score: 25 %
Question 740 Part 3 – Tutor Mode
Meissner's corpuscles are responsible for:

light touch

vibration
pain

temperature

Your answer was CORRECT

Explanation

The cutaneous receptors are as follows:

 Meissner's corpuscles: light touch


 Pacinian corpuscles: pressure and vibration receptors
 Merkel discs: light touch
 Free nerve endings: pain and temperature

984-The lens is derived from:

mesoderm

surface ectoderm

neural ectoderm

neural crest cells

Your answer was INCORRECT

Explanation

The lens is derived from surface ectoderm.

985-Vergence movement:

is limited by the near point and far point of accommodation

is stimulated by sharp images on the retina

is an involuntary eye movement

is more rapid than pursuit movement


Your answer was CORRECT

Explanation

Vergence movement is a tracking movement which is slower than pursuit movement. It is


voluntary but can be stimulated by blurred images on the retina. It is required for
binocular single vision and stereopsis and is limited by the near point and the far point of
accommodation

986-Which statement is FALSE about the corneal epithelium:

it is 50-60 micrometers thick

Dendritic cells decline sharply in density from the periphery towards the centre of the
cornea

MHC Class I cells are rare in the central corneal

it is a stratified non-keratinized squamous epithelium

Your answer was INCORRECT

Explanation

MHC Class II cells are rare in the central cornea because the central cornea is devoid of
immunocompetent cells that contain MHC Class II. Other options are true.

987-Which statement about thrombus is FALSE:

it involves mainly the intrinsic and extrinsic clotting cascade

it can cause embolism

platelets form the first layer of thrombus

it can occur in both arteries and veins

Your answer was CORRECT

Explanation
A thrombus is a solid mass of blood constituents forming within the vascular system, i.e.
both arteries and veins. Unlike simple clotting in which the clotting cascade plays an
important role, thrombus is formed by the interaction between the platelets and the blood
constituents. Fragments of the thrombus may break off into circulation causing
embolism.

988-Rods are responsible for all of the following EXCEPT:

contrast

spatial resolution

motion

brightness

Your answer was INCORRECT

Explanation

The rods are responsible for:

 contrast
 brightness
 spatial resolution

The cones are responsible for:

 fine resolution
 spatial resolution
 colour vision

989-The primary muscle responsible for lid opening is:

Muller's muscle

the levator palpebrae superioris

the orbicularis oculi


the corrugator supercilii

the frontalis

Your answer was INCORRECT

Explanation

The main muscle responsible for lid opening is the levator palpebrae superioris. The
orbicularis oculi is the main muscle responsible for lid closure. The corrugator supercilii
and frontalis are muscles of facial expression. Muller's muscle assists in lid elevation
under sympathetic innervation

990-Which is FALSE regarding cataract:

it is predisposed by mechanical trauma to the lens

it involves the breakdown of crystallins to albuminoids

it is predisposed by ultra-violet radiation

hypocalcaemia has been shown to delay cataract progression

Your answer was INCORRECT

Explanation

Cataract progression is enhanced by any metabolic disturbance that alters the


microenvironment, disrupting the lens fibre cells. This includes diabetes, hypocalcaemia,
ionizing radiation and mechanical trauma.

991-Which statement is FALSE regarding nerve supply to the cornea:

the corneal nerve supply is mainly from the short ciliary nerves

nerve bundles enter the peripheral cornea in a radial manner

individual nerve fibres terminate in the superficial epithelial layers as an intra-epithelial


plexus

nerves must pierce Bowman's layer to enter the corneal epithelium


Your answer was CORRECT

Explanation

The corneal nerve supply is mainly from the long ciliary nerves.

992-A posterior communicating artery aneurysm is most likely to cause:

a seventh nerve palsy

a sixth nerve pasy

a fifth nerve pasy

a third nerve palsy

a fourth nerve palsy

Your answer was INCORRECT

Explanation

A posterior communicating artery aneurysm is most likely to cause a third nerve palsy as
the third nerve on its exit from the brainstem runs alongside and lateral to the posterior
communicating artery.

993-Actinomyces is most likely to cause:

acute retinal necrosis

trabeculitis

keratitis

conjunctivitis

canaliculits

Your answer was INCORRECT

Explanation
Actinomyces is a Gram-positive filament which is a common cause of canaliculitis and
dacrocystitis

994-All statements are true of Schlemm's canal EXCEPT:

it is lined by endothelium

it is a circumferential channel filled with aqueous humour

it is drained by collector channels and aqueous veins

it drains aqueous humour by the non-conventional pathway

the aqueous from Schlemm's canal can join either the deep episcleral venous plexuses or
drain directly into superficial conjunctival veins

Your answer was INCORRECT

Explanation

Drainage of aqueous humour via the Schlemm's canal is described as the conventional
outflow pathway and is responsible for 70-90% of aqueous outflow. The non-
conventional pathway refers to drainage of aqueous via the intercellular spaces between
ciliary muscle fibres and the loose connective tissue of the suprachoroidal space. Other
options supplied above are true

995-A ray of light passing through the nodal point of a thin lens will:

be refracted through the focal point of the lens

pass through undeviated

be refracted parallel to the principle plane of the lens

reflect off the lens by total internal reflection

Your answer was INCORRECT

Explanation

A ray of light passing through the nodal point of a thin lens will pass through undeviated.
It is an important principle in the construction of ray diagrams, as for any object, one can
predict the image produced by drawing 2 lines (a) one passing through the nodal point
undeviated, (b) the other parallel to the principle action passing through the second
principle focus.

996-The great cerebral vein of Galen drains the deep cerebral veins into:

the straight sinus

the superior sagittal sinus

the cavernous sinus

the sigmoid sinus

Your answer was CORRECT

Explanation

The great cerebral vein of Galen drains the deep cerebral veins into the straight sinus.

997-Regarding the total cerebral blood flow:

it shows significant increase if there is an increase in carbon dioxide concentration


in the arterial blood

it shows significant decrease if the mean systemic blood pressure is reduced from 140 to
60 mmHg

it is increased during intense mental activity

it is regulated by the sympathetic nervous system

Your answer was CORRECT

Explanation

The cerebral blood flow is autoregulated and the sympathetic nervous system plays little
role. Autoregulation is very effective in maintaining the blood flow between mean
systemic blood pressure of between 60 to 140 mmHg. In the presence of increased serum
acidity as in raised arterial carbon dioxide concentration, the cerebral blood flow is
increased.
998-The intraocular pressure can fluctuate in all scenarios below EXCEPT:

diurnally

with eye movements

with fluid intake

in idiopathic optic neuritis

Your answer was INCORRECT

Explanation

The intraocular pressure does not fluctuate specifically with optic neuritis. All other
options are true. IOP is higher in winter, higher in the morning, higher when the eye is
moved away from primary position, and higher with increased body fluid.

999-Which is FALSE regarding blot hemorrhages:

they occur in the outer plexiform layer

they can be viewed with the red-free filter on ophthalmoscopy

they occur in diabetic retinopathy

they are larger than dot hemorrhages

Your answer was CORRECT

Explanation

Blot hemorrhages are typical of diabetic retinopathy and are easily visible on
ophthalmoscopy. Often the red-free filter is useful for identifying retinal hemorrhages.
Blot hemorrhages are bigger than dot hemorrhages and are due to bleeding from
capillaries tracking between the photoreceptors and the RPE.

1000-Which of the following is FALSE about ocular drug delivery:

an acidic or alkaline pH can increase drug absorption


ointments have a longer corneal contact time than drops

an acidic or alkaline pH can increase drug clearance

an increased blink rate increases topical drug absorption

Your answer was INCORRECT

Explanation

An increased blink rate reduces the topical drug absorption by increasing clearance. Eye
drop solutions are often made more acidic or alkaline in order to ensure the drug (whether
weak acid or weak base) is more un-ionised and therefore lipid soluble to increase
corneal absorption. However, acidic and alkaline pH will also tend to initiate irritation
and lacrimation, increasing drug clearance
1001-The following statements about restriction endonucleases are true EXCEPT:

they cut DNA at defined sequences

they are natural products of bacteria

they cut double-stranded DNA molecules

they act by breaking the hydrogen bonds between bases

Your answer was INCORRECT

Explanation

Restriction endonucleases act by breaking the sugar-phosphate backbones of the DNA.


Other statements are true.
1002-A lens with a focal length of 25 cm has a power of:

4 dioptres

0.04 dioptres

0.25 dioptres

25 dioptres
Your answer was CORRECT

Explanation

The dioptric power of a lens is equal to the reciprocal of the focal length measured in
meters.

1003-Which statement is FALSE regarding the facial nerve:

it gives off the deep petrosal nerve as a branch to the sphenopalatine ganglion

its fibres exit the brainstem in the cerebellopontine angle

it transmits taste fibres for the anterior 2/3rds of the tongue

it has a sensory component that supplies the outer ear

Your answer was CORRECT

Explanation

The facial nerve gives off the greater petrosal nerve (not the deep petrosal nerve) which
mediates tear secretion. Other options are true.
1004-Which is FALSE regarding the blood supply through the choroid:

the blood flow in the choroid is among the highest in the body

blood drains from the choroid by the vortex veins

the choroidal capillaries are highly fenestrated

the percentage oxygen transfer in choroidal capillaries is high

Your answer was INCORRECT

Explanation

The oxygen transfer in choroidal capillaries is low, with only 5-10% of the oxygen
having been extracted. The blood flow, however, is high at 1400 ml/min per 100g of
tissue, which is higher than the perfusion through the kidney.
1005-Where is the image produced by an object positioned at the focal point F1 of a thin
convex lens:

infinity

inside F2

further from the lens than the object

outside F2

Your answer was CORRECT

Explanation

The image formed by an object positioned at the focal point F1 of a thin convex lens is
virtual, erect and at infinity. We recommend being familiar with drawing ray diagrams by
producing, from the top of any object, 2 lines (a) one passing through the nodal point
undeviated, (b) the other parallel to the principle action passing through the second
principle focus.

1006-All of the following are true of lymph nodes EXCEPT:

B lymphocytes predominate in the follicles of the cortex

T lymphocytes predominate in the paracortex

plasma cells are found chiefly in the paracortex

secondary follicles develop upon antigenic stimulation

Your answer was INCORRECT

Explanation

Plasma cells are found mainly in the medulla. All other statements are true.

1007-The lateral geniculate nucleus is supplied by:

the anterior cerebral artery


the posterior choroidal artery

the posterior cerebral artery

the anterior choroidal artery

Your answer was INCORRECT

Explanation

The lateral geniculate nucleus is supplied by the anterior choroidal artery, which is itself
a branch of the internal carotid artery.

1008-Parasympathetic fibres to the lacrimal gland travel via all EXCEPT:

the deep petrosal nerve

the nervus intermedius

the zygomatic branch of the maxillary nerve

the pterygopalatine ganglion

Your answer was CORRECT

Explanation

The parasympathetic supply to the lacrimal nerve begins in the lacrimal nucleus of the
facial nerve. The preganglionic fibres travel via the nervus intermedius and its greater
petrosal branch to synapse in the pterygopalatine ganglion. The postganglionic fibres
enter the zygomatic branch of the maxillary nerve and travel via the zygomaticotemporal
nerve to innervate the lacrimal gland

1009-Which is TRUE about the orbit:

the nasal bone forms part of the medial orbital wall

the lesser wing of sphenoid forms part of the lateral wall

the palatine bone forms part of the posterior orbital wall

it has a volume of about 200 ml


Your answer was INCORRECT

Explanation

The volume of the orbit is approximately 30 ml.


The greater (not the lesser) wing of sphenoid forms part of the lateral orbital wall.
The nasal bone does not form any part of the orbital wal

1010-Flame hemorrhages occur in which layer of the retina:

the outer plexiform layer

Bruch's membrane

the internal limiting membrane

the inner plexiform layer

the nerve fibre layer

Your answer was INCORRECT

Explanation

Flame hemorrhages follow rupture of a small arteriole with blood tracking into the nerve
fibre layer. The shape and direction of a flame hemorrhage is due to the direction of
passage of the nerve fibres.

1011-Which is NOT true regarding ocular drug delivery:

preservatives can enhance drug delivery

addition of polyvinyl alcohol reduces the viscosity of a drop

particulate suspensions can be used in the delivery of steroids

the corneal epithelium presents a greater barrier to hydrophilic than lipophilic drugs

Your answer was INCORRECT

Explanation
Addition of polyvinyl alcohol will tend to increase the viscosity of a drop, therefore
allowing it to linger longer in the eye. Other options are true.

1012-Which is TRUE regarding the image formed by an object lying within the principle
focus of a concave mirror:

it is inverted

it is enlarged

it lies between the centre of curvature and the principle focus

it is real

Your answer was INCORRECT

Explanation

For an object within the principle focus of a concave mirror, the image formed is erect,
virtual and enlarged; and it lies behind the mirror.

1013-In Best disease there is massive accumulation of which substance in the RPE cells:

lipofuscin

melanin

drusen

myelin

lipid

Your answer was CORRECT

Explanation

Best disease is a hereditary autosomal dominant macular degeneration characterized


histologically by massive accumulation of lipofuscin in the RPE cells with atrophy of the
overlying photoreceptor layer.

1014-Which statement is FALSE regarding Streptococcus pneumoniae:


it is aerobic and non-motile

it is capable of alpha-haemolysis

it is a Gram positive cocci

it is susceptible to phagocytosis

Your answer was INCORRECT

Explanation

Strep pneumoniae produces a capsular polysaccharide which makes it resistant to


phagocytosis. Other options are true.
1015-The basal tear production rate is:

20 microlitres per minute

1.2 microlitres per minute

5 microlitres per minute

10 micolitres per minute

Your answer was INCORRECT

Explanation

Basal tear production is 1.2 microlitres per minute but a massive increase is possible
through reflex secretion induced by mechanical and psychological stimuli.

1016-The endothelial cells of the iris capillaries:

are highly fenestrated

are characterized by many gap junctions between endothelial cells

normally leak fluorescein on angiography

have a thickened basal lamina


Your answer was INCORRECT

Explanation

The iris capillaries form the major site of the blood-aqueous barrier and are structurally
specialized for this purpose. The iris capillaries are non-fenestrated, they are connected
by numerous tight junctions, and they have a thickened basal lamina strengthened by
perivascular collagenous hyalinised layers.

1017-Which is TRUE regarding the image formed by an object outside the centre of
curvature of a concave mirror:

it is virtual

it lies between the centre of curvature and the principle focus

it is erect

it is enlarged

Your answer was INCORRECT

Explanation

For an object outside the centre of curvature of a concave mirror, the image formed is
real, inverted and diminished; and it lies between the centre of curvature and the principle
focus.

1018-The macula lutea corresponds anatomically to:

the area centralis

the fovea

the peripheral retina

the foveola

Your answer was INCORRECT

Explanation
There is some ambiguity between clinical and anatomic terms for the regions of the
fundus. In summary:

 clinical : anatomical
 posterior pole: area centralis (the area between the vascular arcades)
 macula lutea: fovea (1.5 mm diameter area, yellow from xanthophyll)
 fovea centralis: foveola (0.35mm central depression with thickened margins)

1019-Where does RNA splicing occur?

ribosome

lysosome

peroxisome

Golgi body

nucleus

Your answer was INCORRECT

Explanation

RNA splicing occurs in the nucleus. This forms mRNA, which travels out of the nucleus
into the cytoplasm for RNA translation.

1020-Which of the following structures arises from the third pharyngeal pouch:

the parafollicular cells

the thymus

the superior parathyroid gland

thyroid gland

the tympanic membrane

Your answer was INCORRECT


Explanation

The thymus is the only option above arising from the third pharyngeal pouch. The
tympanic membrane arises from the first pharyngeal pouch. The superior parathyroid
gland arises from the fourth pharyngeal pouch. The parafollicular gland arises from the
fifth pharyngeal pouch.

1021-The facial nerve is responsible for all of the following EXCEPT:

motor innervation to the muscles of facial expression

motor innervation to the stapedius

parasympathetic innervation to the parotid gland

taste sensation to the anterior two-thirds of the tongue

Your answer was INCORRECT

Explanation

The facial nerve is responsible for the parasympathetic supply to the pterygopalatine
ganglion which supplies the lacrimal gland (not the parotid), as well as the choroid and
glands in and around the nose and mouth. The facial nerve also supplies motor
innervation to the muscles of facial expression, the stapedius, the stylohyoid and the
digastric. Finally, the facial nerve is responsible for taste sensation to the anterior two-
thirds of the tongue.

1022-Which of the following immunoglobulin classes is/are important in antigen reception


at the surface of lymphocytes in primary immune responses?

1. IgD
2. IgG
3. IgM
4. IgE

2 and 4

1 and 3

1, 2, and 3
1, 2, 3, and 4

4 only

Your answer was INCORRECT

Explanation

IgM, IgG, and IgD can be demonstrated on the surface of virgin B-lymphocytes and are
involved with binding antigen, leading to the activation of the cells, and confer the
capability for anamnestic responses.

1023-Select the FALSE statement below. The optic nerve blood supply:

includes the pial vessels

the majority of capillaries run external to the optic nerve

includes the central retinal vessels and their branches

includes the scleral vessels called the circle of Zinn-Haller

Your answer was INCORRECT

Explanation

The majority of capillaries pierce the nerve and course within the nerve via the glial
septae.

1024-All are involved in the parasympathetic pathway to the lacrimal gland EXCEPT:

the pterygoplatine ganglion

the superior salivatory nucleus

the nervus intermedius

the lesser petrosal nerve

Your answer was INCORRECT


Explanation

The parasympathetic pathway to the lacrimal gland begins in the superior salivatory
nucleus in the pons. Fibres pass via the nervus intermedius of CN VII, the greater
petrosal nerve to the pterygopalatine ganglion, where post-ganglionic fibres leave for the
lacrimal gland.

1025-Which is FALSE regarding a prism:

the angle of deviation is dependent on the refracting angle of the prism

the angle of deviation is dependent on the angle of incidence

the angle of deviation is dependent on the refractive index

the angle of deviation is largest when the angle of incidence equals the angle of
emergence

Your answer was INCORRECT

Explanation

The angle of deviation of a prism is dependent on three factors:

 the refractive index of the prism


 the refractive angle of the prism
 the angle of incidence of light

The angle of deviation is smallest when the angle of incidence equals the angle of
emergence, at which point it is called the angle of minimum deviation.
1026-All of the following statements regarding the culture mediums in microbiology are
true EXCEPT:

Nutrient poor agar with E.coli overlay is useful for acanthamoeba

Lowenstein-Jensen medium is best for mycobacteria

Chocolate agar is best for the growth of Haemophilus and Neisseria organisms

Blood agar is best for anaerobic microbes

McConkey agar is best for Gram negative bacteria


Your answer was INCORRECT

Explanation

Blood agar is best for aerobic microbes. Chocolate agar is a nutrient medium used in
culturing fastidious organisms such as Haemophilus species and Neisseria. It comprises
sheep blood that provides factors X (hemin) and V (nicotinamide adenine dinucleotide)
necessary for Haemophilus growth. McConkey agar is used in identification of lactose
fermenting, Gram-negative enteric pathogens and for inhibiting growth of Gram-positive
organisms. Nutrient poor agar with E. coli overlay provides nutrient for the growth of
acanthamoeba. Lowenstein-Jensen medium is used primarily for mycobacteria culture.

1027-Which is FALSE regarding the use of the cross-cylinder to assess the axis of a trial
lens during a refraction:

the axis of the trial lens is moved towards the axis of positive sign on the cross

the process is repeated until the cross offers equally unacceptable options to the patient

the handle is held in line with the axis of the trial lens

the cylindrical axis is usually assessed before the cylindrical power

Your answer was CORRECT

Explanation

The axis of the trial lens is moved towards the axis of the same sign on the cross-cylinder.
Other options are true.

1028-All of the following insert into the lateral tubercle of Whitnall EXCEPT:

suspensory ligament of eyeball

Whitnall's ligament

lateral canthal tendon

check ligament of eyeball

Your answer was INCORRECT


Explanation

Whitnall's ligament inserts 10 mm above the tubercle of Whitnall.

1029-The image formed by an object lying between the centre of curvature and the
principle focus of a concave mirror is:

it is diminished

it lies between the centre of curvature and the principle focus

it is inverted

it is virtual

Your answer was INCORRECT

Explanation

For an object lying between the centre of curvature and the principle focus of a concave
mirror, the image formed is real, inverted and enlarged; and it lies behind the centre of
curvature.

1030-Which of the following terms is NOT correctly matched:

point mutation: a single nucleotide substitution

frame-shift mutation: deletion of bases leading to incorrect reading during translation

repeat expansion: insertion of a repeated codon of three bases

missense mutation: a nucleotide change resulting in a stop codon

Your answer was INCORRECT

Explanation

A missense mutation is a single nucleotide change resulting in a different amino acid. A


nonsense mutation is a single nucleotide change resulting in a stop codon

1031-The short ciliary nerve arises from:


the ciliary ganglion

the optic nerve

the anterior ciliary nerve

the nasociliary nerve

Your answer was CORRECT

Explanation

The short ciliary nerve arises from the ciliary ganglion and carries sensory (from the
nasociliary), sympathetic and parasympathetic fibres (predominantly from the third nerve
but also from the seventh). The long ciliary nerve, which passes through the choroid and
transmits sensory fibres to the cornea, iris and ciliary body (as well as sympathetic fibres
to the dilator pupillae muscle) is a branch of the nasociliary nerve, itself a branch of the
ophthalmic nerve.

1032-Which is the largest extraocular muscle:

inferior rectus

superior rectus

medial rectus

lateral rectus

Your answer was INCORRECT

Explanation

The medial recuts is the largest extraocular muscle.

1033-Regarding drug metabolism:

lipid-soluble beta-antagonists cause bad dreams more often than watersoluble beta-
antagonists
hepatic drug metabolism often involves conversion of a water-soluble into a more lipid-
soluble drug

drugs with a high affinity for plasma protein have a very large volume of distribution

renal failure significantly increases the plasma protein binding of drugs

Your answer was CORRECT

Explanation

Lipid soluble drugs enters the blood brain barriers more readily and therefore have more
central nervous system side effects. A highly plasma protein binding drug stays in the
vascular component and therefore has a volume of distribution of about 4-5 liters.
Hepatic drug metabolism often converts a lipid-soluble drug into a watersoluble one.
Renal failure causes decreased plasma protein concentration and therefore less binding
for drugs.

1034-All of the following are true of the vitreous EXCEPT:

it is a hydrogel with a water content of 98%

it has a high concentration of hyaluronic acid

it contains mainly type I collagen

it is firmly attached to the pars plana

Your answer was INCORRECT

Explanation

The vitreous contains mainly Type II collagen fibres. It is firmly attached to: the pars
plana via the vitreous base, the posterior lens capsule, around the optic disc, along the
blood vessels.

1035-The circle of least confusion in Sturm's conoid coincides with the focal point of:

the average of the maximum and minimum curvatures

the base curve


the meridian of maximum curvature

the spherical equivalent

Your answer was INCORRECT

Explanation

The circle of least confusion in Sturm's conoid coincides with the focal point of the
spherical equivalent of the toric lens.

1036-At the limbus the following transitions occur EXCEPT:

the random array of collagenous lamellae of the sclera become the regularly arranged
corneal lamellae

Descemet's membrane and Bowman's layer terminate

the conjunctival epithelium becomes the stratified squamous keratinised corneal


epithelium

conjunctival capillaries derived from the anterior ciliary arteries terminate

Your answer was INCORRECT

Explanation

The conjunctival epithelium becomes the stratified squamous non-keratinised corneal


epithelium. Other options are true.

1037-Acetylcholine is released at all of the following sites EXCEPT:

parasympathetic postganglionic neurons

sympathetic postganglionic neurons innervating the iris

parasympathetic preganglionic neurons

sympathetic postganglionic neurons that innervates the sweat glands

sympathetic preganglionic neurons


Your answer was INCORRECT

Explanation

Cholinergic neurons are found at the following:

 all preganglionic neurons


 parasympathetic postganglionic neurons
 sympathetic postganglionic neurons that serve the sweat glands
 sympathetic postganglionic neurons on blood vessels in the skeletal muscle.

Note: the remainder of postganglionic sympathetic neurons are noradrenergic.

1038-Anaphylactic reactions involve all of the following EXCEPT:

Mast cells

Basophils

Ig E

Langhan's giant cells

Histamine

Your answer was INCORRECT

Explanation

Anaphylactic reaction (Type I hypersensitivity) involves the bindings of Ig E (which have


reacted with specific antigen) to mast cells or basophils leading to cell degranulation,
histamine release and anaphylaxis. Langhan's multinucleate giant cells occur in type IV
hypersensitivity (e.g. granuloma formation).

1039-Which is FALSE regarding the lacrimal gland:

the acinus inner wall is lined by stratified columnar cells

the lacrimal gland receives innervation from the trigeminal nerve

a layer of myoepithelial cells surrounds each acinus


the secretion from the lacrimal gland contains IgA

the secretion from the lacrimal gland is primarily proteinaceous

Your answer was CORRECT

Explanation

A lacrimal gland acinus is composed of a single layer of columnar cells whose apices are
directed towards the central lumen. Surrounding the columnar cells are myoepithelial
cells. The secretion is primarily proteinaceous but it also contains lysozymes, lactoferrin
and IgA, which are important in ocular defense. The lacrimal gland receives sensory
innervation from the trigeminal nerve and also parasympathetic innervation originating
from the lacrimatory nucleus via the nervus intermedius and greater petrosal nerve
synapsing in the pterygopalatine ganglion.

1040-Where is the image formed by an object lying within the principle focus of a concave
mirror:

between the centre of curvature and the principle focus

between the principle focus and the mirror

behind the mirror

outside the centre of curvature

Your answer was INCORRECT

Explanation

For an object within the principle focus of a concave mirror, the image formed is erect,
virtual and enlarged; and it lies behind the mirror.

1041-A cross-cylinder is a type of:

toric lens

convex cylindrical lens

concave cylindrical lens


prismatic lens

Your answer was CORRECT

Explanation

The cross-cylinder also known as the Jackson's cross-cylinder is a toric or sphero-


cylindrical lens.
1042-A 15-year-old presents with glandular fever. Which cells are not involved in the
initial presentation of viral antigen to T-helper cells:

macrophages

neutrophils

Kupfer cells

B cells

dendritic cells

Your answer was INCORRECT

Explanation

Neutrophils do not present antigen via class II MHC molecules which mediate the initial
presentation of antigen to T-helper cells. Initial presentation of antigen to naive T-
lymphocytes requires the participation of cells with special properties. These professional
antigen presenting cells express high levels of MHC class II molecules and costimulatory
molecules B7.1 and B7.2. The presentation of peptide antigen accompanied by
costimulatory signals, results in activation and differentiation of the naive T-cell into an
effector lymphocyte. Professional antigen presenting cells include Langerhans cells in the
skin, interdigitating dendritic cells, follicular dendritic cells in lymph nodes, macrophages
(in all forms - e.g. Kupfer cells) and B-cells. Langerhans cells do not express
costimulatory molecules which are needed to fully activate T-cells. These cells are
however able to endocytose antigen and then migrate to the lymphoid organs where they
differentiate into mature dendritic cells. Dendritic cells constitutively express class II
MHC, B7.1 and B7.2 and possess potent co-stimulatory activity. Class II MHC molecules
and co-stimulatory proteins are upregulated on macrophages and B-lymphocytes only
after activation by antigen. B-cells play an important role in antigen presentation during
the secondary immune response, as their high affinity receptor is able to take up and
process small quantities of antigen.

1043-All of the following structures pass through the foramen ovale EXCEPT:
the mandibular nerve

the emissary vein

the accessory meningeal artery

the greater petrosal nerve

Your answer was INCORRECT

Explanation

The mnemonic MALE is useful for remembering the structures that pass through the
foramen ovale: Mandibular nerve, Accessory meningeal artery, Lesser petrosal nerve and
Emissary vein.
1044-A unilateral blind eye is most likely to be caused by:

a stroke affecting the anterior choroidal artery

an aneurysm of ophthalmic artery

a pituitary tumour

a stroke affecting the lateral striate artery

a stroke affecting the anterior cerebral artery

Your answer was INCORRECT

Explanation

An aneurysm of the ophthalmic artery can cause compressive atrophy of the optic nerve
resulting in unilateral blindness. A pituitary tumour would cause a bitemporal hemianopia
while stroke of the anterior choroidal or lateral striate arteries which supply the optic
tracts and radiations would cause a homonomous hemianopia. An anterior cerebral artery
stroke is unlikely to have primary visual symptoms.
1045-Which is FALSE regarding amyloid:

amyloid is an insoluble protein


amyoid demonstrates red birefringence in polarized light

amyloid is deposited in the cornea in lattice dystrophy

Waldenstrom macroglobulinaemia is a systemic form of amyloidosis

Your answer was INCORRECT

Explanation

Amyloid demonstrates apple-green birefringence when stained with Congo red and
examined with polarized light.
1046-A study compared carotid endarterectomy with medical therapy for stroke prevention
over 2 years. 18% of patients in the medical therapy group developed a stroke, compared
with 8% in the carotid endarterectomy group.

What is the number needed to treat over 2 years to prevent 1 stroke?

100 /(18 - 8)

100 / 18

(18-8) / 100

20%

100 /8

Your answer was CORRECT

Explanation

NNT is defined as number needed to treat to prevent 1 event.

NNT = 100/ARR (ARR= Absolute risk reduction)

ARR = |CER - EER|

Where:
CER = control group event rate
EER = experimental group event rate

1047-The following are true about the lacrimal gland EXCEPT:


it is situated within the zygomatic process of the frontal bone

botulinum toxin inhibits lacrimal gland tear production

it is responsible for reflex tear production

the orbital portion is smaller than the palpebral portion

Your answer was INCORRECT

Explanation

The orbital portion is 3-4 times larger than the palpebral portion. Other options are true.

1048-Fibrillins are an important constituent of:

the corneal stroma

the corneal epithelium

the zonules

the sclera

Your answer was INCORRECT

Explanation

Fibrillins are extracellular microfibrils which have an important structural role in long-
range elasticity of tissues. It is a vital component of the lens zonule, the anterior lens
capsule and the vitreous.
1049-

Das könnte Ihnen auch gefallen